Endocrine practice ?'s

Lakukan tugas rumah & ujian kamu dengan baik sekarang menggunakan Quizwiz!

Which of the following is the most common cause of hypoparathyroidism? A Familial hypoparathyroidism B Idiopathic hypoparathyroidism C Severe magnesium depletion D Surgical removal of the parathyroid E Iron deposition in the parathyroid

Surgical removal of the parathyroid Choice D, surgical removal of the parathyroid glands, is the correct answer. Surgery for head and neck cancer, thyroidectomy, and parathyroidectomy are the most common causes of hypoparathyroidism. Choices A, B, C, and E are all causes of hypoparathyroidism that occur more infrequently.

A 38-year-old woman presents for treatment of a minor burn on her left hand. She is nervous, hyperexcitable, and experiencing palpitations. Her pulse is up to 110/min; her blood pressure is 120/80, and you notice a fine tremor of her hands. Upon further questioning, she tells you that she has been losing weight even though she's been eating a lot; she also sweats a lot, especially at night, and she has problems sleeping. She also notes frequent bowel movements, occasional diarrhea, and irregularities in her menstruation. What is the most likely diagnosis? Answer Choices 1 Hyperthyroidism 2 Hypothyroidism 3 Menopause 4 Malignant tumor 5 Pheochromocytoma

hyperthyroidism

A 62-year-old woman presents to her OB/GYN clinic for an annual exam. She reports that she has not felt well for several months, but her family practice physician retired, and she kept hoping her symptoms would improve. She reports stretch marks and a weight gain of about 20 pounds, especially noticeable in her face. She feels fatigued and has no sex drive; she also feels more irritable. She notes increased dark hairs growing on her chin. The patient reports easy bruising, but denies blood in the stools. She denies fever, chills, foreign travel, a change of sex partner, and suicidal thoughts. Her past medical history is significant for asthma, with periodic hospitalizations for asthma attacks. She achieved menopause at the age of 51 years. She has not had any surgeries. She takes prednisone 40 mg daily, an inhaler with fluticasone propionate/salmeterol 2 times daily, and an albuterol inhaler as needed for her asthma. She has no allergies. She is retired and lives at home with her husband; she denies use of tobacco, alcohol and street drugs. BP is 162/92 mm Hg, and pulse is 98. Respiratory rate is 12. Weight is 170 lbs, and height is 63", making her BMI 30.11. On physical exam, the patient has a rounded, full face; she appears fatigued, but she is pleasant and cooperative. Striae are noted on her abdomen, and hirsutism is present on her chin and abdomen. There appears to be a large fat deposit on the upper back. Her abdomen is obese, with some muscle wasting on the patient's extremities. The remainder of her exam is normal. Question What prescription intervention is the most appropriate for this patient's condition? Answer Choices 1 Discontinue inhaled fluticasone propionate/salmeterol 2 Discontinue prednisone 3 Initiate beta-blocker (e.g., metoprolol) 4 Initiate combination hormone therapy (e.g., estradiol/norethindrone acetate) 5 Initiate stimulant for weight loss (e.g., phentermine hydrochloride)

Explanation This patient is presenting with Cushing syndrome, an excess of glucocorticoids; it is most likely iatrogenic, caused by her daily prednisone. The most appropriate prescription intervention for her condition is to discontinue the prednisone. This common steroid is often given for acute flares of chronic diseases. However, chronic daily use can lead to a glucocorticoid excess known as Cushing syndrome. Cushing syndrome is characterized by central obesity, muscle wasting, hirsutism, a 'buffalo hump' on the upper back, 'moon facies', weight gain, hypertension, decreased libido, irritability, and several other possible signs/symptoms. Unrecognized and untreated Cushing syndrome can lead to osteoporosis. Tapering off the prednisone (rather than immediate discontinuation) helps prevent sudden adrenocortical insufficiency. This patient's inhaled fluticasone propionate is also a steroid medication; it is given to reduce inflammation in the airways. (The salmeterol is a long-acting beta-agonist.) This patient has no current respiratory signs or symptoms, so we can assume her asthma is currently controlled. Typically, the inhaled and intranasal steroids are considered low enough strength that they will not lead to corticosteroid excess and Cushing syndrome. The systemic steroid, prednisone, is much more potent, so it is the likely culprit for this patient's symptoms. It would be inappropriate to discontinue her inhaled fluticasone propionate/salmeterol, unless an alternate asthma treatment is started; changing the inhaled asthma medication would not alter her Cushing syndrome symptoms. This patient's blood pressure is elevated, but not at urgently high levels. Cushing syndrome is associated with hypertension. It would be wise to discontinue the prednisone and monitor this patient's blood pressure. With resolution of her Cushing syndrome (and anticipated weight loss), the elevated blood pressure may normalize. If her blood pressure remains elevated, it would not be advisable to initiate a beta-blocker (e.g., metoprolol) due to the common side effect of fatigue; this patient already complains of fatigue. This patient's current condition cannot be attributed to normal menopausal changes. Normal post-menopausal changes include vasomotor symptoms (such as hot flashes and night sweats) and vulvovaginal changes (such as atrophy, vaginal dryness and dyspareunia). Some women may experience some mild weight gain and irritability, but this patient's other symptoms are not related to drops in sex hormones. It may be acceptable to initiate combination hormone therapy (e.g., estradiol/norethindrone acetate) for women with acute menopausal symptoms, but it would not be appropriate in this patient's case. This patient's concerns should not be attributed to a weight-gain only issue. Weight gain is a common side effect from exogenous corticosteroids and should reverse with discontinuation of the medication. A dietician referral could be offered in addition to treatment of this patient's Cushing syndrome. It would be inappropriate to initiate a stimulant for weight loss (e.g., phentermine hydrochloride), especially with elevated blood pressure/hypertension.

Consideration should be given to screening patients with type 1 diabetes mellitus should also be screened for which of the following: A sarcoidosis B Sheehan's syndrome C Sjögren's Syndrome D thyroid disease

sjogren syndrome

A 32-year-old woman presents with paroxysmal episodes of headache, palpitations, and diaphoresis; the episodes last 20 minutes. During 1 episode, her blood pressure was checked and found to be 194/110 mm Hg. These episodes can occur with activity or at rest and she denies known precipitating events. On examination, blood pressure is 120/88 mm Hg, pulse is 82/min, temperature 98.6° F, and respirations 14/minute. Physical assessment, including cardiovascular and neurologic examination, is unremarkable. What is the most likely diagnosis? Answer Choices 1 Hyperthyroidism 2 Anxiety attack 3 Pheochromocytoma 4 Renal artery stenosis 5 Coronary artery disease

Pheochromocytoma Explanation Headache, palpitations, and diaphoresis are the classic triad of a pheochromocytoma crisis. These catecholamine-secreting tumors, most commonly found in young to mid-adult women, present with paroxysmal episodes in over 50% of patients. The episodes are of sudden onset, last minutes to hours, and they sometimes occur as a result of activity affecting the abdomen; often, they occur without a clear precipitating event. Hypertension with tachycardia occurs in most patients, presenting only during attacks in 40% of those patients, while the remaining 60% have sustained hypertension with half of those patients also experiencing hypertensive crises. In the absence of sustained hypertension or a paroxysmal episode, physical exam is often unremarkable. Diagnosis is made based on a 24-hour urine specimen for vanillylmandelic acid (VMA), metanephrines, and free catecholamines. Urine collection should occur when the patient is hypertensive or initiated at the time of a paroxysmal crisis. Associated symptoms during a paroxysmal episode may include apprehension or a sense of doom suggesting an anxiety attack. Although anxiety attacks present as recurrent, unpredictable episodes of intense fear that can be associated with palpitations and sweating, they lack the malignant hypertensive response of a pheochromocytoma. A pheochromocytoma crisis may precipitate chest pain with nausea, vomiting, pallor, and arrhythmias. The catecholamine surge, rather than coronary artery disease, may induce myocardial ischemia, resulting in angina and myocardial infarction. Signs and symptoms of increased metabolic rate associated with a pheochromocytoma such as weight loss, palpitations, and sweating may also suggest hyperthyroidism. The most common cardiac manifestation in thyrotoxicosis is an arrhythmia, such as sinus tachycardia or atrial fibrillation in patients > 50 years of age, and not the malignant hypertension associated with a pheochromocytoma. Renal artery stenosis presents as an abrupt onset of hypertension, malignant and/or refractory, typically associated with abdominal and/or flank bruits. When it occurs in younger women, it is most often the result of fibromuscular dysplasia of the arterial wall. When the onset occurs at > 50 years of age, it is most often the result of atherosclerotic plaques.

A 45-year-old man with a history of neck irradiation for Hodgkin lymphoma at the age of 15 is found to have a 1.5-cm, nontender, firm thyroid nodule. Upon laboratory evaluation, the patient is found to be euthyroid, and fine needle biopsy reveals malignancy. What histologic type is most likely? A anaplastic B follicular C medullary D papillary

D Papillary Thyroid carcinoma often presents as an asymptomatic thyroid nodule. The most common histologic form is papillary carcinoma, representing more than 80% of cases.

A patient was recently diagnosed with acromegaly. He was reading on the Internet that acromegaly is associated with diabetes mellitus. He is especially concerned, since his father was recently diagnosed with diabetes as well. How would you reply to the patient's following question: "What percentage of people with acromegaly develop diabetes mellitus?" A 10% B 25% C 50% D 65% E 80%

25% The correct choice is B, 25%. Growth hormone is a counter-regulatory hormone of insulin, and therefore acts against insulin. This can lead to hyperglycemia, glucose intolerance, and diabetes mellitus in 25% of patients with acromegaly.

A 38-year-old male complains of increasing fatigue, weakness, weight loss, and intermittent nausea, vomiting, and diarrhea over the past few months. He has noted some agitation at times. When this first started he thought that this was related to a virus but the GI symptoms have reoccurred on multiple occasions. Labs show a complete blood count (CBC) within the normal reference range. He is noted to have hyponatremia. On examination you note the multiple areas of hyperpigmentation as seen below. His blood pressure in the office is 100/50, P = 66, T = 97.1˚F. What test would you order to confirm your suspected diagnosis? A dexamethasone suppression test B vasopressin challenge test C radioactive iodine uptake scan D cosyntropin stimulation test E follicular stimulation test

Cosyntropin stimulation test

A new patient is seen in your internal medicine office today. She is coming in to request the removal of several skin tags. She is a 55-year-old woman with a history of untreated acromegaly. A health maintenance plan is set up with the patient, and includes a colonoscopy. This patient is at increased risk for which of the following findings on colonoscopy? A Anal fissures B Ulcerative colitis C Colon polyps D Pseudomembranous colitis E Colonic fistulas

Colon polyps The correct choice is C, colon polyps. Approximately 30% of patients with acromegaly have been found to have colon polyps. These patients also have an increased risk of colon cancer. Patients with acromegaly have not been found to be at increased risk for the other response choices listed here.

A 22-year-old woman presents with an 8-month history of amenorrhea. The patient is also experiencing backaches, headaches, and acne. Physical examination reveals a female patient with a moon-shaped facies, multiple purple striae, and significant central obesity (body mass index of 36). Question What is the most likely diagnosis? Answer Choices 1 Diabetes type II 2 Hypothyroidism 3 Cushing syndrome 4 Hyperthyroidism 5 Diabetes insipidus

Cushings Syndrome Explanation This patient likely is suffering from Cushing syndrome, which is also known as hypercortisolism. Consequences of excessive levels of circulating cortisol, no matter the etiology, will lead to signs and symptoms such as central obesity, thin extremities, a moon face, a buffalo hump, supraclavicular fat pads, protuberant abdomen, oligomenorrhea, and amenorrhea; in men, erectile dysfunction is sometimes seen. Backaches, headaches, hypertension, acne, purple striae, and impaired wound healing may also be found in these patients. Diabetes type II is caused by insulin resistance in tissues to circulating endogenous insulin. Eventually, insulin is unable to be produced by the pancreatic islet cells, and an exogenous supply is required to prevent significant hyperglycemia and its related co-morbidities. Features of these patients include central obesity (specifically visceral obesity), with less fat noted on extremities. Patients will have a history or evidence of acanthosis nigricans, eruptive xanthomas on flexor surfaces, skin tags, chronic skin infections, frequent candidal vulvovaginitis in women, and erectile dysfunction or balanoposthitis in men. Both hypothyroidism and hyperthyroidism are related to dysfunction of the thyroid gland; hypothyroidism is due to failure of the thyroid gland itself or deficient supply of pituitary TSH (thyroid stimulating hormone). Symptoms characteristic of hypothyroidism include weight gain, fatigue, lethargy, depression, weakness, menses irregularities, athralgias, muscle cramps, cold intolerance, dry skin, and constipation; other signs may include thinning hair, brittle nails, bradycardia, or even peripheral edema. Hyperthyroidism is the disease state in which there is an excessive serum level of T3 and or T4 along with suppressed TSH levels. Signs and symptoms may include restlessness, nervousness, heat intolerance, increased sweating, weight loss, palpitations, atrial fibrillation, exophthalmos, or (depending on the severity) even pretibial myxedema. Diabetes insipidus (DI) is an antidiuretic hormone deficiency. Signs and symptoms of DI include an extreme, unquenchable thirst with a very specific craving for ice water. Patients also will have significant polyuria or hypernatremia.

Your patient is taking atorvastatin 40 mg daily for her history of hyperlipidemia. Based on the potential side effects of this medication what labs are indicated for periodic monitoring? A HGB/HCT B AST/ALT C WBC count D TSH/T4 E B12

AST/ALT The correct answer is (B). Atorvastatin is an HMG-CoA reductase inhibitor, which has a potential to cause liver injury. It is recommended that liver enzymes are monitored regularly due to the potential for liver injury. The remaining choices are incorrect because statins are not known to cause changes in these laboratory values and monitoring based on potential adverse reactions to atorvastatin is not recommended.

A 45-year-old man presents with severe pain in the left flank that began in the morning; it comes and goes, lasts 20 minutes, and has an intensity of 10/10. He denies fever or urinary symptoms. He has a cousin with pancreatic cancer. The physical exam (PE) is negative. Laboratory analysis shows UA with uncountable erythrocytes, and high plasma levels of PTH. Question What is the most probable diagnosis? Answer Choices 1 Li-Fraumeni syndrome 2 Von Hippel-Lindau disease 3 Turcot syndrome 4 Cowden syndrome 5 MEN1

Explanation Multiple endocrine neoplasia type 1 (MEN1) is an autosomal dominant tumor predisposition. The diagnosis is often made in patients between 40 - 50 years of age; the disease is characterized by tumors of the parathyroid (83 - 97%), the pancreas (38 - 84%), and the pituitary (18 - 65%). Hyperparathyroidism is the most common manifestation. MEN1 is caused by mutations in the MEN1 gene on chromosome 11q13. The diagnosis is important because the treatment of tumors in patients with this mutation can be different than customary treatment. Li-Fraumeni is an autosomal dominant tumor predisposition syndrome. Mutations in p53 have been identified in 40% of LFL. The tumors appear before 45 years of age. Usually, they are soft tissue sarcomas or osteosarcomas, but they can also manifest as brain cancer, breast cancer, leukemia, and other cancers. There are 3 criteria that are needed to be able to make this diagnosis: the patient had a sarcoma before the age of 45 a first-degree relative had a diagnosis of any cancer before the age of 45 another first-degree or second-degree family member had a tumor before the age of 45 or a sarcoma at any age Von Hippel-Lindau disease is an autosomal dominant tumor predisposition syndrome. The main criterion for the diagnosis is the presence of hemagioblastomas. Patients also can have multiple benign and malignant tumors of the central nervous system, kidneys, pancreas, adrenal glands, and paraganglia. The most malignant tumor that these patients can present with is renal cell carcinoma; it is seen in more than 50% of patients. Another tumor is the pheochromocytoma which is seen in 7 - 20% of patients. This tumor can produce a sudden release of catecholamines and result in death, especially pre or post-surgery. Turcot syndrome is an inherited tumor predisposition. It is characterized by the presentation of colon cancer and brain cancer in the same patient. There are at least 2 types: familial adenomatouspolyposis (FAP) and medulloblastoma caused by APC germline mutations hereditary nonpolyposis colon cancer (HNPCC) and glioblastoma caused by germline mutations in 1 of the DNA mismatch repair genes (e.g., hPSM2 or hMLH1) Cowden syndrome is an autosomal dominant tumor predisposition characterized by the development of multiple mucocutaneous lesions (hamartomas) and benign tumors. Patients also have a predisposition to a variety of neoplasias, particularly breast and thyroid cancers. This is related to mutations in the PTEN tumor suppressor gene.

A 22-year-old woman presents with an 8-month history of amenorrhea. Further questions elicit additional pertinent positives of backaches, headaches, and acne. Physical examination reveals a female patient with a moon-shaped face, multiple purple striae, and significant central obesity (body mass index of 36). Question Considering the given history and physical exam discoveries above, what is the main cause of these findings? Answer Choices 1 Hyperglycemia 2 Hypothyroidism 3 Hypercortisolism 4 Hyperthyroidism 5 Low antidiuretic hormone

Hypercortisolism Explanation This patient likely is suffering from Cushing syndrome, also known as hypercortisolism. Consequences of excessive levels of circulating cortisol, no matter the etiology, will lead to signs and symptoms such as central obesity, thin extremities, a moon face, a buffalo hump, supraclavicular fat pads, protuberant abdomen, and complaints of oligomenorrhea, amenorrhea, or possibly erectile dysfunction in men. Backaches, headaches, hypertension, acne, purple striae, and impaired wound healing may also be found in these patients. Diabetes type 2 is caused by insulin resistance to circulating endogenous insulin in tissues. Eventually insulin is unable to be produced by the pancreatic islet cells, and an exogenous supply is required to prevent significant hyperglycemia and its related co-morbidities. Features of these patients include central obesity (specifically visceral obesity) with less fat noted on extremities. Patients will have a history or evidence of acanthosis nigricans, eruptive xanthomas on flexor surfaces, skin tags, chronic skin infections, frequent candidal vulvovaginitis in women, and erectile dysfunction or balanoposthitis in men. Hypothyroidism and hyperthyroidism both are related to dysfunction of the thyroid gland; hypothyroidism is due to failure of the thyroid gland itself or deficient supply of pituitary TSH (thyroid stimulating hormone). Symptoms exhibited with hypothyroidism are weight gain, fatigue, lethargy, depression, weakness, menses irregularities, athralgias, muscle cramps, cold intolerance, dry skin, and constipation; signs present may include thinning hair, brittle nails, bradycardia, or even peripheral edema. Hyperthyroidism is the disease state wherein there is an excessive serum level of T3 and/or T4 along with suppressed TSH levels. Signs and symptoms may include: restlessness, nervousness, heat intolerance, increased sweating, weight loss, palpitations, atrial fibrillation, exophthalmos, or even pretibial myxedema, depending on the severity. Diabetes insipidus (DI) is antidiuretic hormone deficiency. Signs and symptoms of DI include an extreme, unquenchable thirst with a very specific craving for ice water. Patients also will have significant polyuria or hypernatremia.

In a patient who does not have thyroid disease, an elevated serum thyroid stimulating hormone (TSH) may be found under which of the following conditions? A acute corticosteroid administration B acute psychiatric admission C development of an hCG-secreting tumor D pregnancy E use of amphetamines

acute psych admission

A 53-year-old man is taking a proton pump inhibitor for GERD symptoms, a beta blocker and a thiazide diuretic for hypertension, an SSRI for depression, and an over-the-counter NSAID as needed for aches and pains. He has developed gynecomastia and laboratory studies reveal an elevated prolactin level. If his hyperprolactinemai is due to one of his medications, which is the most likely cause? A the beta blocker B the NSAID C the proton pump inhibitor D the SSRI E the thiazide diuretic

the SSRI Many medications cause hyperprolactinemia, including SSRIs, tricyclic antidepressants, and antipsychotics. Hydralazine and methyldopa, but not beta blockers (A), may also raise prolactin levels; likewise opioids, but not NSAIDS (B). Cimetidine and ranitidine, but not proton pump inhibitors (C) are included among possible pharmaceutical causes. Thiazide diuretics (E) are not know to raise prolactin levels.

An 18-year-old woman comes in for evaluation of "losing weight without meaning to." She also feels weak and in "always in the bathroom." Her appetite is normal but she "can't get enough to drink." Examination shows that she has lost 17# since her last visit a year ago. She is 66" tall and now weighs 120#. She is mildly orthostatic, but no other abnormalities are noted. A random blood sugar done in the office is 260mg/dl. Which results are most likely on measurement of her lipoproteins at this time? A decreased high density lipoprotein level B extremely elevated triglycerides C markedly increased total cholesterol D mildly elevated low density lipoproteins E normal profile

mildly elevated low density lipoproteins In persons with diabetes mellitus, type 1, low density lipoproteins, trigycerides (B), and total cholesterol (C) are likely to be slightly elevatated. High density lipoproteins (A) remain about the same as the patient's baseline. Once the glucose level is controlled, the lipid levels (E) on the profile typically return to normal.

A mother expresses concern for her teen son after feeling a lump in his neck. He has no history of trauma to his neck. Surgical history is negative, and the patient does not take any medications. The mother tells you that thyroid problems run in the family. The patient has not been ill recently. Upon exam you feel a nontender, firm nodule on the right side of his thyroid with associated cervical lymphadenopathy. His serum TSH level is within the reference range. Radionuclide thyroid scanning demonstrates a "cold" nodule in the right side of the thyroid. What is the most appropriate next step in the work up of this patient? A MRI of the anterior pituitary B CT of the thyroid C MRI of the thyroid D Thyroid nodule fine needle aspiration E Emergent thyroidectomy

D Thyroid nodule fine needle aspiration The correct choice is D, thyroid nodule fine needle aspiration. With the advent of fine needle aspiration, it has become much easier, safer, and more reliable to obtain a specimen for biopsy. This patient has several characteristics that increase his risk of malignancy including his gender, young age, firmness of the nodule, and related lymphadenopathy. These, along with the ease of biopsy, suggest this path for diagnostic work up. Not enough information is known to warrant an emergent thyroidectomy, choice E. Choice A, MRI of the anterior pituitary, would be warranted if there was a suspicion of a pituitary cause of the thyroid nodules. Since the TSH is normal and the patient is not presenting with headaches or other pituitary related symptoms, this is not suggested. Choice B, CT of the thyroid, and choice C, MRI of the thyroid, would not provide any additional information after the thyroid scan. They may be helpful prior to any surgery if needed.

A 5-year-old girl presents with a 2-month history of a gradually increasing swelling in the neck. The child is otherwise asymptomatic. There is a similar complaint in most neighboring children. Cauliflower is the main crop in the region. She has no other developmental problems. On physical exam, her thyroid gland is non-tender, soft, and symmetrically enlarged without any palpable nodules or signs of compression. The thyroid function test showed slightly low thyroxine (T4) and normal TSH and triiodothyronine (T3) levels. Her urinary iodine level was also low. Question What is the most likely diagnosis? Answer Choices 1 Toxic nodular goiter 2 Graves' disease 3 Diffuse non-toxic goiter 4 De Quervain thyroiditis 5 Multinodular goiter

Diffuse non-toxic goiter Explanation The clinical presentation, exam, and lab findings are suggestive of diffuse non-toxic goiter. Diffuse non-toxic goiter, colloid goiter, or simple goiter is the name given to the diffuse enlargement of the thyroid gland unaccompanied by hyperthyroidism. Most cases are in the euthyroid stage, although they have passed a transient phase of hypothyroidism due to iodine deficiency. TSH levels are normal or slightly increased. Goiter occurs in 2 forms, either endemic or sporadic. The endemic form occurs due to the lack of dietary iodine or due to the intake of goitrogens like consumption of cassava root, cabbage, cauliflower, turnips, etc. Endemic goiter is most prevalent in high mountainous areas. Sporadic form is more common in the U.S. and in Europe, and the etiology is not known. This is said to be multifactorial in origin. Generally, most simple goiters are asymptomatic, manifesting as a painless symmetric enlargement of the thyroid gland. Spontaneous hemorrhage may cause sudden onset of pain and swelling. Examination of the gland reveals a nontender, soft gland with symmetric enlargement, without any palpable nodules. A low urinary iodine level (<10 μg/dL) is supportive of a diagnosis of iodine deficiency goiter. A thyroid function test generally shows a low total T4, with normal T3 and TSH. Treatment is with iodine or thyroid hormone replacement, which induces variable levels of goiter regression, depending on the duration of swelling and the degree of fibrosis. Multinodular goiter is an end stage of a long-standing simple goiter, and it is characterized by an extreme degree of thyroid enlargement with characteristic nodularity. Most cases are euthyroid, but about 10% cases develop thyrotoxicosis. Large goiters can cause compressive symptoms such as dysphagia, respiratory distress (due to tracheal compression), or plethora (venous congestion). On examination of the thyroid gland, the architecture is distorted with presence of multiple nodules of varying size. A thyroid function test is usually normal. Toxic nodular goiter is characterized by subclinical hyperthyroidism or mild thyrotoxicosis in addition to features of goiter. The clinical presentation of toxic nodular goiter includes tachycardia, nervousness, atrial fibrillation or palpitations, tremor, or weight loss. Plummer disease or toxic nodular goiter differs from Graves' disease by lack of ophthalmopathy and dermatopathy. The thyroid function test shows low TSH level with normal or minimally increased T4 and elevated T3, which is usually of a greater degree than T4. Heterogeneous uptake and multiple regions of elevated and diminished uptake is evident on thyroid scan. Graves' disease is characterized by hyperthyroidism, diffuse thyroid enlargement, and ophthalmopathy. It is an autoimmune disease. Thyroid function test shows low TSH level and elevated total and unbound thyroid hormone levels. De Quervain thyroiditis is characterized by a transient inflammation of the thyroid gland, which may be due to viral infections. Patients generally present with a painful, enlarged thyroid gland, and it is sometimes associated with fever. Thyroid function test evolves through 3 distinct phases: the thyrotoxic phase, the hypothyroid phase, and the recovery phase. The thyrotoxic phase shows elevated T4 and T3 levels and suppressed TSH levels. A high ESR and low radioiodine uptake helps in confirming the diagnosis.

A patient with long-standing, untreated acromegaly is seen in your office with symptoms of severe headaches. After completing a thorough history and physical exam, you order a set of x-rays including a skull series. Which of the following findings would you expect in this patient? A Punched out lesions B Basilar skull fracture C Metastatic bone lesions D Enlarged sella tursica E Thinning of the skull

Enlarged Sella tursica The correct choice is D, enlarged sella tursica. This finding is seen in 90% of patients with acromegaly. Other findings on skull radiographs include thickened calvarium (upper portion of the skull), enlarged mandible, and sinuses. Bony growth is a hallmark of the disease. The pituitary adenoma, which typically causes the disease, can be found in the sella tursica. This disorder doesn't typically metastasize, and is not associated with metastatic bone cancer, as noted in choice C. Choice A, punched out lesions, are commonly associated with Paget's disease. There is no history of head trauma, as would be the case in choice B, basilar skull fracture. As noted earlier, the skull may be thickened. Therefore, choice E, thinning of the skull, would not fit this patient's presentation.

A 56-year-old man presents with signs and symptoms of hyperthyroidism in the presence of a palpable goiter. Laboratory studies are borderline suggestive, but they do not confirm thyrotoxicosis. Further testing includes a Radioactive I 131 Uptake (RAIU) and scan of the thyroid, which reveals a high uptake in a diffuse, uniform pattern throughout the gland. Question What is the most likely diagnosis? Answer Choices 1 Thyroiditis 2 Graves' disease 3 Toxic multinodular goiter (TMNG) 4 Toxic Thyroid Adenoma 5 Thyroid cancer

Graves' Disease Explanation The correct answer is Graves' disease, which is identified on Radioactive I 131 Uptake (RAIU) with an increased amount of iodine uptake consistent with hyperthyroidism and a scan that reveals a diffuse, uniform pattern throughout the gland. This is due to the autoimmune process of Graves' disease, in which circulating Thyroid Stimulating Immunoglobulins affect the entire gland. On physical exam, the gland will be firm and diffusely enlarged, which explains the diffuse uptake of iodine throughout the gland in the typical uniform pattern on RAIU and scan. A toxic thyroid adenoma and toxic multinodular goiter (TMNG) are also etiologies of thyrotoxicosis, and they can present with signs, symptoms, and laboratory values consistent with hyperthyroidism. On thyroid exam, a toxic adenoma is often a palpable thyroid nodule; however, a multinodular goiter will present, just as the name implies, with many fibrotic areas in an enlarged gland. Likewise, a toxic adenoma and TMNG may have an increased uptake on RAIU consistent with thyrotoxicosis, but they are distinguished from Graves' disease on the subsequent scan. A toxic adenoma reveals 1 area of increased uptake, which is localized to the adenoma and is often described as a "hot nodule," indicating the adenoma is hyperfunctioning. TMNG will reveal a heterogeneous pattern of radioactive iodine uptake localized to multiple hyperfunctioning nodules of various sizes throughout the gland. Patients with thyroid cancer most often present asymptomatically, with or without a palpable thyroid nodule and normal thyroid function studies. There is typically a normal RAIU uptake, but the scan reveals an area of decreased iodine uptake localized to the site of the thyroid cancer, which is often referred to as a "cold nodule." Thyroiditis usually occurs during or after a respiratory illness, and it presents acutely with symptomatic pain in the gland; the pain is often associated with fever, elevated WBCs, and sedimentation rate. Changes in serum thyroid hormone levels will depend upon the phase of the disease. Early in the disease, thyroid function is normal; then, hyperthyroidism develops as thyroid hormone is released from the gland after the initial insult. The pituitary gland responds to the excess circulating thyroid hormone by decreasing TSH production. Subsequently, the patient will progress from hyperthyroidism to euthyroidism to hypothyroidism. As thyroid hormone levels drop, the pituitary will respond by increasing TSH production, and the patient will enter the recovery phase, resuming normal thyroid function. RAIU will reveal a normal or low iodine uptake because there is no increased thyroid tissue activity and there is often no pattern visible on scan due to the limited uptake.

Patients prescribed which of the following medications may present with a dorsocervical fat pad, thin extremities, and central obesity, which may mimic Cushing's syndrome? A ACE inhibitors B HIV antiretrovial medications C Loop diuretics D Opioid analgesics E Aminoglycosides

The correct choice is B, HIV antiretroviral medications. Patients using highly active antiretroviral therapy (HAART) can develop partial lipodystrophy and changes in body fat distribution. This complication occurs more commonly with long-term use, and affects 1/3 to 2/3 of patients on therapy for longer than one year. The other medication classes noted are not associated with lipodystrophy. Opioids, choice D, are associated with a decline of cortisol secretion.

A 60-year-old man presented with a mass in the left lobe of the thyroid. Fine needle aspiration was consistent with papillary carcinoma. There was no evidence of locally invasive or metastatic disease. Which of the following treatments is recommended for this patient? A Chemotherapy B External beam radiation C Preoperative radioiodine ablation D Total thyroidectomy

Total thyroidectomy Papillary carcinoma is the most common type of thyroid malignancy. Treatment includes a thyroid lobectomy and isthmusectomy or total thyroidectomy. The decision regarding the extent of the surgery is based on the extent of the disease, the tumor size, and histiologic grade. A poor prognosis is seen in males, patients older than 50 years of age, primary tumors greater than 4 cm in size, tumors that are less well differentiated, or evidence of locally invasive or metastatic disease. Accordingly, the recommended treatment for this patient is a total thyroidectomy. Radioiodine ablation is recommended postoperatively.

In a patient who does not have thyroid disease, an elevated serum thyroid stimulating hormone (TSH) may be found under which of the following conditions? A acute corticosteroid administration B acute psychiatric admission C development of an hCG-secreting tumor D pregnancy E use of amphetamines

acute psychiatric admission About 15% of patients who are admitted for an acute psychiatric illness will have an elevated TSH in the absence of thyroid disease. Acute corticosteroid administration (A), hCG-secreting tumors (C), pregnancy (D), and use of amphetamines (E) are all associated with low TSH levels.

Which of the following statements regarding diabetic medications is/are most correct? A Incretin-mimetics like exenatide commonly result in weight loss. B Thiazolidinediones (TZDs) should be held prior to and for 48 hours after administration of ionidated contrast material. C Sulfonylureas have the highest risk of hypoglycemia of all oral diabetic agents. D Sulfonylureas help preserve beta-cell function. E All of the above.

all of the above Incretin-mimetics act upon the satiety center of the brain and as a result promote weight loss, averaging approximately six pounds. Metformin may cause fatal lactic acidosis when given to patients on concomitantly-administered iodinated contrast dye. Sulfanylureas, in part because of first-pass metabolism, have the highest risk of hypoglycemia of all the oral anti-diabetic agents. TZDs have been found to preserve beta-cell function more so than any other agent.

A 16-year-old girl presents with excessive fatigue and constipation that has developed over the past 3 months. Her eyes have become puffy and her weight has increased approximately 4 lbs. Her skin is dry despite daily moisturizer application, and there is considerable hair loss. Her mother is under treatment for rheumatoid arthritis, and her father has hypertension. On physical examination, her thyroid is diffusely enlarged (twice the normal size) with a rubbery consistency. There is a slight peripheral edema and pallor of skin. Thyroid function tests show free T4 levels of 0.2 ng/dL and a TSH level of 96 mlU/L. Question What is the most likely underlying cause of her disease? Answer Choices 1 Thyroid hormone resistance 2 TSH-producing pituitary tumor 3 Autoimmunity 4 Iodine deficiency 5 Impaired thyroid response to TSH

autoimmunity Explanation This patient has a clinical picture that suggests hypothyroidism, which is confirmed by the low levels of free T4 and elevated TSH. A family history of autoimmune disease puts her at an increased risk to develop autoimmune hypothyroidism. Spontaneous hypothyroidism in iodine-sufficient areas is almost exclusively caused by autoimmunity. Hashimoto's thyroiditis and (in some rare cases) TSH-receptor blocking antibodies can also cause hypothyroidism that may be reversible.These blocking antibodies occupy the TSH receptor on the thyroid cells, preventing the gland from receiving TSH signals to produce T3 and T4. Treatment is levothyroxine replacement (1.6 mg/kg body weight), aiming for TSH levels within the low/normal range. Thyroid hormone resistance is a genetically inherited condition in which mutations in the thyroid hormone receptor (THR) gene render the receptor less interactive with other cofactors in the signaling cascade; there can also be a lower affinity for T3, the active thyroid hormone. Most commonly, these patients are asymptomatic and the condition is detected during a routine laboratory analysis or after a family member is diagnosed. Thyroid hormone levels are elevated along with paradoxically elevated TSH levels. There is no current treatment for this condition. TSH-producing pituitary tumors are rare pituitary adenomas that present more commonly as large lesions producing mass effects (e.g., headache, visual field impairment). Patients have elevated TSH and elevated thyroid hormones. The diagnosis is suspected in a patient with signs and symptoms of hyperthyroidism, elevated T3, free T4, and TSH levels, and a pituitary lesion on magnetic resonance imaging. Iodine deficiency resulting in goiter and/or thyroid dysfunction is currently limited to endemic areas located far from the sea and where salt is not supplemented with iodine. Iodine is the main substrate used in the production of thyroid hormones and is mainly supplemented by diet. In its absence, a decreased production of T3 and T4 will result in increased levels of TSH and the development of a goiter. Treatment is with levothyroxine and an adequate supplementation of iodine. Impaired thyroid hormone response to TSH is a condition in which the thyroid responds poorly to TSH due to mutations in the TSH receptor gene. Patients affected by this disorder present in 2 ways: if the resistance is partial, they present with inadequate production of thyroid hormones in lieu of increased TSH levels; if the resistance is complete, they present with congenital hypothyroidism, decreased response to TSH stimulation, and undetectable thyroglobulin levels. Treatment is levothyroxine replacement.

Which of the following drugs can cause syndrome of inappropriate antidiuretic hormone (SIADH)? A carbamazepine B glyburide C lithium carbonate D metoprolol

carbamazepine Many medications can enhance the release or potentiate the effects of ADH. Carbamazepine may increase ADH release.

A 30-year-old healthy female's labs return showing an HDL of 28 mg/dl, LDL of 132 mg/dl, and total cholesterol of 185mg/dl. She is concerned due to a family history of coronary artery disease (CAD) on her father's side in his 60s. She denies tobacco use. Her blood pressure is 108/50, P-64. She requests advice on what she can due to increase her HDL. What do you advise her? A atorvastatin B cholestyramine C ezetimibe D gemfibrozil E exercise

exercise The correct answer is (E). This patient is at very low risk of CAD based on the 10-year Framingham projections. Based on her < 1%, 10-year risk drug therapy is not indicated. Exercise has been shown to increase HDL. HDL is lower in patients who have a sedentary lifestyle or are obese. Her LDL may also be reduced with exercise, weight loss, and dietary modifications.

Addison's disease, also known as adrenocortical insufficiency, manifests itself by which one of the following? Answer Choices 1 Hyponatremia 2 Truncal obesity 3 Glucose intolerance 4 Hypertension 5 Osteoporosis

hyponatremia Explanation Deficiency of adrenocorticosteroids is the hallmark of Addison's disease, which represents a failure of the adrenal cortex to secrete steroids in adequate amounts to satisfy metabolic needs. 1 of the functions of these adrenal steroids is to regulate sodium metabolism; specifically, they do so by enhancing sodium reabsorption in the presence of hyponatremia or volume depletion. Without this function, a common manifestation of adrenal insufficiency is hyponatremia. The other clinical manifestations listed are typical of the opposite syndrome, hyperadrenocorticism, or Cushing's syndrome.

Your patient is asked to see you in followup for his fasting labs. His total cholesterol = 230 mg/dl, triglycerides = 1200 mg/dl, unable to calculate LDL. Your patient should be advised that due to these findings he is at risk for which of the following? A diabetes mellitus B pancreatitis C gout D diabetes insipidus E hypertension

pancreatitis B The correct answer is (B). Very elevated triglycerides, especially over 1000 mg/dl, increase the patient's risk of acute pancreatitis. This patient requires prompt treatment. Hypertriglyceridemia may be a component of metabolic syndrome but does not directly cause diabetes mellitus or hypertension. Hypertriglyceridemia does not increase the risk of gout and diabetes insipidus.

In addition to insulin and fluid replacement with 0.9% saline, which electrolyte is commonly infused in the type 2 diabetic patient who arrives in the emergency department in a hyperglycemic, hyperosmolar, nonketotic state? A bicarbonate B potassium C calcium D magnesium E sulfate

potassium Insulin not only causes cellular uptake of glucose but also of potassium. Hypokalemia may develop when insulin is infused to correct either a hyperglycemic hyperosmolar state or a diabetic ketoacidosis. Hence, in order to avoid hypokalemia, potassium chloride can be added to a saline solution, as long as the serum potassium is not elevated.

A 36-year-old man with a past medical history of diabetes mellitus presents with weight gain and skin changes. His review of systems is positive for a diminished libido, impotence, depression, cognitive dysfunction, and emotional lability. Lately, his fasting glucose levels have been above normal. His physical exam notes increased adipose tissue in the face, upper back, and above the clavicles. His skin reveals ecchymoses, telangiectasias, and purpura along his back and lower extremities; there is also facial acne and cutaneous atrophy. His abdominal exam reveals the findings in the attached image. Question What is the most appropriate next step in the management of this patient? Answer Choices 1 Obtain a KOH preparation of the skin lesion 2 Begin therapy with ketoconazole 3 Schedule an MRI of the pituitary gland 4 Begin oral prednisone replacement therapy 5 Order a 24-hour urinary free cortisol level

Explanation The correct response is a 24-hour urinary free cortisol level. This patient is demonstrating signs and symptoms consistent with Cushing's syndrome. The diagnosis of Cushing's syndrome due to endogenous overproduction of cortisol requires the demonstration of inappropriately high serum cortisol or urine cortisol levels. Currently, 4 methods are accepted for the diagnosis of Cushing's syndrome: urinary free cortisol level, low-dose dexamethasone suppression test, evening serum and salivary cortisol level, and dexamethasone-corticotropin-releasing hormone test. Urinary free cortisol determination has been widely used as an initial screening tool for Cushing's syndrome because it provides measurement of cortisol over a 24-hour period. A valid result depends on adequate collection of the specimen. Urinary creatinine excretion can be used to assess the reliability of the collection. Urine free cortisol values higher than 3 - 4 times the upper limit of normal are highly suggestive of Cushing's syndrome. A potassium hydroxide (KOH) preparation is a diagnostic test used to assess for the presence of a fungal disorder. Agents that inhibit steroidogenesis (e.g. mitotane, ketoconazole, metyrapone, aminoglutethimide, trilostane, and etomidate) have been used to cause medical adrenalectomy. These medications are used rarely, and they are often toxic at the doses required to reduce cortisol secretion. For instance, ketoconazole's prescribing information was revised to include a black box warning regarding hepatotoxicity, including fatalities and liver transplantation. Therefore, medical treatment should be initiated cautiously; ideally, it should be done in conjunction with a specialist. Ketoconazole has been the most popular and effective of these agents for long-term use and usually has been the agent of choice. However, the FDA has issued a warning that states clinicians should no longer prescribe ketoconazole, except to treat some life-threatening fungal infections; this is due to increased risk for severe liver injury, adrenal insufficiency, and adverse drug interactions. Imaging studies for Cushing's syndrome should be performed after the biochemical evaluation has been performed. The rationale for this is that unguided imaging of the pituitary or adrenal glands may yield a 10% incidence of incidental nonfunctioning pituitary or adrenal adenomas, which may mislead one from proper therapy and surgery. Ideally, the biochemical abnormalities should reconcile with the anatomic abnormalities before definitive therapy is offered. An abdominal CT scan is recommended if a primary adrenal problem is suspected. The presence of an adrenal mass larger than 4 - 6 cm raises the possibility that the mass is an adrenal carcinoma. Corticosteroid therapy (e.g. prednisone) is not appropriate treatment for a patient experiencing hypercortisolism.

A 40-year-old woman presents with a 6-month history of a painless swelling on her neck. On examination, you find a nontender anterior neck swelling that rises with deglutition. Laboratory evaluation reveals normal thyroid function tests. Question What is the most likely diagnosis? Answer Choices 1 Iodine deficiency 2 Zinc deficiency 3 Copper deficiency 4 Folic acid deficiency 5 Cobalamin deficiency

Iodine deficiency Explanation In cases of iodine deficiency, patients can present with an anterior neck swelling that rises with deglutition. This is usually a colloid goiter, as the thyroid hypertrophies as it tries to concentrate iodide in itself. Though most of these patients are euthyroid, some may develop hypothyroidism. Cretinism can develop in infants as a result of iodine deficiency; impaired brain development and fetal growth can also develop. Causes include inadequate dietary intake. Dietary sources of iodine include iodized table salt, seafood, eggs, and dairy products. In cases of zinc deficiency, patients can present with hypogeusia or decreased taste sensation, anorexia, delayed sexual maturation, night blindness, and hair loss. On examination, they have alopecia, growth retardation, delayed sexual maturation, and hypogonadism. Laboratory investigations may reveal hypospermia. Causes include malabsorption states, alcoholism, and prolonged parenteral nutrition. Dietary sources of zinc include beef, liver, eggs, and oysters. Menkes syndrome is an inherited copper deficiency caused by mutations in an X-linked gene. It occurs in male infants and is characterized by intellectual disabilites, kinky hair, hypopigmentation, and vascular aneurysms. Laboratory investigations reveal hypocupremia and decreased circulating ceruloplasmin. Other causes of copper deficiency include severe malabsorption, infants with persistent diarrhea fed on milk diets, copper-free total parenteral nutrition, and excess intake of a zinc salt dietary supplements, which can interfere with copper absorption. Dietary sources of copper include organ meats, oysters, nuts, dried legumes, and whole grain cereals. Patients with folic acid deficiency present with symptoms of anemia such as fatigue, weakness, and syncope. On examination, they are pale. Laboratory investigations reveal macrocytic red blood cells and hemoglobin less than 12 g/dl. Causes include malabsorption and inadequate intake by chronic alcoholics and malnourished individuals, as well as increased demand in pregnancy and chronic hemolytic anemias. Dietary sources of folic acid include green leafy vegetables, liver, and yeast. Patients with a vitamin B12 (cobalamin deficiency) can present with symptoms and signs of anemia, such as fatigue, light-headedness, syncope, and pallor; there are also neurological signs and symptoms, such as ataxia, paraesthesias, and diminished proprioceptive and vibratory sensations in the lower limbs. Laboratory investigations reveal macrocytic red blood cells. Causes include pernicious anemia, blind loop syndrome, fish tapeworm infestation, and vegetarian diets. Dietary sources of vitamin B12 include liver, beef, eggs, and milk.

A 55-year-old man patient presents with tachycardia and heart palpitations. Physical exam shows a multinodular goiter. He does not have obstructive symptoms. He has suppressed TSH and elevated T 3 and T 4 , and a thyroid scan shows multiple functioning nodules. What is the treatment of choice for this patient?

131 I ablation the treatment of choice for multinodular goiter is 131 I ablation. In patients with very large thyroid glands with obstructive symptoms, surgical resection may be the best option.

Patients diagnosed with type 2 diabetes are encouraged to maintain their plasma glucose, to prevent or slow the initiation of chronic complications. What is the maximum level that is within ADA guidelines for a one to two hour postprandial plasma glucose in these patients? A 70 mg/dL B 100 mg/dL C 120 mg/dL D 180 mg/dL E 220 mg/dL

180 mg/dL The correct choice is D, 180 mg/dL. Patients are encouraged to adhere to lifestyle and medication treatment plans in order to help achieve this goal. Other ADA guidelines include maintaining the hemoglobin A1c < 7% or as close to normal (<6%) as possible, blood pressure < 130/80, and preprandial glucose between 90 and 130 mg/dL.

A 38-year-old man presents to the emergency department experiencing a severe headache and heart palpitations. He appears to be anxious and perspiring heavily. On exam, he is found to be tachycardic and his blood pressure is 158/102 mm Hg. His urine catecholamines are increased. If imaging were performed, what is the most likely location where a lesion would be found? A pituitary gland B liver C adrenal gland D testicle E kidney

Adrenal gland Pheochromocytomas produce, store, and secrete catecholamines. They are usually derived from the adrenal medulla, although they may be found in other locations.

Which of the following statements regarding diabetic medications is/are most correct? A Incretin-mimetics like exenatide commonly result in weight loss. B Thiazolidinediones (TZDs) should be held prior to and for 48 hours after administration of ionidated contrast material. C Sulfonylureas have the highest risk of hypoglycemia of all oral diabetic agents. D Sulfonylureas help preserve beta-cell function. E All of the above.

All of the above

A 16-year-old boy with no significant past medical history presents to learn the results of a lipid panel that was performed. The lipid panel was ordered due to periorbital and extensor tendon xanthomas on the patient's body. The patient consumes a low fat and low calorie diet, and he exercises daily. He denies any bothersome symptoms, and his physical exam is otherwise unremarkable. Question What is the most likely mechanism for this patient's metabolic disorder? Answer Choices 1 There is increased production of high density lipoprotein 2 This patient has undiagnosed diabetes mellitus 3 Hypersecretion of triglyceride-carrying chylomicrons from the small intestine 4 A reduction in the amount of LDL receptors on hepatocytes 5 Increased VLDL synthesis by the liver

A reduction in the amount of LDL receptors on hepatocytes LDL (low density lipoprotein) is removed from the circulation by either LDL receptors on hepatocytes or by scavenger cells such as monocytes or macrophages. When there is a decrease in LDL receptors on hepatocytes, or when LDL levels exceed receptor availability, the amount of LDL that is removed by scavenger cells is greatly increased. The LDL receptors are deficient or defective in the genetic disorder known as familial hypercholesterolemia (type 2A). Patients with this primary hypercholesterolemia disorder may be heterozygotes or homozygotes, with homozygotes developing cutaneous xanthomas and myocardial infarctions in childhood. HDL (high density lipoprotein) is synthesized in the liver and transports cholesterol from the peripheral tissues back to the liver; an inverse relationship exists between HDL levels and the development of atherosclerosis. Chylomicrons are involved in the transport of dietary (exogenous pathway) triglycerides and cholesterol that have been absorbed from the gastrointestinal tract, which are then transported to the skeletal muscle and adipose. This patient denies a high triglyceride diet. The production of VLDL (very low density lipoprotein) is increased by the liver as a result of a high calorie diet, obesity, and diabetes mellitus. VLDL represents the primary pathway for transport of endogenous triglycerides produced in the liver; these are then carried to muscle and fat cells. Elevated VLDL levels are not consistent with familial hypercholesterolemia (type 2A). Diabetes mellitus is a cause of secondary hypercholesterolemia, not primary hypercholesterolemia.

An obese 45-year-old woman is requesting medical weight loss therapy. She reports excessive weight gain (37 kgs) over the past 3 years. A detailed history also reveals easy bruising, oligomenorrhea, and increased hair growth in various parts of her body. A thorough examination by the attending doctor shows hypertension (B.P 180/110), truncal obesity with a buffalo hump, and moon face. He also notes hirsutism and pigmentation, with purple abdominal and lower leg striae. Lab reports confirm the diagnosis of Cushing's Syndrome. What substance's high serum levels cause hyperpigmentation in Cushing's Syndrome? Answer Choices 1 Adrenocorticotropic hormone 2 Androgens 3 Cortisol 4 Luteinizing hormone 5 Follicle stimulating hormone

ACTH Explanation Hyperpigmentation in Cushing's Syndrome is due to high serum levels of adrenocorticotropic hormone. Skin pigmentation is a special feature of ectopic ACTH Syndrome. It is a result of alpha melanocyte stimulating hormone (MSH) sequence, which is part of the ACTH molecule. In addition, the rest of the ACTH precursor molecule contains two other MSH sequences. Unlike pituitary tumors secreting ACTH, ectopic tumors have no residual feedback thus there is marked pigmentation. Hyperpigmentation is seen in 4-16% of patients with Cushing syndrome. Additionally, ectopic ACTH is the second most common cause of Cushing syndrome. In patients with pituitary dependent Cushing's disease, treatment with bilateral adrenalectomy may lead to the development of a locally invasive pituitary tumor with high levels of ACTH and hyperpigmentation. Hyperpigmentation is not due to high levels of androgens, cortisol, luteinizing hormone, or follicle stimulating hormone. Some clinical features are more common in ectopic ACTH Syndrome.

A 45-year-old woman presents with recent, unexplained weight loss, and lethargy. She has had several episodes of nausea and vomiting. She admits to craving salty foods. On physical examination, there is a hyperpigmentation to her skin and mucus membranes. Her blood pressure is 85/60 mm Hg. Her laboratory results show hyponatremia and hyperkalemia. Her serum ACTH level is elevated. Question What is the most likely site of the pathology? Answer Choices 1 Pituitary 2 Adrenal medulla 3 Pancreas 4 Adrenal cortex 5 Sympathetic ganglia

Adrenal Cortex Explanation This patient has signs and symptoms consistent with Addison's disease. Addison's disease is primary adrenal insufficiency. The symptoms include weight loss, lethargy, nausea, vomiting, and salt craving. Due to the lack of mineralocorticoids, there will be hyponatremia and hyperkalemia. Addison's disease is primary adrenal insufficiency; therefore, the pathology is located in the adrenal cortex. The primary problem is not in the pituitary. There would be an ACTH deficiency, rather than an elevated ACTH, if the primary pathology were located in the pituitary. In this scenario, the pituitary is responding appropriately by increasing ACTH secretion, thus the elevated ACTH level. Addison's disease is also not due to pathology in the adrenal medulla, pancreas, or sympathetic ganglia.

A 30-year-old man presents with excessive thirst for the past few days. He consumes 3 to 4 liters of water per day but is still thirsty. He has also been passing urine very frequently. He gives a history of a road traffic accident 1 month ago, wherein he hit his head against the dashboard. Urine specific gravity and osmolality are 1.002 and 180 mOsm/kg, respectively. There is an increase in urine osmolality with exogenous antidiuretic hormone (ADH) administration. Question What is the probable diagnosis? Answer Choices 1 Idiopathic central diabetes insipidus 2 Familial central diabetes insipidus 3 Acquired central diabetes insipidus 4 Hereditary nephrogenic diabetes insipidus 5 Acquired nephrogenic diabetes insipidus

Acquired Central DI xplanation The correct answer is acquired central diabetes insipidus. Diabetes insipidus (DI) is a condition of excessive thirst and urination. It may be neurogenic (central) or nephrogenic. Central DI is characterized by decreased ADH secretion, resulting in the person's decreased ability to concentrate urine. Causes include tumors of the pituitary, head trauma, and neurosurgery. It may also be idiopathic or familial. Idiopathic central DI is believed to be an autoimmune condition with destruction of cells in the hypothalamus. Familial central DI is extremely rare. The history of head trauma should raise suspicion of acquired central DI. ADH administration causes an increase in urine osmolality in central DI but not in nephrogenic DI. Nephrogenic diabetes insipidus is caused by a resistance to the antidiuretic hormone (ADH) in the kidney. It may be hereditary, which is rare, or acquired. Acquired nephrogenic DI is caused by lithium toxicity, chronic renal disease, hypercalcemia, etc. The patient has polyuria and polydypsia.

Because of the lack of epiphyseal cartilage, adult bone cannot increase in length when stimulated by an excess of growth hormone; however, they do increase in width by periosteal growth. In adults, what does an increase in growth hormone cause? Answer Choices 1 Gigantism 2 Osteomalacia 3 Acromegaly 4 Rickets 5 Osteoporosis

Acromegaly Explanation In adults, an increase in growth hormone causes acromegaly, a disease in which the bones, mainly the long ones, become very thick. An excess of hormone during growing years causes gigantism. Osteomalacia is caused by calcium deficiency in adults; rickets is caused by calcium deficiency in children. Osteoporosis is the decrease in bone mass caused by decreased bone formation, increased bone resorption, or both.

A 30-year-old female presents to your office for a routine physical exam. She has not seen a health care provider in many years. Upon talking with the patient, you find out that she had been diagnosed with hypertension several years ago, but was unable to afford the antihypertensive medications that were prescribed to her. She has no complaints at this time. Upon exam of the head and neck, you note widened spaces between her lower incisor teeth and a large, fleshy nose. Her skin is oily and she demonstrates mild proximal muscle weakness. Her EKG reveals a left axis deviation and widened QRS. What is the most likely rationale for her clinical presentation? A Diabetes mellitus B Cushing's syndrome C Hypothyroidism D Acromegaly E Clinical depression

Acromegaly The correct choice is D, acromegaly. Patients with acromegaly have an abundance of growth hormone secretion. This leads to excessive growth of many areas of the body including soft tissue. Patients with acromegaly also have an increased incidence of hypertension and left ventricular hypertrophy. None of the other choices will cause this patient's constellation of symptoms. Patients with many endocrine disorders may develop weaknesses as seen in this patient, but the large nose and widely spaced teeth are characteristic of acromegaly.

A 45-year-old male comes into your family practice office for his second follow-up appointment since being diagnosed with type 2 diabetes. He does not complain of any symptoms. He is currently taking metformin 1000 mg BID and his HgA 1c at this visit 7.6%. You ask him about his diet and if he is regularly monitoring his blood glucose. He says that he does not really watch what he eats, but he does check his glucose levels daily. He tells you that on average his fasting plasma glucose (FPG) is usually around 88 mg/dl and his postprandial glucose (PPG) is around 180 mg/dl. What would be the BEST next appropriate step in management for this patient? A Nothing, his levels are within normal limits. B Consider switching to basal insulin therapy to control his FPG. C Add a dipeptidyl peptidase 4 (DPP-4) inhibitor to control his PPG. D Add a sulfonylurea to control his PPG. E Increase his metformin to 1500 mg twice daily.

Add a dipeptidyl peptidase 4 (DPP-4) inhibitor to control his PPG. DPP-4 inhibitors modulated glucagon-like peptide-1 (GLP-1). Their mechanism of action is thought to result from increased incretin levels, especially GLP-1. GLP-1 inhibits glucagon release which, in turn, results in increased insulin secretion, delays gastric emptying, and decreases serum glucose levels. The class is particularly appropriate to utilize in patients who have near-normal HbA 1C and elevated postprandial serum glucose because they work only when food enters the gut and have little to any chance of hypoglycemia.

A 44-year-old woman presents with near-syncope. She states that, over the past month, she has noticed profound fatigue, nausea, and occasional vomiting. She has also experienced a 15-pound weight loss. On physical exam, she appears profoundly weak. Vital signs are: BP 95/55; pulse 110; respiration 14; temp 37.4° C (99.3° F). Orthostatic evaluation shows a positive tilt. HEENT: NCAT, mucous membranes are dry, and areas of pigmentation are noted. Lung exam is clear. Cardiac exam reveals distant heart sounds and tachycardia. Abdominal exam shows mild, diffuse tenderness, without guarding or rebound. The patient's skin is dry and appears very tanned. No bruising or other rashes are noted. Laboratory studies show Chem 7: glucose 80; Na 130; K 5.2; Cl 100; HCO3 21 BUN 55, creat 2.2. With what is the patient most likely afflicted? Answer Choices 1 Bartter syndrome 2 Liddle's syndrome 3 Cushing's syndrome 4 Addison's disease 5 Functional adrenal insufficiency

Addison's Disease Explanation The overall clinical picture of a patient with primary adrenal insufficiency, or Addison's disease, is that of one who is weak and lethargic with loss of vigor and fatigue on exertion and an inability to withstand even minor stresses without shock. The clinical presentation of Addison's disease can be explained on the basis of a deficiency of cortisol and aldosterone and a lack of feedback suppression of ACTH and MSH. Bartter syndrome is hypertrophy and hyperplasia of the cells of the renal juxtaglomerular apparatus, resulting in a saline-resistant metabolic alkalosis. Liddle's syndrome, or pseudohyperaldosteronism, has a clinical picture of hyperaldosteronism; however, there is normal aldosterone secretion, resulting in a saline-resistant metabolic alkalosis. Cushing's syndrome is a constellation of findings due to an excess of cortisol. The most common etiology is iatrogenic from steroid use for the treatment of other diseases. The syndrome is characterized by weight gain, especially on the abdomen, face (moon face), neck and upper back (buffalo hump), thinning and weakness in the muscles of the upper arms and upper legs, in addition to thinning of the skin with easy bruising striae on the abdomen, thighs, breasts, and shoulders. Laboratory abnormalities include hyperglycemia and mild hypokalemia. Functional adrenal insufficiency exists when administration of exogenous steroids results in depression of ACTH secretion.

Your patient comes to the office for a follow up of her atrial fibrillation and hypertension. She has noted that she have been more tired than usual. Laboratory findings include a thyroid-stimulating hormone (TSH) < 0.05 mU/L. What medication is the most likely cause of her laboratory findings? A cardizem B amiodarone C warfarin D dotalol E labetalol

Amiodarone The correct answer is (B). Amiodarone is an antiarrhythmic medication containing iodine that is commonly used in treatment of atrial fibrillation. The use of amiodarone can cause thyrotoxicosis by several mechanisms and may also cause hypothyroidism. In this case the patients suppressed TSH would suggest the presence of amiodarone induced thyrotoxicosis. A high T 3 and FT 4 would support your diagnosis. All the other choices used in the treatment of atrial fibrillation would not cause thyroid dysfunction.

What risk factor(s)/lipid class combination most closely identifies the need to institute pharmacotherapy? Answer Choices 1 An LDL cholesterol of > 110; no history of Coronary Heart Disease (CHD) 2 A Total Cholesterol of <200; Hypertension (BP> 140/90) 3 An HDL cholesterol of >60; cigarette smoking 4 An HDL of <40; Age (men >45yrs, women >55yrs) 5 VLDL of <100; Diabetes

An HDL of <40; Age (men >45yrs, women >55yrs) Explanation The NCEP/ATEP has set forth explicit Risk Factors as well as specific lipid goals/classifications that clearly address what parameters dictate the initiation or cessation of pharmacotherapy. This question illustrates that, in patients with a major risk factor (CHD/Diabetes), the goal of drug therapy is to keep the level of LDL lipids <100. Likewise, drug therapy should be started when the LDL is >130. An HDL of <40 is undesirable, and so is the age factor (>45yrs for men and >55 years for women). This case illustrates the rationale of pharmacotherapy in the presence of 2 or more risk factors. The specific LDL lipid level should be identified to guide the initiation and goal of pharmacotherapy. Drug therapy for patients with 2+ risk factors should be started when the LDL is >130. The LDL goal is <130. A patient with a TC of <200 is already in the desirable range and would not need drug therapy based on lipid classification alone. It should be noted that the TC is only a general marker; therefore, the individual lipid molecules need to be identified via a lipid panel/profile. The presence of 0-1 risk factors (HBP) mandates that drug therapy should be started when the LDL is >190; the goal of therapy is an LDL <160. Once the LDL level is identified, then the decision whether or not to initiate drug therapy can be discussed. An HDL of >60 is very desirable. There is a direct correlation between the level of HDL and the absence of coronary plaque. It should be noted that the LDL level should still be identified in order to ascertain the true lipid picture. Cigarette smoking is only 1 risk factor; therefore, drug therapy should be initiated when the LDL is >190. The goal of therapy is an LDL level of <160. Once the LDL lipid level is identified, the option of drug therapy can be discussed. The VLDL lipid molecule can be a separate risk factor and is usually elevated along with TG. The VLDL level becomes important (VLDL > 200 - 300) when it is accompanied by elevated TG (200-499). As in the above explanations, the LDL lipid level should be identified due to the direct correlation between LDL and CHD. A major risk factor, such as diabetes, mandates that drug therapy be initiated when the LDL is >130. The LDL goal is <100. Once the LDL level is lowered to an acceptable range, the elevated VLDL and TG (especially if TG is >500) should be addressed by initiating either a combination of diet and exercise (will lower VLDL) and/or drug therapy. The Statins have a modest effect on VLDL/TG, whereas niacin and gemfibrozil have been shown to dramatically reduce these lipid levels.

A 60-year-old man is 3 years post-coronary artery bypass grafting. His last lipid profile showed: Lipid Result Triglycerides 165 mg/dL HDL 34 mg/dL LDL 135 mg/dL He is normotensive and almost chest-pain-free on isosorbide mononitrate and metoprolol. Given his lipid profile, how can this patient most appropriately be managed? Answer Choices 1 Fenofibrate, 134 mg daily 2 Simvastatin, 20 mg daily 3 Atorvastatin, 80 mg daily 4 Pravastatin, 20 mg daily 5 Lovastatin, 10 mg daily

Atorvastatin, 80 mg daily Explanation Hyperlipidemia contributes to the accelerated atherogenesis in vein grafts. Thus, aggressive control of hyperlipidemia is vitally important for the patency of the grafts. The target intervention is to obtain the biggest reduction in LDL levels. According to the National Cholesterol Education Program, this patient's Serum triglycerides level falls under borderline high category, LDL under borderline high category, and his HDL cholesterol values are borderline low. Since this patient has deranged values of LDL, HDL, and triglycerides, Atrovastatin 80 mg would be the drug of choice. Atorvastatin in 10 mg doses reduces LDL up to 39% and increases HDL by 6 - 8%; in 80 mg doses, it brings about a 60% reduction in the LDL levels and up to 37% reduction in triglyceride levels. Simvastatin at a dose of 20 mg daily is able to produce a 38% decrease in LDL cholesterol combined with an 8% increase in HDL cholesterol. Fenofibrate would be a better choice if the patient had increased triglycerides because it is effective in treating hypertriglyceridemia. Pravastatin offers only 2% increase in HDL and a 32% decrease in LDL. Lovastatin in a 10 mg dose brings about a 21% reduction in the LDL level and approximately 2 - 9% increase in the HDL level.

Which of the following is the most common cause of primary adrenal insufficiency in the United States? A Tuberculosis B Adrenal hemorrhage C Lymphoma D Autoimmune destruction E Metastatic carcinoma

Autoimmune destruction The correct choice is D, autoimmune destruction. This is responsible for 80% of cases of primary adrenal insufficiency in the United States. All of the other choices can cause adrenal insufficiency, but they are less common. Tuberculosis, choice A, is a common cause of adrenal insufficiency in other areas of the world, where the infection is more common. Bilateral adrenal hemorrhage, choice B, can occur as a complication of sepsis, heparin use, anti-phospholipid syndrome, and after major trauma or surgery. Lymphoma, choice C, and metastatic carcinoma, choice E, are rare causes of adrenal insufficiency.

A 12-year-old girl becomes comatose and is rushed to the hospital by her parents. 2 days before the admission, she went to school feeling ill. She vomited that evening. Her vomiting persisted with only an 8-hour pause during sleep. She is breathing deeply and rapidly; her breath has a fruity odor. Her parents mention that her appetite has increased. She has also been drinking lots of fluids; subsequently, she has been urinating more than normal. Urinalysis reveals 3+ glucose levels and 2+ ketone bodies. Question Based on the above information, what is the etiological cause of this patient's symptoms? Answer Choices 1 Insulin resistance 2 Increase in counterregulatory hormones 3 Autoimmune destruction of B-cells of pancreas 4 Post Epstein-Barr virus infection 5 Autoimmune destruction of pancreatic acini cells

Autoimmune destruction of B-cells of pancreas The etiology of Diabetes Mellitus Type I is autoimmune destruction of pancreatic B-cells. Some common signs and symptoms of Diabetes Type I include hyperglycemia, polyuria, increased thirst, weight loss, increased appetite, and nausea/vomiting. Blood glucose is elevated and urine may show presence of glucose. Diabetic ketoacidosis (as in this patient) may present with ketone bodies in urine and Kussmaul breathing along with other symptoms. Insulin resistance is the etiology of Diabetes mellitus Type II. A counterregulatory hormone associated with insulin is glucagon. Glucagon counters the effects of insulin, but it does not destroy the B-cells of the pancreas. Epstein-Barr virus does not affect the pancreas. The autoantibodies do not destroy the acini cells.

A 50-year-old woman presents with a history of polyuria, polydipsia, muscle aches, bone pain, nausea, and constipation for the past few months. Her past medical history is significant for a pituitary adenoma, peptic ulcer disease, and kidney stones. You are considering a diagnosis of primary hyperparathyroidism. Which of the following lab values would you expect in this patient? A Low serum calcium and elevated serum PTH B Elevated serum calcium and PTH levels C Elevated serum calcium and low serum PTH levels D Low serum phosphate and PTH levels E Elevated serum phosphate and calcium levels

B Elevated serum calcium and PTH levels The correct choice is B, elevated serum calcium and PTH levels. In primary hyperparathyroidism, excess PTH is secreted and stimulates a rise in serum calcium by increasing calcium release from bone, thereby reducing renal clearance of calcium and increasing calcium absorption through the intestine. In choice A, low serum calcium is not seen in hyperparathyroidism. In choice C and D, low serum PTH levels aren't seen in hyperparathyroidism. In choice E, elevated serum phosphate levels aren't seen in hyperparathyroidism. It typically presents with low serum phosphate levels.

A 29-year-old woman presents in July to your office with symptoms of palpitations, sore neck, and excessive sweating, despite using her air conditioner all the time. No surgical or trauma history is noted. She is currently not taking any medications. Vitals include the following: BP = 124/68, pulse = 110 beats per minute, respirations = 18 per minute, and temperature = 101 o F orally. Upon exam, her thyroid is mildly enlarged without nodules, and severely tender. No local erythema or heat is noted. Which of the following lab results would you expect in this patient? A Serum total T4 level = 5.0 ug/dL B Serum TSH level = 0.25 uIU/mL C Sedimentation rate = 15 mm/hr D Free thyroxine index = 8.0 E Positive thyroid stimulating antibodies

B Serum TSH level = 0.25 uIU/mL The correct choice is B, Serum TSH level=0.25 uIU/mL. The reference range for TSH is 0.34 to 4.25 uIU/mL, and therefore the level in this patient is low. This patient is presenting with signs and symptoms of hyperthyroidism, most likely due to subacute thyroiditis. The leaking of thyroid hormone into the circulation causes anterior pituitary suppression and reduced TSH secretion. Choice A is seen in patients with hypothyroidism. Choice C is within the reference range for woman. Since subacute thyroiditis is an acute inflammatory disorder, patients with this disorder will commonly present with an elevated sedimentation rate. Choice D corresponds to a euthyroid situation. It is an estimate of the free thyroid hormone level in the plasma. This result is within the reference range. Choice E is not seen in subacute thyroiditis. They are commonly found in patients with Graves' disease.

A 58-year-old man with a past medical history of diabetes mellitus type II and hypothyroidism presents for a follow up of blood work results that were performed 1 week prior. His low density lipoprotein level was found to be 150 mg/dL; total cholesterol level was 230 mg/dL; high-density lipoprotein was 36 mg/dL; and triglyceride levels were 260 mg/dL. He denies any complaints at this time and is compliant to the only medications he takes, which are Levothyroxine 137 mcg/day and Metformin 500mg twice a day. His labwork demonstrated a euthyroid state and a hemoglobin A1c of 6.1%. Question What is an appropriate step in the management of this patient? Answer Choices 1 Recommend a 300 mg/day cholesterol diet 2 Prescribe aspirin 325 mg every day 3 Begin rosuvastatin 10 mg every day 4 Restrict fiber and fruit intake 5 Prescribe cholestyramine resin daily

Begin rosuvastatin 10 mg every day Explanation This patient should be prescribed a statin. The statin (HMG-CoA reductase inhibitor) class of drugs has revolutionized the treatment of hypercholesterolemia, as they are highly efficacious and very well tolerated. The LDL-C goal for patients with coronary heart disease (CHD) equivalent risk, including patients with diabetes mellitus, should also be less than 100 mg/dL. In patients considered to be very high risk, a goal of less than 70 mg/dL is an acceptable option. Patients with CHD or CHD equivalent are prescribed drug therapy simultaneously with therapeutic lifestyle changes if their LDL-C concentration is greater than or equal to 130 mg/dL. Drug therapy is optional for patients whose LDL-C value is 100-129mg/dL. For rosuvastatin at 5-40mg/d, the LDL-C level is lowered by 45 - 63%. Dietary cholesterol should be limited to less than 200mg/day. Other dietary changes, such as increased soluble fiber found in oat bran or psyllium, may reduce LDL cholesterol by 5-10%. Diets rich in antioxidant vitamins found in fruits and vegetables may be helpful in the reduction of oxidation of LDL, thereby reducing atherogenesis. All patients whose risk from CHD is considered high enough to warrant pharmacologic therapy of an elevated LDL cholesterol should be given aspirin prophylaxis at a dose of 81mg/day unless contraindicated. Cholestyramine is a bile acid sequestrants used primarily as additional therapy in patients with familial hypercholesterolemia who experience inadequate LDL-C lowering with statins.

What is the most common visual field defect occurring in patients with pituitary macroadenoma? Answer Choices 1 Heteronymous hemianopia 2 Bitemporal hemianopia 3 Quadrantanopia 4 Homonymous hemianopia 5 Homonymous hemianopia with macular sparing

Bitemporal Hemianopia Explanation Pituitary macroadenoma compresses the optic chiasma to produce visual symptoms, the most common being bitemporal hemianopia. Local complications of Pituitary tumors: Upward extension: Chiasmal compression and visual field defect Compression of pituitary stalk Lateral extension: Dysfunction of cranial nerves 3,4,6 (diplopia) Stretch of dura mater: Headache Acute infarction / hemorrhage: Pituitary apoplexy Investigations reveal: Increased levels of growth hormone Increased levels of prolactin Hyperglycemia Paradoxical rise of GH levels during an oral glucose tolerance test. Pituitary macroadenomas that are best visualized by MRI or CT scan Management options: Trans-sphenoidal surgery (especially for microadenoma) External radiotherapy to stop growth and decrease GH Octreotide (will lower GH levels but will not stop tumor growth)

A 38-year-old man presents with fatigue, dry mouth, and passing large amounts of urine. He describes his urine as light in color and non-odorous. He says he never had this before. He has always been healthy, and he has never been hospitalized before. During a routine pre-employment screening, his serum electrolytes were investigated; his sodium level was slightly above normal. At that time, he was reassured that it was due to slight dehydration. He was advised to repeat the test after 3 months. When he repeated it about a week ago, it showed more of an increase. He takes vitamin supplements and sometimes uses energy drinks. To avoid going to the bathroom too often, he tries to restrict fluid intake to a minimum, but he says it has never helped. Question What is the most appropriate next step in the management of this patient? Answer Choices 1 Serum osmolarity 2 Brain MRI 3 Urine osmolarity 4 DDAVP (desmopressin test) 5 Blood glucose level

Blood Glucose Level Explanation Diabetes mellitus (DM) should be excluded first in all patients with polyuria. Therefore, checking blood glucose level is the next best step. Should DM be ruled out, you should then determine the correct etiology of the patient's symptoms. Diabetes insipidus (DI) is also on the differential diagnosis. DI is caused by either a deficiency of arginine vasopressin (central DI) or an insensitivity to its effects at the level of the kidney (nephrogenic DI). Symptoms include polyuria, polydipsia, and decreased urine specific gravity. Causes: 1) Central; due to tumor, anoxia, or hemorrhage. 2) Nephrogenic; due to the effect of central drugs, of which lithium is a well-known cause. Differential diagnoses: 1) Diabetes mellitus. 2) Psychogenic polydipsia Workup: 1) First, exclude DM by random blood sugar; then, psychogenic polydipsia should be excluded by water deprivation test (in this case scenario, the patient did it himself). 2) Second, do a desmopressin test to differentiate nephrogenic from central DI. 3) If it is positive, then perform MRI to exclude brain tumors. Treatment: 1) In mild cases, adequate hydration is enough. 2) For central DI, give desmopressin. 3) In cases of psychogenic polydipsia, psychiatric consultation is needed. If drug induced, stop offending drug. The DDAVP test is performed to differentiate central from nephrogenic DI. Brain MRI can be used to detect pituitary tumors that may cause diabetes insipidus. Urine osmolarity is significantly reduced in diabetes insipidus and may be helpful in determining a diagnosis. Although all of these has a role in diagnosis, they are not the best answer to the question.

A 34-year-old African-American woman presents with recurrent vaginal yeast infections. Over the past 2 years, she has had repeated episodes of similar infections that have been only partially responsive to over-the-counter treatments. She has not seen a physician in the 5 years since her last pregnancy, and she denies a history of any major medical illness. She has been moderately obese for most of her adult life; her maximal weight was 240 lb. at a height of 5'1"; however, she has recently had a 15 lb. unintentional loss in weight. She also reports nocturia for the past several months. Physical examination is remarkable for a blood pressure of 155/95 mm Hg, obesity, and findings consistent with vaginal candidiasis. Question What is the most useful test for underlying disease diagnosing in this patient? Answer Choices 1 A 50 g oral glucose tolerance test 2 A 100 g oral glucose tolerance test 3 Blood glucose after an overnight fast (FBG) 4 Point-of-care (POC) Hemoglobin A1c (HbA1c) level 5 Immediate measurement of blood glucose concentration

Blood glucose after an overnight fast (FBG) Explanation This woman's nocturia and her recurrent vaginal candidiasis, which is a common correlate of hyperglycemia in women, are symptoms of diabetes mellitus. Women who are diabetic are more prone to vaginal yeast infections due to vaginal secretions containing greater amount of glucose than normal. Yeast cells are nourished by this excess glucose. Additionally, acute hyperglycemia can affect all major components of innate immunity, thereby impairing the ability of the host to combat infection. The standard means of diagnosing diabetes is measurement of FBG; the measurement should be repeated if the value is greater than 110 mg/dl. The current criteria considered diagnostic of diabetes is the finding of an FBG level of 126 mg/dl on more than one occasion. An FBG of 110 to 126 mg/dl is indicative of abnormal glucose metabolism, with higher values indicating an increased likelihood of progression to diabetes. Immediate measurement of blood glucose concentration is an incorrect response. A determination of random blood glucose level is diagnostic if that level is greater than 200 mg/dl in a symptomatic patient. Random blood glucose values vary considerably more than do FBG values and thus are not as reliable for establishing a diagnosis. Point-of-Care (POC) hemoglobin A1c (HbA1c) level is an incorrect response. Although in 2009, an international expert committee recommended the A1c test as one of the tests available to help diagnose type 2 diabetes and prediabetes, point-of-care (POC) A1c assays are not sufficiently accurate at this time to use for diagnostic purposes. HbA1c as a diagnostic test should be performed using a method that is certified by the National Glycohemoglobin Standardization Program (NGSP) and standardized. Oral glucose tolerance test is an incorrect response. OGTT are quite sensitive in the detection of diabetes, and the 50 g and 100 g tests have a clear place in the diagnosis of gestational diabetes; however, these tests are more expensive and inconvenient for patients than measurement of FBG, and their results are less reproducible. Because abnormalities in fasting and postprandial glycemia tend to progress in tandem, FBG measurement has supplanted the oral glucose tolerance test for the diagnosis of diabetes (except during pregnancy).

To which of the following areas does follicular thyroid cancer most commonly first spread?

Bone and Lung via bloodstream The correct choice is D, bone and lung via bloodstream. Follicular thyroid cancer can spread to regional lymph nodes and distant sites via the blood stream. Choice A, intraglandular metastasis, is seen more commonly in papillary thyroid carcinoma, and choice B, local spread into regional vocal cords, occurs in anaplastic thyroid carcinoma. Choice C, distant lymph nodes, and choice E, local extension into the muscle and trachea, are more commonly seen in patients with medullary thyroid cancer.

For what childhood endocrine and metabolic disorder is the appropriate pharmacologic test (stimulation or suppression) water deprivation? Answer Choices 1 Hypothyroidism 2 GH deficiency 3 Cushing's disease 4 Diabetes insipidus 5 Thyroid medullary carcinoma

DI The water deprivation test can establish the abnormally low osmolality when fluid intake is restricted, and it can document an increase of osmolality in response to long-acting analogue of ADH. These findings confirm the diagnosis of ADH-deficient diabetes insipidus.

A 30-year-old woman has a family history of MEN type 2. Her doctor becomes quite concerned when he notices a nodule on her thyroid. Further work up is done, and she is found to have medullary carcinoma of the thyroid. What tumor marker is most likely to be found in this patient? Answer Choices 1 ADH 2 HCG 3 CA 125 4 Alpha feto-protein 5 Calcitonin

Calcitonin Explanation Calcitonin is normally secreted by the parafollicular cells (C cells) of the thyroid gland. It lowers serum calcium. Calcitonin can be seen with medullary cancer of the thyroid. ADH is antidiuretic hormone. It is normally secreted by the posterior pituitary. It is involved in water homeostasis. Ectopic ADH production can sometimes be seen with lung cancer, intracranial neoplasms, and a few other tumor types. This is the syndrome of inappropriate secretion of ADH (SIADH). HCG is human chorionic gonadotropin. It is normally produced during pregnancy. It can be seen in gestational trophoblastic disease, choriocarcinoma, and germ cell tumors. CA 125 can be found with some ovarian cancer, uterine cancer, pancreatic cancer, lung cancer, breast cancer, and colon cancer. CA 125 can also be seen in some normal conditions, such as menstruation, pregnancy, and other medical conditions. Alpha-fetoprotein is a tumor marker that can be seen in hepatocellular carcinoma, some germ cell tumors, and some other malignancies and conditions.

Which of the following is appropriate in the pharmacologic management of patients with hypoparathyroid tetany? A Aggressive IV hydration B Cinacalcet hydrochloride PO C Calcium gluconate IV D Pamidronate IV E Calcitonin IV

Calcium Gluconate IV The correct choice is C, calcium gluconate IV. In severe hypocalcemia, replacement calcium must be started promptly, as well as airway maintenance and magnesium and vitamin D replacement, as necessary. Choices A, aggressive hydration, B, cinacalcet hydrochloride, D, pamidromate, and E, calcitonin, are all possible treatment options for patients with hypercalcemia.

A 55-year-old patient presents with symptoms of polyuria and is worried that he may have diabetes mellitus. His past medical history includes hypertension and lung cancer. His family history is negative for diabetes mellitus. He works as an accountant and has no history of head trauma. He has an average diet and fluid intake. His fasting plasma glucose is 110 mg/dL and his hemoglobin A1c is 5%. What is the most likely cause of his polyuria? A Cancer related diabetes insipidus B Type 2 diabetes mellitus C Nephrogenic diabetes insipidus D Familial hypothalamic diabetes insipidus E Primary polydipsia

Cancer related Diabetes Insipidus

A 45-year-old man presents with multiple symptoms, including a 2-year history of chronic fatigue, headaches, and joint pain. He was finally prompted to seek care when he noticed an increase in both his hat and shoe size. His past medical history is unremarkable, with no known medical conditions; there is no history of surgery, and he does not take any medications. He lives with his girlfriend, and he works as a building contractor. On physical exam, his facial features appear "coarse"; he has a wide nose, and macroglossia is noted. The remainder of his physical exam is normal, and no observable abnormalities are noted on the patient's head or feet. Several tests are performed and significant findings include: Growth hormone: Elevated Oral glucose tolerance test (OGTT): Elevated glucose levels Insulin-like growth factor-1 (IGF-1): Elevated Question After treatment of this patient's condition is initially completed, for what complication of this condition should he be monitored? Answer Choices 1 Cardiovascular disease 2 Hypogammaglobulinemia 3 Multiple myeloma 4 Osteoporosis 5 Parkinson disease

Cardiovascular Disease Explanation This patient is presenting with acromegaly, a rare disorder of excess growth hormone (GH), most often caused by a GH-secreting pituitary adenoma. Because the changes are insidious, patients may take years to present with symptoms and be diagnosed. The patient and close family members may not notice the acral skeletal growth enlargement, unless it affects the fit of shoes, rings and/or hats. Other possible signs and symptoms of acromegaly include fatigue, headaches, visual field deficits, hypertension, coarse facial features, carpal tunnel syndrome, and joint pain. Treatment may include surgical resection of the adenoma, medications or radiation. Patients with acromegaly are at risk for early death; the primary cause of death is cardiovascular disease. Patients with acromegaly may develop hypertension, cardiomyopathy, and hypertrophy. Hypogammaglobulinemia is an immune disorder in which the affected patient has reduced levels of immunoglobulins and is at risk for infection. Hypogammaglobulinemia, and other immune system disorders, are not identified as acromegaly complications. Multiple myeloma is a plasma cell cancer that affects bone marrow. While acromegaly does affect bone growth, it has not been linked with bone malignancy. Osteoporosis is the demineralization of bone structure, leading to bone weakness and fracture. It is not considered a complication of acromegaly. Parkinson disease is a chronic, progressive neurologic disorder associated with tremor, rigidity, abnormal gait, and mental changes, such as dementia and depression. Patients with acromegaly are not known to be at a higher risk of Parkinson disease.

A 38-year-old man presents with fatigue, dry mouth, and passing large amounts of urine. He describes his urine as light in color and non-odorous. He says he never had this before. He has always been healthy, and he has never been hospitalized before. During a routine pre-employment screening, his serum electrolytes were investigated; his sodium level was slightly above normal. At that time, he was reassured that it was due to slight dehydration. He was advised to repeat the test after 3 months. When he repeated it about a week ago, it showed more of an increase. He takes vitamin supplements and sometimes uses energy drinks. His blood glucose is normal. To avoid going to the bathroom too often, he tries to restrict fluid intake to a minimum, but he says it has never helped. Question DDAVP (desmopressin) injection led to decrease in urine volume and increase in urine specific gravity. What is the most likely diagnosis? Answer Choices 1 Central diabetes insipidus 2 Nephrogenic diabetes insipidus 3 Diabetes mellitus 4 Psychogenic polydipsia 5 Drug-induced polyuria

Central diabetes insipidus Explanation Diabetes insipidus is a disease in which ADH (antidiuretic hormone) from the posterior pituitary is either deficient or the kidneys become insensitive to it. In this patient, since there was a decrease in urine output after desmopressin injection (i.e., kidneys are sensitive to ADH), the diagnosis is central diabetes insipidus. ADH is secreted from posterior pituitary and works on distal renal tubules on specific receptors to reabsorb water, and this occurs in response to osmoreceptors to help keep the body in balance. Symptoms include polyuria, polydipsia, and decreased urine specific gravity. Causes: Central; due to tumor, anoxia, or hemorrhage. Nephrogenic; due to the effect of drugs of which lithium is a well known reason. Differential diagnoses: Diabetes mellitus Psychogenic polydipsia Workup: First, exclude diabetes mellitus by random blood sugar and then exclude psychogenic polydipsia by water deprivation test (in this case scenario, the patient did it himself). Second, perform a desmopressin test to differentiate nephrogenic from central DI. If it is positive, do MRI to exclude brain tumors. Treatment: In mild cases, adequate hydration is enough. In central, give desmopressin. In cases of psychogenic polydipsia, psychiatric consultation is required. In drug-induced cases, stop the offending drug.

A 54-year-old man presents because he has very large urine output and is constantly thirsty. His urine output is many liters per day. He wakes up at night to urinate. His past medical history is remarkable for a 25-year history of a bipolar disorder; it is treated effectively with lithium. His lab results are as follows: TEST RESULTS REFERENCE RANGE BUN 17 mg/dL 10-20 mg/dL Calcium 9.9 mg/dL 8.5-10.5 mg/dL Potassium 4.2 mEq/L 3.5-5.0 mEq/L Sodium 149 mEq/L 135-145 mEq/L Glucose (fasting) 109 mg/dL 65-110 mg/dL The results of his urinalysis are as follows: TEST RESULTS REFERENCE RANGE Urine dipstick blood Negative Negative Urine dipstick glucose Negative Negative Urine dipstick ketones Negative Negative Urine dipstick protein Negative Negative 24 hour urine protein 124 mg/24 hr <150 mg/24 hr Urine osmolality 40 mOsm/kg 50-1400mOsm/kg Urine specific gravity 1.001 1.001-1.035 Urine pH 6.2 4.5-8.5 Question What structure is most involved in the pathophysiology of this condition? Answer Choices 1 Neurohypophysis 2 Adenohypophysis 3 Glomerulus 4 Loop of Henle 5 Collecting duct

Collecting ducts Explanation Collecting dust is the correct response. A known side effect of lithium is nephrogenic diabetes insipidus. This side effect is experienced by a significant minority of the patients taking lithium. Nephrogenic diabetes insipidus results when the collecting duct of the kidney does not respond to the antidiuretic hormone (ADH). The posterior lobe of the pituitary is the neurohypophysis. The neurohypophysis secretes antidiuretic hormone (ADH) and oxytocin. The production of the posterior pituitary hormones actually occurs in the hypothalamus. The anterior lobe of the pituitary is the adenohypophysis. The anterior lobe of the pituitary secretes follicle-stimulating hormone (FSH), luteinizing hormone (LH), prolactin, growth hormone (GH), thyroid-stimulating hormone (TSH), and adrenocorticotropic hormone (ACTH) (and the corticotropin related peptides). The collecting duct is a portion of the nephron. It is also called the collecting tubules. Urine enters the collecting duct from the distal convoluted tubule. The collecting duct is responsive to ADH. The glomerulus is a specialized capillary tuft. The endothelium of the glomerulus is fenestrated. The glomerulus is surrounded by Bowman's capsule. A renal corpuscle consists of the glomerulus and Bowman's capsule. A renal corpuscle is sometimes called a Malpighian corpuscle. Renal corpuscles are seen in the kidney cortex. The loop of Henle is part of the nephron. It is shaped like a hairpin. The proximal convoluted tubule leads into the descending limb of the loop of Henle. The ascending limb of the loop of Henle goes to the distal convoluted tubule.

A 50-year-old man is in persistent cardiac arrest despite 2 electrical shocks and adrenaline medication. While he and his wife played golf at a local course, he collapsed. When the paramedics arrived, he did have a pulse and blood pressure, but he arrested on the way to the hospital. The only medication he takes on a regular basis is an ACE-antagonist for high blood pressure. At the base of his neck you feel a hump, and you see some purple stretch scars on his abdomen; in addition to his obesity, he has a moon face. You draw some blood for electrolytes; the potassium is 1.9. What is most likely causing the patient's symptoms? Answer Choices 1 Cortisol producing tumor 2 Hashimoto thyroiditis 3 Myocardial infarction 4 Cardiomyopathy 5 Pheochromocytoma

Cortisol producing tumor Explanation Trunk obesity, moon face, purple stretch marks on the abdomen, and buffalo hump are typical symptoms of Cushing's syndrome. Cushing's syndrome can be caused by iatrogenic glucocorticoid therapy for M. Addison, skin diseases, inflammatory joint diseases, systemic diseases, hemolytic anemia, kidney diseases, leukemia, and pituitary hypersecretion of ACTH cortisol producing tumors: e.g. adrenal adenoma or small cell carcinoma of the lung. Because of the excessive secretion of corticosteroids, too much aldosterone is produced; this leads to an increased secretion of K+ and H+ ions through the kidney, and it ends in hypokalemia and metabolic alkalosis. The latter are the cause of the cardiac symptoms. Hashimoto thyroiditis, an autoimmune disease, is very often coexisting with M. Addison. The latter is hypocortisolism, the opposite of M. Addison. Symptoms include lethargy, weakness, and hunger for salt. There are no symptoms described, which would make the diagnosis of myocardial infarction or cardiomyopathy likely. Pheochromocytoma, a tumor of chromaffin cells, secretes excessive amounts of catecholamines and causes hypertension. They are mostly found in the adrenal medulla, in contrast to the glucocorticoid producing adrenal adenoma, which are located in the adrenal cortex; they are usually benign. Symptoms include tachycardia, hypotension, tachypnea, flushing, headaches, dyspnea, paresthesias, and constipation.

A 34-year-old woman presents due to a 6-month history of progressive fatigue. She has had to stop running due to lack of energy and notes that she is napping frequently. She also has a poor appetite, dry skin, and notes darkening of her skin over her elbows and knees. Physical examination is significant for orthostatic hypotension and hyperpigmentation of the elbows and knees. Basic metabolic panel reveals the following: Sodium: 125 (135-146 mmol/L) Potassium: 4.8 (3.5-5 mmol/L) Chloride: 78 (98-107 mmol/L) Bicarbonate: 23 (23-32 mmol/L) Glucose: 95 (65-100 mg/dL) BUN:17 (8-25 mg/dL) Creatinine: 0.7 (0.8-1.30 mg/dL) Question What testing is recommended to confirm her diagnosis? Answer Choices 1 Dexamethasone suppression test 2 Plasma fractionated metanephrine 3 Cosyntropin stimulation test 4 Serum CA 19-9 5 Radioactive iodine uptake test

Cosyntropin stimulation test The patient's history, presence of hyperpigmentation, and presence of hyperkalemia are suggestive of Addison's disease, or primary adrenal insufficiency. The cosyntropin stimulation test (also called the ACTH test) is used to assess the function of adrenal gland response. A small amount of cosyntropin (synthetic ACTH) is injected, and the serum cortisol is obtained 45 minutes following administration. Normally, the cortisol level should respond to cosyntropin injection. Patients with Addison's disease lack the normal response to ACTH, so the cortisol level will fail to rise. Dexamethasone suppression test is incorrect. The dexamethasone suppression test is used in the diagnosis of Cushing syndrome, which is caused by hypercortisolism. Plasma fractionated metanephrine is incorrect. Plasma fractionated metanephrines are used in the diagnosis of pheochromocytoma. Serum CA 19-9 is incorrect. Serum CA 19-9 is a tumor marker for pancreatic cancer. Radioactive iodine uptake test is incorrect. The radioactive iodine uptake test is used in the work-up of hyperthyroidism.

A 30-year-old woman presents with depression. She denies any significant past medical history or surgeries. On physical examination, she is found to be obese, with her weight localized to her trunk; her arms and legs are relatively normal. Her abdomen has prominent purple striae. Her face is very round and covered with acne. Her blood pressure is found to be 164/102 mm Hg. Her laboratory results are as follows: TEST RESULTS REFERENCE RANGE BUN 14 mg/dL 7 - 18 mg/dL Calcium 9.6 mg/dL 8.4 - 10.2 mg/dL Potassium 4.1 mEq/L 3.5 - 5.0 mEq/L Sodium 139 mEq/L 135 - 145 mEq/L A further workup is done and she is found to have a pituitary adenoma. What condition is most likely? Answer Choices 1 Addison's disease 2 Conn's syndrome 3 Cushing's syndrome 4 Cushing's disease 5 Nelson's syndrome

Cushing Disease Explanation This patient has a pituitary adenoma that is secreting ACTH. Cushing's disease refers to excessive secretion of ACTH by the pituitary. A pituitary adenoma is the most common cause of pituitary hypersecretion of ACTH. The increase in ACTH secretion results in hyperplasia of the adrenal cortex and a subsequent increase in glucocorticoid release. Characteristics of glucocorticoid excess include depression, truncal obesity, purple striae, round face (moon facies), and hypertension. Addison's disease is primary adrenal insufficiency. The most common cause of Addison's disease is autoimmunity; tuberculosis is another cause. The symptoms seen with Addison's disease include weight loss, lethargy, hyperpigmentation, nausea, vomiting, and hypotension. Hyponatremia and hyperkalemia can be seen because of the lack of mineralocorticoids. When there is decreased stimulation of the adrenal cortex due to a decrease in ACTH, it is called secondary adrenal insufficiency. Conn's syndrome is due to excessive mineralocorticoids, usually due to an adenoma that is secreting aldosterone. Conn's syndrome results in hypernatremia and hypokalemia; there is an associated hydrogen loss, as well. There will be hypertension. A distinction exists between Cushing's disease and Cushing's syndrome. Cushing's disease refers specifically to excessive secretion of ACTH by the pituitary. In Cushing's disease, the excessive ACTH secretion by the pituitary results in an increase in glucocorticoid release by the adrenal glands. Cushing's syndrome refers to the effects of excessive glucocorticoids (other than those due to a pituitary cause); therefore, Cushing's syndrome is due to excessive glucocorticoids. There will be hypertension with both Cushing's disease and Cushing's syndrome. Nelson's syndrome is seen in patients who have had a bilateral adrenalectomy for Cushing's disease; there is the subsequent development, or progression, of an ACTH-secreting pituitary tumor.

A young man presents with difficulty breathing at times. Upon exam you note evidence of a firm, fixed thyroid nodule with extension toward the trachea and surrounding muscles. The patient has a family history of thyroid cancer. You are concerned that the patient may have medullary thyroid cancer. After you confirm your suspicion, which of the following genetic tests would you suggest to his family members? A BRCA 1 B APC C CFTR D ret proto-oncogene E CAG triplet expansion

D ret proto-oncogene The correct choice is D, ret proto-oncogene. Genetic testing to identify this oncogene is available for patients whom you suspect have medullary thyroid cancer and/or pheochromocytoma, and for their asymptomatic families who may be at risk for the same disorders. A mutation in this gene leads to uncontrolled growth of the C cells of the thyroid. Choice A, BRCA 1, is an oncogene that is associated with risk for breast, ovarian, colon, and prostate cancers. Choice B, APC, is an oncogene noted in the familial colon cancer syndrome known as familial adenomatous polyposis. DNA analysis for CFTR mutations, choice C, can identify the gene mutations in patients with cystic fibrosis in the vast majority of instances. Choice E, CAG triplet expansion, is the result of a gene mutation in Huntington's disease.

A 35-year-old man presents with the chief complaint of "peeing a lot." The patient states for the last week, he has urinated at a rate of 4-5 times an hour; he even awakens at night to urinate and states this has severely interfered with his sleeping habits. He further describes significantly intense thirst and a powerful craving for only ice water to quench this thirst, although he never feels that this is accomplished. Pertinent past medical history includes a diagnosis of bipolar disorder at the age of 22; he takes lithium daily for this diagnosis. He is not taking any other medications regularly at this time. Question Given the medical history provided, the patient is most likely experiencing which of the following? Answer Choices 1 Cushing's disease 2 Hypothyroidism 3 Diabetes mellitus 4 Diabetes insipidus 5 Hypoparathyroidism

DI Explanation This patient is suffering from diabetes insipidus (DI). Characteristics of this disease process include an increase in thirst and the passage of large quantities of urine. The underlying pathologic feature of DI is a deficiency of vasopressin or a resistance to available vasopressin. There are a multitude of disease states that lead to DI and the patient above is most likely suffering from an acquired form of vasopressin-resistant diabetes insipidus. Disease states that lead to this form of DI can include pyelonephritis, renal amyloidosis, potassium depletion, Sj?gren's syndrome, use of some medications, and, in the patient's case, the use of lithium. Chronic use of lithium can lead to lithium toxicity and manifests as described in the patient scenario. This is one of the more common causes of DI in adult patients. Hypothyroidism is much lower on the differential due to the signs and symptoms being inconsistent. Findings such as weakness, fatigue, cold intolerance, constipation, weight changes, depression, or menstrual abnormalities would be some of the multiple features evident in patients with hypothyroidism. Hypoparathyroidism is also not as likely. Signs and symptoms of the acute phase of this disease include tetany, muscle cramps, irritability, altered mental status, and convulsions; a tingling sensation around the circumoral area, hands, and feet is almost always present. Cushing's disease is due to manifestations of hypercortisolism secondary to adrenocorticotropic hormone (ACTH) hypersecretion by the pituitary gland. Some features include but are not limited to: central obesity, apparent moon face or buffalo hump, purple striae, muscle wasting, hirsutism, psychological changes, hypertension, weakness, headache, backache, or poor wound healing. None of the signs or symptoms listed matches the patient's history above. Diabetes mellitus (DM) may also be considered in this patient scenario given the symptoms of polydipsia and polyuria described, as well as the strong family history evident. The main cause of this type of diabetes is that patients develop increased tissue insulin insensitivity, also termed insulin resistance. Some key components of the patient's history are not evident in diabetes mellitus. Nocturnal enuresis is not usually a significant chief complaint in patients with DM, although patients with DM may have complaints of polyuria, polydipsia, weakness, fatigue, blurred vision, vulvovaginitis, peripheral neuropathy, or even potentially be asymptomatic.

A 45-year-old woman comes to the office for evaluation of "yeast infection again." Review of the chart shows that she has had six episodes of vaginal candidiasis in the past 18 months. She and her husband have been mutually monogamous during their 23-year marriage; he had a vasectomy following the birth of their second child. Her two children weighed 10# 2 oz. and 10# 4 oz. at delivery. Her last menstrual period occurred one week ago and was slightly heavier than normal, but her preceding two periods were scanty and "off schedule." Her medications include a thiazide diuretic for mild hypertension and a senior multivitamin. She is 62" tall and weighs 198#. Assuming her examination demonstrates candidiasis, this woman needs most urgently to be screened for which of the following? A diabetes mellitus B hyperlipidemia C hypothyroidism D perimenopause E polycystic ovarian syndrome

DM Diabetes causes recurrent vaginal candidiasis in women with uncontrolled glucose. Type 2 diabetes is further suggested by her obesity and history of macrosomia in offspring. Furthermore, thiazide diuretics may increase glucose levels. She needs also to be screened for hyperlipidemia (B), given her weight and probable diabetes, but that is not as urgent. She may be hypothyroid (C), although the clinical picture more strongly suggests diabetes; hypothyroidism is also more likely to cause menorrhagia than scanty periods. She may also be perimenopausal (D), given irregular menses. Polycystic ovarian syndrome (E) is unlikely given no history of infertility.

A 45-year-old woman presents with vaginal itching and irritation that did not resolve with over-the-counter medications. She has not previously sought medical attention for the symptoms. During review of systems, she admits to polyuria, polydipsia, and occasional dysuria, which she attributes to the vaginal irritation. She states she has not had lab work done in over a year, but the last time she did was at a health fair; she was told her fasting glucose was 119 mg/dl. On exam, you note that the patient is 62 inches tall and weighs 243 pounds. Her vital signs are within normal limits except for her blood pressure, which is 138/84 mm Hg. You find that she has white discharge in the vaginal canal; there is beefy-red surrounding skin and similar irritation under her breasts. Question Based on this patient's presentation, what do you suspect is the patient's most serious condition? Answer Choices 1 Obesity 2 Hypertension 3 Sexually transmitted infection 4 Diabetes mellitus 5 Mucocutaneous candidiasis

DM Explanation One of the most common initial manifestations of diabetes mellitus type II in women is candidal infections. Polyuria and polydipsia are also common initial complaints. The patients often do not have ketonuria and weight loss at diagnosis. Many patients are totally asymptomatic. Most people are diagnosed by routine lab work rather than presenting with symptoms. People who are over the age of 40 as well as obese are more likely to develop this disease. Hypertension, dyslipidemia, and atherosclerosis are often associated as well.

A 70-year-old woman presents to the ED with nausea, anorexia, vomiting, fever, chills, and flank pain. Symptoms appeared abruptly after a 2-day history of mild dysuria. Her family reports that about 2 months ago, she started having urinary urgency and frequency and noted excessive urinary production of around 3-4 liters daily. She has a history of hypertension, poorly regulated type II diabetes, and mild cognitive impairment. Her long-term medications are aspirin and metformin, and 2 months ago, her family practitioner included furosemide, statin, and proposed insulin, but she was not willing to take any parenteral medication. Her fear of needles was the reason she gave for not agreeing to get recommended immunizations. On admission, she appears lethargic; her blood pressure is 80/40; pulse rate is 120; respiratory is 18; and temperature is 103.4. Question What is the primary risk factor responsible for her current condition? Answer Choices 1 Diabetes insipidus 2 Diabetes mellitus 3 Diuretic therapy 4 Cognitive impairment 5 Missed immunization

DM Explanation Your patient has a serious systemic infection, most probably pyelonephritis in the settings of poorly regulated diabetes mellitus. In diabetes, infections may precipitate metabolic disturbances, but metabolic disturbances may also facilitate the infection. Metabolic disturbances (hyperglycemia, acid-basic dysregulations) impair humoral immunity and the function of polymorphonuclear leukocytes and lymphocytes. Not only diabetes, but other chronic diseases, increase the risk of acquiring systemic infections. The strongest connection was found in chronic lung disease and chronic kidney disease. Long-standing diabetes may also lead to chronic kidney disease, which could be an additional risk factor in this patient. Although polyuria with urinary frequency and urgency may be caused by diabetes insipidus, it is much more frequently caused by uncontrolled diabetes mellitus. Another reason that your patient may have polyuria is diuretic therapy. Diuretic therapy may contribute to polyuria, but it does not contribute to the development of systemic infection. Patients with mild cognitive impairment can forget to take their medications from time to time, thus contributing to poor regulation of their diseases, but mild cognitive impairment is not a risk factor for systemic infection. Vaccines recommended in seniors (over 65) are: influenza, herpes zoster, diphtheria/tetanus, pertussis, and pneumococcal. Not getting immunized because of the fear of a needle will put your patient at risk for developing those diseases. However, she was not exposed to any of those diseases and has no signs and symptoms of any of them.

A 62-year-old obese woman presents with progressive numbness and tingling in her feet for the past 3 months. On physical examination, the patient is found to have decreased sensation to pinprick and vibration, absence of ankle reflexes, and difficulty with tandem walking. Which is the most common etiology of her symptoms? A diabetes mellitus B alcoholism C vitamin B12 deficiency D spinal cord tumor E rheumatoid arthritis

DM Peripheral neuropathy is a syndrome that is manifested by muscle weakness, paresthesias, decreased deep tendon reflexes, and autonomic disturbances most commonly in the hands and feet, such as coldness and sweating. There are many causes of peripheral neuropathy ranging from metabolic conditions to malignant neoplasm, rheumatoid arthritis, and drug and alcohol use. The increase in non-insulin-dependent diabetes mellitus due to obesity in the American population has increased the incidence of associated disease states

A 54-year-old woman presents to her primary care physician due to persistent fatigue. She notes feeling more thirsty than usual, and she has been getting up several times throughout the night to urinate. Her physician obtains the following labs: Fasting blood glucose: 170 mg/dL 2 hour oral glucose tolerance test: 230 mg/dL Hemoglobin A1C: 9% Question What is the most likely diagnosis in this patient? Answer Choices 1 Type I diabetes mellitus 2 Type II diabetes mellitus 3 Impaired glucose tolerance 4 Diabetes insipidus 5 Diabetic ketoacidosis

DM t2 Explanation The above patient is most likely suffering from type II diabetes mellitus. The diagnostic criteria for diabetes includes a fasting blood glucose greater than or equal to 126 mg/dL, 2 hour oral glucose tolerance test with blood glucose value greater than 200, and a hemoglobin A1C level greater than or equal to 6.5%. The patient's polydipsia and polyuria are also suggestive of diabetes mellitus. Type I diabetes mellitus could also present with the above history and laboratory findings. However, type I diabetes mellitus typically presents in young adulthood, not in middle age. A patient is considered to have impaired glucose tolerance if their fasting blood glucose is 100-125 mg/dL, their 2-hour oral glucose tolerance test is 140 - 199 mg/dL, and their hemoglobin A1C is 5.7-6.4%. The above patient's laboratory values exceed this range. Though a patient with diabetes insipidus would complain of polyuria and polydipsia, they would not have the glucose and hemoglobin A1C findings seen in this patient. While a patient with diabetic ketoacidosis might present with history of polyuria and polydipsia, they would have a higher blood glucose level. The patient might also have associated nausea, vomiting, mental status changes, and a fruity breath odor.

A 28-year-old woman presents for a routine follow up. She is a type I diabetic and is on a multiple daily insulin injections regimen. At her last visit, she mentioned having difficulties with her morning glucose levels. She has been tracking her glucose levels daily. Her pre-breakfast glucose averages 285 mg/dL. At dinnertime, it averages 95 mg/dL, and it averages 68 mg/dL at 3 AM. Question What is the best change to make in her treatment regimen? Answer Choices 1 Increase the evening dose of long-acting insulin 2 Decrease the evening dose of long-acting insulin 3 Decrease the evening dose of short-acting insulin 4 Increase the evening dose of short-acting insulin 5 Increased activity before bedtime

Decrease the evening dose of long-acting insulin Explanation Since her 3 AM level was low, that indicates the patient's early morning hyperglycemia is due to the Somogyi effect. The evening dose of long-acting insulin should be decreased, or she should eat bedtime snacks. 3 mechanisms can account for early morning hyperglycemia in patients with diabetes on insulin: An inadequate dose of long acting insulin at night can be a cause. The Somogyi effect, or rebound hyperglycemia, is a condition often seen in patients with type I diabetes and nocturnal hypoglycemia. The resulting hypoglycemia stimulates the release of counter regulatory hormones leading to increase of glucose levels in the blood. Diagnosis can be made by several plasma glucose analyses (done at 2 AM, 3 AM, and 7 AM). The dawn phenomenon, where the hyperglycemia is secondary to an increased need for insulin in the early morning due to normal early morning surge of growth hormone, which antagonizes insulin action. Checking the blood glucose level at 3 AM can help differentiate between the Somogyi effect and dawn phenomenon: the level will be low in the Somogyi phenomenon. The treatment of the dawn phenomenon involves increasing the evening dose of long-acting insulin. There is no nocturnal hypoglycemia if the night dose of insulin is inadequate.

A 33-year-old man presents with severe thirst and frequent urination; symptoms have been developing over the past month. He claims to consume more than 8 liters of fluids per day. Despite that, he is always thirsty. Fasting blood glucose is 90 mg/dl. On brain MRI, the hyperintensity of the posterior pituitary is absent. What drug is likely used to treat this patient? Answer Choices 1 Triamterene 2 Desmopressin 3 Digoxin 4 Verapamil 5 Propranolol

Desmopressin Desmopressin is a synthetic analog of arginine vasopressin (DDAVP). It acts on the kidney to decrease urine output and reduce the subjective feeling of thirst in patients with diabetes insipidus. DDAVP is usually administered via the mucus membrane of the nose. The dose required to keep the patient in water balance must be determined by measuring plasma sodium and plasma osmolality. Triamterene is a potassium sparing diuretic that produces a mild diuretic effect. Digoxin is used in the treatment of congestive heart failure. Its principal effect is to enhance cardiac output and efficiency and not as a diuretic on the kidney. Verapamil is an antiarrhythmic, antihypertensive drug classified as a calcium channel blocker. The beta-receptor blocking drug propranolol is used as an anti-hypertensive; however, it is not classified as a diuretic.

A 28-year-old man presents with increasing polyuria and nocturia without significant changes in his weight. He has noted increased thirst subsequent to increased urination. Lab results are as follows: Test Result Normal range Glucose 98 70 - 100 mg/dL Sodium 145 135 - 145 mEq/dL Serum osmolality 298 285 - 295 mOsm/kg Urine specific gravity <1.005 Urine osmolality is low, and the urine is poorly concentrated on the water restriction test. Question What treatment would best address this patient's symptoms and condition? Answer Choices 1 Bromocriptine 2 Desmopressin 3 Growth hormone 4 Hydrocortisone 5 Spironolactone

Desmopressin Explanation This patient has subjective and objective findings consistent with diabetes insipidus. Diabetes insipidus is characterized by the decreased production or action of antidiuretic hormone (ADH), which is produced by the posterior pituitary. The body is then unable to concentrate the urine, resulting in polyuria, nocturia, elevated serum osmolality, and decreased urine specific gravity. The treatment of choice for this patient's diabetes insipidus is desmopressin, which is a potent antidiuretic. Bromocriptine is an FDA-approved dopamine-agonist used in the treatment of hyperprolactinemia. Other uses include the treatment of acromegaly and Parkinson's disease. The mechanism of action for bromocriptine includes stimulation of dopamine receptors and inhibition of anterior pituitary prolactin secretion. It has no direct action on the posterior pituitary's secretion of ADH, nor on the kidney's urine-concentrating ability. Because autoimmune destruction of the pituitary is theorized as a cause in idiopathic central diabetes insipidus, there are often deficiencies in growth hormone (GH), thyroid stimulating hormone (TSH), and adrenocorticotropic hormone (ACTH), as well as the deficiency in antidiuretic hormone (ADH). Growth hormone (GH) would be helpful in treating disorders affecting the anterior pituitary's secretion of GH; however, treating this patient with growth hormone will not directly address the polyuria and nocturia caused by his diabetes insipidus. The presentation of a GH deficiency would include short stature, growth failure, hypoglycemia in infancy, and central distribution of body fat. This patient did not have a presentation consistent with GH deficiency. In cases of panhypopituitarism, secretion decreases for all pituitary hormones including ADH and ACTH. In this case, the low ACTH levels would lead to low cortisol levels and necessitate treatment with hydrocortisone. Hydrocortisone would not appropriately address this patient's polyuria and nocturia from his diabetes insipidus. Spironolactone, a potassium-sparing diuretic, inhibits the effects of aldosterone on the renal tubules. Using a diuretic on this patient would only exacerbate the fluid loss and polyuria.

A patient presents with polydipsia and polyuria. Diabetes mellitus is ruled out with a normal plasma glucose and hemoglobin A1c. You are concerned that he may have hypothalamic diabetes insipidus. If you are correct, what treatment would you recommend to this patient? A Lithium B Indomethacin C Metformin D Desmopressin E Fluid restriction

Desmopressin The correct choice is D, desmopression. Patients with hypothalamic diabetes insipidus can't secrete vasopression (or antidiuretic hormone), since the vasopressin producing neurons are dead. Vasopressin analog desmopressin is available in tablets, nasal solution, and parenteral solution for patients with this disorder. Choice A, lithium, can cause nephrotoxicity, and can be a cause of acquired nephrogenic diabetes insipidus. Choice B, indomethacin, is an anti-inflammatory medication used to treat conditions such as gout. Choice C, metformin, is an oral diabetic medication that acts by suppressing hepatic glucose production. Patients must drink sufficient fluids to maintain serum sodium levels since without ADH, they can't conserve water. Therefore, choice E, fluid restriction, is not correct.

A 75-year-old woman presents with frequent leakage of her urine over the past several months; she is also experiencing frequent urinary infections. Leakage is sometimes "continuous", occurring both day and night. She also complains of tingling in her feet and hands, nausea, and a feeling of fullness after eating only a small amount of food. Her past medical history is significant for a 15-year history of diabetes mellitus type II, mild hypertension, and renal insufficiency stage 1. She takes metformin and ACE inhibitors. On examination, you find distended bladder and dysesthesia of stocking and glove pattern; the rest of the examination is within normal limits for her age. Question What is the most likely cause of her symptoms? Answer Choices 1 Pelvic floor muscle weakness 2 Detrusor underactivity 3 Kidney failure 4 Detrusor overactivity 5 Urinary tract infection

Detrusor Underactivity Explanation Having symptoms and signs of sensory disturbance and gastroparesis, your patient most probably has diabetic polyneuropathy that also involves autonomic nervous system. Bladder dysfunction usually appears 10 or more years after the onset of diabetes mellitus. The mechanism is most probably segmental demyelination and impaired nerve conduction. Your patient does not realize that her bladder has reached the maximal storage capacity, and the incontinence is a result of a chronically over-filled bladder. Emptying the bladder is often also not effective because of poor contractibility of a detrusor muscle. Pelvic floor muscles weakness typically presents with leaking a small amount of urine when coughing, sneezing, laughing, or running, but not with continuous leaking. Detrusor overactivity, as seen in spastic bladder, is associated with symptoms of urge incontinence; it is usually associated with frequency and nocturia. The symptoms she has cannot be explained by kidney failure. In addition, patients with chronic kidney disease stage 1 are usually asymptomatic. Retention of the urine contributes to the development of urinary tract infections (UTI). UTI presentation is primarily dysuria, with accompanying urinary urgency and frequency. Your patient has no such symptoms.

A 40-year-old female presents to your office with symptoms of weight gain, hirsuitism, and easy bruising. Past medical and surgical history is noncontributory. She drinks one glass of wine on weekends and does not smoke cigarettes. She takes one multivitamin daily. Upon physical exam, you note facial fullness, central obesity, and thin skin. Which of the following is a valuable biochemical screening test for this patient that will aide in the diagnosis? A Dexamethasone suppression test B Radioactive iodine uptake C Glucose tolerance test D Cosyntropin stimulating test E Plasma fractionated free metanephrines

Dexamethasone suppression test The correct choice is A, dexamethasone suppression test. This patient is presenting with classic signs and symptoms of Cushing's syndrome. The dexamethasone suppression test is a simple test of the hypothalamic-pituitary-adrenal axis, and requires ingestion of oral dexamethasone at nighttime and a blood test in the morning hour, to measure the amount of plasma cortisol. Most patients with Cushing's syndrome demonstrate a lack of normal axis suppression and present with a morning plasma cortisol level >5 mcg/dL. Choice B, radioactive iodine uptake, is used in patients with suspected thyroid disorders. Choice C, glucose tolerance test, is used in patients with suspected diabetes mellitus and in prenatal testing, to investigate gestational diabetes. Choice D, cosyntropin stimulating test, is used to investigate possible adrenal insufficiency. Choice E, plasma fractionated free metanephrines, is used in the diagnostic workup of pheochromocytoma.

A 38-year-old woman presents because she feels that she is "peeing a lot". For the last week, she has urinated at a rate of 4 - 5 times an hour; she even wakes up at night to urinate. She states that this has severely interfered with her sleeping habits. She further describes a significantly intense thirst and a powerful craving for only ice water to quench this thirst, although she never feels that her thirst has been quenched. Pertinent past medical history includes a diagnosis of bipolar disorder when the patient was 20. She takes lithium daily for this diagnosis, and she takes ibuprofen as needed for headaches. She is not taking any other medications regularly at this time. Significant family history includes her mother and father being diagnosed with diabetes mellitus as well as an older sister who was diagnosed with hypothyroidism. Question Given the medical history provided, what is the patient most likely experiencing? Answer Choices 1 Cushing's disease 2 Hypothyroidism 3 Diabetes mellitus 4 Diabetes insipidus 5 Hypoparathyroidism

Diabetes Insipidus Explanation This patient is suffering from diabetes insipidus (DI). Characteristics of this disease process include an increase in thirst and the passage of large quantities of urine. The underlying pathologic feature of DI is a deficiency of vasopressin or a resistance to available vasopressin. There are a multitude of disease states that lead to DI, and the patient is most likely suffering from an acquired form of vasopressin-resistant diabetes insipidus. Disease states that lead to this form of DI can include pyelonephritis, renal amyloidosis, potassium depletion, Sj?gren's syndrome, and the use of certain medications; in the patient's case, the use of lithium resulted in her DI. Chronic use of lithium can lead to lithium toxicity, and it manifests as described in the patient scenario. This is a common cause of DI in adult patients. Hypothyroidism is much lower on the differential due to the signs and symptoms being inconsistent. Findings such as weakness, fatigue, cold intolerance, constipation, weight changes, depression, and/or menstrual abnormalities would be some of the multiple features evident in patients with hypothyroidism. Hypoparathyroidism is also not as likely. Signs and symptoms of the acute phase of this disease include tetany, muscle cramps, irritability, altered mental status, and convulsions; a tingling sensation around the circumoral area, hands, and feet is almost always present. Cushing's disease is due to manifestations of hypercortisolism secondary to adrenocorticotropic hormone (ACTH) hypersecretion by the pituitary gland. Some features include (but are not limited to): central obesity, apparent moon face or buffalo hump, purple striae, muscle wasting, hirsutism, psychological changes, hypertension, weakness, headache, backache, and poor wound healing. None of the signs or symptoms listed matches the patient's history. Diabetes mellitus (DM) may also be considered in this patient scenario given the symptoms of polydipsia and polyuria described as well as her family history. The main cause of this type of diabetes is that patients develop increased tissue insulin insensitivity, which is also termed insulin resistance. Some key components of the patient's history are not evident in diabetes mellitus. Nocturnal enuresis is not usually a significant symptom in patients with DM, although patients with DM may have complaints of polyuria, polydipsia, weakness, fatigue, blurred vision, vulvovaginitis, and peripheral neuropathy; they may even be asymptomatic.

A 42-year-old man presents with fatigue, polyuria, and polydipsia for the past 2 - 3 months. He has a history of hypertension and obesity. He is on 10 mg of amlodipine daily and 75 mg of aspirin daily. His family history is significant for diabetes mellitus in the patient's mother and 2 siblings. On exam, the patient is afebrile; he has a blood pressure of 130/80 mm Hg; he has a weight of 220 lbs, and his height is 66 inches. He has no pallor, icterus, or lymphadenopathy. Lungs are clear to auscultation, and heart sounds are regular. The abdomen is normal, and there is minimal pitting pedal edema. Initial fundus exam is normal. Tests are done, including fasting blood glucose on 2 different days, a basic metabolic panel, and lipid profile. Fasting blood glucose was 136 mg/dL on the 1st day and 140 mg/dL on the 2nd. He is diagnosed with type II diabetes mellitus. Question What is the best strategy for treatment of this patient? Answer Choices 1 Diet control, weight loss, and exercise for 6 - 12 months; then, follow up tests should be performed for further treatment 2 Diet, weight loss, and exercise indefinitely if symptoms resolve 3 Oral hypoglycemic therapy with a sulfonylurea; tests should be repeated every 3 months, and a 2nd agent should be added if fasting blood sugar still above 140 mg/dL 4 Diet, weight loss, exercise, and monotherapy with metformin for 6 months, followed by repeat tests and adjustment of medication dose or addition of sulfonylurea 5 Diet, weight loss, and exercise for 4 - 6 weeks; then, if fasting blood sugar level remains above 126 mg/dL, add metformin

Diet, weight loss, and exercise for 4 - 6 weeks; then, if fasting blood sugar level remains above 126 mg/dL, add metformin Explanation A newly diagnosed patient with type II diabetes mellitus should be given a trial of therapeutic lifestyle changes with strict diet control, weight loss, and a moderately strenuous exercise regimen for 4 - 6 weeks. Afterwards, if the fasting blood sugar level is above 126 mg/dL, metformin should be added. Dietary modifications include low-calorie and low-fat diet with high complex carbohydrates. Some patients with mild diabetes may remain well controlled for a long period of time, but may eventually need medical treatment. Most patients need treatment sooner. Metformin acts by decreasing hepatic gluconeogenesis and increasing peripheral utilization of glucose. It is the drug of choice in type II diabetics who are overweight, since it causes mild to moderate decrease in weight. It is also quite effective in non-obese patients. The common side effects include abdominal pain, diarrhea, a metallic taste in the mouth, nausea, and anorexia. Lactic acidosis is a serious side effect. The contraindications for use of metformin include prior history of lactic acidosis, renal insufficiency with creatinine of 1.5 mg/dL, alcoholism, hepatic insufficiency, sepsis and other severe infections, heart failure, hypoxia and respiratory depression, and concurrent or anticipated use of radiographic material. In normal weight patients, sulfonylureas or biguanides are used. The dose may be increased every 4 - 8 weeks until blood sugar is controlled and hemoglobin A1C has decreased, with a goal of 7.0. Lifestyle changes should be continued for life.

A 34-year-old woman presents with worsening headaches. She says that the headaches are present throughout the day and that she has been feeling nauseous. Lately she has also noticed difficulty in seeing vehicles overtake her on the freeway. She has had several close calls due to this handicap. Her periods, which were previously regular are now irregular, with heavy bleeding every 3 - 4 months. She has also noticed a milky discharge from both nipples. Her pregnancy test is negative. An MRI of the brain shows a mass compressing the hypothalamic pituitary axis. Question What is the best course of treatment? Answer Choices 1 Irradiation of the tumor 2 Surgical excision 3 Catheter ablation 4 Dopamine agonists 5 External beam laser coagulation

Dopamine Agonist The most likely diagnosis is a pituitary adenoma compressing the HP axis. The standard of care at the present time is the use of dopamine agonist drugs, such as bromocriptine and cabergoline. These drugs reduce prolactin levels to normal in 70 - 100% of patients. The presence of the pituitary mass blocks the action of dopamine on the prolactin-secreting cells and increases the secretion of prolactin. Cabergoline is an alternative to bromocriptine with fewer side effects. However, it is a much more expensive drug, and it is usually used when bromocriptine is not tolerated. Pergolide has been taken off the market due to associated heart valve damage with its use. Followup may be done by measurement of the fasting serum prolactin level. Surgery is incorrect, as it is usually indicated only in failure of medical therapy, continued visual field loss despite medication, or failure to tolerate medical therapy. Radiation therapy is incorrect. Although effective, it is seldom used due to the risk of permanent hypopitutarism after treatment. External beam coagulation and ablation are both incorrect. They are distractors that have no role in therapy.

A 24-year-old woman has recently been diagnosed with insulin-dependent diabetes. The disease is being managed on a split dose of 60/40 insulin suspension, which she injects herself at 8:00 a.m. and 5:00 p.m. She was told to call in if she experiences any strange symptoms, which she does this afternoon. At 2:45 p.m., she is not feeling well and notices that her skin is cool and damp. Her hands are shaking and she is very anxious. What do you tell her to do right away, before having somebody take her to your office? Answer Choices 1 Inject 4 IU of her insulin 2 Drink a can of diet soda 3 Drink four ounces of fruit juice 4 Eat a large candy bar 5 Eat a cube of sugar

Drink 4 ounces of fruit juice Explanation The above described symptoms are classical for hypoglycemia. An inexperienced patient needs to be monitored by medical personnel until the blood glucose level is stabilized again. The first priority, though, is to prevent the blood sugar to drop any further and send the patient into a coma. A four-ounce drink of fruit juice should be sufficient. Since diet soda contains only sugar substitutes, it is not going to influence the blood glucose level. A large candy bar could be eaten too, but it may raise the glucose level higher than required. Therefore, the fruit juice is a better choice. 1 cube of sugar is an insufficient amount to raise blood sugar; it will take several cubes. According to the American Diabetes Association, 15-20grams of carbohydrates are recommended. Since the symptoms suggest hypoglycemia, injecting insulin would worsen the situation. If unsure whether the patient is hypo- or hyperglycemic, and there is no possibility for a test, always give sugar first and see if the patient improves. An increase of a blood glucose level of, for example, 350 is not going to hurt the patient; however, lowering a level of 45 is going to send the diabetic into a coma.

A 29-year-old woman presents in July to your office with symptoms of palpitations, sore neck, and excessive sweating, despite using her air conditioner all the time. No surgical or trauma history is noted. She is currently not taking any medications. Vitals include the following: BP = 124/68, pulse = 110 beats per minute, respirations = 18 per minute, and temperature = 101 o F orally. Upon exam, her thyroid is mildly enlarged without nodules, and severely tender. No local erythema or heat is noted. What is the most likely diagnosis in this patient? A Thyroid abscess B Papillary thyroid cancer C Hashimoto's thyroiditis D Graves' disease E Subacute thyroiditis

E Subacute Thyroiditis This patient is presenting with signs and symptoms of hyperthyroidism, most likely caused by subacute thyroiditis. The enlarged and tender thyroid, along with the fever, are classic findings in this condition. Choice A, thyroid abscess, is a localized infection of the thyroid. It would present with heat, swelling, and erythema in the area of tenderness. Choice B, papillary thyroid cancer, commonly presents with either a local non-tender thyroid nodule or as lymph node metastasis. Choice C, Hashimoto's thyroiditis, is a cause of chronic thyroiditis and is associated with a clinical presentation of hypothyroidism. Choice D, Graves' disease, is the most common cause of hyperthyroidism, but does not present with thyroid tenderness and fever.

A 45-year-old woman presents with recent, unexplained weight loss and lethargy. She has had several episodes of nausea and vomiting. She admits to craving salty foods. On physical examination, there is a hyperpigmentation to her skin and mucus membranes. Her blood pressure is 85/60 mmHg. Her laboratory results show hyponatremia and hyperkalemia. Her serum ACTH level is elevated. Question What is causing her hyperpigmentation? Answer Choices 1 Elevated ACTH 2 Hyponatremia 3 Hyperkalemia 4 Elevated mineralocorticoids 5 Elevated glucocorticoids

Elevated ACTH Explanation This patient has signs and symptoms consistent with Addison's disease. Addison's disease is primary adrenal insufficiency. The symptoms include weight loss, lethargy, nausea, vomiting, and salt craving. Because of the low level of adrenocortical hormones, there is an increase in ACTH. Hyperpigmentation is secondary to the elevated ACTH. A portion of ACTH contains melanin-stimulating hormone, thus explaining the increase in pigmentation. Hyperpigmentation would not be associated with any of the other choices.

A 45-year-old woman presents with recent, unexplained weight loss and lethargy. She has had several episodes of nausea and vomiting. She admits to craving salty foods. On physical examination, there is a hyperpigmentation to her skin and mucus membranes. Her blood pressure is 85/60 mmHg. Her laboratory results show hyponatremia and hyperkalemia. Her serum ACTH level is elevated. Question What is causing her hyperpigmentation? Answer Choices 1 Elevated ACTH 2 Hyponatremia 3 Hyperkalemia 4 Elevated mineralocorticoids 5 Elevated glucocorticoids

Elevated ACTH This patient has signs and symptoms consistent with Addison's disease. Addison's disease is primary adrenal insufficiency. The symptoms include weight loss, lethargy, nausea, vomiting, and salt craving. Because of the low level of adrenocortical hormones, there is an increase in ACTH. Hyperpigmentation is secondary to the elevated ACTH. A portion of ACTH contains melanin-stimulating hormone, thus explaining the increase in pigmentation. Hyperpigmentation would not be associated with any of the other choices.

A 38-year-old woman presents with fatigue, amenorrhea, and easy bruising. Examination reveals multiple healing bruises, central obesity, a round face, supraclavicular fat pads, abdominal striae, and hypertension. Question What is an expected finding? Answer Choices 1 Hypoglycemia 2 Hyperkalemia 3 Elevated serum cortisol 4 Decreased urine cortisol 5 Positive cosyntropin stimulation test

Elevated serum cortisol Explanation The correct answer is elevated serum cortisol. The symptoms of fatigue, amenorrhea, and easy bruising in the setting of the above physical exam findings is consistent with a diagnosis of Cushing's syndrome. The patient will have an elevated serum cortisol level. Hypoglycemia is an incorrect response. Cushing's syndrome is associated with hyperglycemia. Hyperkalemia is an incorrect response. Cushing's syndrome is associated with hypokalemia. Decreased urine cortisol is an incorrect response. Cushing's syndrome is associated with elevated urine cortisol. Positive cosyntropin stimulation test is an incorrect response. A positive cosyntropin stimulation test occurs with Addison's disease.

A 33-year-old woman presents due to a 15 pound weight gain over 2 months; there is also muscle weakness, menstrual irregularities, amenorrhea, infertility, skin bruising, memory loss, and periods of depression. She denies any medication use or dietary changes; she has tried to lose weight unsuccessfully through increased exercise. She denies any headache, vision changes, hearing changes, headache, chest pain, abdominal symptoms, polyuria, polydipsia, or breast discharge. Her physical exam reveals a blood pressure of 145/94 mm Hg. Her skin physical exam is remarkable for the findings in the image. Question What is an expected diagnostic finding in this patient? Answer Choices 1 Hyperkalemia 2 Metabolic acidosis 3 Elevated FSH and LH levels 4 Hypoglycemia 5 Elevated urinary free cortisol (UFC) levels

Elevated urinary free cortisol (UFC) levels Explanation This patient is presenting with signs and symptoms consistent with Cushing's syndrome; in this case, it is most likely from an adrenal adenoma source. Expected laboratory findings associated with the syndrome may include an elevated leukocyte count, hyperglycemia, and a hypokalemic metabolic alkalosis, which may occur in patients with markedly elevated cortisol due to cortisol activation of the renal mineralocorticoid receptor. The diagnosis of Cushing syndrome due to endogenous overproduction of cortisol requires the demonstration of inappropriately high serum cortisol or urine cortisol levels. Urinary free cortisol (UFC) determination has been widely used as an initial screening tool for Cushing syndrome because it provides measurement of cortisol over a 24-hour period. Menstrual irregularities, amenorrhea, infertility, and decreased libido may occur in women due to inhibition of pulsatile secretion of luteinizing hormone (LH) and follicle-stimulating hormone (FSH), which is likely due to interruption of luteinizing hormone-releasing hormone (LHRH) pulse generation.

A 45-year-old woman presents with a weight gain of 50 pounds; she attributes the weight gain to her hard-working lifestyle as a corporate attorney. She usually eats out and rarely exercises. She has a fasting blood glucose of 220 mg/dL and has a urinary albumin/g creatinine of 40 mg. These are new findings since her visit 5 years ago. She has been following a high-protein diet, primarily consisting of animal proteins, for the past couple of days. What dietary and lifestyle interventions should you recommend due to her blood glucose and urinary albumin findings? Answer Choices 1 Continue high-protein diet for weight loss and reducing proteinuria, begin exercising 2 Continue present high-protein diet for weight loss and glycemic control, begin exercising 3 Continue present high-protein diet for weight loss, glycemic control, and proteinuria reduction 4 Encourage a low-calorie diet, modest protein and carbohydrate restriction with exercise 5 Discontinue present diet and recommend a low-fat and extremely low-protein diet

Encourage a low-calorie diet, modest protein and carbohydrate restriction with exercise 1 popular diet therapy is the high-protein diet; patients consume >25% of calories from protein sources. Government guidelines typically suggest protein intake to be in the <20% range. Although these diets may assist in weight loss in the short term (≤6 months) by increasing satiety and decreasing postprandial sugar fluxes and insulin surges, their long-term benefit is less clear. Diets suggested for diabetics typically include a combination of food groups including carbohydrates, proteins, and fats. Weight loss typically assists with glycemic control and reduces insulin resistance. Some diabetic patients (1 diagnostic criterion is a random sugar >200 mg/dL) can control their diabetes with diet alone. This patient has obesity, hyperglycemia, and proteinuria, a marker of kidney disease, and she should follow modest reductions in protein intake. The kidney disease outcome initiative (National Kidney Foundation) suggests modest reductions in protein intake. Severe protein restriction is not suggested for diabetics or patients with kidney disease because it may not prevent declines in kidney function; it may contribute to protein, vitamin, and nutrient malnutrition. Although short-term protein loads will increase glomerular filtration rate, long-term, high-protein diets may cause a decline in glomerular filtration rate and kidney function, particularly in patients with diabetes and existing kidney disease. Dietary sources of protein (e.g., chicken, lamb, and beef) may include advanced glycosylation end products (AGE); these are the amino portions of proteins that have reacted through glycation and oxidation to cause the formation of sugar proteins. Consumed AGEs and AGEs created when excess proteins link with excess sugar may lead to cross-linking and scarring of the matrix and glomerular wall portions of the kidney, leading to scarring. Proteins may also promote tissue damage by generating free radicals and causing oxidative stresses. Vegetable-based proteins appear to have lesser effects on renal function than do animal-based proteins.

A 35-year-old man with a 10-year history of type I diabetes presents with early fullness, abdominal pain, stomach spasms, heartburn, nausea, vomiting, bloating, and lack of appetite. Symptoms are getting progressively worse despite the patient's recommended lifestyle and dietary changes. Subsequent isotopic gastric emptying study confirms gastroparesis. Question What initial treatment should be suggested to this patient? Answer Choices 1 Erythromycin 2 Botulinum toxin 3 Misoprostol 4 Amoxicillin 5 A jejunostomy tube

Erythromycin The patient has presented with diabetic gastroparesis, which is a delayed gastric emptying of the stomach. This results in food remaining in the stomach for a longer period of time than normal. Erythromycin is useful in treating gastroparesis due to this pro-motility effect. Botulinum toxin injection into the pyloric sphincter is sometimes used in improving symptoms of nausea and vomiting in patients with refractory diabetic gastroparesis. However, this invasive method can be considered only after all other methods with fewer side effects are exhausted. Misoprostol is used for the prevention of non-steroidal anti-inflammatory drugs-induced gastric ulcers. Your patient has no signs and symptoms of peptic ulcer disease and does not need misoprostol. Amoxicillin in combination with metronidazole and proton pump inhibitor is therapy suggested for uncomplicated cases of ulcers associated with Helicobacter pylori. Your patient has no signs and symptoms of ulcer caused by Helicobacter pylori. A jejunostomy tube is the invasive method that might be considered after all other methods with fewer side effects are exhausted.

A 41-year-old man presents for a physical. He has not had a routine physical in a couple of years. Upon taking a review of systems, you become suspicious that the patient is not present for routine health maintenance alone. The patient admits to various abnormal symptoms that result in the patient receiving an order slip for a 24-hour urine study looking for vanillylmandelic acid (VMA), catecholamines, and metanephrines. Question What triad of symptoms indicated a need for this particular 24-hour urine study? Answer Choices 1 Dry skin, fatigue, and shortness of breath 2 Fatigue, flushing, and constipation 3 Headache, dry skin, and nausea 4 Flushing, constipation, and nausea 5 Headache, palpitations, and diaphoresis

Explanation 'Headache, palpitations, and diaphoresis' is the correct answer. These 3 symptoms together make up the "classic triad" in patients with a pheochromocytoma. A pheochromocytoma is a catecholamine-producing tumor that is most commonly found in the adrenal gland superior to the kidney. It is commonly diagnosed due to hypertension, but it causes less than 1% of the cases of hypertension. The symptom triad listed above has been found to have a specificity of almost 94% and a sensitivity of almost 91% for patients with pheochromocytomas. Patients may also experience other symptoms, including constipation, weight loss, flushing, and dizziness. Since the tumor produces catecholamines, excess production is usually measured by determining the levels of excreted metabolites during a 24-hour period by a 24-hour urine. Abdominal MRI can then be used to confirm the presence of the adrenal tumor. 'Dry skin, fatigue, and shortness of breath' is not the correct answer. Fatigue is not a common symptom in patients with a pheochromocytoma, but it could certainly occur secondary to the pain, anxiety, or shortness of breath. Dry skin is actually the opposite of the common symptom, diaphoresis, which tends to occur in patients with a pheochromocytoma. Shortness of breath is a symptom that patients with pheochromocytoma can experience, but it is not part of the classic triad of symptoms. 'Fatigue, flushing, and constipation' is not the correct answer. Fatigue is not a common symptom in patients with a pheochromocytoma, but it could certainly occur secondary to the pain, anxiety, or shortness of breath. Flushing and constipation can both occur in patients with a pheochromocytoma, but these symptoms are not part of the classic triad, nor do they have the high sensitivity and specificity of the classic triad of palpitations, diaphoresis, and headache. 'Headache, dry skin, and nausea' is not the correct answer. Headache is 1 of the symptoms associated with the classic triad of symptoms that patients with a pheochromocytoma can experience, but dry skin is certainly not a symptom of a pheochromocytoma. The opposite is true, actually, as patients tend to experience diaphoresis. Nausea can occur, but it is not common and is not a part of the classic triad of symptoms. 'Flushing, constipation, and nausea' is not the correct answer. All 3 of these symptoms can occur in a patient with a pheochromocytoma, but none of them are a part of the classic triad of diaphoresis, headache, and palpitations that has a high specificity and sensitivity for pheochromocytoma.

A 27-year-old woman presents with a 2-month history of worsening irritability, fatigue, and weight loss. Her past medical history includes myasthenia gravis, but she is otherwise healthy. Question During her history taking, what positive review of systems symptoms would put hyperthyroidism on your differential list? Answer Choices 1 Excessive sweating, heat intolerance, heart palpitations 2 Hair loss, decreased energy, dry skin 3 Cramping, numbness around mouth, tingling in distal extremities 4 Bone pain, flank pain, anxiety 5 Dizziness, salt craving, chronic diarrhea

Explanation A patient with hyperthyroidism could experience excessive sweating, heat intolerance, heart palpitations, and a myriad of other possible symptoms. Excessive secretion of the circulating free thyroid hormones affects multiple organ systems, resulting in this myriad of potential symptoms. The facts that the patient is female, in her third decade of life, and has another autoimmune disorder are also risk factors for hyperthyroidism. Hair loss, decreased energy, and dry skin are symptoms of hypothyroidism rather than hyperthyroidism. Hypothyroidism is typically an autoimmune disease or a result of prior radiation therapy or thyroid surgery. While some of the symptoms can overlap with those of hyperthyroidism, that is not typically the case with those listed. In fact, a patient with hyperthyroidism is likely to have hyperactivity and irritability rather than decreased energy. They are also more likely to have excessive sweating as opposed to dry skin. Cramping, numbness around mouth, and tingling in distal extremities are symptoms that could occur in a patient with hypoparathyroidism. This disorder is a result of a deficiency of parathyroid hormone from congenital absence, injury, surgery, or other diseases. Most of the symptoms associated with hypoparathyroidism are a result of hypocalcemia. Bone pain, flank pain, and anxiety could occur in a patient who has hyperparathyroidism. This disorder is characterized as a dysfunction in the body's regulatory system for parathyroid hormone. While up to 75% of patients can be asymptomatic, the hypercalcemia associated with hyperparathyroidism can result in the classic complaints of painful bones, renal stones (causing flank pain), abdominal groans, and psychic moans. Dizziness, salt craving, and chronic diarrhea are symptoms that could occur in a patient with Addison's disease. Adrenal gland insufficiency can occur as a result of autoimmune inflammation and multiple other causes, and it is more common in women. Other symptoms that can commonly occur are weakness, fatigue, anorexia, nausea, vomiting, depression, and myalgias.

A 22-year-old woman is started on carbimazole for the treatment of her hyperthyroidism. She has been diagnosed with Graves' disease and has been symptomatic for the last 6 months. She is otherwise healthy and does not take any other medications. You receive a call 2 weeks later from the patient; she tells you that she has had a mild sore throat for the past 2 days and has been feeling like she is coming down with the flu. She states that the symptoms of her hyperthyroidism are a little bit better. Question What should you do? Answer Choices 1 Increase dose of carbimazole 2 Prescribe a small dose of thyroxine 3 Tell her to present ASAP for total and differential blood count 4 Prescribe antibiotics 5 Prescribed saltwater gargles and acetaminophen

Explanation Asking her to come to clinic and have her total and differential blood count done is the correct answer. One uncommon, but well recognized, side effect of antithyroid medications is the potential to cause rapid agranulocytosis, characterized by a very low circulating neutrophil count. Patients usually develop agranulocytosis during the first 3 months of therapy. The drug most commonly associated with it is propylthiouracil. Symptoms most commonly encountered include a fever, sore throat, myalgia, and rigors. Any patient with these symptoms, especially within the first few weeks of initiating therapy, must have their white count checked. The incidence of such side effects is well below half a percent, thus making it economically unviable to routinely monitor counts. The suggested method of treatment once agranulocytosis is proven is the administration of broad spectrum antibiotics with intravenous granulocyte colony stimulating factor. This results in rapid resolution of neutropenia. Increasing her dose of carbimazole is incorrect. That would only worsen the problem. Self administration of thyroxine is incorrect. The patient has hyperthyroidism to begin with and is not exhibiting any signs of hypothyroidism. Prescribing antibiotics is incorrect. While it may be done, the neutrophil count must first be determined in order to rule out neutropenia. If the white count is normal, a mild pharyngitis may not need antibiotics in any case. Prescribing salt water gargles and acetaminophen is incorrect. The neutrophil count must first be determined.

A 60-year-old man you have been treating for congestive heart failure symptoms has been diagnosed with diabetes mellitus. The endocrinologist who made the diagnosis decided to treat the patient with glipizide 5 mg daily since his blood glucose readings are 280 mg/dl. You manage the patient's congestive heart failure with enalapril 10 mg daily, and he takes a furosemide 20 mg daily if he needs it. What kind of effect on the patient's hyperglycemia may be expected in these settings? Answer Choices 1 Decrease of hypoglycemic effect 2 Increase of hypoglycemic effect 3 No hypoglycemic effect 4 Mild hyperglycemic effect 5 Euglycemia interchangeable with hyperglycemia

Explanation Co-administration of ACE inhibitors (captopril, enalapril) and oral sulfonylurea hypoglycemics results in an increase of the hypoglycemic effect, for which more careful monitoring of blood glucose is advised to avoid pronounced hypoglycemia. Co-administration of thiazide, but not loop diuretics, results in a decrease of hypoglycemic effect. There are no documented interactions of furosemide and sulfonylureas.

A 53-year-old man was recently diagnosed with a pituitary adenoma. He has been experiencing excessive thirst and large volume polyuria, as well as headaches and an increase in nocturia from once nightly to 3 to 4 times nightly. Question What medication would be the first-line treatment option given to him? Answer Choices 1 Hydrochlorothiazide 2 Indomethacin 3 Amiloride 4 Desmopressin acetate 5 Glucophage

Explanation Desmopressin acetate (DDAVP) is the correct answer, as this medication is used as the first-line treatment in patients with central or primary diabetes insipidus (DI). Arginine vasopressin (AVP), or antidiuretic hormone, is produced by the pituitary gland and works on the renal tubules within the kidney to concentrate the urine and reduce water loss. DI occurs when there is a deficiency in AVP, and these patients produce large amounts of dilute urine. As a result, patients can experience excessive thirst, polyuria, nocturia, headache, dehydration, and disturbances in vision. Central or primary DI can be diagnosed with 24-hour urine collection or vasopressin challenge test. Once the diagnosis is established, the first line treatment is usually desmopressin acetate, which is a synthetic form of AVP. It has a longer antidiuretic action than AVP and does not constrict smooth muscle. DDAVP is available in oral, nasal, and injectable preparations. While being treated, it is important to control fluid balance in order to prevent dehydration, monitor the patient's weight daily, and monitor electrolytes. Hydrochlorothiazide is not the correct answer, as this medication can be used in cases of nephrogenic DI rather than central DI and as a treatment for hypertension. Nephrogenic DI occurs in the presence of normal AVP production by the pituitary gland with insensitivity to AVP in the kidney. There is a disorder in the renal tubular function that results in the inability to respond to the presence of AVP. In these cases, hydrochlorothiazide can be given in a 25 mg dose either once or twice daily. Indomethacin is not the correct answer. Indomethacin is a non-steroidal anti-inflammatory that is typically used in the treatment of osteoarthritis, rheumatoid arthritis, or ankylosing spondylitis. However, it can also be used in the treatment of nephrogenic DI because indomethacin inhibits prostaglandins, which reduces urine output by decreasing the renal blood flow. This would not be effective in the treatment of central DI. Amiloride is not the correct answer. Amiloride is often added to HCTZ in order to increase the diuretic effects. However, this is a treatment course that is used in nephrogenic DI and not central DI. Glucophage is not the correct answer, as this medication is a biguanide used in the treatment of diabetes mellitus, not diabetes insipidus.

A 6-year-old boy presents due to lethargy, polyuria, nocturnal enuresis, and polydipsia. His mother tells you that he complains of being tired and thirsty all of the time. You note that he has lost 5 pounds since his last visit 6 months ago. Question What is the most likely diagnosis? Answer Choices 1 Diabetes mellitus type I 2 Diabetes mellitus type II 3 Leukemia 4 Lymphoma 5 Diabetes insipidus

Explanation Diabetes mellitus type I can occur at any age, but it most commonly arises in children and young adults; peak incidences are before school age and during puberty. Patients often present with polyuria, polydipsia, weakness, fatigue, thirst, blurred vision, and/or nocturnal enuresis. Type II diabetes and diabetes Insipidus are much less common in children. Leukemia and lymphoma can both have symptoms of fatigue (leukemia more so than lymphoma), but patients do not experience polyuria and/or polydipsia.

A 44-year-old obese woman presents with increased nighttime urination. She has never had issues with having an increased urge to urinate nocturnally before, and it is extremely bothersome to her. She states that she has been waking up at least 3 times a night despite lifestyle modifications designed to help reduce this number. The patient admits to increased fatigue, worsening blurry vision, and 2 vaginal yeast infections in a span of 3 months. Question What is the most likely diagnosis? Answer Choices 1 Hyperthyroidism 2 Hypothyroidism 3 Diabetes type I 4 Diabetes type II 5 Cushing's syndrome

Explanation Diabetes type II is the correct answer for the clinical picture presented. This pathologic state consists of beta cell loss, but it also has other key characteristics that lead to its development; patients develop increased tissue insensitivity, also termed insulin resistance. This insensitivity leads to hyperactivity of the beta cells, producing excessive amounts of insulin, and over time the developed hyperplasia of the beta cells leads to impaired or even impeded function. Signs and symptoms that result from diabetes type II include but are not limited to: polyuria, polydipsia, chronic skin infections, or other indications of insulin resistance. Patients may be asymptomatic at the time of diagnosis. Diabetes should be high on one's differential diagnosis when a patient presents with frequent/chronic vulvovaginitis, which is part of the patient's complaint described above. Diabetes type I is the most common type of diabetes mellitus in patients under the age of 20. Presenting symptoms can include polydipsia, polyuria, significant weight loss, and even a dangerous state of hypergylcemia that is referred to as diabetic ketoacidosis. Over 95% of type I diabetics develop this through autoimmune activity on the pancreatic islet cells; the remaining ~5% is idiopathic. Hypothyroidism may from failure of the thyroid gland itself in secreting efficient levels of the main thyroid hormone free thyroxine, or FT4. Hypothyroidism may also occur because of deficiency of the secretion of thyroid stimulating hormone (TSH), which is secreted by the pituitary gland. Signs and symptoms highly suggestive of this disease state are weight gain, fatigue, lethargy, depression, weakness, cold intolerance, muscle cramps, and constipation. Simply put, hyperthyroidism is a patient suffering from excessive serum levels of T4 and T3, while TSH levels are abnormally low. Signs and symptoms of the disease state include excessive sweating, weight loss, anxiety, palpitation, heat intolerance, palpitations, fatigue, and weakness. Cushing's syndrome, which is also termed hypercortisolism, refers to the manifestations of excessive corticosteroids on the patient. Many of these cases are due to excessive ACTH secretion by a benign pituitary adenoma; however, other causes of excessive corticosteroids may lead to this condition, such as other non-pituitary secreting ACTH neoplasms as well as excessive secretion of t cortisol from the adrenal glands. In up to 30% of cases, the main cause may not be found. There are a multitude of symptoms and signs seen with Cushing's syndrome; they can include a moon facies, buffalo hump, abdominal purple striae, central obesity, menstrual irregularities, and even impaired wound healing.

A 60-year-old man presents with difficulty climbing stairs, dyspnea, and fatigue. He has gained 30 lbs over the past 1 year. On examination, he is found to have edema, pigmentation of the skin, palmar creases, and proximal muscle weakness. Chest X-ray shows an irregular mass in the right upper lobe. Lab values show an increase in evening cortisol levels and an increase in ACTH. Question What should be the next best step in diagnosis? Answer Choices 1 MRI of the brain 2 Adrenal venous sampling 3 Serum ACTH levels 4 High dose dexamethasone suppression test 5 ACTH stimulation test

Explanation High-dose dexamethasone suppression test is the correct answer. Lab values are indicative of raised cortisol and an increased ACTH level. The next step in diagnosis should be to differentiate between a pituitary and peripheral cause of high ACTH. A high-dose dexamethasone suppression test is used to differentiate between Cushing's syndrome caused by pituitary adenoma versus ectopic ACTH production by a lung mass. ACTH of pituitary origin is suppressed by high doses of dexamethasone by negative feedback; ACTH of peripheral origin is not. Adrenal venous sampling is incorrect; however, it is of great utility in patients with primary hyperaldosteronism. MRI of the brain is incorrect. An MRI is appropriate after an endocrine diagnosis is made. If done prior to this, an unacceptably high false positive rate ensues. Serum ACTH level is incorrect. It is already given in the question stem. ACTH stimulation test is incorrect. It is used in adrenal insufficiency, not hyperadrenalism.

A 59-year-old woman presents for a routine annual physical. She is a housewife who does not smoke or drink, and her only significant past medical history is for an appendectomy and radiation treatment for acne as a child. She is post-menopausal, but she uses natural supplements rather than hormone replacement. She is not on any prescription medications. She completed some routine lab work prior to the office visit. The results are entirely within normal limits, including cholesterol panel, glucose, TSH, and T4. Her only complaints are recent dysphagia and sensation of fullness in her neck. Physical examination reveals a large nodule right lobe of patient's thyroid. The patient is referred to an endocrinologist and undergoes an ultrasound of the thyroid, which is followed by a thyroid scan. The results confirm a 1.7cm solid nodule on ultrasound that is deemed 'cold' on thyroid scan. Question Prior to scheduling a fine needle aspiration and based on the information you have, what is the the most likely diagnosis? Answer Choices 1 Hashimoto's thyroiditis 2 Medullary thyroid carcinoma 3 Parathyroid cyst 4 Graves' disease 5 Papillary thyroid carcinoma

Explanation The correct answer is papillary thyroid carcinoma. The patient has 3 risk factors for thyroid cancer, including female gender, age over 40, and history of radiation to face/neck region for acne. Thyroid function tests are typically normal in patients with thyroid cancers. The diagnostic findings of a solid, cold nodule are also suspicious for a thyroid cancer. However, those findings can be seen in benign nodules as well. Given the fact that papillary thyroid carcinoma is the most common type of thyroid cancer (about 60% of all thyroid cancers) and the patient has a history of radiation to face/neck region, the most likely type of thyroid cancer in this case is papillary. Hashimoto's thyroiditis is not the correct answer. While thyroid nodules can be present in cases of Hashimoto's thyroiditis, they are usually "multinodular" rather than a solitary thyroid nodule. Patients would also usually have more complaints upon presentation. In addition, the physical examination is likely to be normal or reveal a goiter with or without multiple nodules. The thyroid function testing would not be normal. Most commonly, the TSH would be elevated in the face of decreased T4. Medullary thyroid carcinoma is not the correct answer, as this is not the 'most likely' diagnosis in this case. The patient is at risk for thyroid cancer, but her history of radiation to face/neck region and the fact that papillary carcinoma is much more common (60% as opposed to 5% of all thyroid cancers) make her type of cancer more likely to be papillary rather than medullary. Parathyroid cyst is not the correct answer. A parathyroid cyst falls under the category of a benign cyst of the thyroid. Thyroid cysts make up only 15 - 25% of thyroid nodules, and benign cysts make up 85% of that group. A parathyroid cyst is 1 of the many types of benign thyroid cysts, and it would not be solid on an ultrasound. Graves' disease is not the correct answer. Patients will typically have more complaints and positive symptoms when they have Graves' disease. In addition, the examination of their thyroid is not likely to have a solitary thyroid nodule. The thyroid function tests would not be normal and are most likely to show a decreased TSH and elevated T4.

A patient presents for routine analysis of cholesterol levels. The results show plasma cholesterol levels of 300-mg/100 ml. You prescribe the drug simvastatin (Zocor). She is reluctant to take drugs to treat her hypercholesterolemia. After further discussion, she agrees to take a vitamin to treat the elevated cholesterol. She also has questions concerning familial hypercholesterolemia. Question Familial hypercholesterolemia is due to what defect? Answer Choices 1 Cholesterol biosynthesis 2 Low density lipoprotein receptors 3 Cholesterol degradation 4 Lysosomal function 5 Cholesterol esterification

Explanation In familial hypercholesterolemia, which is a genetic disorder, the plasma levels of cholesterol and LDL are elevated. In this disease, cholesterol is deposited in many tissues because of the concentrations of cholesterol-LDL in the plasma. The molecular defect results in the absence or deficiency of functional LDL receptors. Therefore, the entry of LDL into the liver and other cells is impaired and all of the deleterious effects of the disease can be attributed to the elevated LDL-cholesterol levels. High levels of cholesterol have 3 major metabolic effects. Cholesterol inhibits the activity of the enzyme β-Hydroxy-β-methyl-glutaryl-CoA (HMG-CoA) reductase, which catalyzes the rate-limiting step of cholesterol biosynthesis. Cholesterol also inhibits the production of low-density lipoprotein (LDL) receptors by suppressing the transcription of the LDL receptor mRNA. The third point of regulation involves the activation of the enzyme acyl-CoA: cholesterol acyltransferase (ACAT), which esterifies cholesterol for storage. The LDL receptors are located in specialized regions of the membrane called coated pits. These coated pits contain a protein called clathrin. Clathrin participates in the endocytosis of LDL receptors, as well as other proteins such as transferrin and sialoglycoproteins. Clathrin forms a closed polyhedral lattice around the coated pit forming a coated vesicle. The coated vesicle then loses its clathrin and fuses with an endosome. In the endosome, the ligand is dissociated from the receptor, and sorting of the different components can occur.

A 63-year-old diabetic woman is seen by her primary care physician. Despite having been an insulin dependent diabetic for over 20 years, her blood sugars remain poorly controlled. No amount of counselling seems to have made a difference to her attitude. At this visit, she complains of left-sided ear pain. She noticed it the previous night; she was unable to sleep with her head resting on the affected side. Since then, the pain has rapidly increased, and she feels feverish. The photograph shows the affected area at the time of the visit. Question What is the best next step in treatment of her condition? Answer Choices 1 Aggressive surgical debridement 2 High-dose crystalline penicillin 3 Intravenous clindamycin 4 Intravenous amikacin 5 Intravenous ceftazidime

Explanation Intravenous ceftazidime is the correct answer. The most common causative organism of malignant otitis externa is the Pseudomonas species. It has a rapid clinical course in diabetics that can be fatal if left untreated. Therapy involves the use of high doses of anti-pseudomonal drugs, such as Ceftazidime. Anti-pseudomonal penicillins, such as Piperacillin and Ticarcillin, are other very effective options. In earlier years, quinolones were used widely, but their routine use for upper respiratory infections has increased rates of Pseudomonal resistance. Surgical debridement is incorrect. With malignant otitis externa, infection spreads rapidly via fascial and vascular planes, and it requires radical debridement of the ear cartilage and temporal bone to prevent its growth; therefore, surgery is restricted to localized slough removal and the removal of bony sequestrum if needed. High-dose crystalline penicillin is incorrect. Pseudomonas is almost completely resistant to penicillin now. Intravenous clindamycin is incorrect. Clindamycin is ineffective against Pseudomonas. It is commonly used in anaerobic infections, diabetic ulcers, MRSA infections, and cases of acne. Intravenous amikacin is incorrect. As an aminoglycoside, it is used in Gram-negative sepsis.

A 38-year-old man presents with fatigue, dry mouth, and passing large amount of urine. He describes his urine as light in color and non-odorous. He says he has never had this before. He has always been healthy, and has never been hospitalized. He had serum electrolytes investigated during a routine pre-employment screening, and his sodium was slightly above normal; however, at that time, he was reassured that it was due to slight dehydration. He was advised to repeat the test after 3 months. When he did about a week ago, it showed more of an increase, and he was also worried about that. He takes vitamin supplements and sometimes uses energy drinks. To avoid going to the bathroom too often, he tries to restrict fluid intake to minimum, but it has never helped. Blood glucose level was measured, and it was normal. DDAVP (desmopressin) injection led to decrease in urine volume and increase in urine specific gravity. Question What is the best treatment? Answer Choices 1 Adequate fluid intake 2 Fluid restriction 3 Diet and exercise 4 Nasal desmopressin 5 Hydrochlorothiazide

Explanation Nasal desmopressin is used to treat central diabetes insipidus. Diabetes insipidus is a disease in which ADH (antidiuretic hormone) from the posterior pituitary is either deficient or not working. ADH is secreted from posterior pituitary and works on distal renal tubules on specific receptors to reabsorb water and this occurs in response to osmoreceptors to help keep the body in balance. Symptoms include polyuria, polydipsia, and decreased urine specific gravity. Causes: Central due to tumor anoxia or hemorrhage. Nephrogenic due to the effect of central drugs of which lithium is well known reason. Differential diagnosis: Diabetes mellitus Psychogenic polydipsia Workup: First exclude diabetes mellitus by random blood sugar then exclude psychogenic polydipsia by water deprivation test (here the patient did it himself!). Then do desmopressin test to differentiate Nephrogenic from central DI. If it is positive do MRI to exclude brain tumors. Treatment: In mild cases, adequate hydration is enough. For Central DI, give desmopressin. In psychogenic polydipsia, psychiatric consultation is needed. If drug induced, stop offending drug. Hydrochlorothiazide combined with indomethacin can be used in Nephrogenic DI as treatment.

An overweight woman presents with a fullness in her neck. TSH, T 3, and T 4 are normal, but the sonogram shows multiple masses on both sides of her thyroid. A fine needle biopsy is done, and it reveals a malignant tumor. What is the most common thyroid malignancy? Answer Choices 1 Follicular adenoma 2 Colloid goiter 3 Polyendocrine adenomatosis 4 Hashimoto's disease 5 Papillary carcinoma 6 Follicular carcinoma 7 Sipple's syndrome 8 Zollinger-Ellison syndrome

Explanation Papillary carcinoma is the most common thyroid gland malignancy. The incidence is greatest among adults 40-years of age or less. Females are more likely to develop this type of thyroid malignancy. It is the least malignant cancer. Polyendocrine adenomatosis is a rare syndrome where adenomas of the endocrine tissue are found in multiple sites. The most common are anterior pituitary gland, islets of Langerhans, and the parathyroid. This can be called the Zollinger-Ellison syndrome. Follicular adenomas are adenomas of the thyroid where the cells are arranged in the form of follicles. This benign epithelial tumor is diagnosed with a biopsy. The colloid goiter is a large soft goiter in which follicles of the gland are distended. Sipple's syndrome is the bilateral medullary carcinoma of the thyroid that is part of an autosomal dominant problem seen with Marfan's syndrome. This is also associated with pheochromocytoma and there is a tendency towards hyperparathyroidism due to adenomas or hyperplasia. Hashimoto's disease, named after a Japanese surgeon, is thyroiditis.

A 58-year-old man presents to discuss his recent annual screening blood work results. His past medical history includes obesity and hypertension. He wants to discuss whether he has additional risk factors for Coronary Heart Disease (CHD). His Total Cholesterol is 240mg/dL, his Low-Density Lipoprotein Cholesterol (LDL) is 191mg/dL, his High-Density Lipoprotein Cholesterol (HDL) is 32mg/dL, and his Triglycerides are 140mg/dL. Question What is the most likely diagnosis? Answer Choices 1 Hypertriglyceridemia with high risk for CHD 2 Normal cholesterol profile with low risk for CHD 3 Hypercholesterolemia with low risk for CHD 4 Hypercholesterolemia with high risk for CHD 5 Hypertriglyceridemia with low risk for CHD

Explanation The patient would be diagnosed with hypercholesterolemia with high risk for Coronary Heart Disease (CHD) for several reasons. His total cholesterol is considered elevated, as it is more than 200mg/dL, and his Low-Density Lipoprotein Cholesterol (LDL) is also elevated, as it is more than 160mg/dL. In addition, his High-Density Lipoprotein Cholesterol (HDL) is less than 35mg/dL, which increases his risk for CHD. In order to be diagnosed with hypercholesterolemia with low risk for CHD, the patient would essentially have to have total cholesterol and LDL levels that are elevated, but not to the extent that his are. In particular, an LDL level above 100mg/dL, but still under 160mg/dL, is considered high, but does not have the same risk as a level that is over 160mg/dL, as his is. In addition, he could have lower risk for CHD had his HDL level been over 60mg/dL. The patient would not be diagnosed with hypertriglyceridemia, with or without risk for CHD, because his triglycerides are actually within normal limits. Triglyceride levels under 200mg/dL are considered desirable. Levels between 200 and 400mg/dL are considered borderline elevated, and levels over 400mg/dL are considered high. The patient would definitely not be diagnosed with a normal cholesterol profile. A normal cholesterol profile would include a total cholesterol under 200mg/dL, an HDL level over 60mg/dL, an LDL level under 130mg/dL, and a triglyceride level under 200mg/dL. Of the 4 serum levels, he only has a normal triglyceride level. The rest are abnormal and thus increase his risk for CHD.

A 25-year old-woman is admitted to the hospital due to a 3-week history of episodic headaches; they are accompanied by palpitations and sweating. The episodes occur several times daily; each episode lasts about 15 minutes. On examination, the patient is diaphoretic and has elevated blood pressure, which prompted her admission. Her blood pressure is currently 220/100 mm Hg. Past medical history is significant for exercise-induced asthma, for which she takes albuterol. Question What test would be useful in the diagnosis of this patient? Answer Choices 1 Cosyntropin stimulation test 2 Dexamethasone suppression test 3 Plasma fractionated metanephrine 4 Electrocardiogram 5 Echocardiogram

Explanation Plasma fractionated metanephrine is the correct response. The patient presented with episodic headache, palpitations, and diaphoresis that was accompanied by hypertension; these symptoms are suggestive of pheochromocytoma. Pheochromocytomas are tumors that originate from the adrenal glands and secrete norepinephrine. This secretion causes hypertension and symptoms, such as headaches, palpitations and diaphoresis. Patients with pheochromocytoma have elevated plasma metanephrine. Cosyntropin stimulation test is an incorrect response. The cosyntropin stimulation test is used in the diagnosis of Addison's disease. Dexamethasone suppression test is an incorrect response. The dexamethasone suppression test is used in the diagnosis of Cushing syndrome. While patients with Cushing syndrome may have hypertension, it typically does not occur along with episodic headaches, palpitations and diaphoresis. Electrocardiogram is an incorrect response. EKG findings are not diagnostic for pheochromocytoma. Echocardiogram is an incorrect response. Echocardiogram is not used in the diagnosis of pheochromocytoma.

A 58-year-old man presents with severe headache, gradual vision loss, weakness, and enlargement of his hands. He has noticed that his wedding ring no longer fits on his ring finger and that his feet feel "too tight" in his current sneakers. He said his voice seems to be deeper, and his skin feels doughy and sweaty. Laboratory testing reveals an elevated insulin-like growth factor (IGF)-I, and there is a failure of growth hormone (GH) suppression following an oral glucose load. His physical exam reveales soft-tissue swelling resulting in bilateral hand and foot enlargement, macroglossia, bitemporal hemianopsia, hyperhidrosis, oily skin, kyphosis, and proximal muscle weakness. His facial appearance is shown in the image. A laboratory work-up demonstrates an elevated insulin-like growth factor (IGF)-I and failure of growth hormone suppression following an oral glucose load. Question What health maintenance recommendation should be provided to this patient? Answer Choices 1 Following diagnosis, performing GH and IGF-I levels is unnecessary 2 Noncompliance to therapy has no effect on survival 3 Routine colonoscopies should be scheduled for this patient 4 A high salt and carbohydrate diet improves overall prognosis 5 Periodic bone density scans are required to identify demineralized bone

Explanation Routine colonoscopies should be scheduled for this patient. This patient's most likely diagnosis is acromegaly. Acromegaly is nearly always caused by a pituitary adenoma. Patients should be advised that, if untreated, one's life span is decreased by an average of 10 years. Patients must receive lifelong follow-up, with regular monitoring of serum GH and IGF-I levels. Serum GH levels over 5 ng/mL and rising IGF-I levels usually indicate a recurrent tumor. Both IGF-I and GH levels correlate with mortality; survival improves greatly if GH and IGF-I can be normalized. Patients must receive lifelong follow-up, with regular monitoring of serum GH and IGF-I levels. Both an initial colonoscopy and echocardiogram are recommended. Colon polyps and mortality from colonic malignancy are more likely to develop in patients with acromegaly. As such, increased surveillance for colorectal cancer is recommended. Based on a cohort study of patients hospitalized for acromegaly (Denmark 1977-1993; Sweden 1965-1993) linked to tumor registry data for up to 28 years of follow-up, individuals with acromegaly have higher rates of small intestine, colon, rectal, kidney, and bone cancer. Patients with acromegaly have increased morbidity and mortality from cardiovascular disorders and progressive acromegalic symptoms. Complications of acromegaly include hypopituitarism, hypertension, glucose intolerance or frank diabetes mellitus, cardiac enlargement, and cardiac failure. Strict control of comorbidities, including diabetes and hypertension, are essential. This patient is not at risk for demineralized bone; a bone density (DEXA) scan is not recommended in patients with acromegaly.

Growth of the epiphyseal cartilage during endochondral bone formation is stimulated by a hormone; if over-secreted, it may result in an individual in which there is an abnormal increase in the length of bones and their density. What is the hormone? Answer Choices 1 Calcitonin 2 Parathormone 3 Corticotrophin 4 Testosterone 5 Somatotrophin

Explanation Somatotropin, or growth hormone (GH), is secreted from the pituitary gland; it stimulates growth in general, but it especially stimulates growth of the epiphyseal cartilage during childhood. Over-secretion of GH during childhood will result in an abnormal increase in the length and density of long bones, causing a clinical condition referred to as gigantism. Calcitonin and parathormone also have major effects on bones; they do so by either depressing or stimulating the deposition of calcium in the matrix of bone, respectively.

A 40-year-old woman presents with swelling in her neck, which has gradually increased in size in the past year. On physical examination, her thyroid gland is symmetrically and diffusely enlarged and non-tender. Her thyroid function tests shows: Serum T4 3.5 micrograms dl Serum T3 30 ng/dl Serum TSH 12 mU/L Radioiodine uptake is decreased. The patient was subjected to thyroidectomy. On microscopic examination, it shows extensive infiltration of the parenchyma by a mononuclear infiltrate containing small lymphocytes, plasma cells, and well-developed germinal centers. The thyroid follicles are small; in many areas, they are lined by Hurthle cells. What is the most likely diagnosis? Answer Choices 1 Hashimoto's thyroiditis 2 Juvenile lymphocytic thyroiditis 3 Atrophic thyroiditis 4 Painless thyroiditis 5 Graves' disease

Explanation Subacute lymphocytic thyroiditis, or painless thyroiditis, is an uncommon cause of hyperthyroidism. It is considered a variant of Hashimoto's thyroiditis, as this disorder is also characterized by the presence of autoantibodies to thyroid globulin and thyroid peroxidase. The symptoms develop over a period of 1 to 2 weeks and last from 2 - 8 weeks before subsiding. Infiltrative ophthalmopathy and other features of Graves' disease are absent. In addition, even though the serum T3 and T4 levels are increased and TSH levels are decreased during the episode of hyperthyroidism, the radioiodine uptake is decreased. Graves' disease, also known as diffuse toxic goiter or Basedow's disease, is characterized by hyperthyroidism, diffuse thyroid enlargement, and infiltrative ophthalmopathy with resultant exophthalmos and infiltrative dermopathy. It is an autoimmune disease and shows elevated levels of T3 and T4 and depressed levels of TSH. However, the radioiodine uptake is increased, and the thyroid scan shows a diffuse uptake of iodine. Refer to the table for additional information. Hashimoto's thyroiditis (also known as autoimmune thyroiditis, diffuse lymphocytic thyroiditis, struma lymphomatosa, or goitrous autoimmune thyroiditis) is characterized by 3 main features: Diffuse goitrous enlargement of the thyroid Lymphocytic infiltration of the thyroid gland Occurrence of thyroid autoantibodies Juvenile lymphocytic thyroiditis, a variant of Hashimoto's thyroiditis, is also known as lymphocytic thyroiditis of childhood and adolescence. It occurs mainly in children and young females and is characterized by a mild enlargement of the thyroid and a lower level of autoantibodies compared to Hashimoto's thyroiditis. The patient is mildly hypothyroid or euthyroid and is symptomless. Atrophic thyroiditis is also a variant of Hashimoto's thyroiditis, and it is also known as idiopathic myxedema or spontaneous hypothyroidism. Instead of enlargement, there is diminution in size of the thyroid, and it is characterized by the presence of autoantibodies.

A 33-year-old man presents for his biannual checkup. He was diagnosed with Type I diabetes at the age of 4 and has been on insulin since then. 2 years ago, he was switched to a continuous subcutaneous insulin infusion system. He exercises regularly, controls his diet, and has had his sugar well controlled for the past few months, checking regularly with home monitoring. His BP is 120/75, pulse 70/min, no JVD, and cardiovascular and respiratory exam are normal. In addition to eye and foot exams, the physician wants to assess the patient's renal function. Question What is the best screening test? Answer Choices 1 Serum creatinine 2 Blood urea level 3 Urine specific gravity 4 24-hour urine protein 5 Urine dipstick for protein

Explanation Testing the urine with a dipstick for microalbumin is the most appropriate screening test for diabetic nephropathy. Results of 2 or 3 tests for microalbuminuria should be more than 30 mg per day or 6-month period to diagnose a patient with diabetic nephropathy. An early morning sample is preferred, although not strictly required. The dipstick measures a spot albumin to creatinine ration in the urine, which is a reliable indicator of albuminuria. Screening for diabetes is recommended at the time of diagnosis with Type II diabetes and 5 years after diagnosis of Type I diabetes. Some authorities recommend earlier screening in individuals with Type I diabetes with poor metabolic control. Without any intervention, over 80% of patients with microalbuminura will develop overt nephropathy within 10 - 15 years. Of this population, roughly 20% will go on to develop renal failure. Diabetes has become the most common single cause of end-stage renal disease (ESRD) in the U.S. and Europe. This is attributed to the following: 1) the increasing prevalence of diabetes; 2) increased acceptance of diabetic patients to dialysis programs that were earlier excluded; and 3) increase longevity of diabetic patients. As per the findings of the Diabetes Control and Complications Trial Research Group, tight blood sugar and blood pressure control are the most important variables to control in order to prevent nephropathy. An HbA1c of <7% and BP of <135/80 mmHg are the standard of care. ACE inhibitors are the drug of choice for hypertension, with non-dihydropiridine calcium channel blockers as the alternative if ACE is contraindicated. At an early stage, the use of ACE inhibitors has been shown unequivocally to delay the progression of nephropathy. Creatinine, blood urea, and urine specific gravity are incorrect. While they are indicators of renal function, none of them are as sensitive or cost effective as measuring a spot urine albumin to creatinine ratio; however, they are measured in order to monitor renal function and assess the side effects of medications, particularly ACE inhibitors and oral hypoglycemic drugs. A 24-hour urine protein is incorrect. It is an excellent test with high sensitivity. However, it requires urine to be collected over 24 hours, which is cumbersome; therefore, it is inappropriate as an initial screening test. It is sometimes used in patients who are dipstick positive in order to quantify the excreted protein.

There are three major carrier proteins for T3 and T4. More than 99% of these transport globulins are responsible for binding and transporting T3 and T4. What transport binding is responsible for the most transportation of T3 and T4? Answer Choices 1 Immunoglobulin A 2 Immunoglobulin B 3 Albumin 4 Thyroxine-binding prealbumin (TBPA) 5 Thyroxine binding globulin (TBG)

Explanation The 3 major binding globulins are albumin, thyroxine-binding prealbumin (TBPA) and thyroxine binding globulin (TBG). Albumin binds approximately 15 to 20% of T4. TBPA binds approximately 10% and is responsible for immediate delivery of T4 and T3 to the cell. TBG is the binding globulin that binds most of T3 and T4 (approximately 70%).

An otherwise healthy patient presents with a small lesion on the lips that, on biopsy, proves to be a mucosal neuroma. The patient's mother had medullary carcinoma of the thyroid. Question In addition to medullary carcinoma of the thyroid, to what condition would this patient be particularly vulnerable? Answer Choices 1 Gastrinoma 2 Insulinoma 3 Parathyroid adenoma 4 Pheochromocytoma 5 Pituitary adenoma

Explanation This is a probable case of multiple endocrine neoplasia, specifically MEN III. Features of this autosomal dominant condition include medullary carcinoma of the thyroid, pheochromocytoma, and oral and intestinal ganglioneuromatosis (including mucosal neuromas). Gastrinomas and insulinomas are found in MEN I. Parathyroid adenomas are found in MEN I and II. Pituitary adenomas are found in MEN I.

Classic symptoms of polyuria, polydipsia, and weight loss in a pediatric patient is likely due to autoimmune damage of the insulin-producing β-cells of the pancreatic islets. Symptoms usually appear gradually, and they occur when at least 80% of the islets have been damaged. Question What is the most likely diagnosis of this pathological process? Answer Choices 1 Acute pancreatitis 2 Hypothyroidism 3 Diabetes type I 4 Diabetes type II 5 Cushing's syndrome

Explanation The above summary is depicting diabetes type I, which is the most common type of diabetes mellitus in patients under the age of 20. Presenting symptoms can include polydipsia, polyuria, significant weight loss, or even a dangerous state of hyperglycemia known as diabetic ketoacidosis. Over 95% of type I diabetics develop diabetic ketoacidosis through autoimmune activity on the pancreatic islet cells; in the remaining ~5% of cases, it is idiopathic. Acute pancreatitis (acute inflammation of the pancreas) does not usually lead to permanent destruction of the pancreatic islet cells. Patients who develop acute pancreatitis are adults with significant gallbladder disease and/or a history of heavy alcohol use. Hypothyroidism may be the result of the thyroid gland's failure to secret efficient levels of the main thyroid hormone free thyroxine, or FT4. Hypothyroidism may also occur because of a deficiency of the secretion of thyroid stimulating hormone (TSH), secreted by the pituitary gland. Although hypothyroidism can virtually affect almost every body system in terms of signs and symptoms, it does not lead to autoimmune damage of the pancreatic islets. Diabetes type II does consist of beta cell loss, but there are also other key characteristics that lead to this disease state. Patients develop increased tissue insensitivity, which is also termed insulin resistance. This insensitivity lead to hyperactivity of the beta cells, producing excessive amounts of insulin; over time, the developed hyperplasia of the beta cells leads to impaired or even impeded function. The majority of the time, this is not due to autoimmune issues. Cushing's syndrome (or hypercortisolism) refers to the manifestations of excessive corticosteroids on the patient. Many of these cases are due to excessive ACTH secretion by a benign pituitary adenoma; however, other causes of excessive corticosteroids may lead to this condition, such as other non-pituitary secreting ACTH neoplasms and excessive secretion of t cortisol from the adrenal glands; in up to 30% of these cases, the main cause may not be found. There are a multitude of symptoms and signs seen with Cushing's syndrome, and glucose tolerance impairment secondary to insulin resistance is one of them.

A 38-year-old man presents with a 15-lbs weight loss over the past couple of months; the weight loss is associated with poor appetite, nausea, fatigue, and irritability. The patient has been training for a triathlon, but he did not experience such pronounced weight loss during previous training regimens. Physical exam reveals a pulse of 60 beats/min, and a BP of 76/58 mm Hg. Abdomen and rectal exams reveal no abnormal findings. A skin exam reveals a dark tan over this body, including his genital region. Question What is the most likely diagnosis? Answer Choices 1 Melanoma 2 Excessive physical training 3 Cushing syndrome 4 Addison's disease 5 Growth hormone deficiency

Explanation The correct answer is Addison's disease. Addison's disease is typically characterized by hypotension, hyperpigmentation, weight loss, anorexia, fatigue, nausea, arthralgias, myalgias, and irritability. It is due to an insufficiency of adrenocortical hormones. Melanoma is a skin cancer that is characterized by a hyperpigmented, irregularly colored and bordered lesion, which is not mentioned in the above patient. Cushing syndrome is due to an excess of endogenous corticosteroid production; it is more common in women, and it includes signs of central obesity, muscle wasting, and thin skin. Excessive physical training is not likely, as it would not cause significant hypotension or tan skin in areas not typically exposed to the sun such as the genital region. Growth hormone deficiency can cause muscle wasting, increased fat mass, fatigue, memory problems, and even depression, but it is not associated with hypotension or hyperpigmented skin.

A 47-year-old man presents for his annual physical exam. His past medical history is not significant, and he is not currently on any medications. He consumes 2 beers weekly and does not smoke. His blood pressure is normal at this visit. His primary care physician orders a fasting lipid panel with the following results: Component Value Triglyceride 135 HDL 50 LDL 220 Question In addition to diet and exercise, what medication should be started on this patient? Answer Choices 1 Cholestyramine 2 Fenofibrate 3 Nicotinic acid 4 Simvastatin 5 Ezetimibe

Explanation The correct answer is Simvastatin. This patient has elevated LDL, decreased HDL, and a normal triglyceride level. Simvastatin, a HMG CoA reductase inhibitor, will decrease LDL and increase HDL levels. The patient has no history of liver disease, which is the main contraindication to using HMG CoA reductase inhibitors. Cholestyramine isa bile acid sequestrant that may be used in combination with HMG CoA reductase inhibitors; however, is not recommended for monotherapy in patients with elevated LDL levels. Fenofibrate, a fibric acid, is primarily used for the treatment of hypertriglyceridemia. The above patient has a normal triglyceride level. Nicotinic acid (niacin) may be used in combination with HMG CoA reductase inhibitors, but it is not typically used in monotherapy for elevated LDL levels. Ezetimibe acts by blocking cholesterol absorption in the intestines. It is currently recommended for those who fail to respond to HMG CoA reductase inhibitor therapy. This patient has newly diagnosed hyperlipidemia; therefore, ezetimibe would not be an appropriate first line treatment.

A 38-year-old woman presents for a consult regarding her thyroid disease. She was recently diagnosed with Graves' disease, and she has not started any treatment yet. Question What finding might you see that is considered severe and is present in about 5-10% of patients? Answer Choices 1 Exophthalmos 2 Conjunctivitis 3 Chemosis 4 Periorbital edema 5 Corneal abrasion

Explanation The correct answer is exophthalmos, which is a bulging of the eye anteriorly out of the orbit. This is considered a severe form of ophthalmopathy and can be present in about 5-10% of patients who have Graves' disease. When patients are treated with high-dose, tapered prednisone, it is possible to see reversal of this finding. However, the treatment works best in patients who are non-smokers. Conjunctivitis is not the correct answer; conjunctivitis is a form of mild ophthalmopathy, and it can be present in 20-40% of patients with Graves' disease. Conjunctivitis is an inflammation of the conjunctiva secondary to a bacterial or viral infection. Chemosis is not the correct answer; chemosis is a form of mild ophthalmopathy that can be present in 20-40% of patients with Graves' disease. It is a term that refers to edema of the outer surface of the eye, and it can be related to allergies or an infection, such as conjunctivitis. Periorbital edema is not the correct answer; it is an ophthalmologic finding that is more commonly found in hypothyroidism, not hyperthyroidism. Corneal abrasion is not the correct answer. It is not associated with thyroid disease. It is more closely associated with some form of trauma or foreign body in the eye.

A 33-year-old woman presents with weight loss in spite of a hearty appetite. At first glance, the physician notices her large eyes. She gives a history of increased anxiety over the last few months; it has been associated with palpitations and diaphoresis. During the examination, the physician notes tachycardia, hypertension, sweaty palms, and trembling outstretched hands. There is a diffuse swelling in the neck with a bruit heard on auscultation. What is the most likely diagnosis? Answer Choices 1 Hashimoto's thyroiditis 2 Hypothyroidism 3 Parathyroid adenoma 4 Lateral aberrant thyroid 5 Graves' disease 6 Hyperthyroidism

Graves' Explanation Graves' disease is an idiopathic autoimmune disorder associated with HLA-B8 and HLA-DR3. It is characterized by hyperthyroidism and a diffuse swelling of the thyroid (which moves with deglutition); hence, it also called diffuse toxic goiter. Hashimoto's thyroiditis and hypothyroidism are associated with lowered levels of thyroxine and its related symptoms, whereas this patient is showing signs of hyperthyroidism. The features that distinguish graves disease from other causes of hyperthyroidism are given below. Lateral aberrant thyroid is a unique presentation of carcinoma thyroid wherein the lymph node metastasis is detected before the primary. The most common clinical features of hyperthyroidism are: GIT: Wt. Loss Fatigue Anorexia Cardiorespiratory: Dyspnea on exertion Palpitations CNS: Emotional lability Tumor Hyperreflexia Dermatological: Palmar erythema Alopecia Increased sweating Reproductive: Amenorrhea / oligomenorrhea Infertility Impotence Decreased libido Other: Heat intolerance Hyper defecation Apathy Clinical features specific to Graves' Disease: Ocular: Lid lag Excess lacrimation Chemosis Exophthalmos Corneal ulceration Diplopia and ophthalmoplegia Papilledema Decrease visual acuity Dermatological: Vitiligo Clubbing Pretibial myxedema Other: Lymphadenopathy Diffuse goitre with bruit

A 35-year-old man with type I diabetes presents for an acute visit with uncontrolled blood sugars with high and low readings throughout the day and night. When he called to make this appointment, he was advised to bring in his blood sugar log and check a few 3:00 am blood sugars. He is currently on 40 units of NPH and 20 units of regular insulin before breakfast, and 20 units of NPH as well as 10 units of regular insulin before dinner. Time 7am 11 am 5 pm 11 pm 3 am Average blood glucose mg/dL 393 210 175 140 50 Question What is the most appropriate insulin change based on his average blood glucose readings? Answer Choices 1 Add 4 units of regular insulin before breakfast 2 Add 2 units of NPH insulin before breakfast 3 Add 2 units of NPH insulin before dinner 4 Eliminate 4 units of NPH insulin before dinner 5 Eliminate 2 units of regular insulin before dinner

Explanation The correct answer is to eliminate 4 units of NPH insulin before dinner. The first step in managing insulin therapy is to eliminate low blood sugars. This patient is experiencing nocturnal hypoglycemia, which stimulates the release of counter-regulatory hormones to increase the blood sugar, thereby resulting in rebound hyperglycemia. This would account for his elevated fasting blood sugar readings. The NPH insulin has an onset of action of 1 - 2 hours and a peak effect in 6 - 12 hours, so decreasing the dinner dose 10 - 20% will decrease the likelihood of 3:00 am hypoglycemia and the subsequent morning rebound hyperglycemia. Eliminating 2 units of regular insulin before dinner may have some effect on the patient's 3:00 am hypoglycemia, but at the expense of higher 11:00 pm blood sugar readings. The regular insulin has an onset of action in 30-60 minutes, with a peak effect at 2-4 hours. The current dinner dose of 10 units of regular insulin is providing rapid-acting coverage for that meal with an acceptable average blood sugar of 140 mg/dL at 11:00 pm. Eliminating 2 units before dinner will result in higher blood sugars at 11:00 pm and suboptimal control. Adding 4 units of regular insulin before breakfast will serve to lower the elevated fasting blood sugars, but will do nothing to correct the nocturnal hypoglycemia, which is the main problem. Adding this extra rapid-acting insulin is a reactive response rather than a proactive step to control blood sugars. Adding 2 units of NPH insulin before breakfast will have minimal effect to correct the elevated morning blood sugars or the nocturnal hypoglycemia. Adding 2 units of NPH insulin before dinner will make the nocturnal hypoglycemia worse.

A 26-year-old woman wants to conceive over the next few months. She is married, has 1 child, and she and her husband have been using condoms and foam for contraception. Her past medical history is significant for hypothyroidism which is treated with levothyroxine (Synthroid) 0.125 mg PO per day. Her last thyroid-stimulating hormone (TSH) level, performed 3 months ago, was normal. Today, she presents with a 6-week history of amenorrhea. A urine pregnancy test is positive. You estimate that she is 6 weeks pregnant. What will she likely need? Answer Choices 1 A decrease in her levothyroxine (Synthroid) dose 2 Checking free T4 levels 3 Adding triiodothyronine 4 An increase in her levothyroxine (Synthroid) dose 5 Checking free T3 levels

Explanation The correct response is an increase in her levothyroxine (Synthroid) dose. This patient is pregnant and hypothyroid.The management of this patient requires careful titration of her levothyroxine (Synthroid) levels to maintain euthyroidism during pregnancy. Thyroid hormone is highly protein-bound, and any medical condition that changes the level of serum-binding proteins will change the level of free thyroid hormone available. For example, conditions that increase the level of serum-binding proteins (e.g., pregnancy and oral contraceptives) will decrease the level of free thyroid hormone. In such cases, the dose of replacement thyroid must be increased. Conditions that necessitate a decrease in replacement doses include aging, chronic liver disease, and protein-losing conditions (e.g., nephropathy and enteropathies). While free T4 levels estimate the level of thyroid hormone available, the variation in levels does not reflect the metabolic needs of patients. TSH assessment provides a better reflection of decreasing levels of available thyroid hormone; however, TSH levels lag behind changes in free T4 levels. TSH should be repeated no earlier than 4 weeks after any changes are made in thyroid doses; it is important to note that it may take 8 weeks for the TSH to stabilize after the change is made. Some patients may note an improvement in mood, memory, and cold tolerance with the addition of triiodothyronine to their regular thyroid replacement. In the patient presented, however, there was no problem with her thyroid replacement before her pregnancy; therefore, it is unlikely that she will need T3 added to her regimen. Free T3 levels are unnecessary in the management of this patient. The increased T4 dose requirement is typically seen by the 1st trimester of pregnancy. Monitoring the TSH regularly throughout pregnancy will allow adjustments to be made in the levothyroxine (Synthroid) dose. The goal of thyroxine replacement is to maintain serum TSH in the normal range. It must be remembered that hypothyroidism during pregnancy is associated with hypertension and premature labor. Dose adjustments upwards are usually in the range of 25 - 50% of the normal daily thyroid replacement dose.

A 25-year-old woman presents because she feels weak and tired all the time. She also reports that she has been amenorrheic for the past 3 months. On examination, she is not pale but she appears fatigued. Her BP is 100/50 mm Hg and a pregnancy test is negative. What other symptoms would you expect from this patient? Answer Choices 1 Hyperhidrosis 2 Insomnia 3 Heat intolerance 4 Diarrhea 5 Decreased axillary hair

Explanation The correct response is decreased axillary hair. The clinical picture is suggestive of hypopituitarism. Hypopituitarism results in decreased levels of growth hormone (GH), gonadotropins (FSH, LH), thyroid stimulating hormone (TSH), adrenocorticotrophic hormone (ACTH), and prolactin. The onset is usually insidious. The clinical features of hypopituitarism depend on the underlying cause and the specific hormones that are deficient. Thyroid stimulating hormone deficiency results in hypothyroidism. Patients present with feeling weak, drowsy, and fatigued. Some may complain of rough skin and decreased sweating. These symptoms may be accompanied constipation and cold intolerance. Adrenocorticotrophic hormone deficiency results in adrenal insufficiency, which is characterized by fatigue, hypotension, and the loss of axillary as well as pubic hair. Gonadotropin deficiency results in amenorrhea and genital atrophy in females, as well as impotence and testicular atrophy in males. Growth hormone deficiency results in growth failure in children, but has no clinically significant effects in adults. Prolactin deficiency results in failure of lactation in the postpartum period. When panhypopituitarism is suspected, all target organ hormone function should be evaluated. Initial evaluation should be aimed at detecting TSH and ACTH deficiencies, because their deficiencies can cause life-threatening conditions: myxedema coma and adrenal crisis, respectively. Serum thyroxine (T4), triiodothyronine (T3), TSH levels, serum cortisol, and ACTH levels are low in hypopituitarism. Gonadotropin levels are low in panhypopituitarism, and GH measurements are generally helpful only in children. Thyrotropin-releasing hormone, insulin, and gonadotropin-releasing hormone can be administered intravenously together; then, TSH, GH, LH, FSH, ACTH, cortisol, prolactin, and glucose levels can be measured. Diagnostic imaging includes skull X-rays of the sella turcica, which may provide evidence of the presence of pituitary tumors. High-resolution CT scans and MRI with contrast media may visualize smaller pathologies. The differential diagnoses for panhypopituitarism include anorexia nervosa, chronic liver disease, and polyglandular autoimmune disease. In anorexia nervosa, the secondary sexual characteristics are maintained even though the patients may be wasted and amenorrheic. Men with chronic alcoholic liver disease may develop testicular atrophy, but they have other features of chronic liver disease, and the serum levels of pituitary hormones are usually normal. Treatment includes replacing the hormones of the hypofunctioning target glands. Treatment of GH deficiency in adults is unnecessary. When hypopituitarism is due to a pituitary tumor, specific treatment must also be directed to the tumor as well as to hormone replacement therapy. The management of such neoplasms may include surgical resection, bromocriptine, or irradiation. Hyperhidrosis, insomnia, heat intolerance and diarrhea are symptoms of hyperthyroidism.

A 29-year-old pregnant woman presents for a prenatal check up. She is 6 months pregnant. She comments that she is constantly tired and is worried about her thyroid function because she has a friend who just had thyroid surgery. On physical examination, her pregnancy is progressing normally. Her thyroid is nonpalpable. No bruit over her thyroid is heard. Her skin is unremarkable. The doctor orders a series of blood tests. When the results come back, her doctor tells her that her abnormal lab results are probably due to a change in the level of thyroxine-binding globulin (TBG). Her lab results are on the chart. TEST RESULTS REFERENCE RANGE Serum total T4 15 µg/dL 5 - 11 µg/dL Free T4 2.5 ng/dL 1.5 - 3.5 ng/dL Question What condition does her blood test suggest? Answer Choices 1 Euthyroidism 2 Thyrotoxicosis 3 Hyperthyroidism 4 Hypothyroidism 5 Myxedema

Explanation The correct response is euthyroidism. Thyroid hormone exists in the circulation, either bound or free. It is the unbound component, which has the metabolic effect. Thyroxine-binding globulin (TBG) binds roughly 70 - 75% of the thyroid hormones. Many conditions, including pregnancy, can affect the level of thyroxine-binding globulin in the serum, but not affect the level of unbound hormone. Since her unbound level of thyroxine is normal and it is the unbound portion that exerts the metabolic effect, this patient probably is euthyroid. Thyrotoxicosis and hyperthyroidism are due to an excess of active thyroid hormone. Hypothyroidism and myxedema are due to a deficiency of thyroid hormone.

A 40-year-old Caucasian woman, with a past medical history of Hashimoto's thyroiditis, type I diabetes mellitus, and pernicious anemia, presents with a 2-year history of insidious and intermittent fatigue, anorexia, involuntary weight loss, nausea, abdominal pain, vomiting, and dizziness associated with positional changes. Her physical exam is noteworthy for postural hypotension, with a maximum systolic blood pressure of 104 in the supine position. A fingerstick determination reveals a blood glucose of 100 mg/dL, and she has a normal urinalysis. She has a low-grade fever and a generalized pigment change to her skin, as noted in the attached image. The comprehensive metabolic panel demonstrates hyponatremia, hyperkalemia, a mild non-anion gap metabolic acidosis, and elevated BUN and creatinine levels. Question What is the most appropriate treatment for this patient? Answer Choices 1 Bromocriptine 2 Hydrocortisone 3 Insulin aspart 4 Metyrapone 5 Octreotide

Explanation The correct response is hydrocortisone. Given this patient's strong autoimmune history, the most likely diagnosis is Addison's disease. The most common cause of Addison's disease is idiopathic autoimmune adrenocortical insufficiency resulting from autoimmune atrophy, fibrosis, and lymphocytic infiltration of the adrenal cortex. Manifestations include insidious hyperpigmentation of the skin (most often prominent on the sun-exposed areas of the skin, extensor surfaces, knuckles, elbows, knees, and scars formed after the onset of disease), progressive weakness, fatigue, poor appetite, and weight loss. Prominent gastrointestinal symptoms may include nausea, vomiting, and occasional diarrhea; dizziness with orthostasis due to hypotension occasionally may lead to syncope. The comprehensive metabolic panel reveals hyponatremia, hyperkalemia, a mild non-anion gap metabolic acidosis, and elevated BUN and creatinine levels. Treatment involves replacement of corticosteroids and mineralocorticoids. The normal adrenal gland output of cortisol is approximately 250 - 300 mg in 24 hours; this amount of hydrocortisone in soluble form (hydrocortisone sodium succinate or phosphate) should be given, preferably by continuous infusion. In patients in acute adrenal crisis, IV access should be established urgently, and an infusion of isotonic sodium chloride solution should be started to restore volume deficit and correct hypotension. Some patients may require glucose supplementation. The drugs that are effective in the treatment of hyperprolactinemia are dopamine agonists; they are the primary physiologic inhibitors of prolactin secretion. Bromocriptine is generally considered to be the agent of choice in the treatment of prolactinoma. Insulin aspart is not appropriate, as the patient is euglycemic; her urinalysis does not reveal any evidence of ketoacidosis. Metyrapone, along with ketoconazole, mitotane, cabergoline, and pasireotide, are medications useful in reducing ACTH or cortisol levels in patients with Cushing's syndrome. Somatostatin and dopamine analogues and growth hormone (GH) receptor antagonists are the mainstays of medical treatment for GH excess syndromes (e.g., acromegaly and gigantism). They are generally used when primary surgery fails to induce complete remission. The most extensively studied and used somatostatin analogue, octreotide, binds to the somatostatin receptor; this inhibits GH secretion.

A 19-year-old man has known type I diabetes on insulin; he is brought to the ER by his family with a history of clouding of consciousness, vomiting, shortness of breath, and weakness. Symptoms have present since the morning. He also has a 2-day history of cough with productive sputum and fever. He is known to be non-compliant with insulin and diet and has had multiple ER visits. At home, his mother checked his blood sugar on his glucometer, which reported a "high level", signifying more than 500 mg/dl. On examination he is found to have a temperature of 101.5° F, pulse 110/min, BP 100/58 mm Hg, and respirations of 24/ minute. Skin and mucosa are very dry. Auscultation reveals crackles at the base of the right lung, tachycardia, and normal bowel sounds. He is drowsy but arousable and oriented. Labs are ordered, and supportive measures are started with fluids and insulin. Question The labs for the above patient show a blood sugar of 950 mg/dl, sodium 129 meq/L, chloride 91 meq/L, potassium 3.2 meq/L, BUN 35 mg/dL, creatinine 1.5 mg/dL, bicarbonate of 18 meq/L, anion gap 20, WBC 13,000/uL, serum ketones positive, and serum osmolality 300 mosm/kg. CXR shows right lower lobe consolidation and EKG shows tachycardia only. The best combination for the correct management of this patient includes fluids and what else? Answer Choices 1 SC insulin every 4 hours, potassium, bicarbonate, and antibiotics 2 Insulin drip, potassium, and antibiotics 3 Bolus insulin IV then SC every 4 hours, potassium, and antibiotics 4 IV insulin every 4 hours, potassium, and antibiotics 5 SC insulin every 2 hours, potassium, and antibiotics

Explanation The correct response is insulin drip, potassium and antibiotics. Therapy for diabetic ketoacidosis includes 4 components: fluids, insulin, electrolyte replacement, and treatment of precipitating factor. The average fluid deficit in such patients is 3 - 6 liters. Replacement is done aggressively, initially with normal saline, then with half normal saline after a few hours. However, too rapid hydration may cause cerebral edema, so should be avoided. For patients in shock, as this one, intravenous insulin is recommended as a bolus of 15 - 20 units followed by infusion of 0.1 units/kg/hr or approximately 5 - 10 units per hour. Subcutaneous insulin is not recommended in patients in shock, since poor perfusion impairs the absorption of insulin. Once the blood sugar falls to 200 - 250 mg/dl and acidosis is resolved, standard insulin therapy is started. Frequent labs and bedside blood sugar testing are done, including the complete metabolic panel and arterial blood gases. Potassium, phosphate and bicarbonate are replaced as needed. Insulin causes metabolism of ketones and regeneration of bicarbonate, so bicarbonate is given only if arterial pH goes below 7.1 or there is life-threatening hyperkalemia, since bicarbonate drives the potassium into the cells. Antibiotics are given for the underlying pneumonia in this case.

A 52-year-old Caucasian woman who has a longstanding history of Addison's disease is scheduled for an elective cardiothoracic surgery in 4 weeks. The planned surgical procedure is expected to produce significant surgical and medical stress. The patient currently takes hydrocortisone 21 mg PO daily in 2 doses; 7 mg is taken in the morning, and 14 mg is taken in the evening. Question What would be the most logical plan of action in regards to her hydrocortisone perioperatively? Answer Choices 1 Keep at the current amount of daily replacement dose 2 Stop hydrocortisone 2 days before the surgery, and restart the daily replacement dose 2 days after the procedure 3 Double the daily replacement dose 2 days before the surgery 4 Intravenous hydrocortisone 75 mg/daily on day of procedure, then taper over next 1 - 2 days to the patient's usual outpatient daily replacement dose 5 Intravenous hydrocortisone 150 mg/daily on day of procedure, then taper over next 2 - 3 days to the patient's usual outpatient daily replacement dose

Explanation The correct response is intravenous hydrocortisone 150 mg/daily on day of procedure, with a taper over next 2 - 3 days to usual daily replacement dose if the postoperative period is otherwise uncomplicated. Cortisol has many critical metabolic and endocrine functions that are necessary for human survival, particularly during times of stress. Everyday health and illness, surgery, anesthesia, trauma, and severe illnesses result in elevated plasma ACTH and cortisol levels. Those that are adrenally insufficient (such as patients diagnosed with Addison's disease) have very little or no function of this type of endocrine function, thereby predisposing them to inappropriate body reactions to stress that can result in devastating morbidities. External corticosteroids are prescribed to these patients to take on a daily basis. For this reason, patients with any type of adrenal insufficiency must have an appropriate treatment plan in regards to their corticosteroid therapy in order for them to produce an appropriate immune response to the stress of surgery. There is no universally accepted regimen for corticosteroid supplementation during the surgical and postoperative time period. A recommended dosing plan should be based on the severity of the surgery stress as well as the medical stress that the patient will most likely endure intra- and post-operatively. Cardiothoracic surgery and its postoperative time period are categorized as severe in terms of the effects on the body. For this reason, the most appropriate plan would be Intravenous hydrocortisone 150 mg/daily on day of procedure, with a taper over next 2 - 3 days to usual daily replacement dose if the postoperative period is otherwise uncomplicated.

A 49-year-old Caucasian man who is well known to your practice presents due to his history of hypertriglyceridemia. He has come in to see you today for evaluation of his recent cholesterol lab values. He has a significant family history of cardiovascular disease; his mother had a heart attack at the age of 57, and his father had open heart surgery at the age of 60. The patient has had low HDL levels in several past cholesterol screening tests. At the last office visit, lifestyle modifications were implemented by the patient. He has been extremely conscientious about his diet; for the past year, he has also been participating in physical activity 6 days a week. In addition, he has significantly limited his alcohol intake. Current lab values for the patient are as follows: total cholesterol of 235 mg/dL, triglycerides of 350 mg/dL, HDL of 35, and a LDL of 275 mg/dL. You decide to initiate pharmacologic therapy in order to treat the patient's cholesterol and triglyceride levels. Question What medication should you prescribe? Answer Choices 1 Simvastatin 2 Ezetimibe 3 Colestipol 4 Colesevelam 5 Cholestyramine

Explanation The correct response is simvastatin. Most patients with high cholesterol levels present with no specific symptoms or signs, although this is a significant contributor to the number 1 killer of adults in the United States, which is cardiovascular disease. There are a multitude of modifiable as well as non-modifiable risk factors present that contribute to overall cholesterol levels. If a patient has any abnormal levels of cholesterol levels, the initial component of any treatment is appropriate patient education with lifestyle modifications, which many times include limiting consumption of alcohol and cholesterol-containing foods, smoking cessation, and increasing physical activity. If a valiant effort of modifiable risk factors is taken and fails to bring cholesterol levels to normal ranges, pharmaceutical intervention should be initiated. The patient described in the scenario above would most benefit from the initiation of simvastatin. This medication has been found to significantly decrease triglycerides; it also causes up to a 40% reduction of LDLs and up to a 10% increase of HDL levels. Financially, it is on the lower end of the spectrum in terms of price per dose. Ezetimibe, cholestyramine, colestipol, and colesevelam are not good choices due to the fact that they have a varying effect on triglyceride levels, which is one of this patient's major abnormalities.

A 33-year-old woman presents with fatigue, weight gain, and non-specific aches and pains for the past year or so. She denies fever, cough, abdominal pain, dysuria, or visual changes, but she has had constipation recently. She feels dizzy on and off, and she grows increasingly tired with each passing day. Her weight gain of 20 pounds or so in the last year has made her depressed and tired. Her best friend repeatedly tells her that her voice is deeper and raspier on the phone. After further questioning, she claims that she has had quarrels with her husband and children regarding the thermostat in her house. She would like to keep it at 80°F, and they want it at 60°F in this month of May. On examination, she has a temperature of 96°F, pulse 56/min, and BP 110/70 mm of Hg. She is 5'3" tall, and her weight is 155 pounds. Lungs are clear, abdomen shows no abnormal findings, and heart sounds are regular. Her skin appears somewhat dry and scaly. Her hair is coarse. You order tests. Question What is the most important test at this point? Answer Choices 1 Metabolic panel 2 Complete blood count and peripheral smear 3 Thyroid stimulating hormone 4 FSH and LH 5 Lipid profile

Explanation The correct response is thyroid-stimulating hormone. This woman has hypothyroidism. She has the classic symptoms, which are weight gain, depression, constipation, myalgias, cold intolerance, fatigue, deep voice, bradycardia, and dry skin and hair. It is a very common condition that is seen mostly in middle aged and elderly women; however, it can also be seen in young patients. 1.5 - 2% women above 60 years and up to 5 - 15% women above 65 years of age may be affected. Hashimoto's thyroiditis is an autoimmune condition, and it is the most common cause of hypothyroidism in the western world, whereas iodine deficiency is the etiological factor in the developing countries. Hashimoto's thyroiditis may present as hypothyroidism, goiter, silent thyroiditis, or post-partum thyroiditis. Other symptoms of hypothyroidism include puffy eyes from periorbital edema, shortness of breath, exercise intolerance, decreased appetite, somnolence, blurred vision, and decreased sweating. Anemia may be present, which can be normocytic normochromic due to decreased red cell mass or, in 10% of cases, autoimmune thyroiditis. Macrocytic anemia due to pernicious anemia may be seen. Menstrual irregularities are common and could be amenorrhea, oligomenorrhea, or menorrhagia. Infertility and early pregnancy loss may be seen. Muscle weakness and carpal tunnel syndrome may occur. TSH will be high and T3, T4 low. TSH is the earliest and most sensitive indicator of primary hypothyroidism. Sensitive third generation TSH assays are now available for easy diagnosis. Anti-thyroid peroxidase (TPO) antibodies and antibodies to thyroglobulin may also be done. Treatment is by replacement of thyroxine for life, since it is a permanent condition. The recommended replacement dose for an average 70 kg adult is 1.6 mcg/kg/day, but the dose may vary from 50-200 mcg daily. In the elderly, it is a good idea to start at a lower dose, since thyroxine can aggravate coronary ischemia. It should be taken in the morning on an empty stomach. Metabolic panel is non-diagnostic, and so is a complete blood count. Anemia may be present occasionally. FSH and LH are done when perimenopause is suspected. Some of her symptoms, such as depression, myalgias, and fatigue, are also seen in perimenopausal women; however, weight gain, cold intolerance, and bradycardia are not seen. Lipid profile is a screening test for adults and should be done in this patient also. It is not a diagnostic indicator of the underlying condition, which is hypothyroidism.

A 32-year-old man presents with multiple symptoms. He states that his feet have become wider, requiring him to purchase shoes in a larger size. His hands have also increased in size, and a friend recently noted that his handshake felt doughy. His facial features have become coarse, and his brow also appears abnormal (as seen in the image). Question What is the most common cause of the patient's condition? Answer Choices 1 MEN-1 syndrome 2 Pituitary tumor 3 Pancreatic tumor 4 Lymphoma 5 Hypothalamic tumor

Explanation The patient most likely has a pituitary tumor. The patient is suffering from acromegaly, the excess secretion of growth hormone (GH). GH typically stimulates growth, and when produced in excess amounts, causes abnormal growth of the soft tissue and skeleton. As a result, patients develop symptoms of increased hand and foot size, a prominent brow, and coarse facial features. Most cases of acromegaly are caused by a pituitary adenoma that secretes excessive amounts of GH. MEN-1 syndrome, pancreatic tumors, lymphoma,and hypothalamic tumors may all cause acromegaly, but they are not the most common causes.

A 39-year-old Caucasian man has a history of hypertriglyceridemia. He is well known to your practice, and he presents today for evaluation of his recent cholesterol lab values. He has a significant family history of cardiovascular disease: his mother had a heart attack at the age of 57; his father had open heart surgery at the age of 60. He has a long history of high total cholesterol and triglycerides levels; low HDL levels have been found in the past few cholesterol screening tests. At the last office visit, lifestyle modifications were implemented by the patient. He has been extremely conscientious about his diet; he has been engaging in physical activity 6 days a week for the last 6 months. Current lab values for the patient are as follows: total cholesterol of 235 mg/dL; triglycerides of 350 mg/dL; HDL of 35; and a LDL of 275 mg/dL. At this current appointment, the patient refuses to initiate pharmacologic treatment and wishes to continue lifestyle modifications to treat his cholesterol levels. He inquires to you what else he could do to make his actions be more effective. Question What advice should you give the patient? Answer Choices 1 Increase consumption of alcohol 2 Increase consumption of saturated fats 3 Increase consumption of fish oils 4 Decrease his aerobic activity 5 Increase consumption of refined carbohydrates

Explanation The patient should increase consumption of fish oils. Most patients with high cholesterol levels present with no specific symptoms or signs, although this is a significant contributor to the number one killer of adults in the United States, which is cardiovascular disease. There are a multitude of modifiable as well as non-modifiable risk factors present that contribute to overall cholesterol levels in each patient. If a patient has any abnormal cholesterol levels, the first component of any treatment is appropriate patient education regarding lifestyle modifications. Modifiable risk factors a patient should be instructed to initiate include the concept of moderation of energy intake and specifically limiting the energy intake from animal fats, which are also known as saturated fats. Body weight control is critical and regular aerobic activity should be encouraged in these patients. Patients should be instructed to avoid high-carbohydrate and refined carbohydrates. Increasing consumption of unsaturated fats is recommended; this has been shown to specifically lower triglycerides. Fish oils have also been found to contain the essential fatty acids docosahexaenoic acid (DHA) and eicosapentaenoic acid (EPA), both with known effectiveness at also lowering triglyceride levels. Limiting consumption of alcohol as well as smoking cessation are also critical components of lifestyle modification initiation. If an earnest attempt at lifestyle modification is taken and the efforts fail to bring cholesterol levels to normal ranges, then pharmaceutical intervention should be initiated.

A 32-year-old woman presents with constipation, weight gain, and dry skin. She has been experiencing the symptoms for a few months. Examination findings include dry rough skin, diffuse thyroid enlargement, bradycardia, and edema of hands and feet. A thyroid profile is performed and shows elevated thyroid stimulating hormone (TSH) and the presence of thyroid antibodies-antithyroid peroxidase (anti-TPO) and anti-thyroglobulin (anti-Tg) antibodies. The tissue biopsy microscopy is revealed in the image, with areas of interest indicated by the arrows. Question What is the most likely cause of these findings? Answer Choices 1 Hashimoto's thyroiditis 2 Riedel's Thyroiditis 3 Subacute painless thyroiditis 4 Suppurative thyroiditis 5 De Quervain's Thyroiditis

Hashimoto's Thyroiditis The presence of antithyroid antibodies and the typical histology points to a diagnosis of Hashimoto's thyroiditis (chronic lymphocytic thyroiditis). It is the most common form of thyroiditis, and most common cause of hypothyroidism in the US. This condition is characterized by thyroid cell destruction by various antibody-mediated immune processes. The gland appears diffusely enlarged and firm. Increased circulating levels of antithyroid peroxidase or anti-thyroglobulin antibodies is a diagnostic feature. Hashimoto's thyroiditis is a histological diagnosis characterized by infiltration of lymphocyte and plasma cells, reduced size of the thyroid follicles containing sparse colloid, and fibrosis along with presence of follicle with oxyphilic change called Hurthle cells. The image shows replacement of thyroid follicles by lymphoplasmacytic infiltrate with germinal centers (indicated by arrows). Riedel's thyroiditis is characterized by dense fibrosis, which results in a stony hard enlargement of the thyroid gland. It is the rarest form of thyroiditis. Histological features of Riedel's thyroiditis include a fibro-inflammatory process involving a portion or the entire thyroid gland and the presence of fibrotic extension into the adjacent structures beyond the thyroid capsule. Subacute painless thyroiditis is a self-limiting disease process having triphasic clinical presentation of hyperthyroidism, hypothyroidism, and return to euthyroid state. Thyroid biopsy shows presence of lymphocytic infiltration. Suppurative thyroiditis is rare and associated with severe anterior neck pain, tenderness, and redness in the region. It is an acute thyroiditis characterized by high fever and a swollen and tender thyroid gland. Lab findings include leukocytosis and increased erythrocyte sedimentation rate (ESR). De Quervain's thyroiditis, also called subacute granulomatous thyroiditis, is a painful enlargement of the thyroid gland. It is believed to be caused by viral infection or a post-viral inflammatory response. The histological finding in De Quervain's thyroiditis includes the presence of multinucleated giant cell granulomas.

A 45-year-old man presents with what he refers to as "anxiety attacks". He describes them as recurring bouts of palpitations, headaches, anxiety, and sweating. One evening, while having dinner with his wife, his wife witnesses an episode. She is concerned and immediately takes his blood pressure. His blood pressure is 195/105 mmHg. The next day, his wife takes him to see their family doctor. On further questioning, he comments that he sometimes gets light headed when he stands up too rapidly. He comments that his mother had similar problems. On physical examination, his blood pressure is 165/90 mmHg and his heart rate is 80 beats/min. A 24-hour collection of his urine test is positive for vanillylmandelic acid. Question What is the most likely diagnosis? Answer Choices 1 Pheochromocytoma 2 Essential hypertension 3 Cushing's syndrome 4 Renal artery stenosis 5 Phenylketonuria

Explanation This patient exhibited signs and symptoms of pheochromocytoma. He has paroxysmal hypertension and paroxysmal episodes of sweating and anxiety. The paroxysmal episodes that he experiences are due to intermittent release of catecholamines from the pheochromocytoma. Postural hypotension can sometimes be seen with pheochromocytoma. Urinary vanillylmandelic acid (VMA) is a metabolite of catecholamines and is found in pheochromocytoma. Essential hypertension, Cushing's syndrome, and renal artery stenosis can all cause hypertension; however, they would not cause paroxysmal hypertension, as this patient had. In addition, VMA would not be found in the urine. Phenylketonuria is not a cause of hypertension. It is a metabolic disease that results in problems processing phenylalanine.

A 32-year-old woman presents with a solitary thyroid nodule. A biopsy confirms that she has thyroid cancer. She subsequently has a thyroidectomy. However, during the procedure, the surgeon accidentally removes most of her parathyroid glands as well. As a consequence, she develops hypoparathyroidism. Her laboratory results are in the chart. What symptom would be consistent with her history and findings? TEST RESULTS REFERENCE RANGE RBC 4.8 x 106/µl 4.5 - 5.7 x 106/µl (male) 3.9 - 5.0 x 106/µl (female) Hematocrit 42 40 - 50 (male) 36 - 44 (female) Hemoglobin 15 gm/dL 13.8 - 17.2 gm/dL (male) 12.1 - 15.1 gm/dL (female) Platelets 252,000/mm3 130,000 - 400,000/mm3 Potassium 4.1 mEq/L 3.5 - 5.0 mEq/L Sodium 144 mEq/L 135 - 145 mEq/L Calcium 7.8 mg/dL 8.5 - 10.5 mg/dL Question What is a sign of her secondary condition? Answer Choices 1 Chvostek's sign 2 Kernig's sign 3 Babinski's sign 4 Kussmaul's sign 5 Quincke's sign

Explanation This patient has hypocalcemia secondary to hypoparathyroidism. Paresthesias can be seen with hypocalcemia due to the increase in neuromuscular irritability. Common sites for the paresthesias are around the mouth and on the fingertips. Tetany can also be seen. Chvostek's sign can be seen with tetany. When tapping on the facial nerve produces contraction on that side of the face, it is called Chvostek's sign. Kernig's sign is a sign of meningeal irritation. The patient lies on his back and his leg is raised and knee bent at a 90-degree angle. When the examiner straightens the knee, if there is pain or resistance to further extension, Kernig's sign is said to be positive. Babinski's sign is an abnormal response to stimulation on the sole of the foot; it results in dorsiflexion of the big toe and fanning of the other toes. Babinski's sign is seen with pyramidal disease. Kussmaul's sign is an increase in venous pressure during inspiration; it can be seen with cardiac tamponade. Quincke's sign (Quincke's pulse) is an alteration seen in nails with each heartbeat. Quincke's sign can be seen with aortic insufficiency.

A 62-year-old woman presents to her OB/GYN clinic for an annual exam. She reports she has not felt well for several months, but her family practice physician retired, and she kept hoping her symptoms would improve. She reports stretch marks and a weight gain of about 20 pounds, which is especially noticeable in her face. She feels fatigued; she has no sex drive, and she feels more irritable. She notes increased dark hairs growing on her chin. The patient reports easy bruising, but she denies blood in the stools. She denies fever, chills, foreign travel, changes in sexual partners, and suicidal thoughts. Her past medical history is significant for asthma, with periodic hospitalizations for asthma attacks. She was menopausal at the age of 51. She has had no surgeries. She takes prednisone 40 mg daily, an inhaler with fluticasone propionate/salmeterol 2 times daily, and an albuterol inhaler as needed for her asthma. She has no allergies. She is retired; she lives at home with her husband, and she denies use of tobacco, alcohol, and street drugs. BP is 162/92 mm Hg, and pulse is 98. Respiratory rate is 12. Weight is 170 lbs, and height is 63", making her BMI 30.11. On physical exam, the patient has a rounded, full face; she appears fatigued, but she is pleasant and cooperative. Striae are noted on her abdomen, and hirsutism is present on her chin and abdomen. There appears to be a large fat deposit on the upper back, and she has an obese abdomen, with some muscle wasting on the patient's extremities. The remainder of her exam is normal. Question What is the most appropriate intervention for this patient's current condition? Answer Choices 1 Counsel the patient about normal post-menopausal symptoms 2 Perform dexamethasone suppression test 3 Refer to dietitian for weight loss 4 Stop the inhaled medications 5 Taper off the daily prednisone

Explanation This patient is presenting with Cushing syndrome, an excess of glucocorticoids; it is most likely iatrogenic, caused by her daily prednisone. The most appropriate intervention is to taper off her daily prednisone. This common steroid is often given for acute flares of chronic diseases. However, chronic daily use can lead to a glucocorticoid excess known as Cushing syndrome. Cushing syndrome is characterized by central obesity, muscle wasting, hirsutism, a 'buffalo hump' on the upper back, 'moon facies', weight gain, hypertension, decreased libido, irritability, and several other possible signs/symptoms. Unrecognized and untreated Cushing syndrome can lead to osteoporosis. Tapering off the prednisone (rather than immediate discontinuation) helps prevent sudden adrenocortical insufficiency. While it would be acceptable to counsel the patient about normal post-menopausal symptoms at her GYN annual exam, her current condition cannot be attributed to normal menopausal changes. Normal post-menopausal changes include vasomotor symptoms (such as hot flashes and night sweats) and vulvovaginal changes (such as atrophy, vaginal dryness and dyspareunia). Some women may experience some mild weight gain and irritability, but the remainder of this patient's symptoms are not related to a drop in sex hormones. When Cushing syndrome is suspected, it is common to perform a dexamethasone suppression test. The dexamethasone is an exogenous glucocorticoid that will negatively feedback on the pituitary and should lower the adrenocorticotropic hormone and result in lowered cortisol levels in the serum. However, while the patient is taking daily corticosteroids, the test will be altered. This patient should be tapered off the prednisone, and if symptoms persist, a dexamethasone test can be done. This patient's concerns should not be attributed to a weight-gain only issue and simply referred to a dietitian for weight loss. Weight gain is a common side effect from exogenous corticosteroids and should reverse with discontinuation of the medication. A dietitian referral could be offered in addition to treatment of this patient's Cushing syndrome, but it should not be the primary treatment. This patient's inhaled fluticasone propionate is also a steroid medication, and it is given to reduce inflammation in the airways. Typically, the inhaled and intranasal steroids at usual doses are considered low enough strength that they will not lead to corticosteroid excess and Cushing syndrome. The systemic steroid, prednisone, is much more potent and is the likely culprit for this patient's symptoms. The albuterol inhaler is a short-acting beta-agonist and should not be discontinued in patients with severe asthma; therefore, it would be inappropriate to stop this patient's inhaled medications.

A 35-year-old man with no significant past medical history presents with multiple skin lesions in various parts of his body. These evolved gradually over the past year. He describes them as small, painless, non-pruritic, and red-yellow, as well as flesh-toned. They are distributed on his buttocks, posterior ankles, and on his knees. There is no discharge to these lesions. He denied any allergies, trauma, history of surgeries, or instrumentation. He further denied recent fever, chills, travel, insect bites, sick contacts, chest pain, cough, abdominal pain, nausea, vomiting, diarrhea, edema, or other rashes. The physical examination was notable for an overweight man with multiple red-yellow papules on the buttocks. Additionally, there were several flesh-toned, firm, nodular lesions distributed over the bilateral Achilles and patellar tendons. All skin lesions were nontender and without discharge. The remainder of the physical exam was unremarkable. Question What is the most likely diagnosis? Answer Choices 1 Hyperlipidemia 2 Squamous cell carcinoma 3 Folliculitis 4 Psoriasis 5 Nummular eczema

Explanation This patient is presenting with hyperlipidemia. Extremely high levels of chylomicrons or VLDL particles (triglyceride level above 1000 mg/dL) result in the formation of eruptive xanthomas. These lesions are described as red-yellow papules, which are commonly found on the buttocks. High LDL concentrations result in tendinous xanthomas on certain tendons, such as the Achilles, patellar, and those on the back of the hand. The lesions associated with squamous cell carcinoma occur primarily in chronic sun-exposed areas and appear as small red, conical, hard nodules that occasionally ulcerate without healing. Folliculitis typically presents as pustules of hair follicles, with symptoms ranging from slight burning and tenderness to intense itching. Psoriatic lesions may be pruritic and are most commonly found on the scalp, elbows, knees, palms, and soles. They are red, sharply defined plaques covered with silvery scales. Pitting and onycholysis are common nail findings. Nummular eczema is a chronic, pruritic inflammatory dermatitis occurring as coin-shaped plaques composed of grouped small papules and vesicles on an erythematous base, which is common on the lower legs. These are seen in atopic individuals.

A 32-year-old woman presents with constipation, weight gain, and dry skin. She has been experiencing the symptoms for a few months. Examination findings include dry rough skin, diffuse thyroid enlargement, bradycardia, and edema of hands and feet. A thyroid profile is performed and shows elevated thyroid stimulating hormone (TSH) and the presence of thyroid antibodies-antithyroid peroxidase (anti-TPO) and anti-thyroglobulin (anti-Tg) antibodies. The tissue biopsy microscopy is revealed in the image, with areas of interest indicated by the arrows. Question What is the most likely cause of these findings? Answer Choices 1 Hashimoto's thyroiditis 2 Riedel's Thyroiditis 3 Subacute painless thyroiditis 4 Suppurative thyroiditis 5 De Quervain's Thyroiditis

Hashimotos The presence of antithyroid antibodies and the typical histology points to a diagnosis of Hashimoto's thyroiditis (chronic lymphocytic thyroiditis). It is the most common form of thyroiditis, and most common cause of hypothyroidism in the US. This condition is characterized by thyroid cell destruction by various antibody-mediated immune processes. The gland appears diffusely enlarged and firm. Increased circulating levels of antithyroid peroxidase or anti-thyroglobulin antibodies is a diagnostic feature. Hashimoto's thyroiditis is a histological diagnosis characterized by infiltration of lymphocyte and plasma cells, reduced size of the thyroid follicles containing sparse colloid, and fibrosis along with presence of follicle with oxyphilic change called Hurthle cells. The image shows replacement of thyroid follicles by lymphoplasmacytic infiltrate with germinal centers (indicated by arrows). Riedel's thyroiditis is characterized by dense fibrosis, which results in a stony hard enlargement of the thyroid gland. It is the rarest form of thyroiditis. Histological features of Riedel's thyroiditis include a fibro-inflammatory process involving a portion or the entire thyroid gland and the presence of fibrotic extension into the adjacent structures beyond the thyroid capsule. Subacute painless thyroiditis is a self-limiting disease process having triphasic clinical presentation of hyperthyroidism, hypothyroidism, and return to euthyroid state. Thyroid biopsy shows presence of lymphocytic infiltration. Suppurative thyroiditis is rare and associated with severe anterior neck pain, tenderness, and redness in the region. It is an acute thyroiditis characterized by high fever and a swollen and tender thyroid gland. Lab findings include leukocytosis and increased erythrocyte sedimentation rate (ESR). De Quervain's thyroiditis, also called subacute granulomatous thyroiditis, is a painful enlargement of the thyroid gland. It is believed to be caused by viral infection or a post-viral inflammatory response. The histological finding in De Quervain's thyroiditis includes the presence of multinucleated giant cell granulomas.

A 28-year-old woman with a 4-year history of end stage renal disease secondary to diabetes has had intact parathyroid hormone (PTH) levels of 600-800pg/ml (ref 150-300 pg/ml) for the past 15 months. She does not take prescribed phosphorus binders with meals; she routinely misses her dialysis treatments, during which she receives intravenous paricalcitol. What radiologic findings are expected in this patient? Answer Choices 1 Normal DEXA scan 2 Cinnamon skull 3 Rugger jersey spine 4 Thumbprinting 5 Quasifractures

Explanation This patient will most likely have a 'rugger Jersey' appearance to her vertebrae if she has a back or skeletal X-ray. She has hyperparathyroidism. End stage kidney disease patients (stage 5 chronic kidney disease) should have PTH levels between 150 and 300 pg/ml. She is likely hyperphosphatemic because of impaired renal phosphorus excretion, inadequate dialysis, and failure to take phosphorus binders when she eats. Hyperphosphatemia stimulates PTH release. Vitamin D (i.e., paricalcitol) can suppress PTH release, but she may be missing adequate doses when she skips her dialysis sessions. Excess PTH leads to abnormal bone remodeling, including excessive resorption. Lucencies may be seen in poorly mineralized areas. 'Pepper pot skull' refers to such scattered lucent findings on skull films; 'cinnamon skull' is not a term used to describe the radiologic findings of hyperparathyroidism. 'Pseudofractures' are the straight lucent bands of poorly mineralized bone seen in hyperparathyroidism, particularly in long bones. Quasifractures are not radiologic findings seen in hyperparathyroidism. DEXA (Dual X-ray absorptiometry) scans are quantitative measures of bone mineralization. Low bone mineralization is inversely related to PTH levels (Brenner). This patient will not likely have a normal DEXA scan because her bones are diseased. DEXA is not a diagnostic test for renal bone disease; measures of PTH, together with calcium, phosphorus, and alkaline phosphatase levels, are better diagnostic tests for renal bone disease. Thumbprinting refers to bowel wall edema noted on CT or abdominal radiographs in certain conditions, such as mesenteric ischemia (Harrison's ch. 279) and toxic megacolon. Ischemia often requires surgical intervention. Toxic megacolon is treated surgically and with antibiotics.

A 52-year-old woman presents to her primary care physician with weight gain and poor wound healing. Examination reveals round face, central obesity with muscle wasting, striate, and hypertension. Imaging reveals a pituitary adenoma. Question What is the recommended treatment? Answer Choices 1 Hydrocortisone 2 Prednisone 3 Transsphenoidal adenoma resection 4 Pituitary radiosurgery 5 Adrenalectomy

Explanation Transsphenoidal adenoma resection is correct. The patient is showing signs of Cushing syndrome (hypercortisolism). Cushing syndrome is a term used to describe the effects of excessive corticosteroid production. Cushing disease refers to hypercortisolism caused by excessive ACTH secretion. Most cases of Cushing disease are caused by a pituitary adenoma. The preferred treatment is transsphenoidal resection. Hydrocortisone and prednisone are incorrect. Both of these medications would worsen Cushing disease. Pituitary radiosurgery and adrenalectomy are both used in the treatment of Cushing disease, but transsphenoidal adenoma resection is the preferred treatment modality.

A 16-year-old boy presents for a physical. The physician notices a large amount of truncal fat and relatively thin limbs. What is the most likely diagnosis? Answer Choices 1 Turner's syndrome 2 Marfan's syndrome 3 Cushing's syndrome 4 Klinefelter's syndrome 5 Male hypogonadism

Explanation Truncal fat with relatively thin limbs is characteristic of Cushing's syndrome. Turner's syndrome is characterized by very short stature and webbed neck. People with Marfan's syndrome have very long limbs, fingers, and toes in proportion to the trunk. Klinefelter's syndrome is a chromosomal abnormality in which males have a eunuchoid habitus. Males with hypogonadism have long limbs in proportion to the trunk.

A 78-year-old woman is diagnosed with a duodenal neuroendocrine tumor. The attending surgeon, prior to undertaking surgery, reviews her past medical and surgical history. She underwent a total parathyroidectomy at the age of 55 for multiple hyperplastic parathyroid adenomas. Since then, she has been on calcium supplementation. She has no other comorbidities. Her younger brother had similar adenomas. CT scans are ordered to look for any metastases, and they reveal a small pituitary adenoma. She undergoes surgery and recovers with no complications. Question What diagnostic measure could have detected this in her youth? Answer Choices 1 Yearly serum calcitonin levels 2 BRCA mutation analysis 3 Wermer's syndrome genetic testing 4 RET protooncogene testing 5 Radio iodine uptake scanning

Explanation Wermer's syndrome genetic testing is the correct answer. Also known as Multiple Endocrine Neoplasia syndrome type 1 (MEN1), it is characterized by the occurrence of varying combinations of more than 20 endocrine and non-endocrine tumors. Endocrine tumors are represented mainly by the 'classic' P-triad originally described by Wermer: parathyroid, pituitary, and pancreatic tumors. Asymptomatic MEN1 mutant carriers need to be clearly recognized because the gene-related mutations confer a high risk of multiple primary cancers, which occur at younger ages, and they affect multiple family members who inherit the cancer-predisposing genetic mutation. A DNA test of the MEN1 gene detects mutations in 80 - 90% of familial MEN1 and in 65% of individuals with simplex MEN1. Yearly serum calcitonin levels is incorrect. Calcitonin levels help to diagnose Medullary carcinoma of the thyroid, but yearly levels are not an effective way of diagnosing MEN1 syndrome. In addition, it is not a preventive measure as Calcitonin levels are elevated only after the development of the malignancy. BRCA mutation analysis is incorrect. It is not associated with MEN1 syndrome. RET protooncogene testing is incorrect. It is diagnostic for MEN2a syndrome. Radioiodine uptake scanning is incorrect. In today's setting, its utility is mainly in the evaluation of hyperthyroidism and in identifying cold nodules.

A 35-year-old woman presents because of weight loss and palpitations. She lost 10 kg over 5 months despite having a good appetite. Her heart pounds and her hands tremble "all the time." She feels hot, is sweating profusely, and has difficulties going to and maintaining sleep; the slightest stimulus wakes her. Her job is suffering because of her nervousness, and her supervisor became concerned because she uses the bathroom 3 - 4 times a day in a need to move her bowels. She thinks that poor sleep quality and frequent bowel movements make her weak; she cannot climb stairs anymore and has to take a rest every 10 steps or so. Physical examination reveals a slim, anxious woman with pronounced stare, fine postural and tremor at rest, and slight proximal weakness. Her thyroid is diffusely enlarged and non-tender; her pulse is 100/min; and the rest of examination is within normal limits. Question What is the most likely diagnosis? Answer Choices 1 Hashimoto's thyroiditis 2 Toxic goiter 3 Grave's disease 4 Plummer's disease 5 Thyroid storm

Explanation Your patient has signs and symptoms suggestive of hyperthyreosis. The most common cause of hyperthyreosis is Grave's disease. It is an autoimmune disease caused by anti thyroid-stimulating hormone (TSH) receptor, can be familial, and is associated with other autoimmune diseases. Grave's disease typically presents with diffuse enlargement of the gland and exophthalmos. Exophthalmos is seen in about half of patients; it is a distinctive feature of Grave's disease and cannot be seen in other thyroid diseases. In the case of inflammation, as in Hashimoto's thyroiditis, the gland will be tender on palpation and there will be no exophthalmos. Patients with toxic goiter usually have a history of nontoxic enlargement of the gland and no exophthalmos. Plummer's disease, or toxic multinodular goiter, is the second most common cause of hyperthyroidismin the US and the most common cause of hyperthyroidism in the developing world, where the population is iodine-deficient. Patients usually have a history of prior nontoxic goiter with multiple nodules in the gland and no exophthalmos. Thyroid storm is a medical emergency. Patients with thyroid storm usually have a history of untreated hyperthyroidism and stress (most commonly trauma and infection) as a trigger. Typical presentation is a hypermetabolic state with high fever, tachycardia, nausea, vomiting, tremulousness, agitation, and changes in mental state.

A 46-year-old man presents for a routine check up. He drinks alcohol occasionally and quit smoking 20 years ago. His BP is 124/76 mmHg. He works as a salesman, but he wants to change to a desk job because he finds the commuting too stressful. His new employer requests that he have a full health check up prior to starting. He is concerned because his maternal grandmother and his father had diabetes. Question What is the best screening test to use? Answer Choices 1 HbA1c 2 Fasting Plasma Glucose 3 Post Prandial Plasma Glucose 4 75 gram Glucose Tolerance Test 5 100 gram Glucose Tolerance Test

Fasting Plasma Glucose Explanation The FPG is the recommended screening test. The OGTT may be necessary for the diagnosis of diabetes when the FPG is normal. A fasting blood glucose level of over 126 is considered diagnostic of diabetes. A level of 110-125 is considered impaired glucose tolerance. The FPG is preferred for screenings because it is faster and easier to perform, more convenient, acceptable to patients, and less expensive. These are the American Diabetes Association Guidelines. HbA1c, Post Prandial Glucose, and OGTT are incorrect. The American Diabetic Association has approved the use of PPBS and HBA1c in the diagnosis of diagnosis of diabetes. However, fasting plasma glucose is preferred for the reasons stated above.

A 39-year-old woman presents for an annual appointment. She has a history of diabetes mellitus, but she is otherwise healthy. Her recent lab work reveals that her serum glucose is well controlled by a combination of lifestyle and 1 medication. Although this is a routine appointment, she does present with recent onset of some unusual symptoms. Physical examination of the patient also reveals some abnormalities. Question What history and physical examination combination would prompt a workup for Addison's disease? Answer Choices 1 Cramping, numbness around mouth, and tingling in distal extremities combined with carpopedal spasm and exaggerated deep tendon reflexes 2 Weight gain, decreased libido, menstrual irregularity combined with moon facies, and increased adipose tissue in neck 3 Fatigue, dizziness, salt craving combined with increased pigmentation of her lips and hand creases, as well as hair loss 4 Bone pain, flank pain, and generalized anxiety combined with hypertension and mental confusion 5 Hair loss, decreased energy, and constipation combined with periorbital puffiness and dry, coarse skin

Fatigue, dizziness, salt craving combined with increased pigmentation of her lips and hand creases, as well as hair loss The correct answer is the combination of fatigue, dizziness, and salt craving with increased pigmentation of her lips and hand creases, as well as hair loss. Addison's disease is the result of adrenal cortex destruction, which causes deficiencies in cortisol, aldosterone, and androgens. Patients can have any combination of various systemic symptoms, including fatigue, weakness, nausea, vomiting, abdominal pain, dizziness, chronic diarrhea, depression, salt craving, and a decreased tolerance for cold. In addition, the physical examination findings can also vary. Some possible findings are hypotension, weight loss, vitiligo, hair loss, and increased pigmentation. The increased pigmentation can occur in the hand creases, dental-gingival margins, buccal and vaginal mucosa, lips, areolas, scars, and pressure points. The increased pigmentation is a result of increased secretion of β-lipoprotein. The combination of weight gain, decreased libido, and menstrual irregularity with moon facies and increased adipose tissue in neck are indicative of a patient with Cushing's disease. Cushing's disease is essentially the opposite of Addison's disease; its signs and symptoms are caused by an excess of adrenocorticotropin rather than the deficiency seen in Addison's disease. Patients with Cushing's disease can also experience hypertension, hirsutism, depression, easy bruising, muscle weakness, obesity, thin skin, skeletal growth retardation, and acne. The combination of cramping, numbness around mouth, and tingling in distal extremities with carpopedal spasm and exaggerated deep tendon reflexes could occur in a patient with hypoparathyroidism. This disorder is a result of a deficiency of parathyroid hormone from congenital absence, injury, surgery, or other diseases. Most of the symptoms associated with hypoparathyroidism are a result of hypocalcemia. The combination of bone pain, flank pain, and anxiety with hypertension and mental confusion could occur in a patient who has hyperparathyroidism. This disorder is characterized as dysfunction in the body's regulatory system for parathyroid hormone. While up to 75% of patients can be asymptomatic, the hypercalcemia associated with hyperparathyroidism can result in the classic complaints of painful bones, renal stones (causing flank pain), abdominal groans, and psychic moans. The combination of hair loss, decreased energy, and constipation with periorbital puffiness and dry, coarse skin is indicative of hypothyroidism. Hypothyroidism is typically an autoimmune disease or a result of prior radiation therapy or thyroid surgery.

A 5-year-old child presents for her kindergarten checkup. The clinician notes that over the past couple of years, her height decreased from the 50th percentile to the 5th percentile. On examination, the clinician also notes truncal adiposity. Her CBC and lead levels were normal. Which of the following is the most likely diagnosis? A growth hormone deficiency B Cushing disease C congenital hypothyroidism D congenital adrenal hyperplasia

GH deficiency Growth hormone (GH) deficiency is defined as a decreased growth velocity, delay in skeletal maturation, absence of other explanations for poor growth (lack of intake), and laboratory tests demonstrating decreased GH secretion. Etiology of GH deficiency can be congenital, genetic, acquired, or idiopathic, which is the most common. Infants usually have a normal birth weight and may have a slightly decreased length. In addition, most infants present with other endocrine deficiencies like hypoglycemia, hypothyroidism, and/or adrenal insufficiency. Children may present with truncal adiposity because growth hormone promotes lipolysis. Serum GH or intrinsic growth factor levels may or may not be decreased. In patients who do not have a demonstrated decrease in these hormones, a trial period with GH is indicated. These patients and positive GH-deficient patients receive a once-daily subcutaneous injection of recombinant human GH. Congenital hypothyroidism typically presents with short stature (typically noted after the 4-month newborn visit), delayed epiphyseal development, delayed closure of fontanelles, and retarded dental eruption in addition to other signs of hypothyroidism. Cushing disease typically presents with truncal adiposity with thin extremities, muscle wasting, decreased growth rate, and moon facies. Laboratory results show elevated adrenocorticosteroids both in urine and serum, hypokalemia, eosinopenia, and lymphocytopenia. Typically, in patients younger than the age of 12, Cushing disease is secondary to administration of ACTH or glucocorticoids. Congenital adrenal hyperplasia typically presents with pseudohermaphroditism in females or salt-losing crisis in males with or without isosexual precocity. There is an increased linear growth and advanced skeletal maturation.

A 34-year-old man presents for his first doctor's appointment. He had always neglected his health, but he finally sought medical attention at his wife's insistence. He explains that he was from and "unhealthy family" and that he was tired of being around doctors while growing up; therefore, he avoided medical attention. His father died at the age of 30 of "very high blood pressure" and "heart failure". His older brother was recently operated on for the removal of a cancer from his neck. Prior to that, his brother had had surgery to remove a mass from his adrenal gland. He wants to know what he can do to be healthy. His blood pressure and BMI are within normal limits. He does not smoke or use alcohol, and he exercises regularly. The physician suspects a possible familial syndrome. Question What is the most appropriate next diagnostic step? Answer Choices 1 An annual colonoscopy starting now 2 Measure T3, T4, and TSH levels 3 Genetic testing for possible familial cancer syndrome 4 Reassurance and suggest he return for yearly physical exam 5 Order a renal ultrasound to look for renal cysts

Genetic testing for possible familial cancer syndrome Explanation The vignette suggests a possible familial cancer syndrome. MEN 2a syndrome fits the picture of 2 generations and multiple members of the family affected. MEN 2a is an autosomal dominant disease, characterized by primary hyperparathyroidism, medullary carcinoma of thyroid, and pheochromocytoma. The father has features suggestive of pheochromocytoma, one of the causes of early onset hypertension and sudden cardiac death. The older brother has an adrenal mass that was excised, indicating a possible pheochromocytoma, and a malignant neck mass, indicating possible medullary cancer of the thyroid. MEN 2a is most commonly caused by mutations in the RET proto oncogene. The mutations are more commonly familial, although they may occur spontaneously. Genetic analysis for the mutation is done as a screening test in particularly high risk families, and a prophylactic thyroidectomy may be advocated. Mutation testing has replaced the regular measurement of serum calcitonin as a screening test. Other tests that may be done include: Serum Ca, parathyroid hormone, and plasma free metanephrine or urinary catecholamine levels Pheochromocytoma localization with MRI or CT A routine colonoscopy is incorrect. The US Preventive Services Taskforce recommends a routine colonoscopy starting at age 50 and repeated every 10 years. With a family history of colonic malignancy, the guidelines suggest earlier screening. However, this patient has no family history of colon cancer and is only 34 years of age. Thyroid function testing is incorrect. There is no indication at the present time for a thyroid function testing. Calcitionin may be measured to screen for early medullary cancer, but the thyroid hormones themselves are of no benefit. Reassurance is incorrect. With such a strong family history, reassuring the patient and sending him home would not be appropriate. Renal ultrasound to look for cysts is incorrect. It may be recommended if suspecting polycystic kidney disease, but there is no history suggestive PKD.

A 35-year-old woman presents with a 2-month history of palpitation and nervousness. She mentions that she always feels hot, even if the weather is cold. Her menses have been irregular lately. She has had no fevers recently. She was also told that her eyes are "weird looking". On examination, her blood pressure is 150/70 mmHg, her pulse rate 89 beats per minute. Her eyes show exophthalmos, and she also has lid lag on looking down. Thyrotoxicosis is suspected. What follows is the thyroid scan result. Question What is the most likely diagnosis? Answer Choices 1 Graves' disease 2 Toxic multinodular goiter 3 Hashimoto thyroiditis 4 Factitious hyperthyroidism 5 Toxic adenoma

Graves' disease Explanation The most likely diagnosis in this patient is Graves' disease. Thyrotoxicosis occurs due to thyroid hormone excess, the etiology of which varies from Graves' disease, toxic multinodular goiter, toxic adenoma, thyroiditis, and even functioning metastasis. Symptoms of thyrotoxicosis include hyperactivity, irritability, dysphoria, heat intolerance and sweating, palpitations, fatigue, weakness, weight loss with increased appetite, diarrhea, polyuria, and menstrual irregularities. Graves' disease is the most common cause of thyrotoxicosis. It is an autoimmune disease in which thyroid-stimulating autoantibodies stimulate thyroid receptors to secrete thyroxine. It is more common in women and can be identified by radionucleotide scan in which the whole thyroid shows increased uptake. Here the patient has all Graves' disease characters: she is middle aged with thyrotoxic characters and her Tc-99 scan shows the characteristic Graves' pattern. Toxic multinodular goiter is another cause of thyrotoxicosis in which there are multiple nodules that can be felt with palpation, and it is shown to be hot or active with the radionucleotide scan. It shows localized or patchy uptake in the scan pictures. Hashimoto thyroiditis is an inflammatory condition that follows viral infection. It is associated with a brief period of hyperthyroidism that is followed by hypothyroidism, which may persist throughout life. On Tc scan, it shows decreased uptake. Factitious hyperthyroidism is due to ingestion of the thyroid hormone. Usually this happens in nurses, physicians, or those who have access to medicine. It can be detected by measuring T3 and T4. There is also decreased uptake on thyroid scan. Toxic adenoma is a benign neoplastic nodule, which actively secretes thyroxine. It shows localized or patchy uptake of Tc 99 on thyroid scan.

Patients with a certain type of dwarfism (Laron dwarf) have normal or elevated levels of growth hormone; however, these patients still have the characteristics of dwarfs. What is the most likely explanation for their condition? Answer Choices 1 Growth hormone does not activate its receptor 2 Insulin-like growth factor receptor is defective 3 Somatostatin levels are very low 4 Insulin-like growth factor is overproduced 5 Insulin-like growth factor is rapidly degraded

Growth hormone does not activate its receptor Explanation Human growth hormone, also known as somatotropin, is a 191 amino acid polypeptide with a molecular weight of 22,000 and is the major hormone regulating growth in humans. It is synthesized as a 28,000-dalton precursor that is not biologically active. The gene for pre-growth hormone is found on chromosome 17. Growth hormone is part of a family of hormones that include prolactin and human chorionic somatomammotropin (human placental lactogen). Of the 3, only growth hormone has growth promoting activity. Growth hormone directly acts on cells via a receptor-signaling pathway to cause differentiation and the release of insulin-like growth factors (IGF's). The IGF's promote cell division. Growth hormone synthesis and secretion are regulated by somatostatin and growth hormone-releasing hormone (GHRH). GHRH is produced by the hypothalamus and acts on the anterior pituitary to promote growth hormone release. Somatostatin (growth hormone release-inhibiting hormone) inhibits growth hormone synthesis and secretion. Therefore, excess somatostatin could lead to decreased amounts of growth hormone. A deficiency in growth hormone causes a form of dwarfism. A 2nd type of dwarfism (Laron dwarf) is caused by defects in the growth hormone receptors. These patients have normal or elevated growth hormone levels but do not secrete IGF's due to the inability of growth hormone to properly signal an increase in IGF secretion. If these patients are administered IGF, growth stimulation is observed indicating the presence of functional IGF receptors.

A 55-year-old man with no significant past medical history presents for a routine evaluation and fasting bloodwork. He does not note any symptoms at this time. His physical examination reveals an obese body mass index with a waist circmference of 120 cm and a blood pressure of 140/90 mm Hg, but he is otherwise unremarkable. His fasting bloodwork is drawn. Question What laboratory finding would qualify a diagnosis of metabolic syndrome in this patient? Answer Choices 1 HDL value of 35 mg/dL 2 Total cholesterol of 230 mg/dL 3 Triglyceride value of 125 mg/dL 4 LDL measurement of 110 mg/dL 5 Fasting plasma glucose of 95 mg/dL

HDL value of 35 mg/dL Explanation The correct response is an HDL value of 35 mg/dl. According to the National Cholesterol Education Program and Adult Treatment Panel III, the diagnostic criteria of the metabolic syndrome include 3 or more of the following: • Central obesity: waist circumference >102 cm (M), >88 cm (F) • Hypertriglyceridemia: triglyceride level ≥150 mg/dL or specific medication • Low HDLcholesterol: <40 mg/dL in men and <50 mg/dL in women, or specific medication • Hypertension: blood pressure ≥130 mm Hg systolic or ≥85 mm Hg diastolic or specific medication • Fasting plasma glucose level ≥100 mg/dL or specific medication or previously diagnosed type II diabetes

A 72-year-old woman presents with a 2-week history of fever, cough, and excessive diuresis. The woman has diabetes mellitus, which is treated with glimepiride (Amaryl). Her fluid and food intake have been poor during this time as well. On physical examination, blood pressure is 98/58 mm Hg; pulse is 112/min; temperature is 100.6° F; and respirations are shallow and regular at 20/minute. On physical assessment, the patient is stuporous; skin and mucous membranes are dry; heart has a regular rate and rhythm without murmurs; and auscultation reveals rales in the left lung base. Question Based on this information, what is most likely causing the patient's condition? Answer Choices 1 Diabetic ketoacidosis 2 Hyperglycemic hyperosmolar state 3 Hypoglycemic coma 4 Lactic acidosis 5 Chronic renal failure

HHS Explanation The correct answer is a hyperglycemic hyperosmolar state (HHS). Most commonly seen in the elderly with type II diabetes mellitus, it is typically preceded by a few weeks of polyuria, weight loss, fatigue, and poor oral intake; this results in a change in mental status that can progress from confusion to lethargy, stupor, and coma. It is associated with dehydration and hyperosmolality that presents on physical examination with hypotension and tachycardia. It is usually precipitated by a serious infection (sepsis or pneumonia, which is most likely the case with this patient) or acute illness (stroke or myocardial infarction) in the postoperative period, or it is due to noncompliance. Lactic acidosis is an accumulation of lactate due to excess formation and decreased utilization. Although an increase in lactic acid may occur in HHS, it is not the cause of this patient's condition. It may be seen in patients with type II diabetes treated with glucophage (metformin) in renal insufficiency or when taken during periods of illness, when bedridden, or in conjunction with intravenous radiographic contrast. It is the most common cause of metabolic acidosis in hospitalized patients; it occurs with excess production (with shock due to poor perfusion), poor utilization (hepatocellular dysfunction), or decreased clearance of lactate (cirrhosis). The metabolic acidosis is accompanied by a characteristic, compensatory hyperpnea with long, deep breaths, which this patient does not exhibit. HHS results in a prerenal azotemia or acute renal failure, which is also a metabolic acidosis, but renal failure is not the underlying cause of this patient's condition. The fact that the patient has had polyuria over the past 2 weeks makes chronic renal failure unlikely. Hypoglycemia has an acute onset when blood glucose is <70 mg/dL, which triggers secretion of counter-regulatory hormones (glucagon, epinephrine, cortisol) and growth hormone to raise blood glucose levels. This response produces adrenergic symptoms such as palpitations, tremor, anxiety, hunger, or nausea. A blood glucose decrease to <45 - 50 mg/dL will result in neuroglycopenia, with symptoms of weakness, confusion, and combativeness, and if untreated, it may result in seizure, coma, or death. This patient had an insidious onset of symptoms that signify hyperglycemia, not hypoglycemia. Diabetic ketoacidosis is seen primarily in type I diabetics, and it is associated with nausea, vomiting, abdominal pain, acetone-odor breath, and Kussmaul breathing.

A 36-year-old woman has a past medical history of diabetes mellitus; she presents with weight gain and skin changes. Her review of systems is positive for menstrual irregularities with extended periods of amenorrhea, infertility, depression, cognitive dysfunction, and emotional lability. Lately, her fasting glucose levels have been above normal. Her physical exam notes increased adipose tissue in the face, upper back, and above the clavicles. Her skin reveals ecchymosis, telangiectasias, and purpura along her back and lower extremities; there is also facial acne and cutaneous atrophy. Her abdominal exam reveals the following image. Question What manifestation is expected in this patient? Answer Choices 1 A waist-to-hip ratio of less than 0.8 2 Increased proximal muscle strength 3 Hyperreflexia 4 Hypertension 5 Increased libido

HTN This patient is demonstrating signs and symptoms consistent with Cushing's syndrome. Hypertension (and possibly edema) may be present due to cortisol activation of the mineralocorticoid receptor, leading to sodium and water retention. Patients may have increased adipose tissue in the face (moon facies), upper back at the base of neck (buffalo hump), and above the clavicles (supraclavicular fat pads); there may be central obesity with increased adipose tissue in the mediastinum and peritoneum as well as an increased waist-to-hip ratio greater than 1 in men and 0.8 in women. Upon CT scan of the abdomen, increased visceral fat is evident. Proximal muscle weakness may be evident. Because of progressive proximal muscle weakness, patients may have difficulty climbing stairs, getting out of a low chair, and raising their arms. Signs of hypothyroidism, such as slow reflex relaxation, may occur from an anterior pituitary tumor whose size interferes with proper thyroid-releasing hormone (TRH) and thyroid-stimulating hormone (TSH) function. Similarly, other pituitary function may be interrupted. Menstrual irregularities, amenorrhea, infertility, and decreased libido may occur in women due to inhibition of pulsatile secretion of luteinizing hormone (LH) and follicle-stimulating hormone (FSH), which is likely due to interruption of luteinizing hormone-releasing hormone (LHRH) pulse generation.

The most common type of thyroiditis in the US is associated with increased circulating levels of antithyroid peroxidase or antithyroglobulin antibodies. The tissue biopsy is revealed in the image. With what condition is it most likely to be associated? Answer Choices 1 Hashimoto's thyroiditis 2 Riedel's Thyroiditis 3 Subacute thyroiditis 4 Suppurative thyroiditis 5 De Quervain's Thyroiditis

Hashimotos Explanation Hashimoto's thyroiditis is also known as chronic lymphocytic thyroiditis. It is the most common form of thyroiditis and probably any thyroid disorder in the US. The gland appears diffusely enlarged and firm. Increased circulating levels of antithyroid peroxidase or antithyroglobulin antibodies are a diagnostic feature. It tends to be familial, and it is 6 times more frequent among females. The image shows Hurtle cells with abundant pink cytoplasm and vesicular nuclei. The red arrow points to the lymphoid cell whereas the black arrow points to a hurtle cell. Riedel's thyroiditis is often a manifestation of a multifocal systemic fibrosis syndrome with a stony hard enlargement of the thyroid gland. It is the rarest form of thyroiditis. De Quervain's thyroiditis is also called subacute thyroiditis; it is an acute, painful enlargement of the thyroid gland. If there is no pain, it is called a silent thyroid. Suppurative thyroiditis is rare and associated with severe pain, tenderness, redness, and fluctuation in the region.

A 36-year-old woman with a past medical history of diabetes mellitus presents to her family medicine office with complaints of weight gain and skin changes. Her review of systems is positive for menstrual irregularities with extended periods of amenorrhea, infertility, depression, cognitive dysfunction, and emotional lability. Of late, her fasting glucose levels have been above normal. Her physical exam notes increased adipose tissue in the face, upper back, and above the clavicles. Her skin reveals ecchymoses, telangiectasias, and purpura along her back and lower extremities, facial acne, and cutaneous atrophy. Her abdominal exam reveals the following image. Question Which of these is correct regarding this patient's condition? Answer Choices 1 Strengthened bone and enhanced immune function are typical. 2 Bilateral adrenal hyperplasia is the leading cause of this presentation. 3 This illness is more commonly encountered in males. 4 In males with this condition, basal gonadotropin function is unaffected. 5 Headaches, polyuria, vision loss, or galactorrhea suggest a pituitary source.

Headaches, polyuria, vision loss, or galactorrhea suggest a pituitary source. This patient is exhibiting signs and symptoms consistent with Cushing's syndrome. Most cases of Cushing syndrome are due to exogenous glucocorticoids. Endogenous glucocorticoid overproduction or hypercortisolism that is independent of ACTH is usually due to a primary adrenocortical neoplasm (usually an adenoma but rarely a carcinoma). ACTH-secreting neoplasms cause ACTH-dependent Cushing syndrome. In 80% of cases, this is due to an anterior pituitary tumor and referred to as classic Cushing disease. Patients with an ACTH-producing pituitary tumor present with headaches, polyuria, nocturia, visual problems, or galactorrhea . Exposure to excess glucocorticoids results in multiple medical problems, including hypertension, obesity, osteoporosis, fractures, impaired immune function, impaired wound healing, glucose intolerance, and psychosis. Bilateral micronodular hyperplasia and macronodular hyperplasia are rare causes of Cushing syndrome. In men, inhibition of LHRH and FSH/LH function may lead to decreased libido and impotence.

A 16-year-old girl presents with papillary thyrocarcinoma. Your surgical plan is bilateral thyroidectomy. The procedure is uneventful. Histopathology indicates that the tumor was completely excised along with the thyroid. You schedule your patient for post-surgical I131 to destroy any remaining tumor cells and thyroid tissue. 2 days post-operatively, your patient has difficulty speaking and swallowing. While you are examining her, she begins to have laryngeal spasms, experiences difficulty breathing, and goes into respiratory arrest. What happened? Answer Choices 1 She had an allergic reaction to the I131 2 Perturbation of certain tumors can stimulate them to grow rapidly. The tumor probably spread to the respiratory centers of the brain 3 Lack of T3/T4 in the blood has slowed her metabolism causing laryngeal spasms 4 Her parathyroid glands were either removed during the procedure or severely damaged 5 Because most anesthetic agents are lipid soluble, she is probably reacting to anesthetic agents that were stored in her body fat and released as she began to breakdown her fat stores

Her parathyroid glands were either removed during the procedure or severely damaged Explanation Lack of parathyroid activity will result in rapid depression in serum concentrations of calcium. I131 does not have the characteristics of an allergen. Although some tumors may grow rapidly when manipulated, papillary thyrocarcinoma is very slow-growing and not likely to metastasize in 2 days. The half-life of T3/T4 in the body is approximately 6 weeks. Although some anesthetic agents may be stored in body fat, they are released in relatively small amounts.

A patient was recently diagnosed with type 1 diabetes mellitus. A treatment plan was initiated, with a combination regimen of insulin. Which of the following types of insulin works well with a rapidly acting insulin, such as insulin lispro, to provide 24-hour coverage for the patient? A NPH insulin B Regular insulin C Insulin aspart D Insulin glargine E Humalog 75/25

Insulin glargine The correct choice is D, insulin glargine. This is the only long acting insulin listed. The combination of a long acting insulin with a rapidly acting insulin provides physiologic insulin replacement to the patient. This regimen provides postprandial control after meals and basal coverage throughout the day and night. Choice A NPH insulin, can be used by itself in two or more injections throughout the day. Choice B, regular insulin, can be used instead of rapid acting insulin, and not in combination with it. Choice C, insulin aspart, is a type of rapidly acting insulin and would not be used in combination with another rapidly acting insulin. Choice E, Humalog 75/25, is a combination insulin preparation with 75% intermediate acting insulin and 25% insulin lispro.

Which of the following sets of lab values is most consistent with the diagnosis of Hashimoto's thyroiditis? A High serum TSH, low serum total T4, and high thyroidal peroxidase antibodies B Low serum TRH, low serum TSH, and low serum free T3 levels C Normal TSH, normal serum total T4, and normal radioactive iodine uptake D Low serum TSH, high serum total T4, and high thyroid stimulating antibodies E High serum TSH, high radioactive iodine uptake, and high serum free T3

High serum TSH, low serum total T4, and high thyroidal peroxidase antibodies The correct choice is A, high serum TSH, low serum total T 4 , and high thyroidal peroxidase antibodies. Hashimoto's thyroiditis is the most common cause of primary hypothyroidism and is autoimmune in nature. The serum thyroid hormone levels are low, secondary to the destruction occurring in the thyroid gland. The negative feedback loop causes the pituitary to respond by increasing production and secretion of TSH. Thyroidal peroxidase, thyroglobulin, and TSH receptor blocking autoantibodies can be found in these patients. Choice B, low serum TRH, low serum TSH, and low serum free T 3 levels, is seen in patients with secondary hypothyroidism relating to pathology in the hypothalamus. Choice C, normal TSH, normal serum total T 4 , and normal radioactive iodine uptake, is seen in patients who are euthyroid. Choice D, low serum TSH, high serum total T 4 , and high thyroid stimulating antibodies, are findings in patients with primary hyperthyroidism (e.g. Graves' disease). Choice E, high serum TSH, high radioactive iodine uptake, and high serum free T 3 can be seen in patients with secondary hyperthyroidism, as a result of anterior pituitary pathology.

A 29-year-old woman presents with a 3-month history of excessive fatigue and weakness. Hyperpigmentation is noted over her knuckles, elbows, and knees. Her labs are significant for mild leukocytosis, hyponatremia, and hyperkalemia. There is no elevation of cortisol on the cosyntropin stimulation test. Question What medication is recommended in this patient? Answer Choices 1 Levothyroxine 2 Hydrocortisone 3 Propylthiouracil 4 Octreotide 5 Progesterone

Hydrocortisone Explanation Hydrocortisone is correct. The patient's history and her lack of cortisol elevation on the cosyntropin stimulation test indicate Addison's disease or primary adrenal insufficiency. Addison's disease is caused by adrenal dysfunction, most commonly from autoimmune disease. The treatment of Addison's disease requires cortisosteroid replacement therapy. Hydrocortisone is the treatment of choice. Levothyroxine is incorrect. Levothyroxine is used in the treatment of hypothyroid. Propylthiouracil is incorrect. Propylthiouracil is used in the treatment of hyperthyroid. Octreotide is incorrect. Octreotide is not used in the treatment of Addison's disease. Octreotide is used in the treatment of intestinal tumors, carcinoid, tumors, and acromegaly. Progesterone is incorrect. Progesterone is used for many conditions, including infertility and amenorrhea. It is not indicated in the treatment of Addison's disease.

A 73-year-old frail-appearing woman is brought in by her daughter who is concerned about her mother's increasingly poor memory. The mother complains of fatigue and weakness so profound "her bones hurt" as well as chronic constipation. She is being treated for depression and osteoporosis, but she is otherwise in good health. Given the lab values, what is the most likely diagnosis? Labs WBC 5.3 thous/ml CBC 4.2 mill/mcl Hct 35% Hemoglobin 12.1 g/dl Glucose 80 mg/dl BUN 15 mg/dl Creatinine 0.6 mg/dl Sodium 140 mEq/L Potassium 4.2 mq/L Chloride 148 mEq/L Phosphate 2.0 mg/dl Calcium 12 mg/dl Alkaline Phosphatase 25 U/L Answer Choices 1 Bone metastasis 2 Multiple myeloma 3 Hyperparathyroidism 4 Hyperthyroidism 5 Addison's disease

Hyperparathyroidism Explanation Painful bones, renal stones, abdominal groans, and psychic moans are the sine qui non-symptoms of hyperparathyroidism. In this case, the arthralgias and myalgias; constipation and depression along with hypercalcemia; and elevated and low serum phosphate levels substantiate the diagnosis. Bone metastasis, multiple myeloma, and some hematological malignancies will present with hypercalcemia, but with an elevated alkaline phosphatase and hematological abnormalities. Addison's disease presents with hyponatremia, hyperkalemia, and sometimes hypoglycemia as a result of a deficiency in the secretion of adrenocortical hormones.

A 25-year-old woman presents because of changes in her eyes, heat sensitivity, constant sweating, and nervousness. She has lost 15 pounds recently. Her doctor suspects that she has Graves' disease. She asks her doctor why her thyroid is enlarged. What is the correct term for the underlying phenomenon in her illness? Answer Choices 1 Hyperplasia 2 Hypertrophy 3 Metaplasia 4 Dysplasia 5 Anaplasia

Hyperplasia Hyperplasia is an increase in the number of cells in the tissue. Hypertrophy is an increase in the organ size. Hypertrophy is not due to an increase in the number of cells; rather, it is due to an increase in the size of the cells. Metaplasia is the replacement of one adult tissue type with another adult tissue type. Dysplasia is a disorderly arrangement to the tissue. Anaplasia is characteristic of malignancy. It refers to cancerous cells being undifferentiated.

A 47-year-old woman presents because she feels "weak and dizzy". She was feeling well up until about an hour ago, at which point she began to feel as if she were going to "pass out." Her husband states that she appeared nervous and confused; she was also trembling. When he asked her what was wrong, she did not seem to understand him. However, she did not seem to lose consciousness. She had been healthy, and she has no significant past medical history. On examination, she is found to have tachycardia and generalized weakness. She is conscious, but slightly disoriented. She is afebrile. Her laboratory tests are normal with the exception of a hemoglobin level of 10.4g/dL and a blood glucose level of 47 mg/dL. The patient receives treatment and improves immediately. Question What is the most likely cause of this patient's symptoms? Answer Choices 1 Hypoglycemia 2 Transient ischemic attack 3 Anxiety disorder 4 Sepsis 5 Seizure disorder

Hypoglycemia Explanation Hypoglycemia results from low blood glucose. The symptoms of hypoglycemia can vary depending on the glucose level. It also varies from person to person. Symptoms can include weakness, pallor, hunger, palpitations, sweating, nervousness, and trembling. These symptoms usually resolve when the glucose levels return to normal. Anyone who has experienced an episode of hypoglycemia describes a sense of urgency to eat and resolve the symptoms. The symptoms are usually easily recognized by patients. If the patient does not eat or take an agent to increase their blood glucose level, the brain will eventually begin to suffer. The patient can develop drowsiness, confusion, behavioral changes, seizures, and coma. Causes of hypoglycemia include insulin resistance in patients who are pre-diabetic, insulin producing tumors, and medications. True hypoglycemia is found in patients with diabetes. Patients who are pre-diabetic may have low blood glucose if they are fasting and their insulin level is high. Diagnosis is usually made through history and physical and is confirmed with laboratory measurements. Plasma insulin, plasma C-peptide, and serum glucose should be checked. Treatment consists of recognizing the symptoms and resolving the hypoglycemia. The patient's symptoms can usually be resolved with food and drink containing sugar. If the patient does not improve after consumption of sugar, he or she should seek medical care for further evaluation. The other choices are incorrect since the symptoms and laboratory results do not match. The patient's labs are normal with the exception of low hemoglobin and blood glucose levels. Transient ischemic attack is an acute episode stroke that last usually less than 24 hours. It can be thought of as a stroke in which the symptoms resolve. It is caused by a temporary occlusion of a blood vessel. Symptoms are not affected by food or drink. Anxiety disorders can result in excessive worry and the feeling of being overwhelmed. Lab results, such as blood glucose levels, would not be affected. Sepsis results from a bacterial infection that has spread to the blood. Patients may have fever, chills, tachycardia, decreased urination, confusion, dizziness, and an elevated white cell count. This patient's white blood cell count was normal. Depending on the type of seizure, seizure disorders can produce disorientation, uncontrollable movements, or staring into space. Unlike symptoms of hypoglycemia, they do not end when the diet is changed.

You are evaluating a 69-year-old female who complains of an intermittent sensation of hot flashes, flushing of her face/chest, and pruritus after starting a new medication for her cholesterol. Which of the following medications is the most likely cause of her symptoms? A niacin B lovastatin C gemfibrozil D ezetimibe E fenofibrate

Niacin The correct answer is (A). Niacin has a characteristic side effect of hot flashes, flushing, and pruritus. These symptoms can be reduced by addition of ASA or a nonsteroidal anti-inflammatory drug (NSAID) if there are no contraindications. The other choices are unlikely to cause this combination of symptoms.

A 21-year-old insulin-dependent diabetic college student is acutely agitated and is verbalizing expletives. Prior to admitting him to an acute psychiatry inpatient service and administering intra muscular chlorpromazine (Thorazine) or haloperidol (Haldol), for what should he be evaluated? Answer Choices 1 Hyponatremia 2 Hypokalemia 3 Hypocalcemia 4 Hypovolemia 5 Hypoglycemia 6 Hypomagnesemia 7 Hypothermia

Hypoglycemia Explanation Hypoglycemia, reduced blood glucose level below the laboratory reference ranges, is most likely to present with a toxic-metabolic state emulating an acute agitation and/or an acute psychosis; this patient demonstrates these symptoms. Given the known history of diabetes, and even in the absence of a history of a insulin-dependent diabetes disorder, the health care provider should check the patient's blood glucose level; if found to be low, glucose should be administered intravenously. Hyponatremia, which is reduced serum sodium below the laboratory references ranges, may present with seizures; however, none are evident in this case. Hypokalemia, which is reduced serum potassium below the laboratory reference ranges, may present with muscular weakness; however, that is not a listed symptom. Hypocalcemia, which is reduced serum calcium below the laboratory reference ranges, may present with neuromuscular hyperirritability; it is not mentioned in this case. Hypovolemia, which is reduced circulating volume, may present with cardiovascular collapse verging on shock. This patient is not in such a condition. Hypomagnesemia, which commonly results from poor nutrition, would be unlikely in this setting. Hypothermia, which is reduced body temperature, may present with slow mentation; it is not a condition pertinent to this case. The core ethical and legal principle is to be reasonably certain that all efforts have been provided. Afterwards, it is important to identify, diagnose, treat, and correct all acute medical issues and crises before simply providing injections of intra-muscular, anti-agitation, and/or anti-psychotic medications that necessarily carry an element of risk and are of no benefit in this setting.

Release of which of the following substances is triggered by pituitary growth hormone and promotes growth of other tissues in the body? A C-pepide B IL-I C IGF-I D Thyroxine E Catecholamines

IGF-I The correct choice is C, IGF-I or insulin like growth factor I. This growth factor leads to increased DNA, RNA, and protein synthesis, which leads to overgrowth of bone, soft tissue, and cartilage. Choice A, c-peptide, is a part of the prohormone of insulin. Choice B, IL-I or interleukin I, is an important cytokine that promotes cell activation. Choice D, thyroxine, potentiates the actions of growth hormone on tissues.

A 50-year-old man with a history of myocardial infarction (MI) is on aspirin, atenolol, and atorvastatin; he presents with pain in the knee joints. He is a diabetic controlled by diet and exercise. He is diagnosed with osteoarthritis, and he is prescribed ibuprofen. Question What is a cause for concern with regard to the ibuprofen prescription for this patient? Answer Choices 1 Ibuprofen increases the risk of hyperglycemia 2 His age is inappropriate to prescribe ibuprofen 3 Ibuprofen reduces aspirin's cardioprotective benefits 4 Ibuprofen decreases atorvastatin's hypocholesterolemic effects 5 Ibuprofen is ineffective in treating osteoarthritis

Ibuprofen reduces aspirin's cardioprotective benefits Explanation Ibuprofen is a nonsteroidal anti-inflammatory drug (NSAID), which when used concomitantly with aspirin, may reduce the latter's cardioprotective effect. Ibuprofen has been shown to inhibit the irreversible platelet inhibition brought about by aspirin. A clinically and statistically significant increase in the risk of mortality in persons using ibuprofen and aspirin, compared to those using ibuprofen alone, has been noted. It is recommended that ibuprofen not be prescribed for the long-term in patients on aspirin. Another strategy is to ingest aspirin 2 hours before ibuprofen. Ibuprofen has not been shown to cause hyperglycemia or interact with atorvastatin. The greater risk of gastrointestinal complications should be borne in mind while prescribing NSAIDS in persons aged 65 years or more. Ibuprofen is 1 of the NSAIDs effective in treating osteoarthritis.

A 7-year-old boy presents with headaches, nausea, and lethargy. According to his parents, the boy has been consuming large amounts of fluids. He also urinates frequently. A previously healthy child, he has no significant past medical history. His symptoms began approximately 3 weeks ago. His parents thought that it was his increased activity level that led to his increased fluid intake. However, over the past few days, he has been becoming more lethargic and is still consuming large amounts of water and juice. On physical exam, the boy appears to be thin, ill-appearing, and lethargic. His eyes appear sunken and his skin is dry. His vital signs are as follows: blood pressure 100/54 mm of Hg, temperature 99.0 degrees Fahrenheit, pulse 120 beats per minute, and a respiratory rate of 22 per minute. Laboratory tests show dilute urine with a specific gravity of 1.002 and a urine osmolality of 199mOsm/kg. His plasma osmolality is 296mOsm/kg. Question Which of the following statements is true? Answer Choices 1 The genetic form of diabetes insipidus is rare and is usually in an autosomal recessive pattern 2 The drug of choice for treatment is chlorpropamide 3 Idiopathic DI is associated with destruction of cells in the hypothalamus, often as part of an autoimmune process 4 Patients with nephrogenic DI have low serum ADH levels 5 An individual with DI cannot normally drink enough fluid to keep up with fluid loss

Idiopathic DI is associated with destruction of cells in the hypothalamus, often as part of an autoimmune process Explanation Idiopathic DI is associated with destruction of cells in the hypothalamus, often as part of an autoimmune process. Lymphocytes infiltrate the posterior pituitary and the stalk. These infiltrates may show up on Magnetic Resonance Imaging (MRI) and help in the diagnosis. In addition, antibodies may form against vasopressin cells. These antibodies can also cause the development of the disease. The drug of choice for the treatment of DI is usually an analogue of vasopressin, such as desmopressin. There is a genetic form of DI that is rare and is inherited in an autosomal dominant pattern. Patients with nephrogenic DI have a normal-to-elevated serum ADH level, and the kidneys fail to respond to exogenous ADH. Under normal circumstances, an individual with DI is able to keep up with fluid loss by increasing their fluid intake. Stress, lack of availability, or illness may make this more difficult, and the signs of dehydration will become evident.

A patient is admitted to the hospital for an acute adrenal crisis. He has a history of chronic adrenal insufficiency and was admitted with severe weakness, nausea and vomiting while fighting a pulmonary infection. Which of the following suggested plans might this patient have forgotten or been unable to do? A Restrict fluid intake during times of metabolic stress B Increase the daily dose of hydrocortisone during times of metabolic stress C Hold the daily dose of hydrocortisone during times of metabolic stress D Add levothyroxine to the daily dose of hydrocortisone during times of metabolic stress E Increase the daily ingestion of proteins during times of metabolic stress

Increase the daily dose of hydrocortisone during times of metabolic stress The correct choice is B, increase the daily dose of hydrocortisone during times of metabolic stress. The cortisol dose should be increased to between 60 and 80 mg/day, to mimic the normal physiologic response of the body. Increased mineralocorticoid therapy is generally not required. Choice A, restrict fluid, would aggravate the potential for the development of dehydration in this patient. Choice C would lead to further cortisol depletion. Choices D and E would not benefit this patient.

A 34-year-old woman requests a prescription for Synthroid (levothyroxine) 125 mcg, which she takes 1 X day. The patient has no medical records, but you are able to ascertain she underwent previous transsphenoidal surgery for an enlarged pituitary gland. She was subsequently placed on thyroid hormone replacement medication and advised she must remain on this as a life-long medication routine. She admits to some fatigue and difficulty concentrating; otherwise, she is in her usual state of health. Further review of syptoms is unremarkable. Physical exam reveals coarse hair, patchy areas of dry skin, no periorbital edema or lid lag, no thyroid enlargement, and ankle reflexes +1/4. The remainder of the physical examination is within normal limits. Subsequent laboratory studies are as follows: Result Reference range TSH 0.25 0.40 - 4.5 mU/L Free T4 0.6 0.8 - 1.8 ng/dL Question What is the best therapeutic intervention? Answer Choices 1 Increase the dose of Synthroid (levothyroxine) 2 Decrease the dose of Synthroid (levothyroxine) 3 Continue the current dose of Synthroid (levothyroxine) 4 Discontinue Synthroid and initiate treatment with Tapazole (methimazole) 5 Obtain a thyroid Iodine131 uptake and scan

Increase the dose of Synthroid (levothyroxine) Explanation The correct choice is to increase the dose of Synthroid (levothyroxine) for the treatment of secondary hypothyroidism. TSH secretion is controlled by hypothalamic thyrotropin-releasing hormone (TRH) and negative feedback from circulating free thyroid hormones FT4 and FT3. It is the most important test in the diagnosis of thyroid disorders. However, in this patient (who previously underwent surgical resection of the pituitary), postoperative TSH secretion was found to be insufficient; this resulted in hypothyroidism secondary to pituitary dysfunction, thus requiring life-long thyroxine replacement. In patients with secondary hypothyroidism, the TSH levels remain low or inappropriately normal (for the circulating thyroid hormone levels), so the dosage of thyroxine is titrated to maintain circulating thyroid hormone levels within the normal range. Since the patient has a low serum free T4, she is hypothyroid; therefore, it is inappropriate to decrease, discontinue, or continue her current dose of Synthroid. Based upon her history, the low serum TSH is the result of surgical intervention, not indicative of hyperthyroidism, so Tapazole and an I131 uptake and scan are not indicated. It would also be prudent to obtain past medical records, including post-operative and recent laboratory results, to ensure the remainder of the pituitary hormone levels are within normal limits. Levels that should be looked at include prolactin (PRL), follicle stimulating hormone (FSH), luteinizing hormone (LH), human growth hormone (HGH), and adrenocorticotropic hormone (ACTH).

A 43-year-old Caucasian woman presents for evaluation of multiple concerns, including: a 6-month history of menstrual irregularities; weight gain with increased abdominal girth; easy bruising without a history of trauma; new "stretch marks" on her torso; mood swings; decreased libido, and weakness in her arms and legs. She had previously regular menses, a bilateral tubal ligation (BTL), and she reports multiple negative home pregnancy tests. She denies hot flashes, night sweats, and frank depression. She denies any health changes, medications, or stressors in relation to these changes. The patient is frustrated that her health fair labs (which include complete blood count, complete metabolic panel, lipid panel and thyroid stimulating hormone) were all normal, and they do not explain her symptoms. Her past medical history is unremarkable, with no chronic conditions and no medication use; her only surgery was the BTL. She lives with her husband and 3 children. She works in retail, and she walks for exercise. She denies use of tobacco, alcohol, and drugs. Her blood pressure is 154/92. Chart review demonstrates weight gain of 15 pounds over 6 months, with normal blood pressures in the past. On physical exam, you observe an overweight woman with an especially rounded, full face. She has a fatty fullness to her neck and some central obesity, but her arms show some muscle wasting. Purple striae are noted on her torso. Her skin also appears thinned, with multiple bruises. Hirsutism is observed on the patient's chin, abdomen, and breasts. The remainder of her exam is unremarkable. Question What would best explain the pathophysiologic basis for this patient's condition? Answer Choices 1 Autoimmune destruction of adrenal glands, causing decreased cortisol production 2 Exogenous administration of corticosteroids, causing increased adrenal production of cortisol 3 Hypoplastic adrenal cortex, causing low serum levels of cortisol 4 Increased adrenocorticotropic hormone (ACTH) secretion from pituitary, causing increased adrenal production of cortisol 5 Low adrenal production of cortisol, causing increased renal retention of cortisol

Increased adrenocorticotropic hormone (ACTH) secretion from pituitary, causing increased adrenal production of cortisol Explanation This patient presents with a multitude of signs and symptoms (weight gain, menstrual irregularity, mood swings, poor libido, moon facies, fat depositions, proximal limb weakness, central obesity, skin thinning, purple striae, and easy bruising) that support a diagnosis of Cushing syndrome. The most common cause of Cushing syndrome is considered iatrogenic, from administering exogenous steroids, such as prednisone. The next most common cause is known as Cushing disease and results from increased adrenocorticotropic hormone (ACTH) secretion from pituitary, causing increased adrenal production of cortisol. This patient should be evaluated with laboratory tests, and then imaging, to look for a pituitary adenoma. Autoimmune destruction of adrenal glands, causing decreased cortisol production, is the most common mechanism for development of Addison's disease. Addison's disease also has multiple, somewhat nonspecific signs and symptoms, but they differ from those of excess cortisol production. A patient with Addison's may have fatigue, weakness, hypotension, hyperkalemia and hyponatremia, hyperpigmentation of the skin, anorexia, nausea, vomiting, diarrhea, and weight loss. As indicated above, exogenous administration of corticosteroids is the most common cause of iatrogenic Cushing syndrome. However, the effect of administering corticosteroids is suppression of (not increased) adrenal cortisol secretion. This is also the rationale for tapering off prescribed corticosteroids, in order to allow the patient's natural adrenal production of cortisol to return to normal. Otherwise, an Addison's-type situation occurs with sudden discontinuation of high-dose steroids. Hypoplastic adrenal cortices would be unable to produce normal cortisol levels, thus causing low serum cortisol levels. Congenital adrenal hypoplasia is another (less common) cause of Addison's disease. Renal retention, or elimination, of cortisol is not a mechanism for controlling cortisol levels in the body. Instead, corticotropin-releasing hormone (CRH) from the hypothalamus and ACTH from the pituitary gland, and to a lesser degree, arginine vasopressin (AVP), regulate physiologic cortisol levels. It would be expected that when low adrenal production of cortisol occurs, there would be an increase in CRH and ACTH in order to stimulate more adrenal production of cortisol.

A 46-year-old woman presents for a follow-up evaluation for her total thyroidectomy; it was performed 1 year ago for Stage II differentiated mixed papillary and follicular carcinoma. Laboratory tests were performed. What change in levels may suggest the recurrence of thyroid malignancy in this patient? Answer Choices 1 Increasing serum thyroglobulin levels 2 Decreasing serum thyroglobulin levels 3 Increasing T4 levels 4 Increasing TSH levels 5 Decreasing TSH levels

Increasing serum thyroglobulin levels Explanation The correct answer is increasing thyroglobulin levels. Detectable thyroglobulin levels may be found in patients with incomplete thyroidectomies and those thyroid remnant ablations with radioactive iodine. While these levels do not necessarily indicate metastatic or residual thyroid cancer, increasing levels over time in the absence of serum anti-thyroglobulin antibodies have been associated with recurrent thyroid malignancy. The presence of anti-thyroglobulin antibodies may invalidate the test results by interfering with the assay. Baseline and stimulated thyroglobulin levels that are greater than or equal to 2 ng/mL need follow up with neck ultrasound and possibly other scanning. Rising thyroglobulin levels are at high risk for occult thyroid cancer metastases. Other scanning may be used to detect this possibility. The other responses: decreasing thyroglobulin levels, increasing, or decreasing T4 and TSH levels are not necessarily associated with recurrent thyroid malignancy.

A 38-year-old Caucasian man with a long-standing diagnosis of acromegaly has come to you with a concern regarding his chance of developing colon cancer. He had read some articles on the internet suggesting that patients with the diagnosis of acromegaly have an increased incidence of developing colon cancer, and this has caused him tremendous worry. The patient has no known personal or family history of diverticular disease, colon polyps, or colon cancer. He had a successful selective trans- sphenoidal surgical resection of a pituitary micro adenoma over 20 years ago and has had routine periodic serum insulin-like growth factor-1(IGF-1) levels drawn that have always been within normal range. Question Based on the information above, what would be the recommendation for performing a colonoscopy in this patient at this time? Answer Choices 1 Initial colonoscopy now and every 5 years thereafter 2 Initial colonoscopy at 40 years old and every 5 years thereafter 3 Initial colonoscopy at 40 years and every 3 years thereafter 4 Initial colonoscopy at 50 years and every 5 years thereafter 5 Initial colonoscopy at 50 years and every 3 years thereafter

Initial colonoscopy at 50 years and every 3 years thereafter Explanation There is conflicting evidence indicating that patients with the diagnosis of acromegaly have an increased risk of developing colon cancer. There is a higher incidence of colon cancer mortality in patients who have a history of uncontrolled growth hormone or IGF1 levels. The key risk factors in this patient population that increase one's chance include the patient being over 50 years old and male, having 3 or more skin tags, having a personal history of colon polyps, and having a family history of colon cancer. The patient in the above scenario has only 1 of these risk factors, as he is male. The current recommendation for screening follows the 2009 guidelines from the Acromegaly Consensus Group, which recommends the following: baseline colonoscopy at the age of 50 and then every 3 years thereafter. This should be strictly followed, especially if the patient has a history of poorly controlled growth hormone levels.

A 48-year-old man presents to the family practice clinic for evaluation of fatigue, weakness, and nausea. He reports that his symptoms have progressively worsened over the last 6 - 8 months. He reports fatigue despite adequate sleep; he is also experiencing an overall feeling of muscle weakness, nausea with occasional vomiting, a weight loss of about 12 pounds, headaches, and muscle aches. His wife thinks he appears tanned year-round, despite a lack of sun exposure. He admits feeling anxious and somewhat irritable, but he denies any major psychosocial or traumatic events surrounding onset of symptoms. Prior to the onset of his symptoms, he was healthy and active. His past medical history reveals no chronic medical conditions, no medication use, and no history of surgery; he does not have any allergies. His family history is significant for thyroid disease in a sister and his mother; there is also a history of diabetes mellitus type I in a brother. He teaches high school, and he lives with his wife and children; he denies the use of tobacco, alcohol, and drugs. On physical exam, he is noted to be hypotensive and hyperpigmented. The remainder of his physical exam is normal. Several labs are performed, and the results are as follows: Urinalysis- Normal Complete blood count- Mildly decreased hemoglobin and hematocrit Comprehensive metabolic panel- Mildly decreased sodium and elevated potassium, rest normal Adrenocorticotropic hormone (ACTH)- Elevated Cortisol (morning level)- Decreased ACTH stimulation test- Decreased cortisol Question Once this patient is diagnosed, what choice represents the most important intervention for his presumed condition? Answer Choices 1 Increase dietary sodium 2 Initiation of antidepressant with anti-anxiety properties 3 Initiation of steroids 4 Refer to oncologist 5 Restrict dietary potassium intake

Initiate Steroids Explanation This patient is presenting with a corticoadrenal insufficiency, which is also known as Addison's disease. In the United States, this rare disease is most often linked with autoimmune causes. The affected patient will have a variety of symptoms related to low glucocorticoid and mineralocorticoid production. Signs and symptoms include fatigue, nausea, vomiting, headaches, anorexia, myalgias, arthralgias, muscle weakness, anxiety, mental irritability, hyperpigmentation, hypotension, hyponatremia, hyperkalemia, anemia, and many more. The mainstay of treatment is initiation of steroids, often with oral hydrocortisone, unless urgent IV treatment is needed. Patients with Addison's disease commonly have hyponatremia and hyperkalemia. They can be instructed to increase dietary sodium, especially in times of exercise and hot weather. However, this alone will not address the underlying adrenal insufficiency, and failure to initiate steroid treatment will risk patient deterioration. With a variety of somatic and psychiatry complaints, a patient presenting like this may be falsely given a psychiatric diagnosis, and he may be treated with initiation of an antidepressant with anti-anxiety properties; however, the anxiety and irritability are symptoms of the adrenal disorder, and they should improve with treatment of the underlying disorder. It would be reasonable to monitor this patient and possibly treat him with antidepressants if his symptoms do not resolve. While Addison's disease is serious and chronic, it is not malignant. This patient's presentation with weight loss, nausea, and fatigue could be suggestive of cancer. However, with the laboratory values indicating adrenal insufficiency, it would be inappropriate to refer the patient to an oncologist. Even as a mild hyperkalemia is indicated on this patient's lab values, restriction of dietary potassium is not an appropriate treatment of his primary condition. The hyperkalemia is a result of the adrenal insufficiency. With appropriate treatment with steroids, such as hydrocortisone, the electrolyte abnormalities are likely to improve without additional intervention.

A 32-year-old Caucasian woman has a past medical history of Hashimoto's thyroiditis, type I diabetes mellitus, and pernicious anemia; she presents with a 2-year history of insidious and intermittent fatigue, anorexia, involuntary weight loss, nausea, abdominal pain, vomiting, and dizziness that is associated with position changes. Her physical exam is noteworthy for postural hypotension, with a maximum systolic blood pressure of 104 in the supine position. Additionally, she has a low-grade fever and a generalized pigment change to her skin, as indicated in the image below. Question What health maintenance recommendation should be provided? Answer Choices 1 Periodic DEXA scans are useful in detecting steroid-induced increased bone density 2 Close clinical monitoring during treatment is unnecessary 3 A decrease in salt intake is necessary in hot weather 4 A reduction of steroid replacement doses is needed in stressful situations 5 Instructions on self-administration of IM hydrocortisone on an as-needed basis

Instructions on self-administration of IM hydrocortisone on an as-needed basis Explanation This patient's most likely diagnosis is primary adrenal insufficiency, which is known as Addison's disease. Treatment involves replacement of corticosteroids and mineralocorticoids. Patients should be instructed on how to give themselves IM injections. They should be given a prescription for parenteral hydrocortisone for use on occasions when oral intake may not be possible or when marked vomiting or diarrhea occurs. A periodic bone dual-energy radiographic absorptiometry scan may be useful in detecting early reductions of bone density and osteoporosis in patients who are inadvertently over-replaced with maintenance steroids. Patients on steroid replacement therapy need to be closely monitored by their primary care physician and by an endocrinologist. Close monitoring for any signs of inadequate replacement (e.g., morning headaches, weakness, and dizziness) and any signs of over-replacement (e.g., cushingoid features) should prompt appropriate dosage adjustment. Patients may need to be advised to increase salt intake in hot weather. Patients should be instructed to double or triple their steroid replacement doses in stressful situations, such as a common cold or tooth extraction.

A 51-year-old woman presents with a long history of blurry vision, palpitations, and weakness. Her lab results are as follows: TEST RESULTS REFERENCE RANGE Calcium 8.9 mg/dL 8.4-10.2 mg/dL Potassium 3.8 mEq/L 3.5-5.0 mEq/L Sodium 144 mEq/L 135-145 mEq/L Glucose 41 mg/dL 70-110 mg/dL You suspect a pancreatic islet cell tumor. What is the most likely diagnosis? Answer Choices 1 VIPoma 2 Insulinoma 3 Gastrinoma 4 Glucagonoma 5 Somatostatinoma

Insulinoma Her low blood glucose levels indicate an insulinoma. The endocrine portion of the pancreas is housed in the islets of Langerhans. Tumors that arise from this area are referred to as islet cell tumors. The specific type of tumor depends on the cell type involved. Typically, the tumor is named based on the hormone it is secreting. An insulinoma is an islet cell tumor. Insulin is produced by the beta cells of the islets of Langerhans. Therefore, an insulinoma is a tumor of the beta cells. An insulinoma is the most frequently occurring islet cell tumor. Hypoglycemia, secondary to the insulin secretion, is a symptom. Verner-Morrison syndrome is due to a VIPoma. This is an endocrine tumor of the pancreatic islets of Langerhans. It secretes vasoactive intestinal polypeptide. Vasoactive intestinal polypeptide is produced by the D1 cells of the islets of Langerhans. Therefore, a VIPoma is a tumor of the D1 cells. Other names for a VIPoma include pancreatic cholera and WDHA-syndrome (watery diarrhea, hypokalemia and achlorhydria). Gastrinomas are most often due to islet cell tumors. On occasion, gastrinomas can arise outside of the pancreas. Peptic ulcers, secondary to the excessive gastrin secretion, are frequent. Gastrinomas and peptic ulcerations are referred to as the Zollinger-Ellison syndrome. A glucagonoma is an islet cell tumor. Glucagon is produced by the alpha cells of the islets of Langerhans. Therefore, a glucagonoma is a tumor of the alpha cells. Diabetes can be seen with a glucagonoma. Other symptoms include a skin rash, weight loss, and anemia. A glucagonoma is a rare islet cell tumor. A somatostatinoma is an islet cell tumor. Somatostatin is produced by the delta cells of the islets of Langerhans. Therefore, a somatostatinoma is a tumor of the delta cells. Somatostatinoma is a rare islet cell tumor.

An 8-year-old boy presents with acute abdominal pain. On examination, there is mild hepatosplenomegaly. Biochemical investigations reveal elevated serum amylase and lipase levels. The plasma sample of the patient shows a thick creamy layer when kept overnight at 4 C. Fasting serum triglycerides level are 2200mg/dl. There have been similar complaints of abdominal pain in the past 4-5 years. The Apo CII levels were normal. What is the most likely diagnosis? Answer Choices 1 Abetalipoproteinemia 2 Apo CII deficiency 3 Familial combined hyperlipidemia 4 Familial hypercholesterolemia 5 Familial type III hyperlipoproteinemia 6 Lipoprotein lipase deficiency

Lipoprotein lipase deficiency Lipoprotein lipase deficiency is characterized by a defect in the degradation of chylomicrons. Lipoprotein lipase is required for the catabolism of triacylglycerol present in chylomicron to free fatty acids and glycerol. Lipoprotein lipase is present in the endothelium of blood vessels of the adipose tissue and muscle. A high serum triglyceride level characterizes this disorder. The serum triglyceride level is usually >1500mg/dl. The serum cholesterol is marginally raised. Lipoprotein lipase may be released from the tissues by administration of intravenous heparin. Diagnosis is confirmed by measuring lipoprotein lipase using specific immunologic techniques. Serum triglycerides levels are markedly elevated (>1500mg/dl). The most frequent presentation is as recurrent abdominal pain accompanied by hepatosplenomegaly. In many cases, patients present with acute pancreatitis. Observation of plasma, after keeping it overnight at 4 C, shows the presence of a creamy layer on the plasma. Management involves the administration of medium-chain triglycerides in the diet that enter the portal circulation directly without formation of chylomicrons. Familial hypercholesterolemia (FH) is an autosomal dominant genetic defect in which there is deficiency of functional LDL receptors. These LDL receptors recognize the Apo B100 present in IDL and LDL. LDL/IDL binds to the receptor and is taken up into the cell by receptor-mediated endocytosis. In FH, there is decreased peripheral uptake of LDL and IDL, which causes a marked increase in the serum cholesterol level. Serum triglycerides are within normal limits. There is a large accumulation of cholesterol in the macrophages of the tendons that causes xanthomas, indicating that the scavenger cells are taking up and degrading large amounts of LDL. Patients manifest with an increased risk of atherosclerosis and coronary artery disease at an early age. There is cholesterol accumulation in the arteries, especially at the root of the aorta, causing aortic stenosis. Apo C II is the apolipoprotein present in chylomicrons that is required for the activation of lipoprotein lipase. Absence of Apo CII is inherited as a recessive disorder. It is characterized by high levels of chylomicrons and serum triglycerides. It is distinguished from lipoprotein lipase deficiency by the assay of lipoprotein lipase after the administration of intravenous heparin. In this disorder, lipoprotein lipase levels are normal; however, Apo CII levels are low. Symptoms of presentation and management of the condition are similar to lipoprotein lipase deficiency. Familial combined hyperlipidemia (Frederick son's type IIB hyperlipoproteinemia) is characterized by an increase in LDL and VLDL. In these patients, there is an increased fasting serum cholesterol and serum triglycerides level. There is an increased risk of coronary artery disease in these patients. Serum cholesterol level is usually in the range of 200-300 mg/dl and serum triglyceride is in the range of 200-500mg/dl. Abetalipoproteinemia is characterized by an absence of lipoprotein species containing apoprotein B, which are LDL, chylomicrons, and VLDL. The levels of triglycerides and total cholesterol are very low (<50mg/dl). The predominant features include malabsorption of dietary fat and steatorrhea. There is also malabsorption of the fat-soluble vitamins. Lack of vitamin E causes progressive degeneration of the CNS, which manifests as neurological abnormalities by the first decade. There is decreased visual acuity and night blindness as a result of vitamin A deficiency. Familial type III hyperlipoproteinemia (dysbetalipoproteinemia) is characterized by increased levels of IDL in circulation. There is also an increase in the chylomicron remnant particles. There is a defect in apolipoprotein E, which is required for the uptake of chylomicron remnants and IDL by the liver. In these individuals, the fasting serum cholesterol and triglycerides are raised. Patients present with an increased risk of premature atherosclerosis and palmar xanthomas.

A 55-year-old woman with a 15-year history of type II diabetes presents for follow-up of her diabetes. Her spot albumin/creatinine ratio was 100 mg/g 4 months ago and was confirmed at 100 mg/g yesterday. Her urinary analysis shows no cells, casts, or blood. Her creatinine is 0.7mg/dl, and her estimated glomerular filtration rate is 95 ml/min/1.73m2. Question What medication(s) should you prescribe to help prevent her progression from micro to macroalbuminuria and to help prevent progressive decline in glomerular filtration rate? Answer Choices 1 Potassium chloride 2 Calcium carbonate 3 Lisinopril 4 Sodium bicarbonate 5 Calcium citrate

Lisinopril Explanation Lisinopril, an ACE Inhibitor, should help decrease albuminuria, prevent progression of diabetic kidney disease from micro to macroalbuminuria, and prevent a decline in glomerular filtration rate. This class of medications has been studied extensively for these purposes. Angiotensin II receptor blockers (e.g., irbesartan) may also reduce urinary albumin to normal levels. Monotherapy with either of these classes of medications should be attempted first in patients with microalbuminuria. This will test tolerance, effectiveness, and adverse reaction such as hyperkalemia. For patients with greater degrees of albuminuria (e.g., 1 g/day), poor response to monotherapy and blood pressure control, and no hyperkalemia associated with therapy, combination therapy should be considered. Combination therapy with both ACE inhibitors (e.g., lisinopril) and angiotensin II receptor blockers (e.g., irbesartan) is used to treat both diabetic and non-diabetic kidney disease. These medications act on different parts of the renin angiotensin system. In combination, irbesartan could block the effect of angiotensin produced by non-ACE pathways and lisinopril could block the production of angiotensin stimulated by irbesartan in a negative feedback system; however, combination therapy is usually preceded by monotherapy. Although combination therapy is currently being used in both diabetic and non-diabetic kidney disease, this therapy still being researched. It is unknown whether monotherapy alone is sufficient. It does not appear to be sufficient in all patients, particularly those with persistent micro and macroalbuminuria despite monotherapy. Sodium bicarbonate is used to treat the metabolic acidosis that commonly occurs secondary to chronic kidney disease. Diseased nephrons eliminate acids poorly, allowing them to build up in the blood stream. Sodium bicarbonate buffers these acids. Sodium bicarbonate is not known to significantly alter progression of proteinuria or decline in glomerular filtration rate. Potassium chloride is a mineral used in the management of hypokalemia. We have no knowledge of hypokalemia in this patient and no indication to use potassium supplements. Potassium is not known to alter the progression of diabetic nephropathy. In fact, this woman is more likely to be hyperkalemic due to her diabetes, which can cause hyporeninism, hypoaldosteronism, and hyperkalemia. Giving potassium to patients already at risk for hyperkalemia is not advised, as hyperkalemia may provoke cardiac arrhythmias. Calcium carbonate and calcium citrate are used to treat the hyperphosphatemia found commonly in patients with chronic kidney disease. They will not alter the progression of diabetic proteinuria.

A 25-year-old man presents with increased urination and thirst. Over the past 3 days, he has been unable to satisfy his thirst and has to urinate up to 20 times per day. He noticed that his urine is very clear and colorless. In general, he feels very weak. He has never had any problems with urination before. His medical history is remarkable for a recently diagnosed psychiatric condition for which he began medical treatment. On physical exam, he appears to be lethargic, dehydrated, and pale. His vital signs are as follows: blood pressure 96/52 mm of Hg, temperature 101.2 degrees fahrenheit, pulse 108 beats per minute, and a respiratory rate of 26 per minute. Question What is a common cause of diabetes insipidus? Answer Choices 1 Lithium 2 Carbamazepine 3 Amitriptyline 4 Valproic acid 5 Vasopressin

Lithium Explanation Lithium, used in the treatment of bipolar disorder, is a common cause of diabetes insipidus. It is freely filtered through the glomerulus and reabsorbed in the proximal tubule along with sodium and water. Even small doses of lithium may cause diabetes insipidus. Lithium can also make the distal renal tubules resistant to the action of vasopressin. This patient had recently started this medication, which resulted in his new symptoms. Carbamazepine, amitriptyline, and valproic acid do not cause diabetes insipidus. Analogues of vasopressin are used to treat diabetes insipidus. Diabetes Insipidus (DI) exists in 1 of 2 forms: central and nephrogenic. In central diabetes insipidus, there is a decrease in the secretion of antidiuretic hormone (ADH). This results in polyuria and polydipsia and the patient has a diminished ability to concentrate urine. Decreased or deficient ADH can be caused by a defect in the hypothalamic osmoreceptors, supraoptic or paraventricular nuclei, or the supraoptico-hypophyseal tract. Nephrogenic DI is defined as a decrease in the ability to concentrate urine due to a resistance to ADH on the kidney. It can be seen in chronic renal insufficiency, lithium toxicity, hypercalcemia, hypokalemia, and tubulointerstitial disease. There is a hereditary form of nephrogenic DI that is rare and is transmitted as an X-linked genetic defect of the V2 receptor gene. ADH's main target is the kidney. It causes an alteration of permeability to water of the collecting tubules in the kidney's medulla and cortex. Water is reabsorbed by osmotic equilibration and is put back into the bloodstream. The actions of ADH are mediated through at least 2 receptors. The first is known as V1. V1 mediates vasoconstriction and enhances the release of corticotrophin. It also takes part in the synthesis of renal prostaglandins. The second one, known as V2, is a mediator of the antidiuretic response. The diagnosis of DI is usually made clinically. If a patient is able to match fluid loss with adequate fluid intake, their condition may appear to be otherwise normal. If they cannot keep up with fluid demand, they will begin to show signs of dehydration (sunken eyes, dry skin and mucous membranes, fever, tachycardia, unintentional weight loss). Other symptoms include headache, fatigue, muscle aches and irritability. A urine specific gravity of 1.005 or less and a urine osmolality less than 200 mOsm/kg is the hallmark of DI. Random plasma osmolality generally is greater than 287mOsm/kg. It is important to rule out other causes such as diabetes mellitus. One way to do this is to use the water deprivation test. A patient is monitored, and his/her water intake is withheld. The patient's weight and urine osmolality are measured on an hourly basis. If two urine osmolality readings are less than or equal to 30 mOsm on two consecutive readings, or if the person's weight decreases by 3% or more, then 5 units of vasopressin is given and another urine sample is checked after an hour. A normal reading will show a urine osmolality that is 2 to 4 times greater than the plasma osmolality. In patients with central DI, urine osmolality can increase by 50% or more. ADH levels will be extremely low in central DI. If a patient has nephrogenic DI, their response to the test will be an ADH level that is either normal or increased. There will be no response to the ADH by the kidney. The disease is very uncommon in the U.S., and men and women are affected approximately equally. The most common causes are head trauma and surgery. There is a genetic form of DI that is rare and is inherited in an autosomal dominant pattern. It is caused by a mutation in the AVP-neurophysin gene. Medications used to treat DI include vasopressin analogues in either oral, nasal, or subcutaneous forms. Other medications used include indomethacin, carbamazepine, thiazide diuretics, and chlorpropamide.

A new patient to the practice reports that his paternal grandfather had a tumor of the pituitary gland, his father had hyperparathyroidism, one uncle had a pancreatic tumor and another a thyroid cancer. His first cousin has "some kind of facial tumors" and another relative had treatment for "producing too much stomach acid." Given this family history, he should be screened for which of the following? A Carney complex B Cowden disease C McCune-Albright syndrome D MEN1 E Sipple syndrome

MEN1 This family history strongly suggests Multiple Endocrine Neoplasia 1, which are inherited in an autosomal dominant manner and involve mutations on the long arm of chromosome 11 (11a13). Carney complex (A) is another syndrome of multiple endocrine neoplasia (MEN) with tumors of the adrenal cortex, pituitary gland, thyroid, and gonads and with hyperpigmentation and cardiac myxomas. Cowden disease (B) is also a syndrome of MEN including thyroid abnormalities, breast cancer and hamartomas. Another MEN, McCune-Albright syndrome (C) is associated with precocious puberty, Cushing syndrome, hyperthyroidism, and acromegaly. Sipple syndrome (E), also known as MEN2a. may develop medullary thyroid carcinoma, pheochromocytomas, or Hirschsprung disease.

A patient seen at the prenatal clinic develops Graves disease at 25 weeks' gestation. Which of the following is the most appropriate treatment? A PTU 100 mg po tid B methimazole 10 to 30 mg po qd C propranolol 80 mg po qid D radioactive iodine therapy (RAI, 131I) E levothyroxine 0.1 mg po qd

PTU 100 mg po tid In nonpregnant patients, PTU and methimazole are the drugs of choice for the management of Graves disease. During pregnancy, PTU has a lower incidence of crossing the placental barrier than does methimazole. It also is excreted into breast milk to a lesser degree than is methimazole. Propranolol will help with the symptoms of Graves but not treat it. It can also cause low birth weight in the infant. RAI is contraindicated in pregnancy. Levothyroxine will worsen a Graves patient's hyperthyroidism

A 22-year-old woman presents with a 2-month history of weight loss; it is occurring despite the woman having a good appetite. She also reports of having frequent bouts of diarrhea. On detailed questioning, she reveals a feeling of heat intolerance and menstrual irregularity. The right lobe of the thyroid is palpably enlarged, and further investigations confirm the diagnosis of hyperthyroidism. What symptom is characteristic of hyperthyroidism? Answer Choices 1 Fatigue and lethargy 2 Palpitations and tremors 3 Slowed speech and movement 4 Thickening and dryness of skin 5 Impaired memory and sleepiness

Palpitations and tremors Explanation The common symptoms of hyperthyroidism include nervousness, palpitation, tremors, weight loss with preserved appetite, heat intolerance, diarrhea, increased perspiration, and decreased menstrual flow. Common signs include tachycardia, systolic hypertension, hyperactivity, atrial fibrillation, lid lag, eyelid retraction, tremors, and hyperreflexia. The causes of hyperthyroidism include Graves' disease (most common cause), toxic adenoma, toxic multinodular goiter, sub-acute thyroiditis, lymphocytic thyroiditis, factitious hyperthyroidism, and struma ovarii. Diagnosis is by complete history and examination in correlation with complete blood count and thyroid profile. Other tests include radioactive iodine uptake, fine needle aspiration cytology (FNAC), and biopsy. Treatment depends on the cause and severity of the disease, the patient's age, comorbid conditions, goiter size, and treatment requirements. The goal of therapy is the correction of the hypermetabolic state with the fewest side effects and the lowest incidence of hypothyroidism. Antithyroid drugs, radioactive iodine, beta-blockers, and surgery are the main treatment options. Fatigue and lethargy, slowed speech and movement, thickening and dryness of skin, impaired memory, and sleepiness are features associated with hypothyroidism.

A 45-year-old man presents with what he refers to as "anxiety attacks". He describes them as recurring bouts of palpitations, headaches, anxiety, and sweating. One evening, while having dinner with his wife, his wife witnesses an episode. She is concerned and immediately takes his blood pressure. His blood pressure is 195/105 mmHg. The next day, his wife takes him to see their family doctor. On further questioning, he comments that he sometimes gets light headed when he stands up too rapidly. He comments that his mother had similar problems. On physical examination, his blood pressure is 165/90 mmHg and his heart rate is 80 beats/min. A 24-hour collection of his urine test is positive for vanillylmandelic acid. Question There is a familial association of this condition with what other disease? Answer Choices 1 Medullary carcinoma of the thyroid 2 Follicular carcinoma of the thyroid 3 Papillary carcinoma of the thyroid 4 Pancreatic islet cell carcinoma 5 Pituitary adenoma

Medullary carcinoma of the thyroid This patient exhibits signs and symptoms of a pheochromocytoma. He has paroxysmal hypertension and paroxysmal episodes of sweating and anxiety. The paroxysmal episodes that he experiences are due to intermittent release of catecholamines from the pheochromocytoma. Postural hypotension can sometimes be seen with pheochromocytoma. Urinary vanillylmandelic acid (VMA) is a metabolite of catecholamines and is found in pheochromocytoma. Pheochromocytoma is 1 of the diseases seen with multiple endocrine neoplasia, type II. Multiple endocrine neoplasia, type II (Sipple's syndrome) consists of pheochromocytoma, medullary carcinoma of the thyroid, and parathyroid hyperplasia. Pituitary adenoma (or hyperplasia), pancreatic islet cell tumors, and parathyroid tumors are components of multiple endocrine neoplasia type I (Wermer's syndrome). Follicular carcinoma of the thyroid and papillary carcinoma of the thyroid are not associated with pheochromocytoma.

A 45-year-old man presents with episodic attacks of headache, recurring bouts of palpitations, anxiety, and sweating. He also gives history of a severe attack 1 week prior; it occurred while he was having wine and cheese with his wife. On further questioning, he comments that he gets light headed when he stands up too rapidly. He comments that his mother had similar problems. On physical examination, his blood pressure is 165/90mmHg and his heart rate is 80/min. A 24-hour collection of his urine tests positive for vanillylmandelic acid. Imaging studies showed bilateral adrenal medullary hyperplasia. Further work-up showed hypercalcemia, hypophosphatemia, and increased parathyroid hormone levels. The positive family history and investigative findings are suggestive of multiple endocrine neoplasia (MEN) in this patient. Question MEN type II is characterized by pheochromocytoma (PCC), hyperparathyroidism and which of the following conditions? Answer Choices 1 Medullary carcinoma of the thyroid 2 Pituitary adenoma 3 Pancreatic islet cell carcinoma 4 Follicular carcinoma of the thyroid 5 Papillary carcinoma of the thyroid

Medullary carcinoma of the thyroid Explanation This patient exhibits signs and symptoms of a pheochromocytoma. Orthostatic hypotension can be seen with pheochromocytoma. He has hypertension and paroxysmal episodes. The paroxysms that he experiences are due to intermittent release of catecholamines from the pheochromocytoma. Urinary vanillylmandelic acid (VMA) is a metabolite of catecholamines and is commonly found in pheochromocytoma. Multiple endocrine neoplasia type II (Sipple's syndrome) is a rare autosomal dominantly inherited syndrome; it is characterized by pheochromocytoma, medullary carcinoma of the thyroid, and primary hyperparathyroidism or parathyroid hyperplasia. In contrast to sporadic PCC, the familial variety within MEN-IIA begins with adrenal medullary hyperplasia. It is multi-centric and bilateral in more than 50% of the cases. Normal adrenal medulla is composed of roughly 85% epinephrine, in comparison; most PCCs predominantly contain norepinephrine. The familial pheochromocytomas are an exception because they secrete large amounts of epinephrine. Diagnosis involves genetic testing; hormonal assay, and imaging tests to locate the tumors. Pituitary adenoma and pancreatic islet cell tumors are components of multiple endocrine neoplasia type I (Wermer's syndrome). Follicular carcinoma of the thyroid and papillary carcinoma of the thyroid are not associated with pheochromocytoma.

A 45-year-old man presents with what he refers to as "anxiety attacks". He describes them as recurring bouts of palpitations, headaches, anxiety, and sweating. One evening, while having dinner with his wife, his wife witnesses an episode. She is concerned and immediately takes his blood pressure. His blood pressure is 195/105 mmHg. The next day, his wife takes him to see their family doctor. On further questioning, he comments that he sometimes gets light headed when he stands up too rapidly. He comments that his mother had similar problems. On physical examination, his blood pressure is 165/90 mmHg and his heart rate is 80 beats/min. A 24-hour collection of his urine test is positive for vanillylmandelic acid. Question There is a familial association of this condition with what other disease? Answer Choices 1 Medullary carcinoma of the thyroid 2 Follicular carcinoma of the thyroid 3 Papillary carcinoma of the thyroid 4 Pancreatic islet cell carcinoma 5 Pituitary adenoma

Medullary thyroid cancer Explanation This patient exhibits signs and symptoms of a pheochromocytoma. He has paroxysmal hypertension and paroxysmal episodes of sweating and anxiety. The paroxysmal episodes that he experiences are due to intermittent release of catecholamines from the pheochromocytoma. Postural hypotension can sometimes be seen with pheochromocytoma. Urinary vanillylmandelic acid (VMA) is a metabolite of catecholamines and is found in pheochromocytoma. Pheochromocytoma is 1 of the diseases seen with multiple endocrine neoplasia, type II. Multiple endocrine neoplasia, type II (Sipple's syndrome) consists of pheochromocytoma, medullary carcinoma of the thyroid, and parathyroid hyperplasia. Pituitary adenoma (or hyperplasia), pancreatic islet cell tumors, and parathyroid tumors are components of multiple endocrine neoplasia type I (Wermer's syndrome). Follicular carcinoma of the thyroid and papillary carcinoma of the thyroid are not associated with pheochromocytoma.

Which of the following sets of disorders is commonly found in multiple endocrine neoplasia (MEN) 2A? A Islet cell tumor; renal cell carcinoma; pheochromocytoma; B Pheochromocytoma; medullary thyroid carcinoma; mucosal neuromas C Medullary thyroid carcinoma; parathyroid hyperplasia; pheochromocytoma D Parathyroid adenoma; islet cell hyperplasia; pituitary adenoma E Visceral lipomas; Marfanoid features; retinal angiomas

Medullary thyroid carcinoma; parathyroid hyperplasia; pheochromocytoma The correct choice is C. The three primary features of MEN type 2A include medullary thyroid carcinoma, parathyroid hyperplasia or adenoma, and pheochromocytoma. In choice A, islet cell tumor and renal cell carcinoma are disorders found in various other MEN syndromes, but not in MEN type 2A. Choice B, pheochromocytoma, medullary thyroid carcinoma, and mucosal neuroma are disorders found in MEN 2B. Choice D, parathyroid adenoma, islet cell hyperplasia, and pituitary adenoma are found in MEN I. Choice E, visceral lipomas, Marfanoid features, and retinal angiomas are features found in various MEN syndromes, but not in MEN 2A.

An 88-year-old female patient has been advised by her primary care physican that she needs a computed tomography (CT) scan of her abdomen and pelvis due to persistent abdominal pain, bloating, and weight loss. She was told that she needs to hold one of her medications the day of the procedure and that she may resume the medication 48 hours later. She can't remember which medication she should discontinue. Which medication listed below should she discontinue temporarily as specified above due to the diagnostic test ordered? A glyburide B glipizide C pioglytizone D metformin E acarbose

Metformin The correct answer is (D). A CT of the abdomen and pelvis requires p.o. and IV iodinated contrast unless ordered specifically without IV contrast. There is an increased risk of acute renal failure with IV iodinated contrast. The risk to the patient may increase with metformin and therefore should ideally be held prior to and for 48 hours after any radiocontrast IV study to avoid the added possibility of lactic acidosis. The other treatments for non-insulin-dependent diabetes mellitus (NIDDM) do not require discontinuation with IV contrast studies for these time periods. However, the am dose of a sulfonylurea (glyburide or glipizide) may be held until after the study that day when the patient resumes eating to avoid hypoglycemia in some patients.

Which of the following pairs of conditions constitute 90% of the causes of adult hypercalcemia? A Vitamin D intoxication and renal failure B Lithium ingestion and hyperthyroidism C Prolonged immobilization and thiazide use D Hypothyroidism and hypoparathyroidism E Primary hyperparathyroidism and malignancy

Primary hyperparathyroidism and malignancy The correct choice is E, primary hyperparathyroidism and malignancy. Together, they constitute 90% of the causes of hypercalcemia. Choices A, B, and C can cause hypercalcemia, but at a lower rate. Choice D, hypothyroidism and hypoparathyroidism, do not cause hypercalcemia.

A 21-year-old woman presents with urinary frequency; her BMI is 41. A urinalysis is positive for glucose. Her random blood sugar is 257 and hemoglobin A1C is 8.5%. She is diagnosed with type II diabetes, diet and exercise are recommended. What is the drug of choice for managing her diabetes? Answer Choices 1 Metformin (Glucophage) 2 Glipizide (Glucotrol) 3 Pioglitazone (Actos) 4 Insulin 5 Acarbose (Precose)

Metformin is the only hypoglycemic agent that has been shown to reduce mortality rates in patients with type II diabetes. Insulin resistance, decreased insulin secretion, and increased hepatic glucose output are the hallmarks of type II diabetes. Metformin decreases hepatic glucose output and, to a lesser extent, sensitizes peripheral tissues to insulin. Metformin can reduce A1C levels by an average of 1 - 2.5%. Metformin has also been shown to slow progression of impaired glucose tolerance to diabetes. Metformin causes weight loss and is not associated with hypoglycemia. Gastrointestinal symptoms associated with metformin can be minimized by starting at a low dose and titrating slowly. It is usually administered 2 times a day. Long-acting preparations that are administered once daily are also available. Metformin rarely causes lactic acidosis, and current guidelines recommend against using metformin when creatinine levels are ≥1.4 in women and ≥1.5 in men. Metformin is approved for use in children over 10 years. Glipizide is a sulfonylurea insulin secretagogue, and it has not been shown to have any effect on cardiovascular morbidity or mortality. Severe hypoglycemia can occur with glipizide. Actos causes fluid retention and weight gain. It increases the incidence of heart failure, but it has not been shown to affect cardiovascular mortality. Insulin is typically introduced when glucose control is no longer possible with oral agents, or when contraindications to oral agents exist. Newly diagnosed patients may also benefit from short-term insulin use if sugars are extremely high. Such patients would typically have A1C levels greater than 9%. Acarbose is an alpha-glucosidase inhibitor and has been shown to reduce cardiovascular events; average A1C reduction is only 0.5 - 1%. It has to be taken 3 times a day with meals; gastrointestinal side effects can be troubling.

A 36-year-old man with a past medical history of diabetes mellitus presents with weight gain and skin changes. His review of systems is positive for a diminished libido, impotence, depression, cognitive dysfunction, and emotional lability. Lately, his fasting glucose levels have been above normal. His physical exam notes increased adipose tissue in the face, upper back, and above the clavicles. His skin reveals ecchymoses, telangiectasias, and purpura along his back and lower extremities; there is also facial acne and cutaneous atrophy. His abdominal exam reveals the following findings. Question What is the most appropriate pharmacotherapeutic treatment for this patient? Answer Choices 1 Ketoconazole 2 Metyrapone 3 Mupirocin 4 Prednisone 5 Tacrolimus

Metyrapone Explanation This patient is demonstrating signs and symptoms consistent with Cushing's syndrome. Medications used in the management of Cushing syndrome include somatostatin analogs (pasireotide), adrenal steroid inhibitors such as metyrapone, ketoconazole, and etomidate, glucocorticoid receptor antagonists (mifepristone), and adrenolytic agents (mitotane). Metyrapone blocks 11-beta-hydroxylase activity, the final step in cortisol synthesis. Therapy is begun at 1 g/d divided into 4 doses and increased to a maximum dose of 4.5 g/d. Adverse effects present from increases in androgen and mineralocorticoid precursors and include hypertension, acne, and hirsutism. Ketoconazole has been the most popular and effective of these agents for long-term use and usually has been the agent of choice. However, the FDA has issued a black box warning that states clinicians should no longer prescribe ketoconazole, except to treat some life-threatening fungal infections; this is due to increased risk for severe hepatotoxicity, including fatalities and liver transplantation, adrenal insufficiency, and adverse drug interactions. Prednisone is not indicated in the treatment of Cushing syndrome. Cushing syndrome is caused by prolonged exposure to elevated levels of either endogenous glucocorticoids or exogenous glucocorticoids. Exogenous use of glucocorticoids should always be considered and excluded in the etiology of Cushing syndrome. Tacrolimus is a topical immunosuppressant agent useful in the treatment of psoriasis, while mupirocin is a topical antibiotic useful in the treatment of topical skin infections such as impetigo.

A 28-year-old woman who was born and brought up just outside of Washington, DC, comes in for evaluation of vague "problems with swallowing." She has no other symptoms except "my neck is bigger than it used to be." Examination reveals only a diffuse, somewhat irregular, nontender enlargement of the thyroid gland with distinct masses palpable within it. What is the most likely diagnosis? A endemic goiter B Graves disease C Hashimoto thyroiditis D multinodular goiter E thyroid carcinoma

Multinodular goiter Multinodular goiter is the most likely in a woman with these findings in the United States. It may be nontoxic as in this case or toxic, i.e., producing excessive thyroid hormones which cause symptoms of hyperthyroidism. Endemic goiter (A), which may present as a simple enlargement of the thyroid or as a multinodular one, is found almost entirely in iodine-deficient areas of the world and is extremely rare in the U.S. In Graves disease (B), the thyroid is enlarged and may exhibit a thrill and a bruit. In addition, the patient would have other signs of hyperthyroidism. The thyroid in Hashimoto thyroiditis (C) is diffusely enlarged and firm with fine nodules. A thyroid carcinoma (E) usually presents as a firm, nontender nodule in the gland.

A 42-year-old woman works full-time as a data entry clerk and often puts in many hours of overtime. She is experiencing numbness and tingling in her right thumb, index finger, middle finger, and half of her ring finger. The numbness and tingling initially comes and goes; however, after a few months, it is constantly present. Question What could be considered a predisposing condition for the most likely diagnosis? Answer Choices 1 Myxedema 2 Hyperthyroidism 3 Hyperparathyroidism 4 Hypoglycemia 5 Cushing syndrome

Myxedema Explanation This patient has symptoms consistent with carpal tunnel syndrome. In carpal tunnel syndrome, the median nerve is compressed in the wrist. The distribution of her complaints indicates distribution of the median nerve. Myxedema is a predisposing condition for carpal tunnel syndrome. Myxedema is seen with hypothyroidism that causes mucopolysaccharides and fluid accumulation in the tissues. This leads to compression of the median nerve. None of the other choices are associated with an increased risk in carpal tunnel syndrome.

A 23-year-old woman is brought to the office due to her "heart racing". Her episodes have been intermittent over the past 2 weeks. She reports feeling tired, and she has not worked for 2 days. She says she had 'the flu' about a month ago and has not quite recovered. On examination, her pulse is 102/min, BP 110/80, and her palms feel sweaty and shaky. All other systems appear to be normal. Her lab investigations are given below: Creatinine: 0.9 mg/dl AST: 45 IU/dl ESR: 68 mm/hr Hemoblobin: 11.9 g/dl TSH: 0.01 mIU/mL T3: 5 mcg/dL T4: 160 mcg/dL Her radioiodine uptake scan shows diffusely reduced uptake throughout the gland. Question What is the best initial therapy? Answer Choices 1 Propylthioruacil 2 Radiolabeled Iodine ablation 3 High dose prednisone 4 NSAIDs 5 Levothyroxine

NSAIDS Explanation The clinical picture is suggestive of subacute thyroiditis, also called DeQuervain's thyroiditis. The etiology is unclear, but it is usually a result of viral infection with coxackie, mumps, and adenovirus, among others. NSAIDs are the drug of choice, with steroids being a backup drug for severe cases. The pathophysiology involves inflammation of the thyroid gland with release of colloid and preformed thyroid hormone. Histology shows infiltration of the gland with mononuclear cells and non-caseous granulomas. Due to destruction of the follicular cells, colloid and thyroid hormones are released into the general circulation, resulting in hyperthyroidism. The hyperthyroid phase lasts as long as the preformed colloid is release. A transient hypothyroid phase while the follicular cells regenerate commonly follows the depletion of colloid. The hallmark of the inflammation is pain, usually described by the patient as being vaguely in the front of the neck and presenting while swallowing, chewing, or radiating to the jaw. Hyperthyroid symptoms commonly manifest as palpitations, sweating, fatigue, weight loss, and diarrhea. Examination typically reveals tenderness over the thyroid gland, which may sometimes be severe. Lab investigations show an elevated ESR, which is evidence of inflammation. Thyroid hormone levels reflect the hyper or hypothyroid phase of the illness. Thyroglobulin levels may be elevated due to the release of colloid. Radio iodine uptake studies show diffusely reduced uptake due to the destruction of cells. This is in direct contrast to Grave's disease, where there is diffusely increased uptake. NSAIDs are usually sufficient for treatment. Rarely steroids are required for severe disease. Propylthiouracil is incorrect. The hyperthyroidism in thyroiditis is due to inflammation and destruction of cells with the release of preformed thyroid hormones, with an eventual return to normal. Administering anti-thyroid medication is unnecessary. Radioiodine ablation is incorrect, as subacute thyroiditis is a reversible condition, with thyroid function eventually coming back to normal. High dose prednisone is incorrect. It may be used in severe thyroiditis not responding to NSAIDs, but should be avoided as a first line medication due to the associated side effects. Levothyroixine is incorrect. This patient is hyperthyroid at present, and thyroxine would exacerbate the problem.

A 45-year-old woman is constantly thirsty and consumes large amounts of water. She has a history of sarcoidosis. In addition, her urine output is in the range of 7 liters per day; she wakes up several times a night to urinate. Upon administration of aqueous vasopressin, her urine osmolarity rises above the value of her plasma osmolarity. Question What structure is most involved in the pathophysiology of the patient's condition? Answer Choices 1 Neurohypophysis 2 Adenohypophysis 3 Glomerulus 4 Loop of Henle 5 Collecting duct

Neurohypophysis Explanation This patient has signs and symptoms of diabetes insipidus. Central diabetes insipidus occurs when there is deficient antidiuretic hormone (ADH) secretion from the posterior pituitary (neurohypophysis). In central diabetes insipidus there will be a low ADH level. Nephrogenic diabetes insipidus occurs when the kidney fails to respond to ADH. The collecting duct is the portion of the kidney that is actually responsive to ADH In nephrogenic diabetes insipidus; ADH levels will be normal to elevated. In a patient with central diabetes insipidus, administration of aqueous vasopressin will result in an increase in urine osmolarity above plasma osmolarity. This will not be the cause in a patient with diabetes of nephrogenic origin. Central diabetes insipidus can be caused by granulomatous disease (e.g., sarcoidosis), head trauma, intracranial neoplasm, and some other conditions. The anterior lobe of the pituitary is the adenohypophysis. The anterior lobe of the pituitary secretes follicle-stimulating hormone (FSH), luteinizing hormone (LH), prolactin, growth hormone (GH), thyroid-stimulating hormone (TSH), and adrenocorticotropic hormone (ACTH) (and the corticotropin-related peptides). The glomerulus is a specialized capillary tuft. The endothelium of the glomerulus is fenestrated; the glomerulus is surrounded by Bowman's capsule. A renal corpuscle consists of the glomerulus and Bowman's capsule. A renal corpuscle is sometimes called a Malpighian corpuscle. Renal corpuscles are seen in the kidney cortex. The loop of Henle is part of the nephron. It is shaped like a hairpin; the proximal convoluted tubule leads into the descending limb of the loop of Henle. The ascending limb of the loop of Henle goes to the distal convoluted tubule.

A 30-year-old woman presents with episodic headache, palpitations, and sweating. She has had multiple episodes, each lasting approximately 20 minutes, over the last few weeks. Her primary care physician noted hypertension on her last several visits, and her blood pressure today is 210/98 mm Hg. She has no past medical history. Question The production of what hormone is associated with her condition? Answer Choices 1 Growth hormone 2 Gastrin 3 Norepinephrine 4 Insulin 5 Cortisol

Norepinephrine Explanation Norepinephrine is the correct response. The patient's history of headache, palpitations, diaphoresis, and hypertension suggests pheochromocytoma. Pheochromocytoma is a tumor that originates in the adrenal glands. Pheochromocytomas produce catecholamines, such as norepinephrine, epinephrine, and dopamine, causing its symptoms. Growth hormone is an incorrect response. Growth hormone may be secreted by pituitary tumors, leading to acromegaly. Acromegaly results in abnormal growth of soft tissue and the skeleton. Patients present with increase in hand and foot size, coarse facial features, and a prominent brow. Gastrin is an incorrect response. Gastrin-producing tumors are associated with Zollinger-Ellison syndrome. Zollinger-Ellison syndrome causes abdominal pain, diarrhea, and ulcers. Insulin is an incorrect response. Rare pancreatic tumors called insulinomas secrete insulin. Insulinomas are associated with symptoms of hypoglycemia. Cortisol is an incorrect response. Cortisol may be overproduced due to pituitary adenomas, resulting in Cushing syndrome. Cushing syndrome is associated with hypertension; patients also present with round face, central obesity, muscle wasting, striae, and muscle wasting.

A 13-year-old boy presents with right breast development over the last 5 to 6 months. He is on the swim team, so this has caused him much consternation. The breast swelling is slightly tender and without any drainage. He has been healthy and denies taking any medications, he denies any substance abuse or trauma, and he is doing well at school. On exam, his height is 63 in (160 cm), 75th percentile and weight is 115 lbs. (53kg) also 75th percentile, BP is 94/68, pulse 68 beats/min. There is slightly tender 7-8 cm of right breast elevation and swelling extending from the areolar that is non-erythematous. The left side is normal. There is no axillary lymphadenopathy. The testes are descended bilaterally and measure 3 cm in size. Pubic hair shows sparse growth of long, downy hair at the base of the penis. There is no axillary or facial hair. He does have scant acne. The mother is very anxious and wants laboratory tests. Which of the following would be the most appropriate initial evaluation to rule out any underlying pathology? Answer Choices 1 Obtain surgery referral 2 Order ultrasound 3 Order LH, FSH, testosterone and estradiol 4 Order a urine drug screen 5 Obtain nutrition consult

Order LH, FSH, testosterone and estradiol Explanation Approximately 70% of pubertal boys develop at least breast buds during adolescence, though some will have more extensive growth. This is probably a result of low testosterone in relation to estradiol levels thus laboratory evaluation should include testosterone, estradiol and gonadotropins to assess levels. This would effectively rule out conditions that could cause markedly abnormal levels, such as Klinefelter syndrome, the most common sex chromosome disorder, and the most common cause of hypogonadism and infertility in men. There are decreased levels of testosterone and elevated follicle-stimulating and luteinizing hormones. Karyotype analysis will reveal 1 or more extra X chromosomes. Physiologic gynecomastia usually resolves by 18 years when adult androgen/estrogen ratios are achieved. The incidence of breast enlargement peaks between the ages of 13-15 years during Sexual Maturity rating or Tanner staging 2 to 3 and may be unilateral or bilateral. Reassurance concerning the benign nature of the condition is usually all that is needed. Gynecomastia that occurs at other times in age or sexual development (excluding newborns) or that evolves rapidly would warrant further investigation. Surgical removal of the enlarged breast glandular tissue should be considered in boys who have had very persistent gynecomastia and who have completed puberty. Ultrasound imaging of the breast can be used to distinguish between solid tumors and fluid-filled cysts. Breast cancer in adolescents is extremely rare. Tumors tend to be slow growing, well differentiated, firm and painless. A urine drug screen would point to possible marijuana use that can be seen with gynecomastia when it is smoked heavily. There is little concern in this patient who is doing well at school, denies substance abuse, and whose parent does not express this concern. Obese pubertal boys commonly present with gynecomastia due to the accumulation of fat in the upper segment of the body both subcutaneously and intra-abdominally. The breasts are often greater in size and more persistent than in usual physiologic gynecomastia. In such a case, a nutrition consult would be indicated; however, on exam this patient was seen to be proportional for both height and weight.

A patient was prompted to visit his health care provider after his wife started to notice that he was not interested in eating, has lost weight, and has been suffering from nausea for the last few weeks. The practitioner notes hyperpigmentation of the patient's skin, although the patient denies any recent sun or tanning salon exposure. Routine non-fasting blood work reveals the following: Sodium = 130 meq/L Potassium = 5.2 meq/L Chloride = 105 meq/L Glucose = 135 mg/dL Hemoglobin = 13.0 g/dL Hematocrit = 39.0 WBC count = 8,000/mm 3 Which of the following physical exam findings would you expect to see in this patient? A Orthostatic hypotension B Wide, purple striae C Central obesity D Full facial features E Exophthalmous

Orthostatic hypotension The correct choice is A, orthostatic hypotension. The first step in the discussion of this patient is the suspected diagnosis of adrenal insufficiency. Patients with this disorder will have an excess of ACTH, which will act like melanocyte stimulating factor on the skin and cause hyperpigmentation. In adrenal insufficiency, aldosterone is deficient, thereby causing a decrease in sodium retention and potassium excretion. Hypotension is found in approximately 90% of these patients, sometimes associated with syncope as well. Choices B, C, and D are found in patients with cortisol excess. Choice E can be found in patients with Graves' disease.

A 42-year-old woman presents for annual examination. She is in excellent health; there are no current medical issues or medications. On examination, a firm, nontender thyroid nodule is palpated. TSH, T3, and T4 are all within normal limits. The patient undergoes a fine needle aspiration. Question What is the most common type of thyroid cancer likely in this patient? Answer Choices 1 Papillary carcinoma 2 Follicular carcinoma 3 Medullary carcinoma 4 Anaplastic carcinoma 5 Metastatic disease from another location

Papillary Carcinoma Explanation Papillary carcinoma is the most common form of thyroid cancer, accounting for approximately 80% of all cases. Follicular carcinoma accounts for approximately 14% of thyroid cancers. Medullary carcinoma is responsible for about 3% of thyroid cancers. Anaplastic carcinoma causes approximately 2% of thyroid cancers. Metastatic disease from another location is a rare presentation of thyroid cancer. It is most commonly seen with lung, breast, renal, and skin cancers.

Psammoma bodies are concentric calcified structures and are found in several tissue pathologies. In the image, psammoma bodies can be seen. Psammoma bodies are characteristic of which of the following? Answer Choices 1 Papillary carcinoma of the thyroid 2 Mucinous cyst adenocarcinoma of the ovaries 3 Medullary carcinoma of thyroid 4 Follicular adenoma 5 Pheochromocytoma

Papillary carcinoma of the thyroid Explanation Single necrotic cells may form a nidus for deposition of calcium. Subsequently, the progressive acquisition of an outer layer of calcium may form a lamellated structure, which resembles grains of sand. This is called a psammoma body. Psammoma bodies are seen in some neoplastic conditions, such as papillary carcinoma of the thyroid, serous cyst adenocarcinoma of the ovary, adenocarcinoma of the endometrium, and meningioma. Rarely, it may be seen in adenocarcinoma of the lung. In the case of the ovary, the possible mode of formation is by neoplastic and histiocytic cellular degeneration. Psammoma bodies (blue arrow) - calcific concretions with well-defined concentric laminations

A 56-year-old woman presents with severe left-sided flank pain. She has been treated for kidney stones twice in the past year. Review of systems is positive for polydipsia, fatigue, overall weakness, and depression. She denies fever, hematuria, vaginal discharge, constipation, or diarrhea. Her past medical history is significant for hypertension, which is controlled on hydrochlorothiazide. Her serum laboratory values are listed in the table. Serum Reference range Result Blood urea nitrogen 8 - 20 mg/dL 19 mg/dL Creatinine 0.6 -1.2 mg/dL 1.1 mg/dL Glucose 60 - 110 mg/dL 82 mg/dL Phosphorus 2.5 - 4.5 mg/dL 1.9 mg/dL Ionized calcium 4.6 - 5.3 mg/dL 5.9 mg/dL Parathyroid hormone, intact 11 - 54 pg/mL 87 pg/mL Question What is the underlying diagnosis contributing to her recurrent nephrolithiasis? Answer Choices 1 Chronic kidney disease 2 Diabetes mellitus 3 Metastatic breast cancer 4 Parathyroid adenoma 5 Thiazide diuretic overuse

Parathyroid Adenoma Explanation Parathyroid adenoma is the correct response. The patient has hypercalcemia that is contributing to her recurrent nephrolithiasis. Patients with hypercalcemia may be asymptomatic or present with "bones, stones, abdominal groans, psychic moans, with fatigue overtones." The most common causes of hypercalcemia are primary hyperparathyroidism and malignancy, accounting for over 90% of cases of hypercalcemia. To determine the cause of hypercalcemia, serum ionized calcium, phosphate, intact parathyroid hormone (iPTH), and urine calcium levels should be evaluated. If malignancy is suspected, appropriate workup should be performed. This patient has elevated ionized calcium, elevated calcium in her urine, and hypophosphatemia, with elevated parathyroid hormone levels consistent with primary hyperparathyroidism. Primary hyperparathyroidism is caused by unregulated PTH secretion. In 80% of cases, it is caused by a single parathyroid adenoma. It is more common in patients over the age of 50 and affects women more than men. Laboratory results show increased serum and urine calcium, increased iPTH, and decreased or normal serum phosphorous. Imaging is not necessary, but it may be used in a preoperative mapping of the parathyroid glands. Medical treatment includes fluids, bisphosphonates, or cinacalcet. Symptomatic patients can be cured with a parathyroidectomy. Calcium and vitamin D supplementation may be required for a reactive secondary hyperparathyroidism occurring after surgery. Patients with chronic kidney disease will have increased blood urea nitrogen and creatinine levels. They develop a secondary hyperparathyroidism due to decreased vitamin D metabolism, resulting in decreased serum calcium levels and increased PTH levels with increased serum phosphorus levels. Patients with hypercalcemia can develop polyuria and polydipsia due to nephrotic diabetes insipidus. These symptoms can be similar to diabetes mellitus (DM). In DM, the serum calcium, phosphorus, and parathyroid levels will be normal with elevated blood glucose levels. Thiazide diuretic use can result in hypercalcemia, but the urine calcium level will be low. PTH and phosphorus levels will be normal. Malignancy should be evaluated as a possible cause of hypercalcemia. If the patient has not undergone routine screenings, they should be done. The PTH level will be low in a patient with malignancy-related hypercalcemia.

Your patient has a history of primary hyperparathyroidism. Recently she has been hospitalized due to obstructing kidney stones. She has had several fractures including her hip, sacrum, and forearm in the past year, all on separate occasions. She is constantly complaining of a lack of energy. What is the recommended treatment for her symptomatic hyperparathyroidism? A surgical removal of the pituitary B high-dose calcium supplementation C parathyroidectomy D thyroidectomy E thiazide diuretics

Parathyroidectomy The correct answer is (C). Primary hyperparathyroidism is most commonly secondary to a single parathyroid adenoma. The recommended treatment for symptomatic primary hyperparathyroidism is parathyroidectomy. High-dose calcium supplementation and thiazide diuretics, choices (B) and (E), can worsen the hypercalcemia associated with hyperparathyroidism. Neither surgical removal of the pituitary, choice (A), nor or a thyroidectomy, choice (D), is an indicated treatment for this condition.

A patient is recovering from having a total thyroidectomy two days ago for medullary thyroid cancer. An extensive neck dissection was required during the surgery. Post-operative lab testing reveals a low serum calcium level. Which of the following clinical presentations will most likely occur in this patient? A Constipation B Anorexia C Polyuria D Bone pain E Paresthesias

Paresthesias The correct choice is E, paresthesias. Circumoral paresthesias are signs of hypocalcemia. Hypocalcemia can occur after any type of neck surgery that may have resulted in destruction of the parathyroid glands. Choices A through D are symptoms of hypercalcemia and may be seen in hyperparathyroidism.

A 36-year-old woman with a past medical history of diabetes mellitus and hepatitis presents to her family medicine office with complaints of weight gain and skin changes. Her review of systems is positive for menstrual irregularities with extended periods of amenorrhea, infertility, depression, cognitive dysfunction, and emotional lability. Of late, her fasting glucose levels have been above normal. Her physical exam notes increased adipose tissue in the face, upper back, and above the clavicles. Her skin reveals ecchymoses, telangiectasias, and purpura along her back and lower extremities, facial acne, and cutaneous atrophy. Her abdominal exam reveals the following image. At this time, she refuses any surgical interventions. Question Which of the following is the most appropriate pharmacologic treatment for this patient at this time? Answer Choices 1 Deltasone (Prednisone) 2 Pasireotide (Signifor) 3 Ketoconazole (Nizoral) 4 Sandostatin (Octreotide) 5 Declomycin (Demeclocycline)

Pasireotide (Signifor) Explanation This patient is demonstrating signs and symptoms consistent with Cushing's syndrome. Pasireotide (Signifor) is a somatostatin analog that binds and activates human somatostatin receptors, resulting in inhibition of ACTH secretion, which leads to decreased cortisol secretion. It is indicated for treatment of adults with Cushing disease in whom pituitary surgery is not an option or has not been curative. Treatment for exogenous Cushing syndrome is gradual withdrawal of glucocorticoid; therefore, treatment with prednisone is inappropriate. Ketoconazole has been the most popular and effective of these agents for long-term use and has been the agent of choice. However, the FDA has issued a warning that states clinicians should no longer prescribe ketoconazole, except to treat some life-threatening fungal infections; this is due to increased risk for severe liver injury, adrenal insufficiency, and adverse drug interactions. Agents that decrease CRH or ACTH release have been studied for the treatment of Cushing disease. Such agents include bromocriptine, cyproheptadine, valproic acid, and octreotide. Currently, use of these agents is investigational. Demeclocycline is not indicated in the treatment of Cushing syndrome; it is used in the treatment of syndrome of inappropriate ADH secretion refractory to furosemide and water restriction.

A 32-year-old Caucasian woman with a past medical history of Hashimoto's thyroiditis, type I diabetes mellitus, and pernicious anemia presents to her family medicine office with a 2-year history of insidious and intermittent fatigue, anorexia, involuntary weight loss, nausea, abdominal pain, vomiting, and dizziness associated with position changes. Her physical exam is noteworthy for postural hypotension with a maximum systolic blood pressure of 104 in the supine position. Additionally, she has a low-grade fever and a generalized pigment change to her skin, as indicated in the image. Question What is the next most important intervention in this patient? Answer Choices 1 Refer the patient for an MRI of the brain 2 Recommend a low salt diet and begin potassium replacement 3 Begin a broad spectrum antibiotic 4 Start metyrapone (metopirone) 5 Perform a rapid ACTH stimulation test

Perform a rapid ACTH stimulation test Explanation This patient's most likely diagnosis is primary adrenal insufficiency, known as Addison's disease. The diagnosis of adrenocortical insufficiency rests on the assessment of the functional capacity of the adrenal cortex to synthesize cortisol. This is accomplished primarily by use of the rapid ACTH stimulation test (cortrosyn, cosyntropin, or synacthen). Imaging of the brain is unnecessary for this patient at this time since Addison's disease is adrenocortical insufficiency due to the destruction or dysfunction of the entire adrenal cortex. The onset of disease usually occurs when 90% or more of both adrenal cortices are dysfunctional or destroyed. This patient's hyperpigmentation signifies adrenal insufficiency, since hyperpigmentation only occurs with primary adrenal insufficiency (not secondary corticosteroid insufficiency, i.e., Hypothalamic-Pituitary-Adrenal Axis (HPA) Dysfunction). It is caused by the stimulant effect of excess adrenocorticotrophic hormone (ACTH) on the melanocytes to produce melanin. The hyperpigmentation is caused by high levels of circulating ACTH that bind to the melanocortin 1 receptor on the surface of dermal melanocytes. A low salt diet and increased potassium intake is inappropriate for this patient since most patients with Addison's disease have hyponatremia and hyperkalemia. Antibiotics are not indicated at this time, as this patient does not have an infectious pathology. Metyrapone (metopirone) is an inhibitor of endogenous adrenal corticosteroid synthesis. It inhibits the 11-B-hydroxylation reaction in the adrenal cortex and is thus utilized as an agent that inhibits steroidogenesis in Cushing syndrome.

You provide care to a 37 kg, 10-year-old boy with Duchenne muscular dystrophy, which was diagnosed 4 years ago. For the past 18 months, he has been treated with prednisolone (0.75 mg/kg/day). At this time, he has limited ambulation, and tendon release surgery is planned. What regarding his perioperative care is true? Answer Choices 1 Forced vital capacity <30% is an absolute contraindication to surgery 2 Optimal anesthesia is delivered with inhalational anesthesia drugs, such as halothane, enflurane, isoflurane, methoxyflurane, sevoflurane, or desflurane 3 Peri-operative steroid coverage with hydrocortisone should be started 1 - 3 days prior to the planned surgery 4 Preoperative echocardiography is a reliable determinant of intraoperative cardiac function 5 Succinylcholine is an appropriate medication for muscle relaxation during surgery

Peri-operative steroid coverage with hydrocortisone should be started 1 - 3 days prior to the planned surgery The rationale for stress steroid coverage is that chronic corticosteroid therapy suppresses the hypothalamic-pituitary-adrenal (HPA) axis, although this has never been well-documented in studies. Patients who are not taking glucocorticoids activate HPA axis with surgery, trauma, and severe illness. Adults who do not experience stress secrete cortisol equivalent to 5 mg/day of prednisone or 20 mg hydrocortisone/day. Generally, the recommended dose has been 3 to 5 times the amount secreted daily, which is up to 200 - 400 mg/day of hydrocortisone. More recent studies suggest that lower doses may be used. Suggested lower doses for major surgery, such as cardiopulmonary bypass, are up to 100 - 150 mg/day hydrocortisone for 2 - 3 days, and 75 - 100 mg/day for moderate procedures, such as orthopedic surgery, for 1 - 2 days. Even when a patient has taken glucocorticoids in the past and has stopped, the HPA axis may recover quickly or not fully recover for 9 - 12 months. Succinylcholine, a depolarizing agent, may induce increased muscle contractions and may cause rhabdomyolysis. With muscle breakdown, creatine phosphokinase, potassium, and myoglobin may be released. The large proteins of the myoglobin may cause renal damage. Elevated potassium levels may cause cardiac arrest. Both succinylcholine and inhalational anesthetics may cause a malignant hyperthermia-like response. High intracellular calcium levels lead to activation of actin and myosin fibers with continuous contractions of the muscles producing muscular rigidity and spasm. A hypermetabolic state occurs in the muscle cells resulting in increased oxygen consumption and metabolic acidosis. Respiratory acidosis may occur with elevated CO2 levels. As a result of the hypermetabolic state, hyperthermia occurs, along with tachycardia and possible cardiac arrhythmia. Nitrous oxide is a safe inhalational agent. Also safe are non-depolarizing muscle relaxants, including pancuronium, cisatracurium, vecuronium, rocuronium, atracurium, and mivacurium. Echocardiography should be done pre-operatively in all patients with Duchenne muscular dystrophy. In the past, patients commonly died from respiratory problems. As care for these conditions has improved, dilated cardiomyopathy has become an increasingly common morbidity and cause of death in these patients. The time course for development of cardiomyopathy is not clearly defined. There may be limited ability to increase cardiac output, with stress resulting in impaired oxygen delivery. Clinical symptoms of cardiac dysfunction may be subtle, vague or unrecognized, as these children have limited physical activity. Scoliosis may limit the usefulness of echocardiography, making it difficult get appropriate windows. In addition, echocardiography may not detect the myocardium's ability to respond to stress. It has been suggested that pre-operative stress echocardiography may be a more reliable test. Intraoperative transesophageal echocardiography, pulmonary arterial catheterization, and pulse contour analysis (PCCO) may be of great benefit for intraoperative monitoring. There is evidence that specific dystrophin gene mutations may be predictive of cardiomyopathy, while other mutations may be protective or inhibitory toward its development1. Forced vital capacity should be determined before surgery. There are no absolute contraindications to surgery based on pulmonary function. Good results have been reported in patients with FVC 20% of predicted, although best prognosis for recovery seems to be with FVC >40% predicted. Pre-operative sleep studies or nocturnal oximetry can be helpful if abnormal, noninvasive nighttime ventilation can be instituted prior to surgery.

A 15-year-old boy presents for a high school basketball physical. After his mother leaves the room, he asks if you can 'take a look at something'. He then lifts his shirt and tells you that he is developing 'man-breasts'. He is very embarrassed. There is firm, mobile, tender tissue palpable under both nipples. This tissue is approximately 75 mm in size on the right side and 1 cm on the left. There is no nipple discharge on either side. The patient's height and weight are 66 inches and 125 lbs (BMI 20.2). Question What is the most important next step in the diagnostic workup? Answer Choices 1 Drug screen and careful medication history 2 Liver function tests and abdominal CT scan 3 Mammogram 4 Physical examination (especially genitalia) 5 Thyroid function tests

Physical examination especially genitalia Explanation The correct response is a physical examination (especially genitalia). Gynecomastia may be either physiologic or pathologic. Physiologic gynecomastia has normal peaks in the neonatal period and in adolescence due to a relative increase in estrogen. Age-related gynecomastia (usually between 50 and 70 years of age) results from an increase in fatty tissue and decrease in hormone levels. Pseudogynecomastia (usually seen in obese men) is enlarged breasts without a palpable mass. The most reassuring aspect of this patient's gynecomastia is his age. If his symptoms were occurring outside the neonatal or adolescent period, or if they were associated with physical or laboratory abnormalities, a more aggressive workup would be warranted. The examiner should pay particular attention to the signs of puberty, particularly testicular size, in order to rule out conditions such as Klinefelter's syndrome (which may present with gynecomastia). Therefore, the most important next step in this patient's workup is a thorough physical examination; this will ensure normal puberty and rule out a testicular mass as a cause of a hormone imbalance and gynecomastia. Physiologic gynecomastia generally resolves within 2 years without any treatment. Both prescription and illicit drugs can cause gynecomastia. Common offenders include some antidepressants, antibiotics, medication used to treat HIV or heart disease, marijuana, anabolic steroids, heroin, and chronic alcohol abuse. Asking about medications and drug use would certainly be an important part of the medical history in the workup of gynecomastia. Liver disease (cirrhosis) is a very common cause of gynecomastia. Liver disease causes impaired metabolism of androstenedione, a precursor of estrogen and testosterone. In addition, patients with liver disease may be treated with spironolactone, a medication that is known to cause gynecomastia. This patient has no history or signs of liver disease. A mammogram would not be indicated in this patient because the breast changes are bilateral and there is a lack of concerning features (e.g., a firm, discrete mass, bloody nipple discharge, and/or skin changes). Breast cancer is rare in men, and this patient likely does not have pathologic gynecomastia. Thyroid disease (both hypo and hyper) can cause gynecomastia. In hyperthyroidism, this is a result of an increased conversion of androgen to estradiol. Hypothyroidism results in a lower level of testosterone with a resulting increase in estradiol production. This particular patient has no signs of symptoms of thyroid disease.

A 54-year-old woman presents with nervousness, a tremor, and irritability. She indicates that she has lost some weight over the last few weeks even though her appetite has increased. She also says that she is feeling feverish. You suspect that she may have a thyroid condition and order the appropriate labs. The lab results are as follows: TSH 7.8 (0.5 - 5.5 mU/L) T4 4.2 (0.8 - 1.8 ng/dl) T3 5.3 (1.4 - 4.2 pg/ml) Question What is the most likely diagnosis? Answer Choices 1 Thyroid storm 2 Pituitary adenoma 3 Graves' Disease 4 Hashimoto's thyroiditis 5 Iodine deficiency

Pituitary Adenoma Explanation The clinical picture is of a pituitary adenoma. The diagnosis is suggestive by clinically evident hyperthyroidism with elevated T4 and T3 and elevated serum TSH levels. Untreated hyperthyroidism may progress into a thyroid storm. Patients may present with tachycardia, fever, agitation, nausea, vomiting, diarrhea, and restlessness or psychosis. Graves' disease would produce decreased TSH and elevated T4 and T3 levels. Hashimoto's thyroiditis would generally produce elevated TSH and decreased T4 and T3 levels. In iodine deficiency or goiter, there is diffuse enlargement of the gland as well as elevated TSH and decreased T4 levels.

A 40-year-old woman presents with anxiety, difficulty sleeping, rapid heartbeat, and a tremor in her hands for the past 3 months. You note the presence of bulging eyes and suspect Graves' disease. You confirm that she has this disease by demonstrating the presence of thyroid stimulating immunoglobulin (TSI), which mimics the action of thyrotropin; you also use thyroid function tests. What gland releases thyrotropin hormone? Answer Choices 1 Adrenal gland 2 Pancreas 3 Pituitary gland 4 Thyroid gland 5 Hypothalamus

Pituitary Gland Explanation Thyrotropin is released by the pituitary gland. The pituitary gland releases thyrotropin/Thyroid Stimulating Hormone (TSH) in response to insufficient levels of thyroid hormone. Communication between the pituitary gland and the thyroid gland through TSH levels controls the levels of thyroid hormone in the blood. If the levels of thyroid hormone are low, then the levels of TSH will rise. The measurement of TSH in the blood is taken as a measure of thyroid function. High levels of TSH would indicate low levels of thyroid hormone. Thyrotropin hormone is not released by pancreas, hypothalamus adrenal, or the thyroid gland. Graves' disease is a type of hyperthyroidism caused by a generalized overactivity of the entire thyroid gland. The patient appears hot and flushed, and the thyroid gland enlarges in this condition. It is believed that Graves' disease is an autoimmune disorder. Antibodies are produced against 4 thyroid antigens, namely thyroglobulin, thyroid peroxidase, sodium-iodide symporter, and the thyrotropin receptor. The antibody against the thyrotropin receptor-TSI is the principal autoantibody of Graves' disease responsible for the manifestations of hyperthyroidism. Autoantibodies are produced against certain proteins on the surface of thyroid cells, stimulating those cells to overproduce thyroid hormones, resulting in the overactive thyroid. The onset of the disease is gradual, and the symptoms may be mistaken for nervousness due to a stressful life situation. Weight loss occurs, and this is followed by other symptoms such as trembling, muscle weakness of the upper arms and thighs, and insomnia. Thyroid function tests and the presence of TSI help in establishing the diagnosis of Graves' disease.

A 35-year-old male presents complaining of increasingly constant headaches, double vision centrally, and a progressive loss of peripheral vision for two weeks. He has no previous headache history and denies any other medical conditions. Physical examination reveals bitemporal hemianopsia without additional neurologic findings. What is the most likely diagnosis? A Acute ischemic stroke B Circle of Willis ruptured aneurysm C Migraine headache D Multiple sclerosis E Pituitary adenoma

Pituitary adenoma Pituitary adenomas, benign neoplasms associated with pituitary hormone secretory changes, may enlarge and become symptomatic. Symptoms are based upon the location and size of the tumor, and may include bitemporal hemianopsia, double vision, color desaturation, and visual acuity loss. Headaches may occur, due to associated pressure changes within the intrasellar space. Additional evaluation should include a T1-weighted MRI, screening laboratory tests, and a full ophthalmologic evaluation. These tests will also help evaluate for potential differential diagnoses, such as those listed. The patient's history is not consistent with an acute ischemic stroke or migraine headache. Although an unruptured aneurysm may have very similar findings to a pituitary tumor, ruptured aneurysms present with acute headache, nausea, vomiting, and potential changes in consciousness. Multiple sclerosis (MS) should remain on the differential for this patient and will also be evaluated through MRI (although the current findings are more consistent with a pituitary adenoma), and additional neurologic findings would be likely with MS.

A 44-year-old woman undergoes surgery for a follicular neoplasm of the thyroid gland. The tumor is successfully resected, and she is transferred to the post-operative ward. Her condition is noted to be stable. On post-operative day 2, she notices a tingling sensation around her lips that runs down her arms. She experiences a cramping sensation in the muscles of her hands and legs, with her fingers going into spasm. An intern on the floor notices the patient's distress and decides to investigate. Question What is the best initial diagnostic test? Answer Choices 1 Serum free T3 and T4 2 Serum TSH 3 Arterial blood gas analysis 4 Serum calcium levels 5 CT scan of the anterior neck

Serum Calcium Levels Explanation The patient has symptoms of hypocalcemia, a possible complication of thyroid surgery. The parathyroid glands are closely related to the thyroid gland and may be inadvertently removed during surgery. If damaged, the serum calcium levels usually drop within 48 hours of surgery. Symptoms of hypocalcemia include muscle pain, fatigue, and muscle cramps if severe. Chvostek's sign, which is the contraction of ipsilateral facial muscles with a light tap over the facial nerve, and spasm of the carpal muscles (Trosseau's sign) may be seen. The best screening test for hypocalcemia is the measurement of ionized calcium in the serum. Calcium exists in equilibrium between the fraction bound to albumin and the free or ionized fraction. The biological effects of calcium are dependent on the ionized fraction. Serum calcium level varies with level of serum albumin (calcium binding protein). Correction for calcium level when albumin is abnormal is as follows: Corrected calcium (mg/dL) = measured total Ca (mg/dL) + 0.8 (4.0 - serum albumin [g/dL]), where 4.0 represents the average albumin level. The risk factors for hypocalcemia after thyroid surgery include large goiters, hyperthyroidism, low pre-operative vitamin D levels, and planned extensive neck dissection. Hypocalcemia is prevented to some degree by the administration of large amounts of calcium, usually per oral, and 1,25 dihydroxy vitamin D. In the event of severe hypocalcemia, calcium gluconate may be used via IV. It is important to check magnesium levels and correct levels prior to discharge, as hypomagnesemia can compound hypocalcemia and make it refractory to correction. Serum TSH, T3, and T4 are incorrect, as thyroid function has no bearing on the patient's symptoms. They are expected to change after thyroid surgery. Typically, TSH levels will rise, and T3 and T4 levels will fall over the ensuing 3 - 6 weeks. An ABG is incorrect. It may demonstrate low Ca levels, but a direct measure of serum calcium in patients that demonstrates symptoms of hypocalcemia is more accurate and effective. A CT scan of the neck is incorrect, as it provides no additional information about the person's calcium metabolism.

A 40-year-old female presents to your office with symptoms of weight gain, hirsuitism, and easy bruising. Past medical and surgical history is noncontributory. She drinks one glass of wine on weekends and does not smoke cigarettes. She takes one multivitamin daily. Upon physical exam, you note facial fullness, central obesity, and thin skin. Which of the following is the most common cause of her symptoms? A. Pituitary Adenoma B. Iatrogenic C. Adrenal Micronodular Hyperplasia D. Adrenocortical adenoma E. Ectopic ACTH syndrome

Pituitary adenoma The correct choice is A, pituitary adenoma. This patient's clinical presentation is typical in Cushing's syndrome. The most common cause of Cushing's syndrome (other than ingestion of oral steroid medications) is Cushing's disease. This disease is caused by a benign, ACTH secreting pituitary adenoma. Choice B, iatrogenic, refers to the ingestion of prescribed (or non-prescribed) oral corticosteroid medications. This is frequently seen in patients requiring long-term oral steroid medications. This patient does not have this type of history. Choice C, adrenal micronodular hyperplasia, and choice D, adrenocortical adenoma, can cause Cushing's syndrome at less frequent incidence. Choice E, ectopic ACTH syndrome, presents more commonly in males with extremely elevated levels of plasma cortical and ACTH. These patients commonly have a positive history of an ectopic source of the ACTH, such as in small cell carcinoma of the lung.

You are reviewing incoming test reports while your supervising physician (SP) is out of town. There are 2 reports for a 46-year-old man; they are shown in the table. Vasopressin challenge test: Positive response Magnetic resonance imaging (MRI) brain: Mild thickening of the pituitary stalk, no masses or lesions Your SP left a note that he planned to initiate desmopressin after the test results confirmed his suspicion. Question What symptom would you expect to find in reviewing the history portion of this patient's chart? Answer Choices 1 Agnosia 2 Diplopia and blurred vision 3 Headaches 4 Memory loss 5 Polyuria and polydipsia

Polyuria and polydipsia Explanation This patient most likely has diabetes insipidus (DI). The most common presenting symptoms are polyuria and polydipsia. Diabetes insipidus is primarily a disorder in which the arginine-vasopressin (AVP) secreting cells are destroyed, causing a deficiency in antidiuretic hormone (ADH). The condition can be controlled with desmopressin, but in patients with very mild symptoms, no treatment may be preferred. Agnosia is the inability to recognize objects. It can be associated with dementia, head trauma, and brain tumors. The vasopressin challenge test, which should reduce thirst and urine output in patients with diabetes insipidus, would not be an appropriate test; desmopressin would not be suggested as a treatment. An MRI may be part of the workup for agnosia, and there could be variable findings. Diplopia and blurred vision should trigger a thorough neurologic exam and eye exam, with a possible MRI for further workup. As is the case with agnosia, the vasopressin challenge test and use of desmopressin have no role in regard to the patient's visual symptoms. Headaches are common, and most do not require specialized testing or imaging. If they present with 'red flags' that might indicate a more serious disorder (additional neurologic symptoms, new onset later in life, etc.), MRI of the brain is reasonable. Vasopressin challenge would not be used. Diabetes insipidus very rarely presents with headaches. Memory loss is not associated with these tests, findings, or treatments. A complaint of memory loss in a 46-year-old should be evaluated with a thorough neurologic exam; psychological testing and brain imaging should be considered as possibilities.

An 18-year-old woman presents with nausea, vomiting, drowsiness, and abdominal pain. She has Kussmaul respirations, ketotic breath, dry tongue, and loss of skin turgor. Her laboratory studies show the following results: sodium, 126 mEq/L; potassium, 3.1 mEq/L; bicarbonate, 8 mEq/L; chloride, 98 mEq/L, glucose, 320 mg/dL; BUN, 26 mg/dL; creatinine, 1.0 mg/dL; pH 7.02; pCO2, 13 mm Hg; and pO2, 86 mm Hg. In addition to intravenous fluid administration, which of the following therapies should be administered as part of initial treatment? Answer Choices 1 Calcium 2 Magnesium 3 Phosphate 4 Potassium 5 Bicarbonate

Potassium Explanation The correct response is potassium. Diabetic ketoacidosis is marked by an absolute insulin deficiency resulting in metabolic acidosis, ketonemia, and hyperglycemia. It is most commonly observed in patients with type 1 diabetes as a result of noncompliance with insulin therapy. Patients in diabetic ketoacidosis (DKA) often have normal or elevated serum potassium levels because of acidosis and dehydration. These normal or elevated potassium levels occur despite a large total body potassium deficit caused by osmotic diuresis and vomiting. Increased urinary output following fluid resuscitation significantly increase potassium losses. Withholding potassium from a patient in severe DKA can potentially result in cardiac arrest, severe dysrhythmias, respiratory arrest, and generalized paralysis. In contrast, bicarbonate therapy is controversial in DKA and may worsen hypokalemia. Several studies have failed to show any benefit with bicarbonate in DKA, even with a pH as low as 6.9. Although most patients have total body depletion of phosphate, immediate phosphate repletion has not been shown to alter the course of DKA. Replacement with magnesium should occur only in the presence of significant hypomagnesemia or refractory hypocalemia or hypokalemia. Calcium is not indicated in this setting in the immediate treatment of DKA

An 18-year-old woman presents with nausea, vomiting, drowsiness, and abdominal pain. She has Kussmaul respirations, ketotic breath, dry tongue, and loss of skin turgor. Her laboratory studies show the following results: sodium, 126 mEq/L; potassium, 3.1 mEq/L; bicarbonate, 8 mEq/L; chloride, 98 mEq/L, glucose, 120 mg/dL; BUN, 26 mg/dL; creatinine, 1.0 mg/dL; pH 7.02; pCO2, 13 mm Hg; and pO2, 86 mm Hg. In addition to intravenous fluid administration, what therapeutic agent is indicated pending further laboratory results? Answer Choices 1 Calcium 2 Dextrose 3 Phosphate 4 Potassium 5 Bicarbonate

Potassium Patients in DKA may be euglycemic, and they often have normal or elevated serum potassium levels because of acidosis and dehydration. These normal or elevated potassium levels occur despite a large total body potassium deficit caused by osmotic diuresis and vomiting. Administration of insulin, correction of acidosis with bicarbonate, and increased urinary output following fluid resuscitation significantly increase potassium losses. Withholding potassium from a patient in severe DKA with a normal potassium level can potentially result in cardiac arrest, severe dysrhythmias, respiratory arrest, and generalized paralysis. In contrast, bicarbonate therapy is controversial in DKA and may worsen hypokalemia. Several studies have failed to show any benefit with bicarbonate in DKA, even with a pH as low as 6.9. Although most patients have total body depletion of phosphate, immediate phosphate repletion has not been shown to alter the course of DKA. Calcium is not indicated in this setting. Dextrose is not indicated because the patient has a normal blood sugar. Relative contraindications to dextrose include hyperosmolar coma and stroke.

A 22-year-old woman presents with amenorrhea for the past 8 months. Further questions elicit additional pertinent positives of backaches, headaches, and acne. Physical examination reveals a female patient with a moon-shaped facies, multiple purple striae, and significant central obesity (body mass index of 36). Question Given this history and physical exam findings, what pharmaceutical treatment should be initiated after the appropriate clinical intervention has occurred? Answer Choices 1 Insulin 2 Levothyroxine 3 Prednisone 4 Propylthiouracil (PTU) 5 Desmopressin

Prednisone Explanation This patient likely is suffering from Cushing's syndrome, which is also known as hypercortisolism. Consequences of excessive levels of circulating cortisol, no matter the etiology, will lead to certain signs and symptoms, such as central obesity, thin extremities, a moon face, a buffalo hump, supraclavicular fat pads, protuberant abdomen, and oligomenorrhea/amenorrhea; in men, erectile dysfunction is a possibility. The most common etiology of Cushing's syndrome is a benign pituitary adenoma secreting excessive ACTH. Primary clinical intervention is transsphenoidal selective resection of the pituitary adenoma. Once this has been completed, patients typically suffer from a suppression of pituitary corticotrophs; therefore, treatment with hydrocortisone or prednisone replacement therapy is necessary for a range of 6 - 36 months post-surgery. Diabetes Type II is caused by insulin resistance in tissues to circulating endogenous insulin. Eventually insulin is unable to be produced by the pancreatic islet cells and an exogenous supply is required to prevent significant hyperglycemia and its related co-morbidities. Features of these patients include central obesity (specifically visceral obesity), with less fat noted on extremities. Insulin, although usually not the common initial medication considered at for patients with diabetes, is one of the many types of pharmaceutical treatments available for these patients. Hypothyroidism and hyperthyroidism both are related to dysfunction of the thyroid gland; hypothyroidism is due to failure of the thyroid gland itself or deficient supply of pituitary TSH (thyroid stimulating hormone). Symptoms exhibited with hypothyroidism are weight gain, fatigue, lethargy, depression, weakness, menses irregularities, athralgias, muscle cramps, cold intolerance, dry skin, and constipation; signs present may include thinning hair, brittle nails, bradycardia, or even peripheral edema. Typical treatment for hypothyroidism is a dose of daily levothyroxine. Hyperthyroidism is the disease state in which there is an excessive serum level of T3 and or T4 along with suppressed TSH levels. Signs and symptoms may include (depending on the severity) restlessness, nervousness, heat intolerance, increased sweating, weight loss, palpitations, atrial fibrillation, exophthalmos, or even pretibial myxedema. The medication that is commonly utilized in treatment of hyperthyroidism is primarily propylthiouracil, which is also frequently referred to as PTU. Diabetes insipidus (DI) is antidiuretic hormone deficiency. A sign/symptom of DI is an extreme, unquenchable thirst with a very specific craving for ice water. Patients also will have significant polyuria or hypernatremia. Pharmaceutical treatment for DI is typically desmopressin.

A 31-year-old man presents with constipation, fatigue, and weight gain of 20 pounds over the last 6 months. He has no significant past medical history. He is a smoker. His family history is notable for a myocardial infarction in his father at the age of 42. His physical examination is unremarkable, except for a weight of 220 pounds and a height of 6 feet. You order laboratory tests, which reveal normal electrolytes, a total cholesterol of 223, and a triglyceride level of 205. His thyroid function tests reveal a thyroxine of 2.1 (4.5 - 11.0), a free T4 index of 1.7 (4.5 - 11.0), and a TSH of 193 (0.35 - 5.50). Question What is the best next step in managing this patient's condition? Answer Choices 1 Telling him to stop smoking and start on a low cholesterol diet 2 Telling him to stop smoking, start on a low cholesterol diet, and start taking an HMG-CoA reductase inhibitor 3 Prescribing oral levothyroxine 4 Prescribing oral levothyroxine and an HMG-Co reductase inhibitor 5 Being admitted to the hospital for intravenous levothyroxine

Prescribing oral levothyroxine Explanation The patient has frank primary hypothyroidism. His thyroid function tests confirm this diagnosis by showing a markedly elevated TSH and a depressed thyroxine level. Elevated total cholesterol levels and triglycerides can be seen in these patients. These abnormalities usually resolve with thyroid replacement. The best way to manage these abnormalities is with thyroid hormone replacement orally (oral levothyroxine). Although his cholesterol is elevated and he is a smoker, telling him to stop smoking and start a low cholesterol diet would not address his underlying disorder. After his thyroid condition has been addressed, a fasting lipid profile should be obtained and smoking cessation counseling should begin. Adding an HMG-CoA reductase inhibitor to diet and smoking cessation or to levothyroxine therapy is premature. Again, a fasting lipid profile should be obtained after thyroxine has been adequately replaced. Admission to the hospital for intravenous levothyroxine is only indicated in cases of myxedema coma where extreme hypothermia, mental status change, seizures, and respiratory depression are seen. These are not present in this patient.

A 54-year-old man is discovered to have hypertension (165/110 mm Hg) on a routine physical exam. On questioning, the patient reveals that he had been suffering from headaches and leg cramps in addition to a feeling of generalized weakness. A careful history further reveals an increased urinary volume. Complete investigations show hypokalemic alkalosis with high serum sodium (154 mEq/L) and low serum potassium (2-4 mEq/L). The ECG shows flattened T waves, long QT, and U waves. The lab reports also show low plasma renin. What is the likely diagnosis? Answer Choices 1 Primary hyperaldosteronism 2 Secondary hyperaldosteronism 3 Primary hypoaldosteronism 4 Secondary hypoaldosteronism 5 Cushing's Syndrome

Primary Hyperaldosteronism Explanation Primary and secondary hyperaldosteronism are both characterized by increased aldosterone production from the adrenal cortex. Primary hyperaldosteronism results from adrenal adenomas (Conn's Syndrome) in 60% of cases. 30% are due to bilateral adrenal hyperplasia. Secondary hyperaldosteronism results from increased activation of the renin-angiotensin system, leading to intensive stimulation of the adrenal cortex. Secondary hyperaldosteronism is generated by extravascular loss of Na+ and water (e.g. cirrhosis, nephrosis, congestive heart failure), renal artery stenosis, or by tumor of the juxtaglomerular cells. Renin secretion from the juxtaglomerular cells is increased under these conditions, leading to increased formation of angiotensin II. Thus, in secondary hyperaldosteronism, both renin and angiotensin levels are high. In primary hyperaldosteronism the renin and angiotensin levels are low, being reduced by the expansion of the extracellular space. Aldosterone stimulates K+ secretion in the kidney; therefore, plasma K+ level is decreased in both primary and secondary hyperaldosteronism. Plasma Na+ level is usually within the physiological range, because the increased Na+ reabsorption is always followed by increased water accumulation. The plasma level of angiotensinogen is normal in both primary and secondary hyperaldosteronism.

A patient describes an insidious onset of vague gastrointestinal symptoms including nausea, vomiting, and abdominal pain. She has been losing weight over the last few weeks as well. Past medical history reveals a new diagnosis of anemia. The patient is not currently taking any medications. Physical exam findings include hypotension and dark freckles with dark, bluish black pigmentation of the mucosal membranes. Serum sodium is decreased, while the potassium and plasma ACTH levels are elevated. What is the most likely diagnosis for this patient? A Acute abdomen B Cushing's syndrome C Primary adrenal insufficiency D Disseminated intravascular coagulation E Diabetic ketoacidosis

Primary adrenal insufficiency The correct choice is C, primary adrenal insufficiency. This disorder presents with symptoms resulting from progressive destruction of the adrenal glands and resultant decrease in aldosterone and cortisol secretion. The presentation can range from subtle to fulminant adrenal crisis. Common clinical findings include hyperpigmentation, weakness, anorexia, nausea, vomiting, diarrhea, and hypotension. Some patients with adrenal insufficiency may initially be treated for an acute abdomen, choice A. In this patient, ACTH levels are known to be elevated, pointing the cause to the pituitary-adrenal axis. Choice B, Cushing's syndrome, presents with signs and symptoms related to an excess in plasma cortisol, such as truncal obesity, hyperglycemia, hypertension, skin changes, and weight gain. Choice D, disseminated intravascular coagulation, is a coagulation disorder presenting with bleeding and thrombosis. Choice E, diabetic ketoacidosis, is an acute complication seen in patients with type 1 diabetes mellitus.

A 60-year-old female presents to her primary care practitioner for her yearly check up. Her past medical history is significant only for a history of repeat kidney stones. She enjoys gardening outside, has no significant family history, and is not taking any medications. Routine mammograms have been normal. During the review of her systems, the patient describes feeling tired lately and has noticed muscle aches over the last few weeks. Initial lab values are shown below: Hemoglobin = 12.0 gm/dL Hematocrit = 36% BUN = 10 mg/dL Creatinine = 0.7 ng/mL Calcium = 12 mg/dL Phosphate = 2.0 mg/dL PTH = 100 pg/mL TSH = 2.0 UIU/mL What is the most likely cause of her symptoms? A Vitamin D deficiency B Hypercalcemia of malignancy C Primary hyperparathyroidism D Factitious hypercalcemia E Hyperthyroidism

Primary hyperparathyroidism The correct choice is C, primary hyperparathyroidism. This patient has the characteristic signs and symptoms of hypercalcemia, along with the lab findings of primary hyperparathyroidism with elevated calcium, low phosphate, and elevated PTH. Other common presenting features include bone pain, paresthesias, polyuria, and gastrointestinal symptoms. Females are affected three times as much as males, and it is typically found accidentally. Choice A, vitamin D deficiency, is unlikely since she enjoys gardening outside in the sun. Vitamin D deficiency can be seen is association with primary hyperparathyroidism, but is not the cause of this disorder. Choice B, hypercalcemia of malignancy, is unlikely with her history of normal mammograms. Serum PTH is typically normal in this disorder as well. Choice D, factitious hypercalcemia, is unlikely since she doesn't take any calcium supplements or other medications. Choice E, hyperthyroidism, can be a cause of hypercalcemia if the patient is suffering from thyrotoxicosis, but this patient's TSH is within the reference range.

A 28-year-old woman presents because she has not gotten her period in 2 months. She is worried that she may be pregnant, but she is concerned because she and her boyfriend broke up about 2 months ago. She does not want to be a single mother. The patient mentions a discharge from both breasts. You order a urinary HCG, which is negative. Question What diagnostic study would you order next? Answer Choices 1 Prolactin level 2 Free T4 level 3 Pelvic ultrasound 4 Thyroid ultrasound 5 Serum calcium

Prolactin level Explanation The clinical picture is suggestive of amenorrhea/galactorrhea caused by increased levels of prolactin. Of women with amenorrhea and galactorrhea, 70% have hyperprolactinemia. So based on statistics, prolactin levels would be the next study to order. Free T4 levels are to measure for hypothyroidism, which can cause hyperprolactinemia. But the patient does not demonstrate signs of hypothyroidism (cold intolerance, skin, hair and nail changes, weight gain, etc.). A pelvic ultrasound would be for detection of pregnancy/ectopic pregnancy, but the HCG is negative. A thyroid ultrasound would be to detect thyroid masses, but this patient does not have a thyroid condition. Serum calcium would be used to detect hyperparathyroidism, which can cause hyperprolactinemia; however, the patient does not demonstrate signs of hyperparathyroidism (not feeling well, abdominal pain, kidney stones, bone pain and mental changes).

Over a period of several months, a 62-year-old man has developed erectile dysfunction. He has no history of neurologic, kidney, or cardiovascular disease or diabetes mellitus. He takes a multivitamin and an occasional ibuprofen for aches and pains. He has never smoked cigarettes, drinks 1-2 glasses of wine with dinner on weekends, and uses no mind-altering drugs. Physical examination is remarkable only for bilateral gynecomastia. What is the most likely diagnosis? A breast cancer B depression C prolactinoma D steroid abuse E testicular cancer

Prolactinoma Men with prolactinomas may experience erectile dysfunction, infertility, and, less commonly, gynecomastia. Breast cancer in men (A) presents as a usually as a unilateral mass. Men with depression (B) may have erectile dysfunction, but not gynecomastia. Steroid abuse (D) is associated with gynecomastia, but the patient would likely be showing other signs and symptoms. Testicular cancer (E), specifically germ cell cancer, is associated with gynecomastia in 5% of cases but this man has no testicular mass or swelling.

A 45-year-old patient came in to see his health care provider today, to discuss the results of his last annual assessment. He was told that he had developed type 2 diabetes mellitus. One of the recommendations from the physician assistant included a visit to an ophthalmologist. The physician assistant was concerned after seeing new capillaries, macular edema, and fibrous tissue within the retina during his funduscopic exam. What type of ocular complication does this patient most likely have at this time? A Background retinopathy B Closed angle glaucoma C Macular degeneration D Diabetic cataracts E Proliferative retinopathy

Proliferative Retinopathy The correct choice is E, proliferative retinopathy. The distinguishing factor in the patient's presentation, which signals this disorder, is the development of newly formed vessels. Proliferative retinopathy is the leading cause of blindness in the United States. Up to 20% of patients with type 2 diabetes have retinopathy at the time of diagnosis. Choice A, background retinopathy, or simple retinopathy includes retinal microaneurysms, hemorrhages, exudates, and edema, without new vessel formation. Choice B, closed angle glaucoma, is relatively uncommon in patients with diabetes, except after cataract extraction. Choice C, macular degeneration, is not associated with diabetes mellitus specifically. Choice D, diabetic cataracts, tends to occur in patients with diabetes earlier than the general population, and may correlate with the severity of the disease.

A 45-year-old patient came in to see his health care provider today, to discuss the results of his last annual assessment. He was told that he had developed type 2 diabetes mellitus. One of the recommendations from the physician assistant included a visit to an ophthalmologist. The physician assistant was concerned after seeing new capillaries, macular edema, and fibrous tissue within the retina during his funduscopic exam. What type of ocular complication does this patient most likely have at this time? A Background retinopathy B Closed angle glaucoma C Macular degeneration D Diabetic cataracts E Proliferative retinopathy

Proliferative retinopathy The correct choice is E, proliferative retinopathy. The distinguishing factor in the patient's presentation, which signals this disorder, is the development of newly formed vessels. Proliferative retinopathy is the leading cause of blindness in the United States. Up to 20% of patients with type 2 diabetes have retinopathy at the time of diagnosis. Choice A, background retinopathy, or simple retinopathy includes retinal microaneurysms, hemorrhages, exudates, and edema, without new vessel formation. Choice B, closed angle glaucoma, is relatively uncommon in patients with diabetes, except after cataract extraction. Choice C, macular degeneration, is not associated with diabetes mellitus specifically. Choice D, diabetic cataracts, tends to occur in patients with diabetes earlier than the general population, and may correlate with the severity of the disease.

What is the definitive treatment for the majority of patients presenting with mild symptoms of hyperthyroidism secondary to subacute thyroiditis? A Subtotal thyroidectomy B Oral methimazole C Symptomatic treatment D Radioactive iodine E Antibiotics

Symptomatic treatment The correct choice is C, symptomatic treatment. Most patients with subacute thyroiditis and symptoms of hyperthyroidism require only symptomatic treatment, with non-steroidal anti-inflammatory medications and/or beta blockers, for any cardiac symptoms including palpitations and tachycardia. Occasionally, patients may require a course of prednisone for this acute inflammatory condition. Most patients will recover spontaneously within a few months. Choices A, B, D, and E are not necessary in this condition. Most cases of subacute thyroiditis are associated with viral infections, and resolve without additional thyroid medications.

A 70-year-old woman with long-standing type II diabetes mellitus presents with complaints of pain in the left ear with purulent drainage. On physical examination, the patient is afebrile. The pinna of the left ear is tender, and the external auditory canal is swollen and edematous. The white blood cell count is normal. Question What organism is most likely to grow from the purulent drainage? Answer Choices 1 Candida albicans 2 Haemophilus influenza 3 Moraxella catarrhalis 4 Pseudomonas aeruginosa 5 Streptococcus pneumoniae

Pseudomonas Aeruginosa Explanation Ear pain and drainage in an elderly diabetic patient must raise concern about malignant external otitis. The swelling and inflammation of the external auditory meatus strongly suggest this diagnosis. This infection usually occurs in older, poorly controlled diabetics, and it is almost always caused by P. aeruginosa. It can invade contiguous structures, including the facial nerve or temporal bone, and it can even progress to meningitis. S. pneumoniae, H. influenzae, and M. catarrhalis frequently cause otitis media, but not external otitis. Candida albicans almost never affects the external ear.

A 28-year-old woman presents because she is concerned about her 2, 4 and 6-year-old children. She doesn't have any symptoms, and there is nothing abnormal in her medical records. However, her father was recently diagnosed with medullary thyroid cancer due to the presence of MEN 2 A, and she is asking if the condition hereditary; if so, she wants to know what can be done in order to prevent her children from developing the malignancy. Question You explain the autosomal dominant trait of the disease; what advice should you provide? Answer Choices 1 RET mutation test in the patient 2 RET mutation test in children 3 Plasma calcitonin levels in children 4 Prophylactic total thyroidectomy 5 Carcinoembryonic antigen test in children

RET mutation test in the patient Multiple endocrine neoplasia type 2A (MEN 2A) is a rare familial autosomal dominant multiglandular syndrome. MEN 2A is diagnosed clinically by the occurrence of 2 or more specific endocrine tumors in a single individual or in close relatives: medullary thyroid carcinoma; hyperparathyroidism (hyperplasia or multiple adenomas), pheochromocytomas (often bilateral); or Hirschsprung disease. Patients with MEN 2A should have genetic testing for the RET proto-oncogene (RET) mutation. RET mutation test identifies about 95% of affected individuals. If the mutation is present, the first-degree relatives may be tested for the specific RET mutation; each kindred has a specific RET codon mutation that determines MEN 2 A variation (e.g., the age of onset or the aggressiveness of medullary thyroid cancer). Both the specific mutation and family history should guide you in the further management. Because of the incomplete penetrance, about 30% of those with such mutations never manifest endocrine tumors. In the affected family members, you may consider regular check-ups for MEN 2-associated pheochromocytomas (24 hours urine for elevated catecholamines and catecholamine metabolites, especially vanillyl-mandelic acid at the earliest age possible), medullary thyroid carcinoma (serum calcitonin or serum calcitonin/pentagastrin-stimulated calcitonin levels), thyroid tumours (ultrasound and fine-needle aspiration), and annual testing for parathyroid abnormalities (simultaneously elevated serum calcium and parathyroid hormone levels with an elevated urinary calcium to creatinine ratio). The results should guide you in the scheduling profilactic thyroidectomy. An RET mutation test should be performed in children if the mother has RET mutation. Such testing is highly accurate and sensitive for presymptomatic identification of at-risk individuals and test for the specific mutation will determine the best next step in order to reduce morbidity and mortality. Plasma calcitonin levels in children to test for the presence of medullary thyroid carcinoma should be considered if they are positive for the specific RET mutation in order to decide the further steps of management. This test is also used to look for the possible recurrence of MTC after treatment. Prophylactic thyroidectomy shall be planned only in the presence of the specific mutation and case histories of family members. Children harboring this mutation are advised to have a prophylactic total thyroidectomy by the age 6 years. While carcinoembryonic antigen (CEA) is often elevated in thyroid cancers, it is not specific; higher than normal levels may also be due to the breast, colon, lung and pancreatic cancers; other causes include liver and gallbladder disorders, heavy smoking, inflammatory bowel disease, lung infections, pancreatitis, peptic ulcer disease etc. However, CEA can be useful after the thyroidectomy if one suspects a reccurrence of the tumor.

What is the definitive treatment of choice for elderly patients diagnosed with Graves' disease? A Beta blocking agents B Levothyroxine C Methimazole D Total thyroidectomy E Radioactive iodine

Radioactive Iodine The correct choice is E, radioactive iodine. This is the treatment of choice in the elderly because it is efficient, easy to take, and inexpensive. Choice A, beta blocking agents, are useful in the treatment of symptoms of hyperthyroidism, such as palpitations, but they are not a definitive treatment for the disorder. Choice B, levothyroxine, is used for thyroid hormone supplementation in patients with hypothyroidism. Choice C, methimazole, is an anti-thyroid drug that has increased toxicity in the elderly and is more useful in younger patients with mild hyperthyroidism. Choice D, total thyroidectomy, has a limited role as a treatment for hyperthyroidism, and is associated with increased morbidity in the elderly.

A 19-year-old woman presents with weight loss, tiredness, and a decreased appetite. She mentions some nausea, and she vomited "once", but she denies diarrhea, illicit alcohol, and drug use, palpitations, tremor, and skin changes. Her urine dips positive for glucose and ketones. What would you likely see on physical exam? Answer Choices 1 Rapid deep breathing 2 Respiratory rate is unchanged 3 Moist mucous membranes 4 Heart rate is decreased 5 Respiratory rate is decreased

Rapid deep breathing Explanation The patient described here likely has diabetic ketoacidosis, a form of metabolic acidosis. Her respiratory response to metabolic acidosis should be rapid deep breathing (Kussmaul breathing), leading to respiratory alkalosis. Symptoms of diabetic ketoacidosis may be highly variable; symptoms include nausea, vomiting, thirst, polyuria, abdominal pain, and shortness of breath. Physical findings may include hypotension (secondary to volume depletion), tachycardia, dry mucous membranes, hypercapnia, abdominal tenderness, and lethargy. In patients with known diabetes, precipitating events may include inadequate insulin intake, infections (e.g., urinary), myocardial infarctions, and pregnancy; history and related diagnostic testing should focus on these precipitating factors. In patients with both known and new onset diabetes, suspicion of diabetic ketoacidosis warrants an evaluation of serum glucose, ketones, and basic chemistries. Potassium is often elevated due to cellular shift and may fluctuate with insulin treatment; bicarbonate is often decreased secondary to the acid buffering, and urea nitrogen is often elevated due to volume depletion. Management focuses on correcting salt and water deficits over the first 24 hours (first with saline and then with 1/2 normal saline when hemodynamically stable); administering intravenous insulin (0.15 units/kg bolus, then 0.1 unit/kg/hour); and frequent monitoring of serum glucose, potassium and bicarbonate, and acetone during treatment.

A 50-year-old woman has type 2 diabetes mellitus; she has been taking 500 mg of metformin daily for approximately 1 year. She has a hemoglobin A1c of 6.5%. She states that she carefully watches her diet, walks 1 mile almost every day, and monitors her blood sugar levels in the evening and before eating breakfast. Her blood sugar levels are often higher in the morning than they were the previous evening even though she does not eat anything. To what process can this be attributed? Answer Choices 1 Release to the bloodstream of glucose from glycogen in muscle 2 Release to the bloodstream of glucose from glycogen in the liver 3 Release to the bloodstream of glucose (from gluconeogenesis) from amino acids occurring in the liver 4 Continuous absorption of glucose into the bloodstream formed by the digestive system 5 Release to the bloodstream of glucose (from gluconeogenesis) from lactate occurring in the muscle

Release to the bloodstream of glucose from glycogen in the liver Explanation The correct response is the release to the bloodstream of glucose from glycogen in the liver. The major source of glucose for the body during an overnight fast is the glucose released from glycogen stored in the liver. As blood glucose levels decrease, glucagon levels increase. The binding of glucagon to receptors in the liver triggers the activation of a cascade, resulting in the activation of the enzyme glycogen phosphorylase. This enzyme catalyzes the release of glucose 1-phosphate from glycogen. The glucose-1-phosphate is converted to glucose-6-phosphate by the action of phosphoglucomutase; the resulting glucose 6-phosphate is converted to glucose by glucose-6-phosphatase. The glucose formed is then released to the bloodstream; therefore, even though the patient did not eat overnight, the action of glucagon on the liver generates glucose which could increase the patient's serum blood glucose. Although muscle contains large amounts of glycogen which can be broken down, the muscle lacks the enzyme glucose 6-phosphatase; therefore, glucose 6-phosphate remains in the muscle to be used to generate ATP for muscle contraction. Most individuals have adequate liver glycogen stores to provide glucose for an overnight fast; they will not be breaking down protein to generate amino acids which could be used for gluconeogenesis (the generation of glucose from non-carbohydrate sources).

A 50-year-old male is seen with a routine check-up. He is concerned about the possibility of developing diabetes mellitus. He has a negative family history of diabetes. He has no signs or symptoms of diabetes and he is not overweight. Without any risk factors for diabetes, what is the recommended screening protocol for this patient according to the American Diabetes Association (ADA)? A screen all men over 25 years of age every five years B screen all men over 35 years of age every two years C screen everyone over 45 years of age every three years D no screening is necessary without risk factors E no screening is necessary without a family history of diabetes

Screen everyone over 45 yo every 3 years The correct choice is C, screen everyone older than 45 years of age every three years. In addition, the ADA recommends screening for younger people if they are overweight and have at least one additional risk factor, such as positive family history, hypertension, and/or vascular disease. The other choices are not recommended by the ADA for screening the general population for diabetes mellitus.

A 64-year-old man with chronic renal failure presents for a follow up. His physical examination is unremarkable. His pulse, BP, and temperature are as follows: 80/min, 130/90 mm Hg, and 98.9 F. The patient's lab results are as follows: Total serum Ca++: 7.5 mg/dl Serum ionized Ca++: 3.5 mg/dl (reference range- 4.5-5.3 mg/dl) Serum phosphate: 5.2 mg/dl Serum PTH (parathyroid hormone): 13 pmol/L (reference range- 1.0-6.8 pmol/L) Serum urea: 40 mg/dl Serum creatinine: 2.2 mg/dl Question What is the best explanation for the patient's results? Answer Choices 1 Tertiary hyperparathyroidism 2 Primary hyperparathyroidism 3 Primary hypoparathyroidism 4 Secondary hyperparathyroidism 5 Secondary hypoparathyroidism

Secondary Hyperparathyroidism Explanation The candidate in this case is a patient with chronic renal failure who presents for follow up. His serum urea and creatinine are elevated. The patient's serum calcium is decreased. The decreased serum total and ionized calcium in the presence of increased PTH is consistent with secondary hyperparathyroidism. The presence of chronic renal failure suggests that the parathyroid hyperfunctioning is arising in the setting of impaired renal function. In chronic renal failure, hyperphosphatemia and decreased renal production of dihydroxy vitamin D3 leads to hypocalcemia. This stimulates the parathyroid glands to increase PTH secretion in order to increase the level of serum calcium by increasing intestinal absorption and bone resorption. In spite of the increased level of PTH, both the total and ionized serum calcium are decreased. These findings are consistent with the diagnosis of secondary hyperparathyroidism in the setting of chronic renal failure. Primary hyperparathyroidism is excluded by decreased serum calcium levels. Tertiary hyperparathyroidism is marked by hypercalcemia and elevated PTH levels, which are not present in this patient. Hypoparathyroidism is excluded by the increased levels of PTH values.

A 40-year-old patient presents to your walk-in clinic with symptoms of hyperhydrosis, oily skin, daytime sleepiness, and snoring. Upon exam, you note large fleshy heel pads and hands with sweaty palms. The patient also has coarse facial features. When asked, the patient isn't aware of any major changes in her face or body. She has not seen another health care provider in many years and has not kept up with any health care maintenance schedule. The patient lives with her ill mother and is not currently employed. Which of the following screening tests would best aid in the diagnosis of this patient? A CT of the chest and abdomen B Thyroid scan C 24-hour urine for catecholamines D Serum calcitonin level E Serum IGF-I level

Serum IGF I level The correct choice is E, serum IGF-I level. Age and gender matched levels of IGF-I are elevated in patients with acromegaly. IGF-I is the mediator of most of the effects of GH on the body, and lead to the proliferation of bone, cartilage, and soft tissue. Although GH levels may be elevated in patients with acromegaly, they are secreted in a pulsatile fashion and are not consistently elevated. Serum GH levels are not the best screening test for acromegaly. Choices A, a CT of the chest and abdomen, and B, a thyroid scan, are expensive imaging studies that are not usually used as screening tests. They also have no role in the routine workup of patients with suspected acromegaly. Choice C, 24-hour urine for catecholamines, is a test that can be used in the work up of patients with suspected pheochromocytoma. Choice D, serum calcitonin levels, are associated with medullary thyroid cancer and other thyroid disease.

A 40-year-old woman presents for check-up. She is married with 2 sons, and she experiences occasional constipation; she attributes it to her diet. She mentions having mild anxiety. She does not smoke; she consumes alcohol only recreationally, and she has a body weight of 85 kg. She is well nourished and communicates well. Lung, cardiac, abdominal, and neurological examinations are unremarkable. Her thyroid is normal in size and consistency. Her menses are normal, and her drug history is negative, except for "hormone pills" for contraception. Her family history is remarkable for a mother with hypothyroidism and osteoporosis. Question What screening option is appropriate for this woman at present? Answer Choices 1 Follow-up 2 Free T3 assay 3 Total T4 assay 4 Serum TSH assay 5 Total T4 and T3 resin uptake assays

Serum TSH assay Explanation The American Thyroid Association recommends that adults, particularly women, high-risk individuals, and those with symptoms and signs attributable to thyroid dysfunction, should be screened for thyroid dysfunction by measuring serum TSH concentration beginning at the age of 35 years; the screening should be repeated every 5 years (Ladenson et al., 2001). The absence of classic manifestations of hypo- or hyperthyroidism or lack of risk factors would not defer a screening TSH assay.

What is the most sensitive test available for the screening and detection of early thyroid dysfunction? A Radioactive iodine uptake B Serum T3 resin uptake C Serum total T4 level D Serum TSH level E Thyroid scan

Serum TSH level The correct choice is D, serum TSH level. Very small changes in serum TSH level can provide clues that there are changes in the functioning of the hypothalamic-anterior pituitary-thyroid axis. The test is easier and less expensive than any thyroid imaging tests. All of the other choices can be used in the work up of patients for thyroid dysfunction, but they are less sensitive, and many are more expensive and more invasive.

Over the past year, a 43-year-old man has been experiencing episodes of palpitations, anxiety, and sweating. Multiple endocrine neoplasia (MEN) syndrome runs in his family. On physical examination, his blood pressure is elevated. His body habitus is normal. His mucosal membranes are unremarkable. A 24-hour collection of his urine tests positive for vanillylmandelic acid. He is diagnosed with a pheochromocytoma. What is the most likely diagnosis? Answer Choices 1 Wermer's syndrome 2 Sipple's syndrome 3 Sturge-Weber syndrome 4 Mucosal neuroma syndrome 5 Carcinoid syndrome

Sipple's Syndrome Explanation Sipple's syndrome (multiple endocrine neoplasias, type IIA) consists of pheochromocytoma, hyperparathyroidism, and medullary carcinoma of the thyroid. Multiple endocrine neoplasias refer to a group of familial syndromes in which various pathologies of the endocrine glands can be found. Multiple endocrine neoplasias, type I (Wermer's syndrome): Hyperparathyroidism Pancreatic islet cell tumors Pituitary tumors Multiple endocrine neoplasias, type IIA (Sipple's syndrome): Hyperparathyroidism Pheochromocytoma Medullary carcinoma of the thyroid Multiple endocrine neoplasias, type IIB (mucosal neuroma syndrome): Pheochromocytoma Medullary carcinoma of the thyroid Mucosal neuromas Marfanoid body habitus Sturge-Weber syndrome usually includes a facial port wine stain and seizures. There can also be other neurologic findings, such as weakness. Carcinoid syndrome presents with flushing, respiratory wheezing, and diarrhea. Carcinoid syndrome is due to the activity of tumors from neuroendocrine cells.

Consideration should be given to screening patients with type 1 diabetes mellitus should also be screened for which of the following: A sarcoidosis B Sheehan's syndrome C Sjögren's Syndrome D thyroid disease

Sjögren's Syndrome Type 1 diabetes mellitus (T1DM) is an autoimmune disease. As such, patients have a significantly higher risk of other autoimmune diseases, including celiac and thyroid disease. Most recommendations include screening for both diseases in patients with T1DM.

A 63-year-old woman presents with shortness of breath, cough, and proximal muscle weakness of 1-month duration. On clinical exam, she is noted to have a blood pressure of 156/102 mm Hg, facial flushing, mild hirsutism, truncal obesity, marked proximal muscle weakness of both the upper and lower extremity, and hyperpigmentation over the palms and back of the neck. Laboratory exam reveals hypercortisolism and increased ACTH. Which of the following would be the most likely primary diagnosis in this patient? A lymphoma B ovarian cancer C renal cell carcinoma D small cell lung carcinoma

Small cell lung cancer Tumor cells may secrete hormones that have the same biologic actions as the normal hormone. This patient's symptoms are consistent with adrenocorticoid hyperfunction. The most common cause of ectopic ACTH syndrome is small cell lung carcinoma. This should be suspected in any patient with risk factors for lung cancer.

A 13-year-old boy has a 3-year history of type I diabetes mellitus without significant problems; he presents for an acute visit. The patient states his morning glucose readings have risen from an average of 100 mg/dL to over 200 mg/dL over the past 2 weeks. He is currently taking 25 units of glargine (Lantus) at bedtime and 8 units of aspart (Novolog) before meals. The patient states that he has been compliant with his insulin and diet. His mother states that he recently has been having nightmares and night sweats, but he denies any other complaints. ROS and physical exam are otherwise unremarkable. What is the patient most likely experiencing? Answer Choices 1 A tonic seizure 2 Diabetic ketoacidosis 3 Somogyi effect 4 Dawn phenomenon 5 The onset of puberty

Somoygi Explanation The correct answer is that the patient is most likely experiencing the Somogyi effect. The Somogyi effect begins with an episode of nocturnal hypoglycemia; this triggers the body's counter-regulatory response, secreting glucagon, epinephrine, and cortisol plus growth hormone to raise the blood sugar, resulting in elevated morning levels or rebound hyperglycemia. The nocturnal hypoglycemia is due to hyperinsulinemia as a result of insulin therapy in conjunction with decreased evening caloric intake or extra calories utilized for activity, especially in the evening hours. The symptoms of nocturnal hypoglycemia (restlessness, profuse sweating, and nightmares) occur most often when the patient is asleep; it is possible that only a parent or partner may notice them. Patients also tend to awaken with a severe morning headache. Adrenergic symptoms of hypoglycemia, such as palpitations, tremor, anxiety, hunger, and nausea, are the result of the counter-regulatory hormones secreted at blood glucose levels < 70 mg/dL; they may be noticeable by the patient. Neuroglycopenia, with symptoms of weakness, confusion, and combativeness, occurs if the blood glucose falls below 45 - 50 mg/dL; if untreated, they may result in seizure, coma, or death. Checking blood sugars at 3:00 am or continuous glucose monitoring are useful to detect nocturnal hypoglycemia. This patient would benefit from correcting the nocturnal hyperinsulinemia with a bedtime snack, limiting evening exercise, decreasing the dinner dose of aspart, or decreasing the daily dose of glargine if his basal blood sugars, fasting, and pre-meal are low. The Dawn phenomenon also results in morning hyperglycemia, but this is due to the physiologic secretion of growth hormone in the early morning hours, which produces insulin resistance. In this case, there is insufficient insulin to overcome this inherent insulin resistance, so patients arise with elevated morning blood sugars. This can also be documented by checking 3:00 am blood sugars or continuous glucose monitoring, and it is corrected by adding insulin to the evening or nighttime regimen. The onset of puberty would result in accelerated secretion of growth hormone, resulting in morning hyperglycemia, but it would not explain the symptoms of nocturnal hypoglycemia. Tonic seizures occur most frequently in childhood and during sleep. Although his mother did not specifically note any tonic (sustained) or clonic (rapid contraction/relaxation) activity, other forms of seizures can present with night terrors. Likewise, untreated hypoglycemia can result in seizures. Although a seizure disorder would be included in the differential diagnosis, it would not explain the recent onset of morning hyperglycemia. The symptoms of diabetic ketoacidosis include nausea, vomiting, abdominal pain, polyuria, acetone-odor breath, and Kussmaul breathing.

A 33-year-old woman presents with multiple symptoms; they have been worsening over several months. She notes fatigue, loss of appetite, weight loss, abdominal pains, nausea, as well as an "achiness" throughout her muscles and joints. The patient's family has noticed her skin appears very tanned, despite lack of sun exposure; she also uses more salt on food than ever before. The patient has stable hypothyroidism and takes daily medication. She denies any fever/chills, bowel changes, recent travel, psychosocial changes, or other triggers for her various symptoms. Her past medical history is remarkable for hypothyroidism. She takes 88 mcg of levothyroxine. She has had 3 pregnancies and a tubal ligation as her only surgery. She has no allergies; she denies the use of tobacco, alcohol, and drugs. She works in retail. Her family history is unremarkable. Vitals are normal with a BP of 104/64 mm Hg. On physical exam, the patient does have some hyperpigmentation, especially in the nipple/areola region. The remainder of her exam is normal. Weight loss of 8 lbs. is noted from a prior visit. Fasting, morning labs are drawn with results below. Urinalysis - Normal Complete blood count - Normal Comprehensive metabolic panel- Mildly decreased sodium and elevated potassium, rest normal Thyroid stimulating hormone (TSH)- Normal (1.8 uIU/mL) Adrenocorticotropic hormone (ACTH)- Elevated (>200 pg/mL) C-reactive protein- Normal Cortisol (morning level)- Decreased (<3 mcg/dL) Question What pharmacologic intervention is the most appropriate for this patient's condition? Answer Choices 1 Decrease the levothyroxine dose 2 Increase the levothyroxine dose 3 Start furosemide 4 Start hydrocortisone 5 Start sodium bicarbonate

Start hydrocortisone This patient is presenting with Addison disease, a deficiency in the adrenal corticosteroids. The low cortisol and the elevated ACTH confirm this. The symptoms of Addison disease are many and can be confused with other conditions, including some thyroid conditions. They may include: fatigue, weakness, abdominal pain, nausea, anorexia, weight loss, anxiety, irritability, depression, arthralgias and myalgias. Addison disease is a considered an autoimmune condition, and it is often linked with other autoimmune disorders, such as hypothyroidism. Once the condition is identified, the treatment of choice is to start hydrocortisone. The treatment may be IV in acute conditions; oral treatment is appropriate in less urgent cases, and it is usually lifelong. This patient reports stable hypothyroidism and compliant medication-taking. Furthermore, her TSH result is normal, so it would be inappropriate to decrease the levothyroxine dose or increase the levothyroxine dose. Some common symptoms of hypothyroidism include fatigue, depression, weight gain, constipation, and nail/hair changes. Symptoms of over-replaced thyroid medication can be anxiety, palpitations, and/or weight loss. In certain instances, when hyperkalemia (elevated potassium) is identified, one treatment may be to start furosemide, due to its effects on renal potassium excretion. However, this patient's hyperkalemia is mild, and once the underlying condition (Addison disease) is treated, the hyperkalemia should resolve without other measures. This patient has a mild hyponatremia (low sodium); likewise, it should resolve with appropriate replacement of steroids, such as hydrocortisone. It would be inappropriate to start sodium bicarbonate for several reasons. This medication is not intended to raise serum sodium. (Hyponatremia has many potential causes, and fluid status needs to be addressed. Intravenous saline is often part of acute treatments of hyponatremia.) Furthermore, the sodium bicarbonate is primarily used for acidosis, and as an antacid, it is not appropriate for hyponatremia.

A 55-year-old man presents for a follow-up on recent cholesterol tests. He has a past medical history of 2 coronary stents placed 3 days ago after presenting to the emergency department with chest pain and a 40 pack-year history of smoking (he quit smoking 2 years ago). He was not on any medications when he presented to the ER. Cholesterol tests reveal: total cholesterol 200 mg/dL; LDL 100 mg/dL; HDL 40 mg/DL; and triglycerides 395 mg/dL. Question What is the most appropriate treatment to reduce future cardiovascular risk in this patient? Answer Choices 1 Fibrates 2 Fish oil 3 Nicotinic acid 4 Plasmapheresis 5 Statin

Statin Explanation In patients with mild to moderate hypertriglyceridemia (150-500 mg/dL), the main indication for therapy is reduction of cardiovascular (CV) risk. Statins are the best therapy for CV risk reduction, and although they are not the most effective medication for reducing elevated triglycerides, they should be the first line therapy for a patient with a high CV risk. Many studies have shown that fibrates, nicotinic acid, and fish oil can lower triglyceride levels, but they do not lower the cardiac risk as much as statins. This patient is a high-risk cardiac patient because of his recent history of blocked coronary arteries, and his best option at this time is treatment with a statin drug. Plamapheresis is reserved for patients with severe hypertriglyceridemia who are at risk for pancreatitis.

A 49-year-old man presents to the office complaining of general malaise with muscle aches, anorexia, fever, and severe pain over his anterior neck radiating to his ears. He states that he was ill about 2 weeks ago with a sore throat, but it resolved within a few days. On palpation, the thyroid gland is enlarged and tender. His laboratory workup shows a high T4 level and increased erythrocyte sedimentation rate (ESR). What is the most appropriate therapy for this patient's disease? A levothyroxine sodium B PTU therapy C radioiodine ablation D surgery E supportive therapy only

Supportive therapy only This is subacute, painful thyroiditis. This is a self-limiting disorder that at most requires symptomatic therapy. In mild cases, analgesics (ASA) are sufficient for pain relief and to decrease the inflammation. Prednisone may bring more relief if needed. Transient hypothyroidism should be treated as well.

A 30-year-old woman with a past medical history of Sheehan's syndrome and a craniopharyngioma, for which she has followed up with her neurologist, presents to her primary care office with increasing polyuria, urinary frequency, enuresis, and nocturia. She states that as busy office worker, she finds it difficult to drink water throughout the day and has developed an "unquenchable thirst." She also finds that her sleep is disturbed and, as a result, has mild daytime fatigue and somnolence. She denies a family and personal history of diabetes mellitus, thyroid dysfunction, illicit drug use, smoking, and alcohol use. She denies fever, chills, polyphagia, weight changes, vaginal discharge, hematuria, or abnormal urinary odor. Question What physical exam finding would be most likely in this patient? Answer Choices 1 Suprapubic distension 2 Hypertension 3 Adnexal mass 4 Cervical motion tenderness 5 Diminished vibratory sensation

Suprapubic distension Explanation This patient's presentation and past medical history are suggestive of central diabetes insipidus. Decreased secretion or action of antidiuretic hormone usually manifests as diabetes insipidus, a syndrome characterized by the production of abnormally large volumes of dilute urine. The 24-hour urine volume is >50 mL/kg body weight, and the osmolarity is <300 mosmol/L. The polyuria produces symptoms of urinary frequency, enuresis, and/or nocturia, which may disturb sleep and cause mild daytime fatigue or somnolence. It also results in a slight rise in plasma osmolarity that stimulates thirst and a commensurate increase in fluid intake (polydipsia). Overt clinical signs of dehydration are uncommon unless fluid intake is impaired. The physical examination findings vary with the severity and chronicity of DI. The examination findings may be entirely normal. Hydronephrosis, with pelvic fullness, bladder enlargement, flank pain or tenderness, or pain radiating to the testicle or genital area may be present. Unless the thirst mechanism is impaired or access to fluid is restricted, dehydration is not seen. Aside from an enlarged bladder, no specific signs of diabetes insipidus exist. Hypotension, not hypertension, would be anticipated with significant and chronic dehydration. Adnexal masses and cervical motion tenderness reflect underlying gynecological illnesses not present in this patient. Diminished vibratory sensation suggests peripheral neuropathy, which is not a known manifestation of diabetes insipidus.

A 69-year-old woman with a 2-year history of asymptomatic hyperparathyroidism presents with a 1-week history of generalized anxiety and intermittent confusion. 6 months ago, she had normal results on serum and urine protein electrophoresis, mammography, radiography of the chest, and ultrasonography of the neck. The immunoreactive parathyroid hormone level was 5 times the normal level, and the ionized serum calcium level was also increased. She has no other major medical problems. Question Which of the following is the preferred treatment for patients with this presentation? Answer Choices 1 Surgical exploration of the neck 2 Low-calcium diet with oral phosphate supplementation 3 Oral bisphosphonate therapy 4 Subcutaneous calcitonin therapy 5 Estrogen therapy

Surgical exploration of the neck Explanation Over 90% of cases of hypercalcemia in the elderly are caused by primary hyperparathyroidism or malignancy. The rest are caused by medications (e.g. thiazide diuretics), vitamin D intoxication, sarcoidosis, renal failure, hyperthyroidism, and milk-alkali syndrome. Most patients with primary hyperparathyroidism are asymptomatic and are identified incidentally. These patients should be monitored every 6 to 12 months. Most patients with primary hyperparathyroidism do not have disease progression; therefore, immediate therapy is unnecessary. Symptoms of primary hyperparathyroidism manifest in the form of fatigue, weakness, alterations in mental status, hypertension, kidney stones, and gastrointestinal symptoms such as anorexia, nausea, and constipation. Osteoporosis is also common finding. This patient now has symptoms of anxiety and confusion that warrant intervention. Surgery in the form of bilateral neck exploration, and parathyroidectomy is the preferred method of treatment in patients with symptomatic hyperparathyroidism. Medical management is an alternative to surgery in asymptomatic patients, patients who refuse surgery, and those who are considered poor surgical candidates. Adequate hydration should be undertaken, and medications that increase serum calcium levels (e.g., thiazide diuretics, lithium) should be avoided or discontinued. Maintaining adequate vitamin D intake is also important; vitamin D deficiency stimulates parathyroid hormone secretion and further bone resorption. Oral bisphosphonates, such as alendronate, can be used in patients with primary hyperparathyroidism in order to help preserve bone density; however, this is not the preferred treatment method. Subcutaneous calcitonin is also effective in this regard, but it does not treat the primary cause of the disorder. Oral phosphate supplementation and a low calcium diet can help lower serum calcium levels, but surgery remains the most definitive treatment. In postmenopausal women, estrogen replacement therapy suppresses bone turnover, decreases urinary calcium excretion, and increases bone mineral density. However, estrogen therapy also increases the risk of stroke, coronary artery disease, and breast cancer. Therefore, it is not first-line therapy for primary hyperparathyroidism in patients who are good surgical candidates.

2 weeks after an upper respiratory infection, an adolescent presents with diarrhea, sweating, and increased heart rate. Physical examination reveals a tremor and a swollen, tender, and painful thyroid gland. Pulse rate is 110/min and blood pressure is 130/60 mm Hg. Refer to the attached laboratory studies. What is the appropriate treatment? Answer Choices 1 Symptomatic (NSAID, beta-blocking drug) 2 L-Thyroxin sodium 3 Thyroidectomy 4 Methimazole 5 Propylthiouracil

Symptomatic (NSAID, beta-blocking drug) The correct response is symptomatic (NSAID, beta-blocking drug). The clinical picture indicates hyperthyroidism caused by subacute thyroiditis. Sweating, diarrhea, elevated pulse rate, and wide pulse pressure with higher systolic and lowered diastolic pressures are characteristic signs of hyperthyroidism. Subacute thyroiditis usually follows a viral infection and has a benign course. A very low radioiodine uptake, while serum T3 and T4 are high, confirms the diagnosis of subacute thyroiditis. This reflects follicular cell injury and the consequential inability to trap iodine. Symptomatic treatment with non-steroidal anti-inflammatory drugs and beta-blockers is sufficient. Thyroid function returns to normal within a few weeks. The benign course of the disease makes other, more active, treatments of hyperthyroidism unnecessary.

An adult patient was recently diagnosed with type 2 diabetes mellitus. She met with her diabetic educator to discuss suggested changes to her diet and exercise regimens. What percent of her total daily calories should be in the form of carbohydrates? A 5% to 15% B 20% to 25% C 35% to 40% D 45% to 65% E 70% to 80%

The correct choice is D, 45% to 65%. The American Diabetes Association recommends a diet with 45% to 65% carbohydrates, 25% to 35% fats, and 10% to 35% protein.

A 40-year-old woman presents with anxiety, difficulty sleeping, rapid heartbeat, and tremor in her hands. You note the presence of bulging eyes and suspect Graves' disease. What blood levels should be taken so that the disease can be confirmed? Answer Choices 1 Thyroid stimulating hormone 2 Thyroid peroxidase 3 Protein-bound iodine 4 Thyroglobulin 5 Thyrotropin-releasing hormone

TSH Explanation Graves' disease is a type of hyperthyroidism caused by a generalized over-activity of the entire thyroid gland. The patient appears hot and flushed, and the thyroid gland enlarges in this condition. It is believed that Graves' disease is an autoimmune disorder. Antibodies are produced against certain proteins on the surface of thyroid cells, stimulating those cells to overproduce thyroid hormones. In this condition antibodies are produced against the thyrotropin receptor, thyroglobulin, thyroid peroxidase, and sodium-iodide symporter. The circulating autoantibodies continuously stimulate the thyroid gland via the thyrotropin receptor. Associated suppression of the pituitary thyrotropin level is due to increased production of thyroid hormones. The onset of the disease is gradual, and the symptoms may be mistaken for nervousness due to a stressful life situation. Weight loss occurs, and it is followed by other symptoms, such as trembling, muscle weakness of the upper arms and thighs, and insomnia. The pituitary gland releases Thyroid Stimulating Hormone (TSH) in response to insufficient levels of thyroid hormone. Communication between the pituitary gland and the thyroid gland through TSH levels controls the levels of thyroid hormone in the blood. If the levels of thyroid hormone are low, then the levels of TSH will rise. The measurement of TSH in the blood is taken as a measure of thyroid function. In Graves' disease, there is a suppressed level of thyrotropin along with elevated levels of free T4 or T3 hormone levels. The new generation (III generation) assay of TSH is very sensitive and has revolutionized diagnosis of Graves' disease by providing accurate measurements of very low TSH levels. Suppression of TSH is an early and highly sensitive marker of thyrotoxicosis. Estimation of thyroid peroxidase, protein-bound iodine, thyroglobulin, and thyrotropin-releasing hormone are not as reliable as TSH assay.

A 43-year-old woman has an upper respiratory viral infection, with runny nose, fever, sneezing, and congestion. 2 days later, she notices pain in the front of her neck with swelling in that area. She notices more fatigue, palpitations, sweating, and anxiety; as a result, she has been unable to go to work. She has a temperature of 100.2 F, pulse 106/min, BP 110/70 mm of Hg, and respirations 16/minute. She looks nervous and tired. Neck exam reveals a tender vague mass anteriorly. Oropharynx is erythematous without exudates. No other masses are appreciated in the neck bilaterally. Lungs are clear and the abdomen normal. She is alert and oriented, and other than a fine tremor in her hands, no neurological deficit can be elicited. Lab work is ordered. What is the next best step in the work up of this patient? Answer Choices 1 Rapid streptococcal test and if negative, throat culture 2 TSH, T4, ESR 3 Serum thyroglobulin and thyroid ultrasound 4 Heterophile antigen test 5 Complete blood count with peripheral smear

TSH, T4, ESR Explanation This patient has a viral infection followed by infection of the thyroid gland, causing subacute thyroiditis, which is also known as de Quervain's syndrome. Neck pain and a tender mass in the neck anteriorly indicate thyroid inflammation. Sometimes, the swollen thyroid may cause obstructive symptoms. Sudden release of thyroid hormone may cause transient hyperthyroidism; this is the case in this patient who is experiencing nervousness, tremors, palpitations, and sweating. Lab tests show low TSH, high free T4, high ESR, high C reactive protein, and raised serum thyroglobulin. Treatment is symptomatic; anti-inflammatory medication, steroids, and beta-blockers should be given for hyperthyroid symptoms. A reasonable approach is to relieve the pain with high doses of a non-steroidal, such as aspirin or naproxen. If the symptoms are not relieved in 2 - 3 days, prednisone at 30 - 40 mg daily should be given in the absence of contraindications. The usual treatment can last for 4 - 8 weeks. Once the pain is controlled, prednisone should be tapered. If the patient has severe pain, a steroid can be started right away. In the absence of relief from these 2 modes of treatment, the diagnosis should be questioned. Symptoms from release of thyroid hormones need not be treated if they are mild; however, if they are more severe, propranolol or atenolol help in symptomatic relief from anxiety, tremors, and palpitations. The differential diagnosis of this case includes group A beta hemolytic streptococcal infection, infectious mononucleosis, cervical lymphadenitis, and occasionally scrofula. In streptococcal infections of the throat, tonsils are enlarged, exudates are seen, and cervical lymphadenopathy is obvious. Infectious mononucleosis is also characterized by high fever, pharyngitis, and lymphadenopathy, especially posterior cervical, with hepatosplenomegaly in many cases. Fatigue is prominent without tremors, anxiety, or palpitations. The heterophile antibody test is positive, with atypical lymphocytes in the peripheral smear. Scrofula is a tuberculous lymphadenitis caused by mycobacterium scrofulaceum. It is characterized by single gradually enlarging mass in neck on 1 side; the mass is non-tender, rarely indurated, erythematous, or with a draining sinus. Cervical lymphadenitis has multiple etiologies and can be caused by viral, bacterial, fungal, or (rarely) mycobacterial organisms. Multiple lymph nodes are usually palpable, and they may be tender. The underlying condition is usually obvious.

Patients with suspected familial hypercholesterolemia have serum cholesterol levels > 300 mg/dL and are at increased risk of atherosclerosis. Which of the following physical exam findings are nearly pathognomonic for familial hypercholesterolemia? A Tendon xanthomas B Lipomas C Bouchard's nodes D Carotid bruits E Visceral obesity

Tendon xanthomas The correct choice is A, tendon xanthomas. These are depositions of cholesterol rich substances that can present in any tendon as a mass-like lesion. They are most commonly found in the Achilles, patellar, and hand extensor tendons. Choice B, lipomas, are benign, soft, moveable subcutaneous tumors made from fat cells. Choice C, Bouchard's nodes, are painless nodules on the PIP joints, commonly seen in patients with osteoarthritis. Choice D, carotid bruits, may be heard with auscultation of the neck during the physical exam in patients with artherosclerosis of the carotid arteries. Choice E, visceral obesity, is a risk factor for diabetes and atherosclerosis, but is not pathognomonic of familial hypercholesterolemia.

A 45-year-old overweight man presents for a routine physical. He relates that he is always thirsty and urinates frequently at night. You order several blood tests, including a test for hemoglobin A1C. The levels of hemoglobin A1C level are 8.0, and you conclude that he has type II diabetes mellitus. You prescribe Glyburide, a sulfonylurea drug. After 3 months, his hemoglobin A1C level is 7.5. You add the drug Metformin, which drops the hemoglobin A1C to 6.8. What is the primary cause of the reduction in the patient's hemoglobin A1C level? Answer Choices 1 Reduction of fatty acid oxidation by metformin 2 The ability of metformin to decrease the production of glucose by the liver 3 Increased processing of proinsulin to insulin 4 Increased incorporation of GLUT4 into adipose tissue plasma membranes 5 Reduced clearance of glyburide from patient's serum

The ability of metformin to decrease the production of glucose by the liver Explanation Metformin (Glucophage®) is a hypoglycemic drug that is effective only in the presence of insulin because it acts to increase the action of insulin. It does not alter insulin production. One important mechanism by which metformin works is decreasing the production of glucose by the liver. Metformin acts by activating the AMP-activated protein kinase (AMPK), a major cellular regulator of lipid and glucose metabolism. In addition, glucose utilization by muscle is increased. Given in combination with a sulfonylurea, metformin lowers blood glucose concentrations more than either drug alone. This treatment is not effective for patients with insulin-dependent diabetes (Type I) because the beta cells are not viable. Type II diabetes mellitus, also called non-insulin-dependent diabetes mellitus (NIDDM), has a slow onset and is typically seen in patients over the age of 40. Insulin is present, but it is not properly utilized. Controlling diet can often control NIDDM. In cases where diet does not lower blood glucose levels, drugs called oral hypoglycemic agents (such as the sulfonylureas and metformin) are prescribed. Sulfonylurea agents (glyburide, glipizide, and glimepiride) act to lower blood glucose levels by causing the pancreas to secrete more insulin and by allowing target cells to better use the insulin. The main mechanism of action of the sulfonylurea drugs is to stimulate the release of endogenous insulin from the beta cells of the pancreas. The major insulin responsive glucose transporter, GLUT4, is not affected directly by metformin. The conversion of proinsulin to insulin occurs in the secretory granules as they mature, and it is regulated by glucose levels. Metformin has been shown to have no effect on the oxidation of fatty acids such as oleate.

A 39-year-old woman presents for an annual appointment. She has a history of diabetes mellitus, but she is otherwise healthy. Her recent lab work reveals that her serum glucose is well controlled by a combination of lifestyle and 1 medication. Although this is a routine appointment, she does present with recent onset of some unusual symptoms. Physical examination of the patient also reveals some abnormalities. Question What history and physical examination combination would prompt a workup for Addison's disease? Answer Choices 1 Cramping, numbness around mouth, and tingling in distal extremities combined with carpopedal spasm and exaggerated deep tendon reflexes 2 Weight gain, decreased libido, menstrual irregularity combined with moon facies, and increased adipose tissue in neck 3 Fatigue, dizziness, salt craving combined with increased pigmentation of her lips and hand creases, as well as hair loss 4 Bone pain, flank pain, and generalized anxiety combined with hypertension and mental confusion 5 Hair loss, decreased energy, and constipation combined with periorbital puffiness and dry, coarse skin

The correct answer is the combination of fatigue, dizziness, and salt craving with increased pigmentation of her lips and hand creases, as well as hair loss. Addison's disease is the result of adrenal cortex destruction, which causes deficiencies in cortisol, aldosterone, and androgens. Patients can have any combination of various systemic symptoms, including fatigue, weakness, nausea, vomiting, abdominal pain, dizziness, chronic diarrhea, depression, salt craving, and a decreased tolerance for cold. In addition, the physical examination findings can also vary. Some possible findings are hypotension, weight loss, vitiligo, hair loss, and increased pigmentation. The increased pigmentation can occur in the hand creases, dental-gingival margins, buccal and vaginal mucosa, lips, areolas, scars, and pressure points. The increased pigmentation is a result of increased secretion of β-lipoprotein. The combination of weight gain, decreased libido, and menstrual irregularity with moon facies and increased adipose tissue in neck are indicative of a patient with Cushing's disease. Cushing's disease is essentially the opposite of Addison's disease; its signs and symptoms are caused by an excess of adrenocorticotropin rather than the deficiency seen in Addison's disease. Patients with Cushing's disease can also experience hypertension, hirsutism, depression, easy bruising, muscle weakness, obesity, thin skin, skeletal growth retardation, and acne. The combination of cramping, numbness around mouth, and tingling in distal extremities with carpopedal spasm and exaggerated deep tendon reflexes could occur in a patient with hypoparathyroidism. This disorder is a result of a deficiency of parathyroid hormone from congenital absence, injury, surgery, or other diseases. Most of the symptoms associated with hypoparathyroidism are a result of hypocalcemia. The combination of bone pain, flank pain, and anxiety with hypertension and mental confusion could occur in a patient who has hyperparathyroidism. This disorder is characterized as dysfunction in the body's regulatory system for parathyroid hormone. While up to 75% of patients can be asymptomatic, the hypercalcemia associated with hyperparathyroidism can result in the classic complaints of painful bones, renal stones (causing flank pain), abdominal groans, and psychic moans. The combination of hair loss, decreased energy, and constipation with periorbital puffiness and dry, coarse skin is indicative of hypothyroidism. Hypothyroidism is typically an autoimmune disease or a result of prior radiation therapy or thyroid surgery.

A patient was recently diagnosed with type 1 diabetes mellitus. A treatment plan was initiated, with a combination regimen of insulin. Which of the following types of insulin works well with a rapidly acting insulin, such as insulin lispro, to provide 24-hour coverage for the patient? A NPH insulin B Regular insulin C Insulin aspart D Insulin glargine E Humalog 75/25

The correct choice is D, insulin glargine. This is the only long acting insulin listed. The combination of a long acting insulin with a rapidly acting insulin provides physiologic insulin replacement to the patient. This regimen provides postprandial control after meals and basal coverage throughout the day and night. Choice A NPH insulin, can be used by itself in two or more injections throughout the day. Choice B, regular insulin, can be used instead of rapid acting insulin, and not in combination with it. Choice C, insulin aspart, is a type of rapidly acting insulin and would not be used in combination with another rapidly acting insulin. Choice E, Humalog 75/25, is a combination insulin preparation with 75% intermediate acting insulin and 25% insulin lispro.

A patient is being treated for hypothyroidism. His condition has been stable for the past year. What blood test should be ordered and monitored yearly in this patient? A Total T4 B T3 resin uptake C Thyroid releasing hormone D Thyroid stimulating hormone E Free T3

The correct choice is D, thyroid stimulating hormone. This test will help to monitor patient adherence with thyroid hormone supplementation, as well as to fine tune the dose so that the TSH remains within the reference range. Choices A, B, and E can be used in the work up of patients for primary hypothyroidism, but alone each test is not helpful to monitor chronic disease in patients. Choice C, serum thyroid releasing hormone, is used more commonly when investigating secondary hypothyroidism

A 40-year-old patient presents to your walk-in clinic with symptoms of hyperhydrosis, oily skin, daytime sleepiness, and snoring. Upon exam, you note large fleshy heel pads and hands with sweaty palms. The patient also has coarse facial features. When asked, the patient isn't aware of any major changes in her face or body. She has not seen another health care provider in many years and has not kept up with any health care maintenance schedule. The patient lives with her ill mother and is not currently employed. Which of the following screening tests would best aid in the diagnosis of this patient? A CT of the chest and abdomen B Thyroid scan C 24-hour urine for catecholamines D Serum calcitonin level E Serum IGF-I level

The correct choice is E, serum IGF-I level. Age and gender matched levels of IGF-I are elevated in patients with acromegaly. IGF-I is the mediator of most of the effects of GH on the body, and lead to the proliferation of bone, cartilage, and soft tissue. Although GH levels may be elevated in patients with acromegaly, they are secreted in a pulsatile fashion and are not consistently elevated. Serum GH levels are not the best screening test for acromegaly. Choices A, a CT of the chest and abdomen, and B, a thyroid scan, are expensive imaging studies that are not usually used as screening tests. They also have no role in the routine workup of patients with suspected acromegaly. Choice C, 24-hour urine for catecholamines, is a test that can be used in the work up of patients with suspected pheochromocytoma. Choice D, serum calcitonin levels, are associated with medullary thyroid cancer and other thyroid disease.

A 43-year-old woman is found to have a palpable thyroid nodule. It is 1.5 cm in size, and it is located in the right lobe without regional lymphadenopathy. Upon questioning, the patient denies noticing this or any increase in the size of her thyroid. She denies hoarseness, a personal or family history of thyroid disease, and thyroid cancer. Question What is the most common form of thyroid cancer for which she is at risk? Answer Choices 1 Follicular 2 Papillary 3 Medullary 4 Hürthle cell 5 Anaplastic

The correct response is papillary. Papillary thyroid cancer (PTC) is the most common type of thyroid cancer in the United States; it has a prevalence rate of 80 - 90% of all thyroid cancers. It arises from the follicular epithelial cells of the thyroid, as does follicular thyroid cancer (FTC), which is the 2nd most frequently occurring thyroid cancer (accounting for 5 - 15% of all cases). Together, they are defined as well-differentiated thyroid carcinomas; when identified early they have a good, and frequently curable, prognosis. Psammoma bodies (PBs) are concentric lamellated calcified structures, observed most commonly in papillary thyroid carcinoma (PTC) Hürthle cell carcinoma (oncocytic) is a variant of FTC. Also arising from the follicular epithelial cells is undifferentiated (anaplastic) thyroid carcinoma; this accounts for < 5% of thyroid cancers and carries a worse prognosis than the well-differentiated histologic forms. It is aggressive and often refractory to treatment. Medullary thyroid cancer (MTC) arises from the parafollicular cells (C cells), which are calcitonin-producing and account for 5 - 10% of thyroid carcinomas. 75% of MTC patients will have sporadic cases, whereas 25% will have hereditary forms such as multiple endocrine neoplasia 2A (MEN 2A), MEN 2B, or familial (FMTC). MEN 2A is associated with hyperparathyroidism, a high risk of MTC, and pheochromocytoma. MEN 2B is associated with MTC, an increased risk of pheochromocytoma, and mucosal neuromas on the lips, tongue, and GI tract.

A 43-year-old woman was diagnosed with type II diabetes mellitus 6 months ago; she presents for follow up. The patient is presently asymptomatic. She currently receives oral hypoglycemics. The patient has no other significant medical history. Her glucose, acetone, lactate, and glycated hemoglobin are performed, and they are reported as follows: Serum Lactate 0.5 mEq/L Serum Acetone 0.4 mg/dl Fasting serum Glucose 130 mg/dl Glycated Hemoglobin 6% What is the best way to describe the patient's diabetic state? Answer Choices 1 The diabetic state is in good glycemic control 2 The diabetic state is not well controlled and the patient needs addition of another oral hypoglycemic agent 3 The diabetic state is not controlled and the patient's oral hypoglycemic dose needs to be increased 4 The patient's diabetic state is not in control and requires initiation of insulin therapy 5 The patient's diabetic state is well controlled and requires reduction in the dose of the oral hypoglycemic drug

The diabetic state is in good glycemic control

A 30-year-old woman presents to the office with polyuria, fatigue, and a chronic white vaginal discharge with vaginal pruritis. She has been having the discharge off and on for the past 6 months with recurrent treatment failures. Which of the following is the most likely diagnosis? A type 2 diabetes mellitus B hyperthyroidism C hypothyroidism D diabetes insipidus

Type 2 DM Polyuria, polydipsia, and fatigue are all findings that can be consistent with both type 1 and type 2 diabetes. Any woman who presents with a chronic vaginal discharge or chronic vaginal pruritis should be screened for type 2 diabetes.

A 35-year-old woman presents with a 2-month history of palpitation and nervousness. She mentions that she always feels hot, even if the weather is cold. Her menses have been irregular lately. She has had no fevers recently. She was also told that her eyes are "weird looking". On examination, her blood pressure is 150/70 mm Hg, and her pulse rate 89 beats per minute. Her eyes show exophthalmos, and she also has lid lag on looking down. Thyrotoxicosis is suspected. What follows is the thyroid scan result. Question What is the most likely diagnosis? Answer Choices 1 Graves' disease 2 Toxic multinodular goiter 3 Hashimoto thyroiditis 4 Factitious hyperthyroidism 5 Toxic adenoma

The most likely diagnosis in this patient is Graves' disease. Thyrotoxicosis occurs due to thyroid hormone excess, the etiology of which varies from Graves' disease, toxic multinodular goiter, toxic adenoma, thyroiditis, and even functioning metastasis. Symptoms of thyrotoxicosis include hyperactivity, irritability, dysphoria, heat intolerance and sweating, palpitations, fatigue, weakness, weight loss with increased appetite, diarrhea, polyuria, and menstrual irregularities. Graves' disease is the most common cause of thyrotoxicosis. It is an autoimmune disease in which thyroid-stimulating autoantibodies stimulate thyroid receptors to secrete thyroxine. It is more common in women and can be identified by radionucleotide scan in which the whole thyroid shows increased uptake. Here the patient has all Graves' disease characters: she is middle aged with thyrotoxic characters and her Tc-99 scan shows the characteristic Graves' pattern. Toxic multinodular goiter is another cause of thyrotoxicosis in which there are multiple nodules that can be felt with palpation, and it is shown to be hot or active with the radionucleotide scan. It shows localized or patchy uptake in the scan pictures. Hashimoto thyroiditis is an inflammatory condition that follows viral infection. It is associated with a brief period of hyperthyroidism that is followed by hypothyroidism, which may persist throughout life. On Tc scan, it shows decreased uptake. Factitious hyperthyroidism is due to ingestion of the thyroid hormone. Usually this happens in nurses, physicians, or those who have access to medicine. It can be detected by measuring T3 and T4. There is also decreased uptake on thyroid scan. Toxic adenoma is a benign neoplastic nodule, which actively secretes thyroxine. It shows localized or patchy uptake of Tc 99 on thyroid scan.

A 28-year-old woman presents with nervousness and palpitations associated with heat intolerance. On examination, there is no evidence of thyromegaly, but there is a palpable nodule that is "hot" on a thyroid scan. The TSH was low and T3 and T4 were both elevated. Which of the following is the recommended treatment for this patient? A Propylthiouracil (PTU) B Thyroid lobectomy C Total thyroidectomy D Radioiodine ablation

Thyroid Lobectomy In Graves' disease, the thyroid is diffusely enlarged in contrast to a toxic adenoma in which the thyroid is normal sized but with a palpable nodule. Surgery is the treatment of choice for a toxic adenoma. Surgical treatment of a toxic adenoma is a thyroid lobectomy and isthmusectomy. A subtotal or total thyroidectomy is indicated for toxic multinodular goiters or Plummer disease. Thionamides and radioiodine ablation are not effective therapies for toxic adenomas.

A 70-year-old woman presents with a 1-week history of palpitations, dyspnea, and generalized weakness. She also gives history of decreased oral intake and weight loss. The patient has no significant previous medical history. On admission, the patient is afebrile. Pulse is 130/min, BP is 100/68 mm Hg, and RR is14/min. Chest X-ray is normal, and electrocardiogram shows an atrial fibrillation. Skin appears warm and smooth. Question What is the next diagnostic step that would be useful in management? Answer Choices 1 Carcinoembryonic antigen (CEA) test 2 Gastroscopy 3 Colonoscopy 4 Thyroid tests 5 Adrenal functional tests

Thyroid Tests Explanation Other than sinus tachycardia occurring in patients with hyperthyroidism, atrial fibrillation is the most common cardiac arrhythmia. It occurs in 10 -15% of patients with hyperthyroidism and increases significantly in patients over 60 years of age. In the elderly, atrial fibrillation occurs in 25 - 30% of patients with hyperthyroidism. Thyrotoxic atrial fibrillation is associated with increased risk of thromboembolic complications involving the central nervous system. Therefore, it is imperative to perform thyroid function tests to rule out hyperthyroidism in all cases of new-onset atrial fibrillation. The mainstay of treatment in patients with atrial fibrillation and hyperthyroidism is restoration of euthyroid status by using antithyroid drugs. Restoration of the euthyroid status is often associated with conversion to sinus rhythm. However, the rate of conversion is slower in the elderly. Treatment options for atrial fibrillation are rate-control agents, oral anticoagulants, and defibrillation. Other tests, such as CEA antigen test, gastroscopy, colonoscopy, and adrenal function tests, would not be useful for ascertaining the cause of atrial fibrillation.

A mother expresses concern for her teen son after feeling a lump in his neck. He has no history of trauma to his neck. Surgical history is negative, and the patient does not take any medications. The mother tells you that thyroid problems run in the family. The patient has not been ill recently. Upon exam you feel a nontender, firm nodule on the right side of his thyroid with associated cervical lymphadenopathy. His serum TSH level is within the reference range. Radionuclide thyroid scanning demonstrates a "cold" nodule in the right side of the thyroid. What is the most appropriate next step in the work up of this patient? A MRI of the anterior pituitary B CT of the thyroid C MRI of the thyroid D Thyroid nodule fine needle aspiration E Emergent thyroidectomy

Thyroid nodule fine needle aspiration The correct choice is D, thyroid nodule fine needle aspiration. With the advent of fine needle aspiration, it has become much easier, safer, and more reliable to obtain a specimen for biopsy. This patient has several characteristics that increase his risk of malignancy including his gender, young age, firmness of the nodule, and related lymphadenopathy. These, along with the ease of biopsy, suggest this path for diagnostic work up. Not enough information is known to warrant an emergent thyroidectomy, choice E. Choice A, MRI of the anterior pituitary, would be warranted if there was a suspicion of a pituitary cause of the thyroid nodules. Since the TSH is normal and the patient is not presenting with headaches or other pituitary related symptoms, this is not suggested. Choice B, CT of the thyroid, and choice C, MRI of the thyroid, would not provide any additional information after the thyroid scan. They may be helpful prior to any surgery if needed.

A patient is being treated for hypothyroidism. His condition has been stable for the past year. What blood test should be ordered and monitored yearly in this patient? A Total T4 B T3 resin uptake C Thyroid releasing hormone D Thyroid stimulating hormone E Free T3

Thyroid stimulating hormone The correct choice is D, thyroid stimulating hormone. This test will help to monitor patient adherence with thyroid hormone supplementation, as well as to fine tune the dose so that the TSH remains within the reference range. Choices A, B, and E can be used in the work up of patients for primary hypothyroidism, but alone each test is not helpful to monitor chronic disease in patients. Choice C, serum thyroid releasing hormone, is used more commonly when investigating secondary hypothyroidism.

A 42-year-old teacher presents with a 1-year history of hand tremor. She tells you that it appears to be only in her right hand and comes and goes. It is especially embarrassing for her because she writes on the board in front of her students. She denies pain, motor weakness, and abnormal sensation in her right hand. She reports that her father's hand used to "shake" when he drank his coffee. Otherwise, she feels well, and she denies other symptoms. She has not noticed memory, incoordination, or balance problems. Her past medical history is unremarkable, with 3 uncomplicated pregnancies and deliveries; she has no chronic medical conditions. Except for a mild tremor on the patient's right hand when held extended, her physical exam is unremarkable. Her neurological exam is otherwise normal. Question What lab/diagnostic study would be most appropriate for this patient? Answer Choices 1 Computed tomography (CT) of the head 2 Electromyography (EMG) 3 Magnetic resonance imaging (MRI) of the head 4 Serum ceruloplasmin 5 Thyroid stimulating hormone (TSH)

Thyroid stimulating hormone (TSH) would be the most appropriate test at this time. This patient presents with a history and physical consistent with benign essential (or familial) tremor. A TSH would help rule out hyperthyroidism as the cause of this patient's tremor. It is non-invasive, readily accessible, and relatively inexpensive. Unless the history suggests a more serious cause of tremor, no further testing is recommended. A CT or MRI of the head would be recommended if the patient presented with more neurologic findings on exam or a history concerning for tumor growth, such as new headaches. Both tests are quite expensive, and they are unnecessary in cases of benign essential tremor. EMG is done in evaluation of multiple nerve and muscle disorders, such as carpal tunnel syndrome, muscular dystrophy, and myasthenia gravis. Nerve conduction can be measured on EMG. EMG is not recommended or routinely done in the evaluation of essential tremor. Serum ceruloplasmin is useful in the evaluation of the relatively rare Wilson's disease. The tremor of Wilson's disease is a 'wing-beating tremor' and several other findings, such as dysarthria, dystonia, splenomegaly, pallor due to anemia, and Kayser-Fleischer rings (brown to grey-green rings around the cornea), are often present. This patient's history does not suggest Wilson's disease.

A 35-year-old woman presents with weight loss, tremor, and palpitations. She also feels hot, is sweating, and has sleep difficulties and frequent arousals, in addition to frequent bowel movements (3-4 times a day), nervousness, and weakness. Physical examination reveals a slim anxious female with pronounced stare, fine postural and tremor in rest, and slight proximal weakness. Her thyroid is diffusely enlarged and non-tender; her pulse is 100/min; and the rest of examination is within normal limits. Her TSH is low, and free T4 is high. Question What is the most accurate diagnostic test for hyperthyroidism in this patient? Answer Choices 1 MRI of pituitary gland 2 Free T3 3 Fine needle biopsy 4 Color Doppler flow 5 Thyrotropin receptor antibody assay

Thyrotropin receptor antibody assay Explanation Your patient has signs and symptoms of Grave's disease. Grave's disease is an autoimmune disease caused by anti thyroid-stimulating hormone (TSH) receptor. Thyrotropin receptor antibody assay is not required for the diagnosis of Grave's disease, but it will confirm the diagnosis. Sometimes this test is needed to establish the cause of exophthalmos (e.g., in the absence of thyrotoxicosis) or to predict fetal or neonatal thyrotoxicosis. Among the choices offered, it is the most accurate test for Graves' disease. You can order an image of pituitary gland in secondary hyperthyroidism, when both TSH and T4 are high; that is not the case in your patient. You can order free T3 levels in the case of hyperthyroidism when TSH is low and T4 normal and you think that patient might have T3 thyrotoxicosis. Usually those patients will have mild disease, but, otherwise, the clinical picture may be similar to other forms of hyperthyroidism. Elevation of T3 alone can also produce symptoms of thyrotoxicosis, but there will be no exophthalmos. Fine needle biopsy is indicated to rule out or confirm cancer of the thyroid gland. Patients with thyroid cancer usually are euthyroid and have normal TSH and non-functional ("cold") nodule. Color Doppler flow exam may help when a possible nodule is detected on physical exam or may show hypoechogenicity to confirm the diagnosis of Hashimoto's thyroiditis or demonstrate intense vascularity in the case of Graves' disease. However, ultrasound of thyroid gland rarely has a value in the diagnosis of hyperthyroidism and is not specific.

A 42-year-old woman comes in with a "lump in her neck." When she was a teenager, she underwent radiation treatment for "some kind of tumor" in her neck. Ultrasound reveals a 1.5 cm lesion in the left lobe of the thyroid; biopsy confirms papillary carcinoma. Thorough evaluation reveals no evidence of metastasis. Which of the following is the initial treatment of choice for her? A chemotherapy B lobectomy with isthmectomy C radiation therapy D radioactive iodine therapy E total thyroidectomy

Total thyroidectomy Total or near-total thyroidectomy is indicated for this woman. No chemotherapy (A) is available for thyroid cancer. Lobectomy with isthmectomy is reserved for papillary carcinoma that is less than 1 cm in size in persons with no history of radiation exposure and no evidence of metastasis. Radiation therapy (C) is used to treat bone metastasis and anaplastic carcinoma. Radioactive iodine therapy (D) may be used following thyroidectomy to ablate any remnant of the gland and to treat cancer that has metastasized or is otherwise high risk.

A 22-year-old woman presents with an 8-month history of amenorrhea. On further questioning, she admits to backaches, headaches, and acne. Physical examination reveals a female patient with a moon-shaped facies, multiple purple striae, and significant central obesity (body mass index of 36). Question Based on the history and physical exam findings, what is the most appropriate initial clinical intervention? Answer Choices 1 Radiation 2 Chemotherapy 3 Oral prednisone 4 Transsphenoidal selective resection 5 Lumpectomy

Transsphenoidal selective resection Explanation This patient likely is suffering from Cushing's syndrome, which is also known as hypercortisolism. Consequences of excessive levels of circulating cortisol, no matter the etiology, will lead to certain signs and symptoms, such as central obesity, thin extremities, a moon face, a buffalo hump, supraclavicular fat pads, a protuberant abdomen, oligomenorrhea, amenorrhea, and/or (in men) possibly erectile dysfunction. Backaches, headaches, hypertension, acne, purple striae, and impaired wound healing may also be found in these patients. The most commonly used clinical intervention is transsphenoidal selective resection of the adenoma. An adenoma of the pituitary gland is one of the more common etiologies of Cushing's syndrome. Pituitary corticotrophs remain suppressed after this surgery despite the pituitary function returning to normal; for this reason, patients require 6 - 36 months of oral prednisone therapy after surgery. Radiation, chemotherapy,and lumpectomy are not indicated treatment plans when dealing with the type of tumor that leads to Cushing's syndrome.

A female patient presents with a history of hypertension and low plasma HDL. She is asking you if she has metabolic syndrome. She does not have diabetes and she is not obese. Which of the following NCEP ATP III criteria would be needed to confirm that diagnosis? A LDL > 70 mg/dL B LDL > 90 mg/dL C Triglycerides > 100 mg/dL D Triglycerides ≥ 150 mg/dL E Triglycerides > 300 mg/dL

Triglycerides ≥ 150 mg/dL The correct choice is D, triglycerides ≥ 150 mg/dL. The NCEP ATP III criteria for metabolic syndrome include three or more of the following: Central obesity with waist circumference > 102 cm in men and > 88 cm in women Hypertriglyceridemia with serum triglycerides ≥ 150 mg/dL, or the patient is taking medication for hypertriglyceridemia Low HDL level < 40 mg/dL in men and < 50 mg/dL in women, or the patient is taking medication for low HDL Hypertension with blood pressure ≥ 130 mm systolic or ≥ 85 mm diastolic, or the patient is taking medication for high blood pressure Fasting plasma glucose ≥ 100 mg/dL, or a positive diagnosis for diabetes mellitus, or the patient is taking medication for hyperglycemia

Consideration should be given to screening patients with type 1 diabetes mellitus should also be screened for which of the following: A sarcoidosis B Sheehan's syndrome C Sjögren's Syndrome D thyroid disease

Type 1 diabetes mellitus (T1DM) is an autoimmune disease. As such, patients have a significantly higher risk of other autoimmune diseases, including celiac and thyroid disease. Most recommendations include screening for both diseases in patients with T1DM.

A 25-year-old woman is seen today in your office for vague abdominal pain, nausea, anorexia, weight loss, anxiety, and dizziness. Her past medical history is significant for type 1 diabetes mellitus, and her family history is significant for hypothyroidism in several family members. A review of systems reveal a history of amenorrhea. Upon exam, you note hyperpigmentation of her skin and areas of vitiligo, but no mucocutaneous candidiasis. You are not surprised to find that her serum ACTH level is elevated and her serum cotisol is low. Which of the following syndromes should be investigated in this patient? A Type 1 polyglandular autoimmune syndrome type 1 B Type 2 polyglandular autoimmune syndrome type 2 C Multiple endocrine neoplasia type 2A D Multiple endocrine neoplasia type 1 E Metabolic syndrome

Type 2 polyglandular autoimmune syndrome type 2 The correct choice is B, type 2 polyglandular autoimmune syndrome. This patient is presenting with signs and symptoms of adrenal insufficiency including abdominal pain, nausea, anorexia, vomiting, weight loss, anxiety, and hyperpigmentation. Type 2 polyglandular autoimmune syndrome presents most commonly in young women between 20 and 40 years old, with evidence of adrenal insufficiency, type 1 diabetes mellitus, and autoimmune thyroid disease. Because of the strong family history of thyroid disease, it would be prudent to conduct an investigation into this disorder, starting with a serum TSH level. Primary ovarian failure and vitiligo may be symptoms of the autoimmune polyglandular syndrome as well. Choice A, type 1 autoimmune polyglandular syndrome, presents more commonly in childhood with adrenal insufficiency, hypoparathyroidism, and mucocutaneous candidiasis. Choices C and D are inherited disorders, characterized by the development of several different types of endocrine organ neoplasias. Choice E, metabolic syndrome, includes a constellation of several metabolic disorders that increase the risk of cardiovascular disease and diabetes mellitus in the patient.

A 25-year-old woman is seen today in your office for vague abdominal pain, nausea, anorexia, weight loss, anxiety, and dizziness. Her past medical history is significant for type 1 diabetes mellitus, and her family history is significant for hypothyroidism in several family members. A review of systems reveal a history of amenorrhea. Upon exam, you note hyperpigmentation of her skin and areas of vitiligo, but no mucocutaneous candidiasis. You are not surprised to find that her serum ACTH level is elevated and her serum cotisol is low. Which of the following syndromes should be investigated in this patient? A Type 1 polyglandular autoimmune syndrome type 1 B Type 2 polyglandular autoimmune syndrome type 2 C Multiple endocrine neoplasia type 2A D Multiple endocrine neoplasia type 1 E Metabolic syndrome

Type 2 polyglandular autoimmune syndrome type 2 The correct choice is B, type 2 polyglandular autoimmune syndrome. This patient is presenting with signs and symptoms of adrenal insufficiency including abdominal pain, nausea, anorexia, vomiting, weight loss, anxiety, and hyperpigmentation. Type 2 polyglandular autoimmune syndrome presents most commonly in young women between 20 and 40 years old, with evidence of adrenal insufficiency, type 1 diabetes mellitus, and autoimmune thyroid disease. Because of the strong family history of thyroid disease, it would be prudent to conduct an investigation into this disorder, starting with a serum TSH level. Primary ovarian failure and vitiligo may be symptoms of the autoimmune polyglandular syndrome as well. Choice A, type 1 autoimmune polyglandular syndrome, presents more commonly in childhood with adrenal insufficiency, hypoparathyroidism, and mucocutaneous candidiasis. Choices C and D are inherited disorders, characterized by the development of several different types of endocrine organ neoplasias. Choice E, metabolic syndrome, includes a constellation of several metabolic disorders that increase the risk of cardiovascular disease and diabetes mellitus in the patient.

A 32-year-old woman presents with weight gain, acne, and purple striae. Question What test should be ordered to confirm the most likely diagnosis? Answer Choices 1 Serum electrolytes 2 Urinary free cortisol level 3 Blood LH/FSH ratio 4 Blood androgen level 5 Urinary osmolality

Urinary free cortisol level Explanation Cushing syndrome is due to an elevation of corticosteroids in the blood stream; it is diagnosed by measuring urinary free cortisol. Cushing presents with moon faces, facial plethora, supraclavicular fat pads, buffalo hump, truncal obesity, and purple striae. Hypertension, impaired glucose tolerance, and osteopenia may be seen. Causes of Cushing syndrome include administration of exogenous corticosteroids such as prednisolone, pituitary adenoma, adrenal adenoma or carcinoma, and exogenous production of ACTH (such as oat cell tumor of lung). Urinary free cortisol estimates the 24-hour cortisol in urine. A value about 3 - 4 times higher than normal suggests Cushing syndrome. Values higher than normal, but not 3 - 4 times, and values within the normal range will need repeat testing in suspected cases. Other tests are available, such as the dexamethasone suppression test. During this test, 1 mg of dexamethasone is given at 11 pm, and the cortisol level is measured at 8 am on the subsequent morning. Serum electrolytes, androgen level, luteinizing hormone level, and urine osmolality are not useful in the diagnosis of Cushing syndrome.

A 37-year-old woman presents with increased thirst. She states that she is always thirsty and has cravings for ice water. She has been urinating frequently, typically with large amounts, and notes nocturia. She finds herself unable to restrict her fluid intake. Urinalysis shows pale urine with osmolarity of 200 mOsm/L and no presence of bacteria, glucose, or ketones. Question What testing would be appropriate in this patient? Answer Choices 1 Oral glucose tolerance test 2 Vasopressin challenge test 3 Cosyntropin stimulation test 4 Dexamethasone suppression test 5 Radioactive iodine uptake test

Vasopressin challenge test Explanation Vasopressin challenge test is correct. The patient is showing signs of diabetes insipidus. Diabetes insipidus is caused by one of two mechanisms: (1) impaired release of antidiuretic hormone (ADH) from the hypothalamus (central diabetes insipidus) or (2) impaired response of the kidneys to ADH (nephrogenic diabetes insipidus). The vasopressin challenge test is performed by obtaining urine measurements before and after administering desmopressin acetate (a synthetic vasopressin). Those with central diabetes insipidus will have an increase in urine osmolality following desmopressin administration. Those with nephrogenic diabetes insipidus will not have any change in urine osmolality following desmopressin administration. Oral glucose tolerance test is incorrect. The oral glucose tolerance test is used in the diagnosis of diabetes mellitus. Cosyntropin stimulation test is incorrect. The cosyntropin stimulation test is used in the diagnosis of Addison's disease (adrenal insufficiency). Dexamethasone suppression test is incorrect. The dexamethasone suppression test is used in the diagnosis of Cushing syndrome (hypercortisolism). Radioactive iodine uptake test is incorrect. The radioactive iodine uptake test is used in the work-up of hyperthyroidism.

A 30-year-old female presents to your office for a routine physical exam. She has not seen a health care provider in many years. Upon talking with the patient, you find out that she had been diagnosed with hypertension several years ago, but was unable to afford the antihypertensive medications that were prescribed to her. She has no complaints at this time. Upon exam of the head and neck, you note widened spaces between her lower incisor teeth and a large, fleshy nose. Her skin is oily and she demonstrates mild proximal muscle weakness. Her EKG reveals a left axis deviation and widened QRS. What is the most likely rationale for her clinical presentation? A Diabetes mellitus B Cushing's syndrome C Hypothyroidism D Acromegaly E Clinical depression

acromegaly The correct choice is D, acromegaly. Patients with acromegaly have an abundance of growth hormone secretion. This leads to excessive growth of many areas of the body including soft tissue. Patients with acromegaly also have an increased incidence of hypertension and left ventricular hypertrophy. None of the other choices will cause this patient's constellation of symptoms. Patients with many endocrine disorders may develop weaknesses as seen in this patient, but the large nose and widely spaced teeth are characteristic of acromegaly.

A 30-year-old man is brought to the emergency department by his wife for abdominal pain, nausea, vomiting, and diarrhea. She says he has been getting "worse and worse" for at least the past two months. He is "too tired" and his "muscles hurt too much" to play golf, formerly their favorite leisure activity. He has been depressed and reticent, instead of his usual talkative self. He won't eat much of anything, even when she cooks his favorite meals and she is sure he has lost considerable weight because his clothes "hang off him." He has refused medical evaluation until today, when the abdominal pain of approximately a week increased and he began having the vomiting and diarrhea. Initial impression is that the man is quite tanned, although it is the middle of winter. Closer examination reveals dark pigmentation in his skin folds and on the buccal mucosa. This presentation most strongly suggests which of the following? A adrenocortical insufficiency B diabetic ketoacidosis C hypercortisolism D hyperglycemic hyperosmolar state E hypothyroidism

adrenocortical insufficiency This is a fairly classic presentation of adrenocortical insuffiency (Addison's disease). Diabetes mellitus, type 1, that has led to ketoacidosis (B) could also present with weight loss, nausea, and vomiting; but is also accompanied by hyperphagia and polydipsia, and does not have the hyperpigmentation associated with Addison's disease. Patients with hypercortisolism (Cushing Syndrome) (C) present with central obesity, hirsutism, thin skin, poor wound healing, and a host of other problems including emotional lability. Those with hyperglycemic hyperosmolar state (D) are typically older, have a high body mass index, and present with lethargy, confusion, and dehydration. Patients with hypothyroidism (E) usually have weight gain and constipation along with lethargy, fatigue, and weakness.

A 23-year-old patient with type 1 diabetes mellitus (DM) has been having difficulty sleeping at night. Usually around 3 am the patient will wake up feeling sweaty, nauseated, and tachycardic. He has recorded the following blood glucose levels: 10 PM- 90 mg/dL 3 AM- 40 mg/dL 7 AM- 200 mg/dL What advise is the best for this patient? A stop eating a bedtime snack B increase the evening regular dosage C decrease the evening Lente dosage D exercise before going to bed at night

decrease the evening Lente dosage

Your patient states she has been gaining weight for no apparent reason over the past year and is concerned that she might have hypothyroidism. What other historical information would support a diagnosis of hypothyroidism? A anxiety B diarrhea C depression D palpitations E heat intolerance

depression The correct answer is (C). Depression is a common presenting symptom of hypothyroidism. Weight gain can occur with hypothyroidism. Other symptoms may be weakness, fatigue, and menorrhagia. Hoarseness may also be a presenting symptom. The other choices are common symptoms of hyperthyroidism.

42-year-old female complains of weight gain (especially in her abdomen) over the past 8 months. She also has noted that her skin bruises easily. Her husband has noted she seems to be very moody lately and she is worried about their relationship. Furthermore, her hair seems to be getting thinner and she is now getting acne like she had in her teenage years. She wonders if this is due to menopause since her periods have stopped suddenly about a year ago. On physical examination her BP = 170/50, P = 82, T = 98.2˚F. You note the following findings on examination (see picture). What diagnostic test is indicated initially to confirm your suspected diagnosis? A cosyntropin stimulation test B MRI pituitary C dexamethasone suppression test D 24-hour urine for protein E serum protein electrophoresis

dexamethasone suppression test The correct answer is (C). The patient's symptoms are consistent with a diagnosis of Cushing's syndrome (or disease). Her physical examination findings of hypertension and abdominal obesity with the classic purple striae also support the diagnosis. The initial diagnostic test of choice would be the dexamethasone suppression test. If the test is positive, further confirmatory testing is done which would also help to identify the cause. An MRI of the pituitary is appropriate if further testing suggests the possibility of a pituitary adenoma as the cause of the Cushing's syndrome, but is not used as an initial diagnostic test for Cushing's disease. A cosyntropin stimulation test, choice (A), is indicated for the diagnosis of Addison's disease.

A 34-year-old woman presents with worsening headaches. She says that the headaches are present throughout the day and that she has been feeling nauseous. Lately she has also noticed difficulty in seeing vehicles overtake her on the freeway. She has had several close calls due to this handicap. Her periods, which were previously regular are now irregular, with heavy bleeding every 3 - 4 months. She has also noticed a milky discharge from both nipples. Her pregnancy test is negative. An MRI of the brain shows a mass compressing the hypothalamic pituitary axis. Question What is the best course of treatment? Answer Choices 1 Irradiation of the tumor 2 Surgical excision 3 Catheter ablation 4 Dopamine agonists 5 External beam laser coagulation

dopamine agonist he most likely diagnosis is a pituitary adenoma compressing the HP axis. The standard of care at the present time is the use of dopamine agonist drugs, such as bromocriptine and cabergoline. These drugs reduce prolactin levels to normal in 70 - 100% of patients. The presence of the pituitary mass blocks the action of dopamine on the prolactin-secreting cells and increases the secretion of prolactin. Cabergoline is an alternative to bromocriptine with fewer side effects. However, it is a much more expensive drug, and it is usually used when bromocriptine is not tolerated. Pergolide has been taken off the market due to associated heart valve damage with its use. Followup may be done by measurement of the fasting serum prolactin level. Surgery is incorrect, as it is usually indicated only in failure of medical therapy, continued visual field loss despite medication, or failure to tolerate medical therapy. Radiation therapy is incorrect. Although effective, it is seldom used due to the risk of permanent hypopitutarism after treatment. External beam coagulation and ablation are both incorrect. They are distractors that have no role in therapy.

One of your patients is requesting your help. He has seen three physicians in the past 3 months and is still having symptoms. He states, "I keep having these episodes of feeling like I am going to die. Out of the blue I feel real nervous, I get a splitting headache, break out in a sweat, and even feel like I am trembling. Sometimes I feel my heart beating out of my chest but don't have any pain or shortness of breath." He has had numerous tests including a cardiac stress test, multiple EKGs, complete blood count (CBC), basic metabolic panel (BMP), thyroid tests, and computed tomography (CT) scans of his head, which were normal. His last physician told him that he had an anxiety disorder and should try medications; he declined. His only medical problem is hypertension, which has worsened recently. He is currently taking lisinopril 20 mg QD, amlodipine 10 mg QD, and HCTZ 25mg QD. His physical examination is unremarkable except a BP of 190/92, P = 74. What is the best test to order to confirm your suspected diagnosis? A dexamethasone suppression test B sleep study C renal artery ultrasound D plasma fractionated free metanephrines E thyroid uptake scan

he correct answer is (D). The patient's symptoms and uncontrolled hypertension with a previous negative evaluation for cardiac, electrolyte, or thyroid causes highly suggests a pheochromocytoma as the possible cause. Plasma fractionated free metanephrines is a very sensitive test for the diagnosis of pheochromocytoma. Another test to consider would be evaluation of the total urinary metanephrines. A dexamethasone suppression test, choice (A), is used to r/o Cushing's syndrome, which is unlikely due to the normal physical examination findings. A sleep study, choice (B), (suggesting sleep apnea) and renal artery ultrasound, choice (C), (suggesting renal artery stenosis) are used to diagnose secondary hypertension, but would unlikely explain all the symptoms in the scenario. A thyroid uptake scan, choice (E), is not indicated since there is no indication of abnormal thyroid testing suggesting hyperthyroidism, or examination stated findings suggestive of a thyroid nodule.

You are evaluating an 80-year-old female for the first time. She has a history of mild Alzheimer's disease, for which she takes Aricept. She states that she feels fine but her daughter feels she is depressed and has been complaining of not feeling well. Her daughter admits that the patient has a history of primary hyperparathyroidism. What laboratory results would be most consistent with her diagnosis of hyperparathyroidism? A high serum calcium B low intact PTH C low cortisol D low urine calcium E high cortisol

high serum calcium The correct answer is (A). The hallmark of primary hyperparathyroidism is a high serum calcium and high intact PTH. A low intact PTH is consistent with hypoparathyroidism. The urine serum calcium is usually high in primary hyperparathyroidism. Cortisol is related to endocrine conditions affecting the adrenal cortex.

A young child and his parents have been adhering to the treatment plan for type 1 diabetes, as discussed with their health care provider. It includes a change in diet, as well as blood glucose and ketone monitoring. They noticed that the amount of insulin needed decreased after the first two weeks. What is this time period commonly called? A Postprandial control phase B Glucose tolerance time C Pre-diabetic period D Mature onset diabetes of youth E Honeymoon phase

honeymoon phase The correct choice is E, honeymoon phase. During this time, some pancreatic beta cell function may recover, although within eight weeks to two years most patients will show absent or negligible pancreatic beta cell function. Choice A, postprandial control phase, choice B, glucose tolerance time, and choice C, pre-diabetic period, are not true time periods. Choice D, mature onset diabetes of youth, is a subgroup of autosomal dominant inherited disorders, characterized by diabetes in non-obese older children that are not ketosis prone and generally do not need insulin therapy to control their disease. This type of diabetes accounts for up to 5% of diabetes in North America and Europe.

Your supervising physician asks you to advise him which finding is least likely to be suggestive of a thyroid malignancy in your 49-year-old female with a small palpable thyroid nodule. Which of the following choices would be least likely to suggest malignancy in this patient? A ultrasound showing lesion with microcalcifications B ultrasound showing a lesion of > 1 cm C hot nodule on 123I uptake scan D ultrasound showing a solid lesion E cold nodule on 123I uptake scan

hot nodule on 123I uptake scan C The correct answer is (C). A hot nodule, which is a hyperfunctioning thyroid nodule, suggests a benign etiology. The other choices, including ultrasound findings of microcalcifications, solid lesions, and lesions > 1 cm, should increase your index of suspicion for possible malignancy. Cold nodules are nonfunctioning thyroid nodules, which should increase your suspicion, especially in combination with suspicious ultrasound and/or clinical examination findings.

A 30-year-old patient presents 2 months postthyroidectomy. The patient has had symptoms of increased irritability, muscle spasms, and hair loss for the past month. On physical examination, a positive Chovstek sign is noted. Which of the following is the most likely diagnosis? A hypothyroidism B hypopituitarism C hypoparathyroidism D hypogonadism

hypoparathyroidism Hypoparathyroidism commonly presents following thyroidectomy surgery. This patient has classic signs and symptoms of a low calcium level and hypoparathyroidism. Chovestek sign is a physical exam finding that is positive after tapping in front of the ear in the facial nerve region. When doing this, the muscle contracts. When the calcium level is low, this occurs. Hypothyroidism can occur following a thyroidectomy but the symptoms are not the same.

A 14-year-old boy presents to the emergency department with his parents. He has a history of type 1 diabetes, and has had bronchitis for the last few days. He is now presenting with difficulty breathing, worsening fatigue, polydipsia, and polyuria. His last fingerstick glucose at home this morning was 350 mg/dL. Which of the following patterns of breathing are characteristic of this complication of diabetes? A Cheyne stokes respiration B Bradypnea C Biot breathing D Kussmaul breathing E Painful respiration

kussmaul breathing The correct choice is D, Kussmaul breathing, which is deep regular breathing or hyperpnea. It can be seen as a compensatory action of metabolic acidosis and hypoxia. Choice A, Cheyne-Stokes respiration, is a waxing and waning pattern of rate and volume that includes periods of apnea. This can be seen in patients at high altitudes, and with severe left sided heart failure or neurologic disease. Choice B, bradypnea, is noted with a slower than usual respiratory rate and can be seen with use of CNS depressant drugs, uremia, or structural intracranial lesion. Choice C, Biot breathing, is an uncommon variant of Cheyne-Stokes respiration, with periods of apnea alternating with a series of equal breaths that end abruptly. It can be seen in patients with meningitis. Choice E, painful respiration, is relatively normal in pattern, but interrupted by pain during breathing from such disorders as pleurisy, fractured ribs, or subphrenic inflammation.

A 43-year-old Caucasian woman presents for evaluation of menstrual irregularities over the past 6 months, despite of history of prior regular menses. She had a bilateral tubal ligation (BTL) and reports multiple negative home pregnancy tests. The patient also notes weight gain and increased girth in her abdomen. She reports easy bruising without a history of trauma; she also notes the new development of "stretch marks" on her torso, and there is also weakness in her arms and legs. She denies any health changes, medications, or stressors in relation to these changes. She denies hot flashes, night sweats, and frank depression; however, she admits to some mood swings and poor libido. The patient is frustrated that her health fair labs (which include complete blood count, complete metabolic panel, lipid panel, and thyroid stimulating hormone) were all normal and do not explain her symptoms. Her past medical history reveals no chronic conditions; she does not take any medications, and she has no known drug allergies. Her only surgery was the BTL. She lives with her husband and 3 children. She works as a retail clerk and walks for exercise. She denies the use of tobacco, alcohol, and drugs. Her blood pressure is 154/92. Chart review demonstrates weight gain of 15 pounds over 6 months. She has had normal blood pressures in the past. On physical exam, you observe an overweight woman with an especially rounded, full face. She also has a fatty fullness to her neck region, and there is some central obesity; however, her arms show some muscle wasting. She has purple striae on her torso. Her skin also appears thinned, with multiple bruises. Hirsutism is observed on the patient's chin, abdomen, and breasts. The remainder of her exam was unremarkable. Testing confirms elevated ACTH and high cortisol levels in the evening and after dexamethasone suppression, as well as a pituitary macroadenoma as the cause. Question What prescription medication may be indicated after the surgical removal of her tumor? Answer Choices 1 Aldosterone 2 Insulin 3 Levothyroxine 4 Parathyroid hormone 5 Prolactin

levothyroxine Explanation This patient presents with a multitude of signs and symptoms (weight gain, menstrual irregularity, mood swings, poor libido, moon facies, fat depositions, proximal limb weakness, central obesity, skin thinning, purple striae, and easy bruising) that support a diagnosis of Cushing's syndrome or Cushing's disease. The most common cause of Cushing syndrome is considered iatrogenic, from administering exogenous steroids, such as prednisone. The next most common cause is known as Cushing's disease, and it results from increased adrenocorticotropic hormone (ACTH) secretion from pituitary, which causes increased adrenal production of cortisol. This patient's tests indicate Cushing's disease, with a pituitary macroadenoma. Surgery is indicated. If damage occurs to the anterior pituitary, it can affect hormone secretion. Often, replacement of levothyroxine, estrogen, progesterone, and possibly desmopressin may be needed. Aldosterone, which regulates sodium balance, is a hormone produced by the adrenal glands. Rarely, Cushing's disease can be caused by an adrenal neoplasm, but those cases are considered non-ACTH dependent. This patient would not be expected to have any adrenal issues following pituitary surgery. Insulin, which regulates glucose metabolism, is produced in the pancreas. This patient's Cushing's disease and subsequent pituitary surgery should not lead to the requirement of insulin prescription. Parathyroid hormone, which regulates calcium balance, is produced by the parathyroid glands. These glands can be damaged during thyroid surgeries and require subsequent management of calcium. However, this patient's parathyroid hormone should not be altered by her pituitary surgery. Prolactin, which primarily regulates milk production, is produced by the anterior pituitary gland. This patient would potentially experience problems with prolactin production following pituitary surgery if she were lactating. However, she has had surgical sterilization; therefore, she has no need for prolactin production.

A 42-year-old woman has experienced recent weight gain, heavy periods, fatigue, cold intolerance, and constipation. She has a rough voice, and her rate of speech is slow. Physical exam is significant for an enlarged thyroid, slow reflexes, and the presence of brittle and coarse hair. She denies any history of bipolar disease or treatment with lithium. Laboratory tests show an elevated TSH and low free T 4 . What is the most appropriate treatment for this patient? A propylthiouracil (PTU) B levothyroxine C surgical resection D radioiodide ablation

levothyroxine This patient's signs and symptoms are consistent with hypothyroidism. Treatment of choice is levothyroxine, which is partially converted in the body to T 3 . Significant increases are seen within 1 to 2 weeks, with maximum levels reached in 3 to 4 weeks.

A 50-year-old Caucasian man is in persistent cardiac arrest despite 2 electrical shocks and adrenaline medication. He had collapsed on a local golf course while playing with his wife. When the paramedics arrived, he had a pulse and blood pressure, but he arrested on the way to the hospital. The only medication he takes on a regular basis is an ACE inhibitor for high blood pressure. At the base of his neck, you feel a hump; you note purple stretch scars on his abdomen; in addition to his obesity, he has a moon face. You draw some blood for electrolytes. What do you expect to find? Answer Choices 1 Low potassium 2 High potassium 3 High sodium 4 Low sodium 5 Low magnesium

low potassium Trunk obesity, moon face, purple stretch marks on the abdomen, and buffalo hump are typical symptoms of Cushing's syndrome due to chronic exposure to corticosteroids. Other symptoms are muscle wasting and weakness, thin and atrophic skin, poor wound healing, easy bruising, hypertension, osteoporosis, renal stones, glucose intolerance, psychiatric problems, and virilism in females. Due to increased secretion or administration of ACTH, too much aldosterone is produced. The latter results in increased secretion of K+ and H+ ions through the kidney, and ends in hypokalemia (low potassium) and metabolic alkalosis. Cardiac symptoms are usually minimal until serum potassium levels are <3 mEq/l. ECG changes include ST-depression; decreased and sometimes negative T; U-wave, and extrasystole (remember: "no pot [assium], no t [ea], but 'U' "). In severe hypokalemia, you can find premature ventricular and atrial contractions, as well as arrhythmia, which can lead to cardiac arrest. Hyperkalemia presents in the ECG as shortening of the QT-interval and tall, peaked T-waves and then proceeds to nodal and ventricular arrhythmia, widening of the QRS complex, PR prolongation, disappearance of P-wave, and finally degeneration of the QRS complex to sine wave and asystole. Clinical symptoms are arrhythmia, paresthesias, hyporeflexia, muscle weakness, obstipation, and acidosis. Hypernatremia is usually generated when water losses exceed sodium losses in combination with inadequate water intake. The symptoms are CNS dysfunction-like confusions, seizures, coma, hemorrhages, and thrombosis. Hyponatremia is usually caused by renal retention of water, while sodium is excreted. Typical symptoms are confusion, lethargy, and altered mental status. Cerebral edema, brain herniation, coma, and death can occur. Hypomagnesemia is often caused by inadequate intake. The symptoms include tetanus, tremor, seizures, nausea, vomiting, lethargy, weakness and mental alteration.

A 56-year-old woman is being seen for regular assessment and monitoring of her type 2 diabetes mellitus. She has been following a strict diet and exercise plan for 2 years with the addition of metformin 6 months ago for an increased HgA 1c level. Her HgA 1c at today's visit is 7.1. What is the appropriate management for this patient? A add exenatide to her current therapy B change her oral therapy to rosiglitazone C add insulin to her current therapy D maintain her current therapy and recheck in 6 months

maintain her current therapy and recheck in 6 months D The HgA 1c goal for this patient is less than 6.5, with action at a level of greater than 8.0. The appropriate action at this time is to continue her current therapy and reassess in 6 months.

which of the following is the most common cause of short stature worldwide? A Acromegaly B Malnutrition C Prader-Willi syndrome D Congenital growth hormone deficiency E IGF-I receptor deficiency

malnutrition The correct choice is B, malnutrition. When not associated with chronic diseases, this is the most common cause of short stature worldwide. Children with malnutrition commonly present with failure of weight gain before growth rate decreases. A dietary history is key to the diagnosis, as well as a history of any parasites in the local area. Choice A, acromegaly, is a disorder of growth hormone excess. Choice C, Prader-Willi syndrome, choice D, congenital growth hormone deficiency, and choice E, IGF-I receptor deficiency, have been found to cause short stature, but are not seen as commonly as malnutrition.

A 78-year-old male returns to the FP office for a follow up of non-insulin-dependent diabetes mellitus (NIDDM) as a new patient to you, although he has been an office patient for the past year. He denies any problems this visit and says his blood sugars are in the 90-120 mg/dl range. He is currently taking the medications listed in the following choices. You receive his labs and note that his creatinine is 2.0 mg/dl and on the previous few labs this creatinine was also in the 1.8-2.0 mg/dl range. What medication should be discontinued? A glipizide B metformin C omeprazole D sitagliptin E atenolol

metformin The correct answer is (B). Metformin is contraindicated in this diabetic patient with chronic renal failure due to an increased risk of lactic acidosis and should be discontinued. Sitagliptin requires a dosing adjustment in renal failure but is not contraindicated in this patient. Glipizide (sulfonylurea), omeprazole (a PPI for GERD), and atenolol (a beta blocker for hypertension) are not contraindicated in this patient. Beta blockers should be used with caution in diabetics due to the potential of masking symptoms of hypoglycemia, but are not contraindicated.

Your patient is a 77-year-old male with a history of hypertension. For about the past 50 years has smoked a pipe daily. He feels great but admits that his cholesterol was elevated for the past 5 years, and has declined treatment. His best friend died of an myocardial infarction last week and the patient now agrees to treatment for his hyperlipidemia. His LDL is 285 mg/dl and HDL is 30 mg/dl. You decide to put him on simvastatin 80 mg QD. Prior to initiation, you advise the patient to notify you about which of the following potential side effects of this medication? A cough B double vision C myalgias D elevated blood pressure E restlessness

myalgias The correct answer is (C). Myalgias are common side effects of statins, which may result in a patient discontinuing the medication. If the patient develops significant myalgias a CPK may be ordered to rule out myositis and if elevated the statin may need to be discontinued. Elevated liver enzymes may occur, which may result in discontinuation of the medication. The other options are unlikely side effects of statins.

Psammoma bodies are concentric calcified structures and are found in several tissue pathologies. In the image, psammoma bodies can be seen. Psammoma bodies are characteristic of which of the following? Answer Choices 1 Papillary carcinoma of the thyroid 2 Mucinous cyst adenocarcinoma of the ovaries 3 Medullary carcinoma of thyroid 4 Follicular adenoma 5 Pheochromocytoma

papillary carcinoma Single necrotic cells may form a nidus for deposition of calcium. Subsequently, the progressive acquisition of an outer layer of calcium may form a lamellated structure, which resembles grains of sand. This is called a psammoma body. Psammoma bodies are seen in some neoplastic conditions, such as papillary carcinoma of the thyroid, serous cyst adenocarcinoma of the ovary, adenocarcinoma of the endometrium, and meningioma. Rarely, it may be seen in adenocarcinoma of the lung. In the case of the ovary, the possible mode of formation is by neoplastic and histiocytic cellular degeneration. Psammoma bodies (blue arrow) - calcific concretions with well-defined concentric laminations

A 31-year-old woman is being evaluated for irregular, infrequent menstrual periods. On further questioning, she complains of headaches, fatigue, and breast discharge. She takes ibuprofen only occasionally. Which of the following labs would most likely be elevated in this patient? A BUN and creatinine B luteinizing hormone (LH) and follicle-stimulating hormone (FSH) C oxytocin D prolactin E TSH

prolactin This patient's symptoms are consistent with a pituitary adenoma. Prolactinomas account for about half of all functioning pituitary tumors and may secrete PRL, GH, and ACTH.

What is the definitive treatment of choice for elderly patients diagnosed with Graves' disease? A Beta blocking agents B Levothyroxine C Methimazole D Total thyroidectomy E Radioactive iodine

radioactive iodine

A 5-year-old girl is seen in your office with a several week history of increased thirst, weight loss, and blurred vision. She has a positive family history for diabetes mellitus, hypertension, and stroke. Her urine dipstick chemical testing reveals positive glucose and negative ketones, protein, blood, and nitrites. Which of the following laboratory test results would support a diagnosis of diabetes mellitus in this patient? A random plasma glucose > 200 mg/dL B random urine glucose dipstick > 1+ C plasma hemoglobin A1c < 7% D fasting plasma glucose > 110 mg/dL E 2-hour postprandial plasma glucose > 135 mg/dL

random plasma glucose > 200 mg/dL

A 12-year-old boy is being seen for concerns of development of breast tissue. Upon physical exam, he is noted to have a firm, slightly tender mass under the left areola. What is the most appropriate action at this time? A referral to pediatric surgery for resection B measurement of serum hCG C measurement of testosterone and estrogen levels D reassurance and observation

reassurance and observation Type 1 idiopathic gynecomastia in adolescent men presents with a firm mass under the areola ("breast bud") typically during sexual maturation stages (SMR), stages II to III. This is a result of normal estrogen and androgen activity at the breast tissue level. Appropriate action is observation and to reassure the patient that the condition will likely resolve in 1 to 2 years.

A 50-year-old male is seen with a routine check-up. He is concerned about the possibility of developing diabetes mellitus. He has a negative family history of diabetes. He has no signs or symptoms of diabetes and he is not overweight. Without any risk factors for diabetes, what is the recommended screening protocol for this patient according to the American Diabetes Association (ADA)? A screen all men over 25 years of age every five years B screen all men over 35 years of age every two years C screen everyone over 45 years of age every three years D no screening is necessary without risk factors E no screening is necessary without a family history of diabetes

screen everyone over 45 years of age every three years The correct choice is C, screen everyone older than 45 years of age every three years. In addition, the ADA recommends screening for younger people if they are overweight and have at least one additional risk factor, such as positive family history, hypertension, and/or vascular disease. The other choices are not recommended by the ADA for screening the general population for diabetes mellitus.

A 50-year-old male is seen with a routine check-up. He is concerned about the possibility of developing diabetes mellitus. He has a negative family history of diabetes. He has no signs or symptoms of diabetes and he is not overweight. Without any risk factors for diabetes, what is the recommended screening protocol for this patient according to the American Diabetes Association (ADA)? A screen all men over 25 years of age every five years B screen all men over 35 years of age every two years C screen everyone over 45 years of age every three years D no screening is necessary without risk factors E no screening is necessary without a family history of diabetes

screen everyone over 45 yo every 3 years The correct choice is C, screen everyone older than 45 years of age every three years. In addition, the ADA recommends screening for younger people if they are overweight and have at least one additional risk factor, such as positive family history, hypertension, and/or vascular disease. The other choices are not recommended by the ADA for screening the general population for diabetes mellitus.

An extremely heavy 12-year-old girl comes to the practice with her grandmother for new patient evaluation, bringing old records with her. Her blood pressure today is mildly elevated. Which of the following parameters will help determine whether her overweight and elevated blood pressure are due to Cushing syndrome (adrenocortical hyperfunction) rather than exogenous obesity? A advanced skeletal maturity B heavy thighs and legs C pinkish striae D short stature E slightly increased growth rate

short stature Children with Cushing syndrome typically have short stature, while those who are obese due to exogenous factors have normal or tall stature. Likewise, they tend to have delayed skeletal maturity (A), truncal obesity with thin extremities (B), purplish striae (C), and a slowed growth rate (E), while obese children have advanced maturation, heavy extremities, pinkish striae, and an increased growth rate.

A 28-year-old woman presents with nervousness and palpitations associated with heat intolerance. On examination, there is no evidence of thyromegaly, but there is a palpable nodule that is "hot" on a thyroid scan. The TSH was low and T3 and T4 were both elevated. Which of the following is the recommended treatment for this patient? A Propylthiouracil (PTU) B Thyroid lobectomy C Total thyroidectomy D Radioiodine ablation

thyroid lobectomy In Graves' disease, the thyroid is diffusely enlarged in contrast to a toxic adenoma in which the thyroid is normal sized but with a palpable nodule. Surgery is the treatment of choice for a toxic adenoma. Surgical treatment of a toxic adenoma is a thyroid lobectomy and isthmusectomy. A subtotal or total thyroidectomy is indicated for toxic multinodular goiters or Plummer disease. Thionamides and radioiodine ablation are not effective therapies for toxic adenomas. (Coe, 2006, pp. 404-406) Coe NPW. Surgical endocrinology: thyroid gland.

Following a total thyroidectomy for papillary carcinoma, a 72-year-old man develops a heart rate of 140 and a temperature of 104.8F. He vomits almost continuously and has severe diarrhea. He is disoriented and mildly combative. Electrocardiography demonstrates sinus tachycardia. What is the most likely diagnosis? A pulmonary embolism B pneumonia C sepsis D thyroid storm E wound infection

thyroid storm Although rare, thyroid storm or crisis can occur following thyroid surgery, administration of radioactive iodine, or a stressful illness. It is characterized by extreme tachycardia, vomiting, diarrhea, dehydration, delirium, and high fever. Pulmonary embolism (A) may include tachycardia, along with chest pain and shortness of breath, but is unlikely to include GI symptoms. Pneumonia (B) and sepsis (C) are certainly possible, although the extreme nature of her findings more strongly suggest thyrotoxicosis. Wound infection (E) would more likely to present with localized tenderness and a less marked fever.

A 43-year-old obese man presents for a health maintenance visit. On physical exam, it is noted that his waist circumference is 106 cm and blood pressure is 148/92 mm Hg. Which of the following fasting laboratory levels would suggest a diagnosis of metabolic syndrome (syndrome X) in this patient? A HDL of 45 mg/dL B LDL of 180 mg/dL C triglyceride of 190 mg/dL D glucose of 100 mg/dL

triglyceride of 190 mg/dL Metabolic syndrome is found in approximately 25% of Americans. It is defined as three or more of the following findings: waist circumference of greater than 102 cm in men or greater than 88 cm in women; serum triglyceride level of at least 150 mg/dL, HDL level of less than 40 mg/dL in men or less than 50 mg/dL in women; blood pressure of at least 130/85 mm Hg; and serum glucose level of at least 110 mg/dL.

A 42-year-old woman has a developed a solid and quite firm thyroid mass that is approximately 1 cm diameter by palpation. She does not have any hoarseness, difficulty breathing or swallowing, or symptoms of thyroid disease. She has never been exposed to radiation and has no history of cancer, nor does she have a family history of thyroid or other cancers. A thyroid panel is within normal limits. What is the next step in the care of this patient? A fine-needle aspiration biopsy of the nodule B observation to see if the nodule increases in size C radionuclide thyroid scan D resection of the nodule E ultrasonography of the neck

u.s of the neck

An 8-year-old child presents with acute fever. There is redness of the skin in the neck area. The thyroid gland is extremely tender to palpation. What diagnostic test could confirm the most likely diagnosis? Answer Choices 1 Serum T4 concentration 2 Serum TSH measurement 3 T4 and TSH measurements 4 24 hour uptake of radioactive iodine 5 Ultrasound of neck

ultrasound Explanation The symptoms are highly suggestive of acute suppurative thyroiditis. In that case, serum T4, TSH, and 24-hour uptake of radioactive iodine are normal. Ultrasound of the neck is a useful and non-invasive way to confirm the diagnosis. If an abscess is identified, ultrasound-guided needle aspiration can provide a sample for identification of the bacterial organism responsible for the infection. The most common organisms are Staphylococcus aureus and Streptococcus species.

Your patient returns to your office for a follow up for non-insulin-dependent diabetes mellitus (NIDDM). Her HgA1c in the office is 6.4%. She is concerned about developing kidney disease from her diabetes and requests that you test her for this. What initial screening test should you order that would provide clues to potential diabetic nephropathy allowing for treatment to slow the disease progression? A 24-hour urine for protein B serum BUN/CR C urine microscopic D urine microalbumin E serum protein

urine microalbumin The correct answer is (D). An easy office dipstick or laboratory test for urine microalbumin should be done initially and periodically on diabetic patients who are at risk for diabetic nephropathy. Treatment should be initiated if microalbuminuria is found to slow disease progression. A urine microscopic for renal casts may be helpful if the patient has symptoms of kidney disease, but is not an initial screening test. Serum BUN/CR and GFR are useful tests for patients with known diabetic nephropathy to indicate the stage of chronic renal failure but is not elevated early in the disease progression, before urine microalbumin. A 24-hour protein is not indicated in this case as an initial screening test.

A 10-year-old child is seen with his parents for a routine check up. During the review of symptoms, his parents mention that their son has been extremely thirsty and is going to the bathroom to urinate frequently. The patient agrees. The parents are concerned that their son has developed diabetes mellitus. The family history is negative for diabetes mellitus, but the mother has a history of familial hypothalamic diabetes insipidus. Screening blood work includes a CBC, hemoglobin A1c, and renal function tests, all of which are within the reference range. Which of the following serum analytes would you expect to be deficient? A Sodium B Glucose C Thyroxine D Prolactin E Vasopressin

vasopressin The correct choice is E, vasopressin. The reader must first understand that the patient's symptoms are classic for diabetes insipidus, with the increased thirst, frequency, and polyuria. These symptoms are also seen in diabetes mellitus, but this would be less likely since the patient doesn't have a family history of diabetes mellitus, and his hemoglobin A1c is not elevated. Choice A, sodium, should be within range, as long as the boy is able to drink when thirsty; otherwise, he would become hypernatremic. Choice B, glucose, is incorrect. Although the signs and symptoms also fit the pattern of diabetes mellitus, the glucose would most likely be elevated in this disorder, and not low. Choice C, thyroxine, is under control of TSH from the anterior pituitary gland, and choice D, prolactin, is secreted from the anterior pituitary gland. Vasopression is secreted from the posterior pituitary gland.

A 70-year-old woman who was found barely responsive at home by her daughter is brought to the emergency department. Evaluation reveals that she is in a hyperglycemic hyperosmolar state with a severe fluid deficit. Treatment is initiated with vigorous saline rehydration and a continuous infusion of insulin. At what point should her glucose be added to her treatment? A when her condition becomes stable B when her urine output reaches 50 mL/hour C when her blood glucose reaches 250 mg/dL D if she develops hypokalemia E if she begins to spill ketones in her urine

when her blood glucose reaches 250 mg/dL In hyperglycemic hyperosmolar states, the serum glucose rapidly corrects with fluid administration alone. However, with vigorous rehydration, glucose may fall precipitously and lead to severe hypoglycemia. To avoid this, glucose should be added to water, half-normal, or normal saline as soon as the patient's blood glucose is less than or equal to 250mg/dL. She should continue to receive insulin IV until she is stabilized (A) when it can be switched to subcutaneous administration. The goal of fluid therapy in this patient is restoring her urine output to 50 mL per hour (B) or more. Because insulin drives potassium into the cells and can cause hypokalemia (D), potassium chloride should be given unless the patient has chronic kidney disease or oliguria. Persons in a hyperglycemic hyperosmolar state typically do not spill ketones (E) the way persons with diabetic ketoacidosis do.


Set pelajaran terkait

World Geo First 25 US State Capitals Quiz 9/27/23

View Set

NUR-227 PEDS TEST 2 (Immunizations (Sherpath))

View Set

ch 2 Family centered care and cultural considerations

View Set

Using your Hardware and Software

View Set

Maternity: Women's Health/Disorders and Childbearing Health Promotion Set#1

View Set

CIS - Networking Fundamentals - 150 - Quiz 3

View Set